AAP CME Cardiology

Lakukan tugas rumah & ujian kamu dengan baik sekarang menggunakan Quizwiz!

A 17-year-old adolescent girl had a cardiac arrest while competing in the 100-meter breaststroke at her high school natatorium. She was resuscitated by a bystander who performed cardiopulmonary resuscitation and received a shock from an automated external defibrillator. She arrived at the nearby children's hospital and underwent an electrocardiogram (Figure ). Of the following, the MOST likely cause of her aborted sudden death event is a mutation in A.KCNQ1 B.MYH7 C.PKP2 D.RYR2

A.KCNQ1 For the patient in this vignette, the most likely cause for her aborted sudden death is long QT syndrome (LQTS), which affects approximately 1 in 2,500 people and predisposes them to having ventricular arrhythmias, particularly torsades de pointes. To date, at least 16 mutations are known to cause different types of LQTS, with LQTS type 1 being the most common. A mutation in KCNQ1 is a potassium channel mutation that predisposes a person to having LQTS type 1. Individuals with LQTS type 1 are at risk of developing arrhythmias in particular situations, including exercise—particularly swimming. Patients with LQTS type 2 (KCNH2) tend to have events when hearing sudden loud noises, experiencing extreme emotions, or being startled, whereas patients with LQTS type 3 (SCN5A) tend to have events during sleep. The electrocardiogram (Figure ) demonstrates a prolonged QT interval. Mutations in MYH7 predispose a person to develop hypertrophic cardiomyopathy. Hypertrophic cardiomyopathy is the most common cause of sudden death in young athletes in the United States. Most common electrocardiographic changes include ventricular hypertrophy, abnormal ST, and T-wave changes. Affected individuals can develop asymmetrical hypertrophy of the interventricular septum. These people are at risk of experiencing sudden death secondary to ventricular arrhythmias. Risk factors for sudden death in people with hypertrophic cardiomyopathy include: Severe septal hypertrophy (> 30 mm) Unexplained syncope Nonsustained ventricular tachycardia Family history of sudden death Abnormal blood pressure response to exercise Mutations in PKP2 predispose an individual to develop arrhythmogenic right ventricular cardiomyopathy (ARVC). Patients with ARVC develop fatty infiltration of the myocardium and are at risk of experiencing ventricular arrhythmias and sudden death. Most patients develop symptoms in the third and fourth decade of life, but events can occur before those ages. Electrocardiographic changes that can be seen in patients with ARVC are Epsilon waves (terminal notching) in V1 and T-wave inversion in V1 to V3. Cardiac magnetic resonance imaging can demonstrate the fatty infiltration seen in patients with ARVC. Mutations in RYR2 predispose a person to develop catecholaminergic polymorphic ventricular tachycardia. Patients with this condition are at risk of developing ventricular arrhythmias, in particular bidirectional ventricular tachycardia, during high adrenergic states. Baseline electrocardiograms in patients with catecholaminergic polymorphic ventricular tachycardia are normal. Exercise stress test or epinephrine challenges can bring out the classic bidirectional ventricular tachycardia seen in patients with this condition. Although not one of the choices, Brugada syndrome is another genetic condition that can predispose patients to sudden death. Ventricular arrhythmias tend to occur during sleep, febrile illnesses, or hyperthermia. One of the most common gene abnormalities is a loss of function in SCN5A. The classic electrocardiographic findings include right bundle branch block pattern with ST segment elevation in V1 to V3. PREP Pearls The most common genetic causes of sudden cardiac death in the young include hypertrophic cardiomyopathy, arrhythmogenic right ventricular cardiomyopathy, long QT syndrome, catecholaminergic polymorphic ventricular tachycardia, and Brugada syndrome. A mutation in KCNQ1 affects the potassium channels and predisposes an individual to develop long QT syndrome type 1.

You are asked to evaluate a male newborn who failed pulse oximetry screening. Saturations are 80% in room air. He appears comfortable with normal work of breathing. On auscultation his lung fields are clear and there is an S1-coincident murmur along the the left lower sternal border. There is no hepatomegaly. The cardiac silhouette on chest radiograph appears enlarged. Echocardiography is performed (Video 1 and Video 2). What additional cardiac abnormality is MOST likely to be present? A.atrial septal defect B.partial anomalous pulmonary venous return C.pulmonary atresia D.ventricular septal defect

A.atrial septal defect The newborn in this vignette has Ebstein anomaly of the tricuspid valve. Echocardiography is the gold standard for diagnosis and demonstrates the salient feature: apical displacement of the hinge point of the septal leaflet, which is best evaluated in the apical 4-chamber view. The measured distance from the insertion point of the anterior mitral leaflet to the hinge point of the tricuspid septal leaflet, corrected for body surface area, is the displacement index. An index greater than 8 mm/m2 is consistent with the diagnosis. Tricuspid regurgitation is typically severe and audible on examination. An atrial septal defect is the most common associated abnormality and permits interatrial shunting that results in cyanosis. Associated defects that are less common than an atrial septal defect include pulmonary stenosis, pulmonary atresia, functional pulmonary atresia, ventricular septal defect, and mitral valve prolapse. Additional associations are left ventricular noncompaction, bicuspid aortic valve, aortic valve atresia, coarctation of the aorta, and patent ductus arteriosus. Ebstein anomaly is a complex, heterogeneous lesion that can present at any age. Embryologically, there is failure of delamination of the tricuspid valve leaflets, primarily the septal and inferior/posterior leaflet. These leaflets do not fully separate from the underlying ventricular wall, which results in varying degrees of apical displacement of the septal leaflet with anterior rotation of leaflet tissue towards the right ventricular outflow tract. In the most severe cases, the hinge point of the septal leaflet is not visualized in the 4-chamber imaging plane (Video 3 ), because the septal leaflet is rotated so far anteriorly as to reside within the right ventricular outflow tract. Age at presentation often correlates with severity of the tricuspid valve disease. Diagnosis in utero, especially at less than 32 weeks' gestation, is associated with high perinatal mortality. Presentation in infancy occurs due to cyanosis and/or heart failure. Poor prograde flow through the right ventricle and severe tricuspid regurgitation with right-to-left atrial level shunting results in cyanosis. As pulmonary vascular resistance falls over the first few weeks after birth, prograde flow through the right ventricle improves, which may lessen the degree of cyanosis. Older patients may come to medical attention because of exertional fatigue, murmur, or arrhythmias; ventricular preexcitation can be seen in up to 30% of patients. PREP Pearls Due to failure of leaflet delamination in Ebstein anomaly, the septal hinge point of the tricuspid valve is displaced inferiorly (> 8 mm/m2) towards the right ventricular apex and is rotated anteriorly towards the right ventricular outflow tract. In Ebstein anomaly, there is a wide spectrum of anatomic severity, pathophysiology, symptoms, and natural history. The most common associated abnormality is an atrial septal defect (> 80%).

A 12-year-old boy with behavioral problems is referred for cardiac evaluation before starting another psychiatric medication. He has no history of palpitations, chest pain, or syncope. There is no family history of cardiac disease, arrhythmias, or sudden death. You obtain an electrocardiogram (ECG; Figure 1). Of the following, based on his EKG in Figure 1, which arrhythmia below is he most at risk for A. Figure 94.2 B. Figure 94.3 C. Figure 94.4 D. Figure 94.5

C. Figure 94.4 The patient in the vignette has a baseline electrocardiogram (ECG; Figure 1) with a corrected QT interval of 480 milliseconds. The corrected QT interval is obtained by dividing the measured QT interval by the square root of the preceding R-R interval. In general, a normal QTc interval is less than or equal to 440 milliseconds, however, normal values differ based on age and gender. Women and adolescent girls often have a longer QTc interval, up to 460 milliseconds. Infants, particularly newborns, may also have a longer QTc interval. A prolonged QT interval is the result of abnormal cardiac repolarization. This can be because of many causes, including electrolyte abnormalities or alterations in ion channel function secondary to drugs or genetic mutations. This patient may have drug-induced (or acquired) long QT syndrome, because he is taking several psychiatric medications. Many classes of psychiatric medications affect the function of ion channels, leading to prolongation of the QT interval. Drug-induced QT prolongation is usually the result of the blockade of the rapid delayed rectifier potassium current. Blockade of this current delays phase 3 rapid depolarization, prolonging repolarization. It is also possible that the patient has congenital long QT syndrome, secondary to a gene mutation in a potassium or sodium channel involved in cardiac repolarization. His negative family history is reassuring, but does not rule out this possibility. The diagnosis could be clarified by stopping or decreasing the medications that prolong the QT interval and repeating his ECG. The presence of a prolonged QT interval, particularly greater than 500 milliseconds, significantly increases the risk for torsade de pointes (TdP), as pictured in Figure 4 (Response C). TdP is a form of polymorphic ventricular tachycardia characterized by alterations in the QRS amplitude that lead to the appearance of QRS complexes twisting along a line. Abnormal repolarization and variations in the speed of repolarization in different areas of the myocardium create a substrate that can lead to TdP by a re-entrant mechanism. TdP often occurs following a pause. Early after-depolarizations are more common with prolonged repolarization and increase the occurrence of ventricular extrasystoles which are followed by a pause. TdP can degenerate into ventricular fibrillation or can spontaneously terminate. Figure 3 (Response B) shows sustained supraventricular tachycardia. Figure 2 (Response A) shows atrial fibrillation. Prolonged QT interval does not typically affect atrial conduction or lead to supraventricular tachycardia or atrial fibrillation. Figure 5 (Response D) shows monomorphic ventricular tachycardia. Monomorphic ventricular tachycardias are typically the result of an abnormal automatic focus in the ventricles or a reentrant circuit located in 1 area of the ventricles leading to QRS complexes of similar morphology. Because of the features discussed before, this is not typically the result of prolonged QT interval. PREP Pearls Prolonged QT interval can result from electrolyte imbalances, drug effects, or gene mutations that lead to abnormal repolarization. Prolonged QT interval increases the risk for torsade de pointes.

You are presenting the case of a 1-week-old infant with cyanotic heart disease at a surgical conference. He is the product of a full-term pregnancy complicated by gestational diabetes. He is receiving prostaglandin E1. He weighs 3,100 g and his oxygen saturation is 83% in room air. His right ventricular angiogram is reviewed (Figure 1 ). Of the following, the BEST treatment for this infant is A. placement of bilateral systemic artery to pulmonary artery shunts B. radiofrequency perforation of the pulmonary valve and balloon valvuloplasty C. right ventricular outflow tract reconstruction and ductal ligation D. unifocalization of aortopulmonary collaterals

C. right ventricular outflow tract reconstruction and ductal ligation For the infant in the vignette, right ventricular outflow tract reconstruction and ductal ligation is the best treatment option. The patient has pulmonary atresia and ventricular septal defect. Delineation of the sources of pulmonary blood flow is necessary to plan for surgical intervention. The angiogram (Figure 2) demonstrates confluent normal-sized pulmonary arteries. The ductus arteriosus is the only source of pulmonary blood flow. There are no major aortopulmonary collateral arteries (MAPCAs). The aberrant right subclavian artery from the left aortic arch is an incidental finding. This patient has a favorable anatomy for complete repair, which would include reconstruction of the right ventricular outflow tract, VSD closure, and ligation of the ductus arteriosus. An alternative to complete repair would be a staged repair. The pulmonary vasculature is often very complex in pulmonary atresia and ventricular septal defect. Confluent true pulmonary arteries are present in approximately 70% of cases. The presence of a ductus arteriosus is associated with the presence of true pulmonary arteries. The patent ductus arteriosus typically arises from the underside of the aortic arch, as seen in this patient. It also can arise from the innominate artery. The presence of bilateral ductus arteriosus is associated with discontinuous pulmonary arteries. Sources of pulmonary blood flow include the ductus arteriosus or MAPCAs. Individual lung segments may receive flow from the true pulmonary artery, MAPCAs, or both. MAPCAs are present in 30% to 65% of patients and they are usually multiple. They most commonly originate from the descending thoracic aorta but also can arise from the subclavian arteries, abdominal aorta, or coronary arteries. MAPCAs are prone to stenosis, which may develop with time. The goal of treatment is to separate the pulmonary and systemic circulations. There are 3 major components of repair and they often are not accomplished in 1 procedure. Reconstruction of the right ventricular outflow tract is most often accomplished with a right ventricle-to-pulmonary artery conduit. The second component is reconfiguring the pulmonary blood flow. The size of the true pulmonary arteries and the presence of MAPCAs influence the complexity of this portion of the repair. The difficulty of the repair increases with the complexity of pulmonary vasculature. The third component is closure of the ventricular septal defect. Placement of bilateral systemic artery to pulmonary artery shunts is not appropriate because this patient has confluent pulmonary arteries. That procedure may be appropriate for discontinuous and hypoplastic pulmonary arteries. Radiofrequency perforation of the pulmonary valve and balloon valvuloplasty is a treatment option for pulmonary atresia with intact ventricular septum. It is not the standard of care for this disease. The lack of MAPCAs in this patient makes unifocalization unnecessary. PREP Pearls Pulmonary atresia and ventricular septal defect is a complex lesion that often has complex pulmonary artery vasculature. Identification of all the sources of pulmonary blood flow is crucial to the management of pulmonary atresia and ventricular septal defect cases.

A primigravida woman at 22 weeks' gestation with a spontaneous singleton pregnancy is seen for fetal echocardiography indicated by abnormal results from screening ultrasonography in her primary obstetrician's office at 18 weeks' gestation. The fetal echocardiography reveals a rudimentary left ventricle (Figure) and retrograde flow in the ascending aorta. Of the following, the developmental abnormality MOST likely associated with the underlying diagnosis is A.disruption of endocardial cushion formation B.interrupted incorporation of pulmonary vein into left atrium C.posterior and leftward deviation of septum primum D.posterior and rightward deviation of septum primum

C.posterior and leftward deviation of septum primum The fetal cardiac diagnosis in this case is hypoplastic left heart syndrome (HLHS), which occurs when the structures of the left side of the heart including the mitral valve, left ventricle, aortic valve, and aorta are incompletely formed and inadequate to support the systemic circulation. In gestation, the defect is characterized by the presence of a diminutive or absent left ventricular chamber and markedly underdeveloped or atretic mitral and aortic valves. These abnormalities lead to abnormal flow patterns, notably left-to-right atrial level shunting through the foramen ovale and retrograde flow in the ascending aorta supplied by the ductus arteriosus. Depending in part on whether the effect of disruption of normal development is evident early or late in gestation, multiple factors may be associated with the development of HLHS. When the disruption occurs early in gestation, it typically is caused by posterior and leftward deviation of the superior edge of septum primum resulting in abnormal attachments to the left atrial wall. These result in the development of an intact atrial septum or a small foramen ovale that may close over time and restricted atrial-level shunting. Because the fetal left ventricle is primarily filled by flow through the foramen and is dependent on that filling for normal formation, this perturbation of flow into that chamber leads to impairment of its development as well as of its associated structures. Ultimately shunting in the fetal heart with HLHS is reversed from the normal right-to-left fetal shunt. The return from the pulmonary veins enters into the left atrium, crossing the small foramen ovale into the right atrium, the right ventricle to the pulmonary artery, and then the majority of flow through the ductus arteriosus into the descending aorta. Depending on the severity of left heart obstruction, retrograde flow into the ascending aorta may be present, where it may provide the only supply to the cerebral and myocardial circulations. It has been shown in animal models that if the right atrial size is limited after the left atrium has been ligated and anterograde flow into the left ventricle restricted, then physiologic development of the left ventricle can be restored as flow is forced right to left across the atrial communication. Later in gestation the left ventricle's growth and development may be compromised by altered flow dynamics that are caused by intrinsic abnormalities in the formation of the left heart valves, particularly the aortic valve, which is derived from endocardial cushions in the outflow tract of the primitive looped heart tube. Significant obstruction to flow across the aortic valve causes shear stresses in the developing ventricle that lead to abnormal growth and remodeling with the cavity of the ventricle being diminished and the walls becoming abnormally thick. Eventually these changes may result in endocardial fibroelastosis and a physiologic state of restrictive cardiomyopathy. In this condition, both the left ventricular end diastolic and left atrial pressures rise, leading to decreased shunting through the foramen ovale and less flow into and worsening underdevelopment of the left heart. Pulmonary venous abnormalities are also associated with later gestation abnormalities in left ventricular growth and development. Early in gestation there is limited flow through these vessels but as the pulmonary circulation increases in the third trimester and the foramen ovale becomes relatively restrictive, the left ventricular preload becomes more dependent on pulmonary venous return. There are multiple potential developmental abnormalities that may lead to hypoplastic left heart syndrome in a fetus. Some of these are dependent on when in gestation the underlying defect is expressed. When the disease is evident early in gestation with already profound underdevelopment of the left ventricle, the most likely cause is abnormal flow into the left heart caused by restriction at the atrial septum that itself is caused when the superior edge of the septum primum is deviated posteriorly and leftward and forms abnormal attachments in the left atrium. In other cases, particularly those seen later in gestation after a normal-sized left ventricle had been identified previously, primary abnormalities in the aortic valve or the pulmonary veins may be responsible. With increasing understanding of the embryologic abnormalities responsible for HLHS and the time in gestation at which they occur, advances in fetal intervention may be possible to circumvent the development of at least some cases of this disease. PREP Pearls Restricted atrial shunting and underfilling of the left ventricle is the most likely early cause of hypoplastic left heart syndrome. Later in gestation after a normal-sized left ventricle had been identified previously, primary abnormalities in the aortic valve or the pulmonary veins may be responsible.

A 6-month-old infant presents to the emergency department with poor feeding and decreased energy for the past few hours. He has no fever or history of infection. On examination he weighs 8 kg (50th percentile), and his heart rate is 224 beats/min, respiratory rate is 30 breaths/min, blood pressure is 86/50 mm Hg, and oxygen saturation is normal. He is crying, with good air entry. He has tachycardia, but heart sounds are otherwise normal. No hepatomegaly is noted. An electrocardiogram is obtained (Figure). Of the following, the MOST concerning complication for this tachycardia is A. medication toxicity B. recurrent episodes of supraventricular tachycardia C. sudden cardiac arrest D. tachycardia-induced cardiomyopathy

D. tachycardia-induced cardiomyopathy Patients with supraventricular tachycardia (SVT) can have variable presentations. Older patients may have symptoms of palpitations, whereas infants often display nonspecific findings of decreased energy and poor feeding. Obtaining an electrocardiogram (ECG) is necessary to aid in the diagnosis whenever there is tachycardia. In sinus tachycardia, it is uncommon for the heart rate to exceed 220 beats/min, so consideration has to be made for an arrhythmia as in this vignette. One can refine the differential for SVT by assessing the RP intervals. Short RP tachycardia could be caused by atrioventricular nodal reentrant tachycardia or an atrioventricular accessory pathway. Long RP tachycardia could be caused by sinus tachycardia, ectopic atrial tachycardia (EAT), or permanent junctional reciprocating tachycardia (PJRT). The ECG in the vignette shows SVT with a long RP and abnormal P-wave axis. The abnormal P-wave axis rules out sinus tachycardia. The ECG in this vignette is from a patient with PJRT. PJRT is often an incessant slow tachycardia with retrograde P waves in leads II, III, and aVF. However, based on ECG alone, it may be hard to determine whether this is EAT or PJRT. EAT can show 1:1 conduction but often has rate variability whereas PJRT only displays 1:1 conduction. Both present with abnormal P-wave axis. The concern with PJRT is for tachycardia-induced cardiomyopathies from the incessant nature of this type of tachycardia which was observed in 18% of patients with PJRT in a recent series. Medication choices for treating PJRT, like many SVTs is variable depending on the medical center and the physician involved, with beta-blockers being the most commonly chosen. From case reports and case series, oral propranolol appears to have a favorable safety profile in children. The more serious reported complications in the pediatric population include bradycardia, hypotension, hypoglycemia and bronchospasm. Hypoglycemia in infants has being reported even in fairly low doses. Infants should be monitored for hypoglycemia and appropriate parental counseling should be undertaken. Ablation can be successful for PJRT. Sudden cardiac arrest is rare in this lesion and preceded by tachycardia-induced cardiomyopathy. PREP Pearls Supraventricular tachycardia with long RP tachycardia could be from ectopic atrial tachycardia or permanent junctional reciprocating tachycardia. Long RP tachycardia can cause tachycardia-induced cardiomyopathy.

A 10-year-old girl with a history of myocarditis and mild left ventricular dysfunction presents for follow-up examination. At her last visit, 3 months ago, her echocardiogram showed worsening of the left heart dilation. The left ventricular ejection fraction was 45%. She began treatment with lisinopril 5 mg daily after her last visit. She denies any shortness of breath, activity intolerance, lightheadedness, syncope, or palpitations since her last examination. She had an upper respiratory tract infection around the time of her last visit. Her rhinorrhea and congestion have since resolved, but her mother notes a persistent nonproductive cough. Her pediatrician tried a course of antibiotics and allergy medications, but with no resolution of her symptoms. The cough sometimes makes it difficult to sleep. There were no crackles on examination, and a chest radiograph showed clear lung fields. Of the following, the next BEST step and management option for this patient is to: A. continue lisinopril and follow up in 1 year B. continue lisinopril and add furosemide C. stop lisinopril D. stop lisinopril and start enalapril E. stop lisinopril and start losartan

E. stop lisinopril and start losartan Angiotensin-converting enzyme inhibitors (ACEIs) block the conversion of angiotensin I to angiotensin II, which leads to decreased vasoconstriction and fluid retention. ACEIs also block the conversion of bradykinin, a substance that causes bronchoconstriction and cough, to its inactive metabolite. Angiotensin receptor blockers (ARBs) act further down the renin-angiotensin-aldosterone pathway, by blocking the action of angiotensin II at its receptors. This results in decreased vasoconstriction and fluid retention, but does not inhibit the metabolism of bradykinin; in theory, ARBs should not lead to a chronic cough as seen with ACEIs. The patient in the vignette developed a dry cough after the initiation of ACEI therapy. Although the cough could have other causes, its timing and poor response to interventions make it likely to be a side effect of the ACEI. The cough is likely to persist if lisinopril is continued. Although not life-threatening, the cough is unpleasant and may lead to drug noncompliance. Cough usually stops within a few weeks of stopping the medication. The patient still has ventricular dysfunction, therefore, it is not ideal to discontinue afterload reduction. Enalapril is also an ACEI, and the side effect of cough is seen in all medications of this class. Losartan is an ARB, so it should be effective as an afterload reducer, but may be less likely to cause cough. Studies have shown similar efficacy between ACEIs and ARBs as afterload-reducing medications. Studies comparing side effect profiles have been mixed; some have shown decreased incidence of cough with ARBs, and others show equivalent incidence of cough with both drugs. Given the mixed data on side effects, an ARB trial is warranted to see if she tolerates it better than the ACEI. If the cough persists, discontinuation of both drugs and repeat trial of the ACEI or ARB can be attempted. Several drugs have been used to suppress ACEI-induced cough, but none was shown to be universally effective. PREP Pearls Angiotensin-converting enzyme inhibitors can cause cough because of inhibition of bradykinin conversion. Angiotensin receptor blockers have a similar efficacy as afterload-reducing medication, but are less likely to cause cough.

A 7-month-old female infant presents to the outpatient clinic with a new-onset murmur that was first heard during her 6-month health supervision visit. Overall, she looks well and her parents deny any concerning signs or symptoms. She has a normal heart rate and blood pressure, and pulse oximetry is 100% on room air. Her respiratory rate is 30 breaths/min. Auscultation reveals a slightly harsh grade II to III/VI systolic ejection murmur over the left and right sternal borders, but otherwise is unremarkable. Electrocardiography demonstrates QRS axis of -18 degrees, prominent P waves of 3.5 mm in voltage, and the QRS shows left ventricular hypertrophy with strain. Echocardiography reveals the findings shown in Figure 1 and Figure 2. Ventricular function is normal. Of the following, the clinical test MOST likely to result in an early intervention is A. ambulatory monitoring for 24 hours that demonstrates 5 runs of nonsustained ventricular tachycardia, from 4 to 15 beats in duration B. cardiac magnetic resonance imaging demonstrating a mass with a strong hyper-intense shell on gadolinium delayed imaging C. genetic test showing a defect of the TSC1-hamartin gene D. a magnetic resonance imaging scan showing a second tumor of similar size noted in the right ventricular apex E. a tissue biopsy demonstrating large, glycogen-filled vacuolated cells with some "spider cells" noted

A. ambulatory monitoring for 24 hours that demonstrates 5 runs of nonsustained ventricular tachycardia, from 4 to 15 beats in duration Ambulatory monitoring for 24 hours would be the preferred test for the infant in the vignette who has an intracardiac tumor, which is likely to be a rhabdomyoma or a fibroma. The electrocardiogram demonstrates evidence of tumor burden with left axis deviation, left atrial enlargement, and left ventricular strain. Ventricular tachycardia in the setting of a cardiac tumor is potentially life-threatening and should be addressed promptly. Fibromas tend to progress over time and in this setting should be resected. Rhabdomyomas may also present this way, though typically they are multiple. Ventricular tachycardia is an indication for tumor resection. Magnetic resonance imaging (MRI) could be obtained, but would not change the need to intervene early. The presence of a second mass would further cloud the picture because rhabdomyomas usually are multiple, whereas fibromas rarely are multiple. Thus, in the setting of multiple intracardiac tumors, further diagnostic evaluation may be warranted and therapy can be delayed in the absence of life-threatening arrhythmia or hemodynamic compromise. Although the clinical vignette could potentially describe a rhabdomyoma, if biopsy information were available and consistent with a rhabdomyoma, it would not result in early intervention. This is because most rhabdomyomas resolve over time. Intervention would not be indicated based on the information available in the vignette. The cardiac MRI would not necessarily result in early intervention. The cardiac MRI findings described are characteristic of a fibroma. However, the diagnosis of fibroma itself is not an indication for surgical therapy. Although surgical resection is usually recommended, it is not urgent in the absence of life-threatening arrhythmias or hemodynamic compromise. A diagnosis of tuberous sclerosis is not indicated. A genetic test showing a defect in the TSC1-hamartin gene would be present in tuberous sclerosis, and thus the mass is likely to be a rhabdomyoma. Most rhabdomyomas resolve over time, therefore intervention would not be indicated based on the available information. PREP Pearls Rhabdomyomas typically are multiple, associated with tuberous sclerosis, and regress over time. Fibromas typically are single and tend to increase in size. Indications for surgery of fibromas and rhabdomyomas include hemodynamic derangement or life-threatening arrhythmias

A 9-hour-old boy who had a prenatal diagnosis of pulmonary atresia with intact ventricular septum is undergoing cardiac catheterization. The child was delivered at term after an uncomplicated pregnancy, with a birthweight of 3,850 g. In the delivery room, his pulse oximetry reading was 80% in a fraction of inspired oxygen of 100%. Umbilical venous and arterial lines were placed and he given prostaglandin E1 therapy. His initial mean arterial blood pressure was 40 mm Hg. Physical examination revealed a single second heart sound with a II/VI holosystolic murmur at the left lower sternal border. His liver edge was palpable 4 cm below the right costal margin. Echocardiography confirmed the prenatal diagnosis. Arterial blood gas measurement (100% oxygen) showed a pH of 7.21, partial pressure of carbon dioxide of 38 mm Hg (5 kPa), and partial pressure of oxygen of 40 mm Hg (5.3 kPa). Volume was administered to maintain adequate blood pressure. Catheterization data are shown in the Table. Saturation (%) Pressure (mm Hg) Systolic/diastolic Mean Superior vena cava 43 X 20 Right atrium 47 x 22 Right ventricle 45 94/23 X Left atrium 82 X 6 Left ventricle 74 58/7 X Femoral artery 72 59/25 36 Of the following, based on the hemodynamic information, the BEST next step in management is A. balloon atrial septostomy B. balloon pulmonary valvuloplasty C. stent placement in the patent ductus arteriosus D. surgical placement of an aortopulmonary shunt

A. balloon atrial septostomy Balloon atrial septostomy is the best next management option for the patient in the vignette, who is critically ill with pulmonary valve atresia and an intact ventricular septum. The presence of hypotension, hepatomegaly, and metabolic acidosis are the result of poor cardiac output. The echocardiographic images are consistent with a small atrial level communication. The atrial septum bows leftward, and no obvious defect in seen in the 2-dimensional images. More importantly, the hemodynamic data show a markedly elevated mean right atrial pressure with mean pressure gradient of 16 mm Hg between the right and left atria. The presence of metabolic acidosis is also concerning for poor cardiac output. Obstruction at the atrial septal level in patients with pulmonary valve atresia with intact ventricular septum is uncommon and estimated to be present only 5% to 10% of the time. It can be critical if not relieved. Most patients with this anatomy have a large enough atrial communication to allow adequate systemic venous return to the left atrium. The lack of an egress from the right ventricle results in all of the systemic venous return crossing the atrial septum to maintain adequate cardiac output. Balloon atrial septostomy is an effective therapy for obstruction at the atrial septum, which does not prevent more definitive treatment for this condition. A larger atrial communication should improve the hemodynamics and result in patient stability for the next intervention. This vignette does not give enough information to allow a decision about further management options. It is unclear whether this patient is a good candidate for a 2-ventricle repair. The key determinants for that decision include the tricuspid valve size (Z score), coronary artery anatomy and perfusion, and right ventricular size. Decompression of the right ventricle would be appropriate in a patient whose parameters are closer to normal. In a patient who is felt to be a poor candidate for 2-ventricle repair surgery, an aortopulmonary shunt is the standard therapy. Alternatively some centers will place a stent in the patent ductus arteriosus. This would not be the next step for the child in the vignette. PREP Pearls Although rare, severe obstruction occurs at the atrial septum in pulmonary valve atresia with intact ventricular septum. Balloon atrial septostomy may be considered in pulmonary atresia with intact ventricular septum for decompression of a hypertensive right atrium before surgery.

A 27-year-old primigravida woman presents for evaluation of fetal bradycardia. She is carrying a singleton fetus at 24 weeks of gestation. She has undergone testing for autoantibodies to the intracellular ribonucleoproteins Ro (SS-A) and La (SS-B), but the results are not yet available. The heart appears normal, with the exception of the findings indicated in Figure 1 (fetal ventricular rate is 75 beats/min). There is no valve regurgitation, pericardial or pleural effusion, or ascites. The umbilical vessel flow is shown in Figure 2. The ductus venosus flow is given in Figure 3. Heart size and biventricular function appear normal. Of the following, the MOST appropriate therapeutic strategy at this time is A. frequent follow-up with no specific therapy currently indicated B. if anti-Ro/La+, begin therapy with dexamethasone and salbutamol C. immediate therapy with prednisone D. immediate therapy with salbutamol

A. frequent follow-up with no specific therapy currently indicated The fetus has high-grade atrioventricular block (AVB), at least 2:1 second degree heart block, and probably complete heart block. In this fetus, there is high-grade heart block in a structurally normal heart. In the absence of structural heart disease, fetal atrioventricular (AV) block is strongly linked to the transplacental passage of maternal autoantibodies to SSB/La and SSA/Ro ribonucleoproteins. These antibodies are prevalent in almost 2% of pregnant women and may induce immune-mediated cardiac and extracardiac inflammation, AV node damage, and endocardial fibroelastosis in a susceptible fetus. Complete AV block develops in 1% to 2% of fetuses exposed to maternal anti-Ro/La antibodies, typically between 20 and 28 gestational weeks. Complete heart block may be present with an atrial rate that happens to be approximately double the ventricular rate; in such cases, it can be very difficult to distinguish complete heart block from second degree heart block. Studies have shown that the risk of intrauterine fetal death is higher if AV block is diagnosed less than 20 weeks of gestation, if ventricular rate is less than or equal to 50 to 55 beats/min, if hydrops is present, and/or if left ventricular function is impaired. There is some controversy regarding utility of steroid therapy. Jaeggi et al describe a lower mortality rate in fetuses treated with steroids compared with untreated historical control subjects. However, in recent large series, transplacental treatment with steroids and betamimetics do not appear to impact mortality, though an effect on ventricular rate is possible. Recent American Heart Association guidelines state: "Unlike complete heart block (CHB) resulting from congenital malformation of the conduction system, immune-mediated block may benefit from in utero treatment with fluorinated steroids, intravenous infusion of γ-globulin (IVIG), or both. Reported benefits of dexamethasone (4-8 mg/d) include reduction of inflammation, reversal or stabilization of incomplete block, and improvement or resolution of hydrops or endocardial fibroelastosis. Important complications of dexamethasone that have been reported include growth restriction, oligohydramios, ductal constriction (conveyed also by the collagen vascular disease itself), maternal diabetes mellitus, and central nervous system side effect. Despite these potential complications, a trial of dexamethasone for second-degree AV block or first-degree AV block if there are additional cardiac findings of inflammation (echogenicity, valve regurgitation, cardiac dysfunction, effusion,etc) may be considered to prevent progression to CHB, although its usefulness is not well established. Dexamethasone treatment of fetuses with established CHB and no heart failure may also be considered with the goal of improving survival or reducing the incidence of dilated cardiomyopathy, although its usefulness has not been established given that studies to date have been retrospective and nonrandomized and have had incomplete follow-up." In the fetus in the vignette, there are no additional findings of inflammation and frequent monitoring is reasonable. It is advisable to assess the mechanical PR interval in fetuses of mothers who are known SS-A/SS-B+, as heart block is believed to be progressive in affected fetus. That is, a prolonged PR interval (first degree heart block) can progress to second degree and then third degree (complete) heart block. Mechanical PR intervals are measured from simultaneous mitral and aortic Doppler waveforms, from the onset of left atrial contraction (mitral A-wave) to the onset of left ventricular ejection (aortic pulse wave). Normative values have been published (Wojakowski A et al, 2009). In cases of first degree heart block, steroid therapy may be indicated to prevent progression to complete heart block. Salbutamol is a β-sympathomimetic that can be used to increase the fetal heart rate. As fetal mortality is greater if the fetal heart rate is less than or equal to 50 to 55 beats/min, it remains in the arsenal of potential therapies. However, in the vignette, the heart rate is 75 beats/min and thus it is not indicated currently. Prednisone and prednisolone are nonfluorinated steroids and thus will not cross the placenta to effect the fetus. They are not used to treat fetal heart block. PREP Pearls Complete heart block can develop in structurally normal heart in presence of maternal antibodies to SSB/La and/or SSA/Ro ribonucleoproteins. Therapy is controversial; fluorinated steroids cross the placenta and are used in some cases β-sympathomimetics are used to increase fetal heart rate, typically to achieve a heart rate greater than 55 beats/min.

You are reviewing the catheterization data from a 9-year-old girl with unrepaired congenital heart disease who recently emigrated from East Africa. She was born at approximately 30 weeks of gestation and was hospitalized for the first 4 months after birth. Physical examination reveals a small girl with cyanosis, a right ventricular heave, and a I/VI systolic ejection murmur at the left upper sternal border. Her second heart sound is single. Her liver edge is palpable 2 cm below the right costal margin. There is no edema. An angiogram was recorded (Figure). Hemodynamic data were obtained and are shown in the Table. Of the following, the BEST next step in the management of this patient is to A. initiate bosentan therapy B. initiate furosemide therapy C. recommend surgical intervention D. recommend transcatheter intervention

A. initiate bosentan therapy This patient with an unrepaired patent ductus arteriosus has pulmonary hypertension with severely elevated pulmonary vascular resistance that does not change with vasodilator therapy. The best next step in this situation is to initiate pulmonary vasodilator therapy. Initiating bosentan therapy as a pulmonary vasodilator is the best option. Bosentan is an oral endothelin-receptor antagonist that promotes pulmonary vasodilation. Small trials have shown beneficial effects in patients with pulmonary hypertension even in the presence of severely abnormal hemodynamics. Understanding how to analyze the hemodynamic data is important to determine the best therapeutic options for this patient. However, determining which patients are candidates for surgical repair can be difficult. Catheterization-derived criteria have been suggested to determine operability of patients with pulmonary hypertension and unrepaired congenital heart disease. A baseline pulmonary vascular resistance (PVR) of less than 6 Wood units*m2 and a pulmonary vascular resistance to systemic vascular resistance (SVR) of less than 0.3 are suggestive of a favorable outcome. Pulmonary vasodilator testing with oxygen and nitric oxide is recommended if the resting PVR is between 6 to 9 Wood units*m2. The following hemodynamic criteria are predictive of a favorable outcome from surgical repair: a PVR reduction of 20%, PVR:SVR reduction of 20%, final PVR less than 6 Wood units*m2 and final PVR:SVR resistance of less than 0.3. The calculation for resistance for blood flow is based on the Poiseuille law: R= (Pin - Pout)/Q where R is the resistance, Q is the flow, Pin is the inflow pressure, and Pout is the outflow pressure. In pediatric cardiology, the Wood unit is used as the vascular resistance unit. It is indexed to the body surface area because of the considerable size range seen in pediatrics. The normal range for PVR is 1 to 3 Wood units*m2. The formula used to calculate the pulmonary vascular resistance is as follows: Rp = (PAp - PVp)/Qp where Rp is the pulmonary vascular resistance, PAp is the mean pulmonary artery pressure, PVp is the mean pulmonary capillary wedge pressure, and Qp is the pulmonary blood flow. For this question, the pulmonary blood flow is first calculated, followed by the pulmonary vascular resistance (L/min per m2). The calculation for pulmonary blood flow is based on the Fick principle for room air: Qp = VO2/(PVO2-PaO2) = VO2/{(PVsat - PAsat)(o2 capacity)} = VO2/{(PVset - PAset)[(Hgb(13.6)]} where VO2 is the oxygen consumption; PVO2 and PaO2 represent the pulmonary vein and pulmonary artery O2 saturations, respectively; and Hgb is the hemoglobin. Multiplying the hemoglobin by 13.6 allows the oxygen capacity to be calculated. This number is used because each gram of hemoglobin in a liter of blood will combine with 13.6 mL of oxygen when the hemoglobin is fully saturated. In room air, the amount of dissolved oxygen is minimal and can be ignored. In high-oxygen conditions, the dissolved oxygen should be included in the calculations. This requires multiplying the partial pressure of oxygen (in millimeters of mercury) by 0.03 because 1 L of blood contains 0.03 mL of oxygen at an oxygen tension of 1 mm Hg: Qp=VO2/{[(PVsat)(Hgb)(13.6) + (0.03)(PVO2)] - [(PAsat)(Hgb)(13.6) + (0.03(PaO2)]} Using these formulas and the data provided, the PVR can be calculated for each condition. For the calculation of pulmonary blood flow in room air, the left atrial saturation can be used as the pulmonary vein saturation and dissolved oxygen can be ignored: Room Air Qp=137/{0.97-0.73)(14.5 x 13.6)} = 137 = 2.89/{(0.24)(197.2)} L/min/m2 For the resistance calculations, the mean left atrial pressure is used: Rp= (PAp-PVp)/Qp = (68-12)/2.89 = 19 Wood units * m2 100% Oxygen Although the partial pressure of oxygen in the pulmonary artery is included, it could be omitted without a significant change in the pulmonary vascular resistance. [Question 123.3] Nitric Oxide [Question 123.4] Diuretic therapy is used for the symptomatic relief of fluid retention. The description of this patient does not include signs of fluid retention, such as hepatomegaly and edema. In addition, diuretics should be administered with caution to avoid decreased cardiac output resulting from decreased preload. A patient of this age with severely elevated and nonreactive pulmonary vascular resistance is unlikely to survive surgical or catheter-based corrective treatments. A net right-to-left shunt in the presence of a PVR approaching 20 Wood units*m2 can only predict an unfavorable outcome after surgery. PREP Pearls When testing pulmonary vascular reactivity with supplemental oxygen, the dissolved oxygen must be included in oxygen content calculations. Therapy aimed at promoting pulmonary vasodilation may improve quality of life in patients with Eisenmenger syndrome.

You are seeing a 5-month-old former 28-week preterm infant in clinic. Her parents report rapid breathing. In addition to an increased right ventricular impulse, S2 is loud and the liver is enlarged. Her oxygen saturation on room air is 94%. Her weight has dropped from the 25th percentile to the 10th percentile. You perform echocardiography (Video 1, Video 2,and Figure 1). She undergoes cardiac catheterization. Of the following, the set of hemodynamic data that is MOST consistent with the above physical examination, vitals and echo findings is A. Right Atrium Right Ventricle Right Pulmonary Artery Right Pulmonary Capillary Wedge Left Atrium Femoral Artery 8/9/8 60/5/9 59/22/39 8/10/8 9/8/7 84/45/58 B. Right Atrium Right Ventricle Right Pulmonary Artery Right Pulmonary Capillary Wedge Left Atrium Femoral Artery 9/8/7 58/0/9 55/23/37 11/12/11 8/5/6 57/23/34 C. Right Atrium Right Ventricle Right Pulmonary Artery Right Pulmonary Capillary Wedge Left Atrium Femoral Artery 10/9/8 70/0/10 70/30/48 18/19/18 19/21/18 74/34/49 D. Right Atrium Right Ventricle Right Pulmonary Artery Right Pulmonary Capillary Wedge Left Atrium Femoral Artery 4/3/3 24/0/4 23/8/16 9/10/9 6/5/4 50/30/38

B. Right Atrium Right Ventricle Right Pulmonary Artery Right Pulmonary Capillary Wedge Left Atrium Femoral Artery 9/8/7 58/0/9 55/23/37 11/12/11 8/5/6 57/23/34 The patient in this vignette has pulmonary hypertension secondary to pulmonary vein stenosis. When enough veins are involved, patients will usually present in infancy with tachypnea, recurrent pulmonary infections, cyanosis, hemoptysis, and/or failure to thrive. There may be evidence of pulmonary hypertension (increased right ventricular impulse, accentuation of pulmonary closure with or without a pulmonary ejection click) and right-sided heart failure (hepatomegaly). Stenosis of the individual pulmonary veins, in isolation, is a rare anomaly. It can occur as a localized stenosis at the junction of one or more pulmonary veins to the left atrium or in a diffuse form where there is tubular hypoplasia over a significant intrapulmonary and extrapulmonary distance. Lesions of one vein might be asymptomatic and incidentally discovered, whereas the diffuse form can be quite malignant. Acquired pulmonary vein stenosis can be seen in former premature infants, as reported by Drossner et al. Elevations in pulmonary venous pressure eventually lead to the development of pulmonary arterial hypertension. The greater the number of pulmonary veins obstructed and the more severe the obstruction, the worse the pulmonary hypertension. Flattening of the interventricular septum with systolic bowing into the left ventricle (Video 1) indicates the presence of significantly elevated right ventricular pressures. When right ventricular hypertension is seen, a thorough assessment of downstream structures (pulmonary valve, branch pulmonary arteries, pulmonary veins, left atrium, mitral valve) is imperative to rule out an anatomic obstruction. All 4 pulmonary veins must be visualized and interrogated with both color and spectral Doppler. Pulmonary venous flow is usually evaluated in apical or parasternal views. The normal Doppler pattern is phasic and low velocity with 2 antegrade signals (S and D waves correlating to peak antegrade flow during ventricular systole and diastole, respectively) and a retrograde signal (Ar wave) representing retrograde flow during atrial contraction (Figure 2). Obstructed pulmonary veins should be suspected if there is continuous, high-velocity flow with loss of phasicity (loss of end-systolic and end-diastolic deceleration towards baseline (Video 2 and Figure 1). In addition to confirming the presence of pulmonary hypertension, cardiac catheterization will show an elevated pulmonary artery wedge pressure on the affected side and a normal left atrial pressure. The pressure gradient from pulmonary artery wedge to left atrium is analogous, but not always equal to, the measured mean gradient on echocardiography (catheterization and echocardiography usually being performed at different times and under different conditions). Elevated right ventricular end diastolic pressure may result in right-to-left atrial level shunting. The patient in this vignette had physical examination and echocardiographic findings of pulmonary hypertension along with an abnormal pulmonary venous Doppler. The hemodynamic data in Response Choice B are most consistent with pulmonary vein stenosis with secondary pulmonary hypertension. The mean gradient from wedge to left atrium is 5 mm Hg, and right ventricular pressure is systemic. Data in Response Choice A are from a patient with idiopathic pulmonary arterial hypertension. However, this can also be seen in a former premature infant with pulmonary hypertension without pulmonary vein stenosis. Right ventricular pressures are elevated (approximately 70% systemic), but no gradients are observed from the right ventricle to pulmonary artery (ruling out pulmonary valve stenosis) or from pulmonary capillary wedge to left atrium (ruling out pulmonary vein stenosis). Data in Response Choice C are from a patient with a large nonrestrictive ventricular septal defect, with equalization of ventricular pressures, pulmonary arterial hypertension, and left atrial hypertension. This example highlights how the wedge pressure can be a "surrogate" of left-sided heart pressures when there is no pulmonary vein or mitral stenosis. Although the wedge pressure is high, it reflects the increased filling pressure from the large left-to-right shunt. Wedge and left atrial pressures are equivalent in the absence of pulmonary vein stenosis. Data in Response Choice D are from a patient with pulmonary vein stenosis (wedge to left atrium gradient is 5 mm Hg), but there is no associated pulmonary hypertension. This data set, which was obtained in a patient with incidentally discovered and asymptomatic pulmonary vein stenosis, is inconsistent with the echocardiography and examination findings in this vignette. PREP Pearls Doppler echocardiography of the pulmonary veins often alerts the clinician to the presence of pulmonary vein stenosis; turbulent, high-velocity, continuous, nonphasic flow patterns in individual pulmonary veins will be seen. In pulmonary vein stenosis, cardiac catheterization often reflects pulmonary hypertension, with elevated pulmonary artery wedge pressure on the affected side (or sides) and normal left atrial pressure. Premature infants may have an increased risk of pulmonary hypertension and pulmonary vein stenosis.

In a population of 4,000 people, 200 have a given disease and the rest are well. A test has 99% sensitivity and 99% specificity for detecting the disease. The positive predictive value of the test is A. 5% B. 84% C. 95% D. 99%

B. 84% The sensitivity of a clinical test refers to the ability of the test to correctly identify those patients with disease. Thus, if 200 people in the population have the disease and the test is 99% sensitive, the test will correctly diagnose 198 out of 200 patients as positive. The specificity of a clinical test refers to the ability of the test to correctly identify those patients without the disease. Thus, if 3,800 patients in the population do not have disease and the test is 99% specific, the test will correctly diagnose 3,762 out of 3,800 patients as negative (Table). Sensitivity and specificity are independent of the population being tested. Positive predictive value (PPV) and negative predictive value (NPV), however, are both dependent on the prevalence of the disease in the population being tested. Table. Positive and Negative Test Results. Disease Present Disease Absent Test Positive 198 (TP) 38(FP) Test Negative 2(FN) 3,762(TN) TP = True Positive. FN = False Negative. FP = False Positive. TN = True Negative. The PPV = (TP)/(TP + FP) The NPV = (TN)/(TN + FN) In a population where there is high prevalence of disease (ie, 50% or more of the population), a positive result from a test with 99% sensitivity will have a PPV of nearly 99%. If the prevalence of disease in the population is high, the NPV of the test decreases. Conversely, in a population where there is low prevalence of disease (ie, less than 50% of the population), a negative result from a test with 99% sensitivity will have a NPV of greater than 99%. If the prevalence of disease in the population is low, the sensitivity of the test does not change, but the PPV of the test drops because the number of false positives in the population increases as the prevalence decreases. Using the calculation for PPV = (TP)/(TP + FP) PREP Pearls Sensitivity and specificity are independent of disease prevalence. The positive predictive value and negative predictive value of a test are dependent on disease prevalence

The attending physician in the neonatal intensive care unit (NICU) is requesting a consultation on a 2-day-old neonate who underwent omphalocele repair and peripherally inserted central catheter (PICC) placement in the morning. The baseline echocardiogram taken before surgery showed a structurally normal heart with a patent foramen ovale and patent ductus arteriosus that are appropriate for age. After the surgery, the heart rate was recorded at 180 beats/min. Currently the monitor shows a heart rate of 240 beats/min, and a 12-lead electrocardiogram (ECG) is ordered in the NICU (Figure 1). A dose of adenosine was given and the ECG was reviewed (Figure 2 ). Of the following, the BEST next step in management for the neonate in the vignette is A. a higher dose of adenosine B. D/C cardioversion C. propranalol D. verapamil

B. D/C cardioversion The electrocardiogram (EKG) strips show atrial flutter (evidenced by the saw tooth waves) at 2:1 atrioventricular (AV) conduction. The flutter waves have been unmasked by the dose of adenosine in the second EKG (Figure 2). Atrial flutter is a rare but well-tolerated asymptomatic arrhythmia in the newborn period, usually present within the first 2 days after birth. The arrhythmia occurs when there is reentry within the atrial muscle (intra-atrial macro-reentry circuit)—a single reentry circuit generating atrial flutter. The atrial rate during the flutter may vary widely among patients. Generally an atrial rate of up to 500 with 2:1 or variable AV conduction is common in infants. The AV node, which is not part of the reentrant circuit, protects the ventricle by blocking the AV conduction. Usually no association is found with structural heart defects, though severe atrial dilation from Ebstein anomaly/AV valve regurgitation from AV septal defect can precipitate flutter. When diagnosed in the fetus, this arrhythmia can present with congestive heart failure and hydrops fetalis, based on the duration of the tachycardia and the degree of AV block. The diagnosis of atrial flutter in a fetus can be delayed when there is high degree of AV block. The ventricular function normalizes once sinus rhythm is restored in the fetus. The conversion to sinus rhythm can be accomplished by antiarrhythmic therapy through placental transmission. Atrial flutter in neonates responds well to D/C cardioversion, transesophageal pacing (TEP), and/or antiarrhythmic therapy. Of the response choices listed, D/C cardioversion is the most effective way of treating atrial flutter with more than 80% conversion rate to sinus rhythm. The choice of therapy should be based on the patient's hemodynamic status and duration of the tachycardia. Instances of spontaneous conversion to sinus rhythm have been reported in the literature. Recurrence is uncommon, but can occur in the presence of additional arrhythmias like supraventricular tachycardia, atrial ectopic tachycardia, or atrial fibrillation. These patients usually need antiarrhythmic therapy for conversion to sinus rhythm and may require long-term maintenance therapy. Generally, patients who have isolated atrial flutter that responds to D/C cardioversion without recurrence do not require long-term maintenance therapy. The baseline ECG in the vignette does not show flutter waves in a clear way. However, the flutter waves are unmasked in the second strip after AV block by adenosine. Because this step is already accomplished, adenosine would not be the correct choice. Adenosine helps to diagnose the underlying rhythm because it would not convert atrial flutter to sinus rhythm. Atrial flutter in a neonate responds well to D/C cardioversion, TEP, or antiarrhythmics like digoxin, sotalol, or amiodarone. Propranalol and verapamil are not used in the management of neonatal atrial flutter. PREP Pearls Atrial flutter can be precipitated by the lower position of the peripherally inserted central catheter line. Neonatal atrial flutter responds well to D/C cardioversion without recurrence. Untreated fetal atrial flutter with rapid conduction may cause congestive heart failure and hydrops fetalis.

In the catheterization laboratory, after inserting a pigtail catheter in the arterial sheath, you hooked it to the pressure tubing and are trying to interpret the tracing (Figure 1 ). The attending physician takes over, and while you turn your back to clean a wire, she makes an adjustment, resulting in the pressure tracing (Figure 2 ). She then asks you what she did to correct the pressure tracing. Of the following, the MOST likely explanation for the difference in the tracing is that A. blood is drawn back into the catheter B. the loose connection between the catheter and pressure tubing is tightened C. the pigtail catheter is replaced with an end-hole catheter D. the transducer is "zeroed" to ambient air pressure

B. the loose connection between the catheter and pressure tubing is tightened Understanding and recognizing potential causes of artifacts is paramount when interpreting pressure tracings. The pressure tracing in Figure 1 illustrates overdampening. This common artifact is often caused by a loose connection, a significant inclusion of air or clot in the system, or a kinked catheter. Although the most likely site of a poor connection is between the catheter and the pressure tubing, any connection in the system can be responsible. If the connections are tight, inclusions of air or clot can be fixed by withdrawing the inclusion involved and flushing the catheter and tubing. If a kinked catheter cannot be straightened, it should be replaced with a new catheter. Tightening a loose connection is the only provided explanation to correct overdampening. Underdampening or "fling" is another common artifact caused by small or microbubbles in the fluid column. This artifact produces a spike at the peak systolic pressure and possibly the end-systolic pressure. These pressures are exaggerated with a higher peak systolic pressure and end-systolic pressures below the baseline. Underdampening can be corrected by flushing the tubing with vigorous tapping with a metal instrument. The tubing should be disconnected from the catheter before flushing. The appearance of underdampening can be improved by drawing blood back into the catheter. This maneuver does not correct the problem: it only smooths out the artifact. Removing the microbubbles is the only solution that results in obtaining an accurate pressure tracing. Exchanging one type of catheter for another would not correct overdampening. Recalibration or rezeroing the system will not correct overdampening, it will only change the reference point for all subsequent measurements. The quality of catheterization acquired pressure data is directly related to the integrity of the fluid column from the catheter tip to the transducer. Knowledge of common artifacts and corrective measures is required to ensure that there are no disruptions in this system. PREP Pearls Poor quality pressure tracings can result from either an overdampened or underdampened system. The ability to recognize and correct artifacts is important to obtain accurate pressure measurements.

A 12-year-old boy presents to clinic for evaluation of dyspnea with exercise. He has a history of Kawasaki disease as a young child with coronary artery aneurysms. He was on aspirin for several years and may have been on coumadin also, though details are unclear. He has been off of all medications for at least the past 3 years. He was taken out of his family's home 1 year ago following multiple exposures to potentially noxious gases, as his home was used for methamphetamine production. He currently resides in foster care. The family history is unremarkable. His physical examination is unremarkable; in particular, his lungs are clear and his heart sounds normal. Chest radiograph and electrocardiogram are normal. An echocardiogram is performed which demonstrates normal cardiac structure and function, and proximal coronary arteries appear normal, though distal coronary arteries are not well seen. There is mild tricuspid regurgitation (physiologic) with a regurgitant velocity of 3.1 m/sec. The ventricular septal contour is normal. The next BEST step on the management of this patient is: A. 24-hour Holter monitoring B. dobutamine stress echocardiogram C. exercise stress echocardiogram with metabolic assessment and spirometry D. metabolic assessment and spirometry E. methacholine challenge with spirometry

C. exercise stress echocardiogram with metabolic assessment and spirometry Accepted indications for stress echocardiography include dyspnea of possible cardiac origin, valvular heart disease, hypertrophic cardiomyopathy, and risk stratification for patients with known or suspected coronary artery disease. Pulmonary hypertension (PH) is also an indication in select cases. Risk of stress echocardiogram may include angina, syncope, sudden death, ischemia, and exercise-induced arrhythmias, based on the patient's underlying factors. The vignette describes a case of shortness of breath on exertion in a patient with history of significant coronary artery disease due to Kawasaki disease (KD), as well as new-onset PH. His history of KD puts him at risk for coronary artery stenosis. His KD risk seems likely to be more than minimal, as he was on medications for several years. He has a documented exposure to gases from methamphetamine production, which is known to put one at risk for PH. There is increased tricuspid regurgitation (TR) velocity of 3.1 m/sec, which predicts right ventricle systolic pressure of 38 mm Hg above right atrial pressure, consistent with mild PH. As such, stress echocardiography is indicated for multiple reasons. It should be performed to assess for coronary artery stenosis resulting in regional wall motion abnormalities with exercise. It also can be used to assess for worsening PH with exercise. An elevated tricuspid regurgitation gradient and/or worsening ventricular septal flattening with exercise would support a diagnosis of exercise-induced pulmonary hypertension. In this vignette, spirometry also is indicated to assess for exercise-induced bronchospasm, which may contribute to his symptoms. Response choice A is not the best answer because, while a 24-hr Holter is useful to assess for arrhythmias that can cause dyspnea, in the absence of palpitations and with a normal electrocardiogram (ECG), it is not the best choice here. Response choice B is not the best answer, as dobutamine stress echocardiography is a method to assess myocardial contractility in patients with coronary artery disease who cannot complete an exercise stress test. However, the choice of stress test method is based upon the patient's ability to perform an exercise protocol, the presence of baseline ECG abnormalities that could interfere with the interpretation of the exercise ECG, and whether or not it is important to localize ischemia or assess myocardial viability. In patients who can reach greater than or equal to 85% of their predicted maximal heart rate, symptom-limited treadmill or bicycle exercise is preferred, as it provides the most information concerning patient symptoms, exercise capacity, cardiovascular function, and the hemodynamic response during activity. Response choice D is not the best because, as mentioned previously, the echocardiography component will address regional wall motion, TR gradient, and septal flattening. Response choice E is not the best. A methacholine challenge is used to diagnose bronchoconstriction as may be seen with asthma; however, this patient has an elevated TR velocity that needs to be addressed. PREP Pearls To assess exercise-related symptoms, it is helpful to perform testing during the exercise state. Exercise stress testing is preferred over drug challenge because it provides the most information concerning patient symptoms, exercise capacity, cardiovascular function, and the hemodynamic response during usual forms of activity Pulmonary hypertension is an indication for stress echocardiography in select cases.

A 17-year-old adolescent, with hypoplastic left heart syndrome status post fenestrated lateral tunnel Fontan, presents to the emergency department with hemodynamically stable atrial flutter. You tell the patient and his parents that you would like to perform a transesophageal echocardiogram (TEE) under anesthesia before possible cardioversion, and you need to get their consent for the procedure. Of the following, the MOST common risk of a TEE in this patient would be A. arrhythmia B. esophageal trauma C. hoarseness D. infection

C. hoarseness Transesophageal echocardiography (TEE) is usually indicated in congenital heart disease (CHD) patients status post Fontan palliation who present with stable atrial arrhythmias prior to cardioversion in order to assure there is no evidence of intracardiac thrombus, particularly within the Fontan pathway that could potentially embolize to the pulmonary circulation, or in the case of a fenestrated Fontan, to the systemic circulation. Transesophageal echocardiography is considered a very safe procedure in both children and adults, with a reported incidence of serious complication rate of well below 1%. There are few large studies of the rates of TEE complications in children; however, there are larger studies of the risks of TEE in adults. The most commonly reported adverse effects are hoarseness and dysphagia. These may occur either as a result of insertion of the TEE probe itself or as a result of intubation for the TEE procedure: most TEEs in children are performed while the patient is intubated, either in the intensive care unit setting or as part of an intraoperative procedure under anesthesia or deep sedation. Hoarseness has been reported in about 10% of adult patients undergoing TEE. Minor injuries to the oropharynx can occur, including lacerations, loose or chipped teeth, and accidental extubation in those patients who are intubated during TEE. More severe gastrointestinal complications, including esophageal or gastric laceration, perforation, and bleeding, have been reported in up to 1% of adults following TEE, and may be more common than was previously reported, as more than half of these complications are only noted 24 hours or more following a TEE procedure. Esophageal perforation has been reported in several neonates and young infants. Other uncommon risks of TEE include arrhythmias (estimated risk 0.1% to 1.5%), infection, hypotension, and pulmonary complications (bronchospasm, laryngospasm, and airway compression) (all less than 1% risk). Particular TEE complications can occur with specific forms of CHD, such as obstruction of a pulmonary venous confluence in cases of total anomalous pulmonary venous return due to the common position of the pulmonary venous confluence behind the left atrium, often adjacent to the esophagus. Transesophageal echocardiography probe insertion can also cause compression of an aberrant subclavian artery arising from the descending aorta that courses posterior to the esophagus. This latter anatomic abnormality can be diagnosed during TEE if there is "dampening" of an arterial line trace present in the extremity supplied by that aberrant artery (eg, of a right radial arterial line in a patient with a left aortic arch and aberrant right subclavian artery). Finally, although infection is reported as a rare risk of TEE, antibiotic administration for endocarditis prophylaxis is not recommended prior to TEE, according to the 2007 American Heart Association prevention of endocarditis guidelines, even for patients such as in this vignette who would be considered high risk for endocarditis (those with palliated, cyanotic CHD). Specific high-level disinfection guidelines for TEE probes should be rigorously adhered to, as for all flexible endoscopes. PREP Pearls Transesophageal echocardiography (TEE) is generally considered a safe procedure. The most common risks of TEE include hoarseness and dysphagia, with other uncommon risks including oral or esophageal injury, bleeding, infection, and arrhythmia. According to 2007 American Heart Association guidelines, endocarditis prophylaxis is not recommended prior to TEE, including for patients with palliated cyanotic congenital heart disease.

You are evaluating a 15-year-old adolescent boy. He has a 1-year history of shortness of breath and chest discomfort during basketball practice. These episodes do not last long, but they have been getting worse. Recently, he passed out during a training session. He does not smoke. His maternal grandfather required cardiac surgery for a valve replacement in his late 60s. The patient has a systolic ejection murmur that is best heard between the apex and left sternal border and radiates to the suprasternal notch. When he squats from the standing position, the murmur is almost gone. Of the following, the MOST likely diagnosis is A. catecholaminergic polymorphic ventricular tachycardia B. dilated cardiomyopathy C. hypertrophic cardiomyopathy D. long QT syndrome

C. hypertrophic cardiomyopathy In obstructive hypertrophic cardiomyopathy (HCM), the left ventricular outflow tract murmur diminishes in intensity on squatting from the standing position because of increased left ventricular preload. Hypertrophic cardiomyopathy is the most common genetic cardiomyopathy. It occurs in 1 of 500 people, but it often goes undiagnosed. It is the most common cause of sudden cardiac death in young individuals and young athletes. Hypertrophic cardiomyopathy is an autosomal dominant disease that is often caused by mutations in genes encoding sarcomere proteins. It is characterized by myocyte hypertrophy and myocyte and myofibrillar disarray resulting in ventricular hypertrophy and diastolic dysfunction. Left ventricular outflow tract obstruction occurs in 25% to 30% of patients. Increased myocardial oxygen demand, increased heart rate, impaired diastolic filling, and worsening of the left ventricular outflow tract obstruction can cause symptoms during exertion. An acute intensification of obstruction during exercise can cause syncope. Elevation in left atrial and pulmonary vascular pressures may be responsible for dyspnea. Thickening of coronary arteries, impaired coronary vasodilation, and the increased myocardial oxygen demands of massively hypertrophied myocardium can cause angina pectoris. Patients with HCM have a double apical impulse, a prominent a wave in jugular venous pulse, a double carotid pulse, a laterally displaced and force apical impulse and S3 and S4. The double carotid pulse (also known as bisferiens pulse) results from progressive left ventricular outflow tract obstruction during systole, resulting in a mid-systolic decrease in the pulse followed by a secondary increase. The systolic ejection murmur of left ventricular outflow tract obstruction typically is a harsh systolic ejection crescendo-decrescendo murmur that is best heard between the apex and left sternal border and radiates to the suprasternal notch. The murmur and the gradient across the left ventricular outflow tract diminish with maneuvers that increase the preload (eg, Mueller maneuver, squatting) or afterload (eg, handgrip). Conversely, the murmur and the gradient increase with maneuvers or conditions that decrease preload (eg, Valsalva maneuver, nitrate administration, diuretic administration, standing, dehydration) or afterload (eg, vasodilators like inhaled amyl nitrate). An increase in the intensity of murmur during the Valsalva maneuver is a characteristic feature of HCM. Almost all other cardiac murmurs decrease in intensity during the Valsalva maneuver. During the Valsalva maneuver, increased intrathoracic pressure results in decreased venous return to the right side of the heart and reduced left ventricular filling. In HCM, an underfilled left ventricle results in an increased left ventricular outflow tract gradient, as the ventricular cavity is smaller and the left ventricular outflow tract is narrow. A holosystolic murmur at the apex and axilla can be heard in the presence of mitral regurgitation. A small proportion of patients can have a diastolic decrescendo murmur of aortic regurgitation. Catecholaminergic polymorphic ventricular tachycardia is a severe genetic arrhythmogenic disorder characterized by adrenergically induced ventricular tachycardia manifesting as syncope and sudden death. Prevalence of catecholaminergic polymorphic ventricular tachycardia is 1 in 10,000 individuals. The cardiac ryanodine receptor gene (RyR2) and the cardiac calsequestrin gene (CASQ2) are responsible for this condition. Patients with catecholaminergic polymorphic ventricular tachycardia have normal cardiac structure and function, so this condition is unlikely to be the cause of the clinical presentation in this vignette. Dilated cardiomyopathy is characterized by ventricular dilatation and impaired systolic function. It has a prevalence of 1 in 2,500 individuals. Patients with dilated cardiomyopathy may experience heart failure, arrhythmia, and premature death. Patients exhibit signs and symptoms of heart failure, due to volume overload or low cardiac output. Although syncope, angina, and sudden cardiac death can occur in patients with dilated cardiomyopathy, symptoms of heart failure predominate. These patients usually exhibit fatigue, orthopnea, paroxysmal nocturnal dyspnea, and increasing edema, weight, or abdominal girth. The examination findings are characterized by tachycardia, jugular venous distension, prominent P2, S3 and S4 and gallops. A pansystolic murmur of mitral regurgitation can be heard at the apex in some patients. The patient in this vignette has no signs of heart failure and the murmur is an ejection systolic murmur, so dilated cardiomyopathy is not the correct response. The cardiac anatomy, function, and examination findings are usually normal in patients with congenital long QT syndrome, which is a hereditary disease of cardiac ion channels characterized by a prolongation of the QT interval and by a high risk of life-threatening arrhythmias. Disease prevalence is estimated to be 1 in 2,500 live births. The syncopal episodes in long QT syndrome can be precipitated by stress and physical activity and are caused by torsade de pointes, a polymorphic ventricular tachycardia with a characteristic twist of the QRS complex around the isoelectric baseline, often degenerating into ventricular fibrillation. PREP Pearls Hypertrophic cardiomyopathy is the most common genetic cardiomyopathy, but it often goes undiagnosed. It is the most common cause of sudden cardiac death in young individuals and young athletes. Left ventricular outflow tract obstruction occurs in 25% to 30% of patients with hypertrophic cardiomyopathy. In hypertrophic cardiomyopathy, the murmur and the gradient across the left ventricular outflow tract diminish with maneuvers that increase the preload (eg, Mueller maneuver, squatting) or afterload (eg, handgrip). The murmur and the gradient increase with maneuvers that decrease preload (eg, Valsalva maneuver, nitrate administration, diuretic administration, standing) or afterload (eg, vasodilator administration). In hypertrophic cardiomyopathy, myocardial hypertrophy with abnormalities of the mitral valve apparatus leads to a dynamic subvalvular left ventricular outflow tract obstruction. The gradient of aortic valve stenosis is fixed and develops secondary to obstruction at the valvar level. The systolic murmur in aortic valve stenosis increases with squatting and decreases with standing and isometric muscular contraction, such as the Valsalva maneuver, which helps distinguish it from obstructive hypertrophic cardiomyopathy

A 4-year-old patient with tricuspid atresia and normally related great vessels returns from the operating room after third stage single ventricle palliation with a lateral tunnel fenestrated Fontan. The anesthesiologist reports that the patient's Fontan pressure was 16 mm Hg, yielding a transpulmonary gradient of approximately 8 to 9 mm Hg after coming off of cardiopulmonary bypass. Approximately 6 hours after the patient returns from the operating room, urine output has decreased, and lactic acid has increased from 1.5 mg/dL to 3.8 mg/dL. A central venous catheter in the right internal jugular vein has a good wave form and blood return, and now measures 20 mm Hg. An echocardiogram reveals a fenestration gradient of 7 to 8 mm Hg and no obvious obstruction in the Fontan circuit. Of the following, the BEST next step in the management of this patient is A. extubate the patient to nasal cannula B. give a 10 mL/kg bolus of normal saline C. start an infusion of a phosphodiesterase III inhibitor D. start inhaled nitric oxide E. take the patient for a hemodynamic cardiac catheterization

C. start an infusion of a phosphodiesterase III inhibitor The vignette tests the reader's understanding of the postoperative physiology of the total cavopulmonary anastomosis or Fontan circuit. The normal transpulmonary gradient (TPG) is typically 10 mm Hg or less. The TPG equals the Fontan pressure minus the atrial pressure. This patient presents with a normal TPG. Although the initial atrial pressure or single ventricle end diastolic pressure is not provided, the reader should extrapolate from the information given that the patient's initial atrial pressure or single ventricle-end diastolic pressure should be approximately 7 to 8 mm Hg (atrial pressure = Fontan pressure - TPG). Approximately 6 hours after the surgery, the patient has evidence of low cardiac output, as exhibited by increasing lactate and decreased urinary output. In addition, the Fontan pressure is now 4 mm Hg higher than upon initial presentation. Causes of elevated Fontan pressures in this scenario would include fenestration obstruction, Fontan obstruction, pulmonary hypertension, and/or decreased ventricular compliance and function (Table ). The echocardiogram demonstrates a patent fenestration and a patent Fontan circuit. While Fontan obstruction cannot be completely excluded by echocardiogram, low velocity laminar flow demonstrated in the Fontan circuit makes obstruction less likely. Consequently, taking the patient to the catheter laboratory would not be the first choice or intervention. The echocardiogram does reveal that the fenestration gradient is 7 to 8 mm Hg. The fenestration gradient is a surrogate for the TPG. The central venous line in the right internal jugular (ie, superior aspect of the Fontan circuit) has a pressure of 20 mm Hg. With this information, the atrial pressure can be extrapolated to be 12 to 13 mm Hg (atrial pressure = Fontan Pressure - fenestration gradient). In the setting of indices consistent with low cardiac output and an increasing atrial pressure, the most likely etiology is decreased single ventricle function due to poor ventricular compliance post-cardiopulmonary bypass. As a result, the most appropriate intervention would be to start the patient on therapy that would provide both inotropic support and lusitropic support, such as a continuous infusion of a phosphodiesterase III inhibitor such as milrinone. With an elevated Fontan pressure and elevated atrial pressure, the patient does not need additional volume. Pulmonary hypertension can cause elevated Fontan pressures and poor cardiac output. However, in that scenario, atrial pressure would remain the same as Fontan pressure increases. Consequently, the patient would be expected to have a TPG that increases to greater than 10 mm Hg and not stay the same. Positive pressure ventilation increases intrathoracic pressure and negatively impacts the passive flow in the Fontan circuit. Consequently, early extubation is typically the goal strategy for post operative Fontan patients. However, given that the patient returned on positive pressure ventilation and had normal hemodynamics, it would be less likely that positive pressure ventilation would be the cause of the changes now seen in this patient. PREP Pearls In a patient with a fenestrated Fontan, the fenestration gradient is a surrogate for the transpulmonary gradient pressure. The differential for elevated Fontan pressure includes fenestration obstruction, worsening ventricular compliance/function, pulmonary hypertension, Fontan baffle obstruction, and pulmonary vein obstruction.

The neonatal intensive care unit has asked you to consult on a 25-week-gestation infant who is now 12 weeks old. The infant received mechanical ventilation for 2 weeks after delivery and subsequently transitioned to noninvasive positive pressure ventilation. He underwent patent ductus arteriosus ligation at 4 weeks of age. Two weeks ago, he was weaned to room air and has been progressively working on tube feeds. Since weaning from oxygen, the neonatologist has noted that the infant has become more tachypneic with more pronounced subcostal retractions. A chest x-ray shows changes consistent with chronic lung disease (Figure), but no significant change from before. An echocardiogram demonstrates moderate ventricular septal flattening and tricuspid valve regurgitation of 3.5 m/sec. Of the following, the NEXT best intervention for this patient would be to A. observe the infant B. start inhaled nitric oxide C. start oxygen via nasal cannula D. start sildenafil treatment

C. start oxygen via nasal cannula Pulmonary hypertension associated with chronic lung disease is a common sequela of extreme prematurity. Long-term exposure to mechanical ventilation, the presence of left-to-right shunts (such as patent ductus arteriosus), and premature lung development all contribute to the presence of pulmonary hypertension. Often interventions to treat the chronic lung disease, such as inhaled corticosteroids, will be helpful in treating the pulmonary hypertension as well. In addition, elimination of any factors that exacerbate pulmonary hypertension such as gastroesophageal reflux and obstructive sleep apnea, are imperative in the treatment regimen. The natural history is often marked by improvement in pulmonary hypertension as the chronic lung disease improves. The infant in the vignette has been weaned off oxygen and is now demonstrating symptoms of pulmonary hypertension in the form of tachypnea and increased retractions. His echocardiogram also has 2 hallmarks of pulmonary hypertension: flattening of the interventricular septum and elevated right ventricular pressure as measured by tricuspid regurgitation velocity. Given the clinical and echocardiographic changes in the infant, initiating therapy would be indicated in this case. Oxygen is a potent pulmonary vasodilator and is often used both diagnostically and therapeutically for pulmonary hypertension. The 2015 European Society of Cardiology and European Respiratory Socitety guidelines for the treatment of pulmonary hypertension recommend oxygen therapy as part of the initial therapy regimen for supportive care of the patient with moderate pulmonary hypertension. For the infant in the vignette, initiation of low-flow oxygen via nasal cannula may be a very quick, noninvasive, and durable treatment. He could be discharged from the hospital with nasal cannula oxygen and weaned off as his pulmonary hypertension improves. An echocardiogram within 1 to 2 weeks of oxygen initiation would be indicated to measure objective improvement in the pulmonary hypertension. Inhaled nitric oxide is also a potent pulmonary vasodilator but its use is generally limited to acute pulmonary hypertensive crises. The infant in the vignette is in the process of transitioning to more chronic care; though inhaled nitric oxide could potentially improve his pulmonary hypertension it does not provide a durable long-term treatment. Sildenafil, a phosphodiesterase-5 inhibitor, has become a mainstay in treating pediatric pulmonary hypertension. It is a first-line medication and provides symptomatic improvement in patients. It is recommended for severe pulmonary hypertension and for patients with moderate pulmonary hypertension who have failed supportive therapy including oxygen. It is used frequently in pulmonary hypertension associated with chronic lung disease. Although it would be useful in the patient in the vignette, a trial of oxygen would be indicated first to monitor responsiveness and avoid chronic medication administration given the potentially transient nature of the pulmonary hypertension. If oxygen therapy in addition to other pulmonary medications were to fail, then sildenafil should be considered as the next line of treatment. PREP Pearls Pulmonary hypertension associated with chronic lung disease is common and often responds well to oxygen therapy and pulmonary treatment.

A 16-year-old adolescent boy has a witnessed out-of-hospital cardiac arrest at a football game. Bystander cardiopulmonary resuscitation was started. Emergency medical services was called and places an automated external defibrillator on arrival. A shockable rhythm is detected. A shock is administered with return of a palpable pulse. Cardiopulmonary resuscitation is continued, and the patient is brought to the emergency department. On arrival to the emergency department, he has a heart rate of 137 beats/min, blood pressure of 92/63 mm Hg, and oxygen saturation of 96%. In the emergency department, he has a second arrest with the rhythm shown in the Figure. Defibrillation is performed with 4 J/kg with no return of spontaneous circulation. Repeated defibrillation is performed and intravenous epinephrine is administered with no return of spontaneous circulation. Of the following, the BEST next step in the management of this patient is administration of A.amiodarone bolus B.calcium chloride bolus C.magnesium bolus D.procainamide bolus

C.magnesium bolus The rhythm strip demonstrates torsades de pointes (TdP), which is the classic arrhythmia that is seen in patients with a prolonged QTc. Prolonged QTc can be congenital (long QT syndrome) or acquired. Acquired causes for prolonged QTc can be secondary to medications, electrolyte abnormalities (hypokalemia, hypomagnesemia, hypocalcemia), anorexia, and neurological abnormalities. Torsades de pointes ("twisting of the points"), which was first described by François Dessertenne in 1966, is a type of polymorphic ventricular tachycardia in which the QRS complexes are twisting around the isoelectric baseline. For patients with hemodynamically unstable TdP, the first-line therapy is electrical defibrillation with 2 to 4 J/kg. For patients with shock-refractory TdP or with multiple nonsustained episodes, treatment with intravenous magnesium sulfate is indicated. If untreated, TdP can degenerate into ventricular fibrillation and cause sudden death. Amiodarone and procainamide are both QT-prolonging antiarrhythmics and should be avoided in this situation to prevent further QT prolongation, which would increase the risk of TdP. In cases where defibrillation and magnesium do not convert TdP, lidocaine can be used. Once the patient is stable and if the underlying cause for their TdP is long QT syndrome, β-blockers should be used to prevent further episodes of TdP. Placement of an implantable cardioverter defibrillator should be considered for patients with recurrent TdP episodes despite appropriate medical therapy or as secondary prevention in patients with a prior aborted sudden death event. Left thoracoscopic sympathectomy can also be a treatment option for patients with drug-refractory long QT syndrome. Calcium chloride can be used for membrane stabilization in patients with hyperkalemia or for blood pressure support in hypotensive patients. PREP Pearls Torsades de pointes is the classic arrhythmia seen in patients with long QT syndrome and drug-induced QT prolongation. Magnesium bolus should be given in shock-refractory torsades de pointes.

A 6-week-old male infant born after a normal pregnancy and delivery to healthy parents presents with a history of failure to thrive, head lag, weakness, lethargy, and difficulty in nursing. On examination, he is non-dysmorphic and severely hypotonic, the liver is palpable 3 cm below the right costal margin, and he is taking shallow breaths. His laboratory findings are shown in the Table. Laboratory Test Patient Result (SI Value) White blood cell count 4,500/µL (4.5 ´ 109/L) Polymorphonuclear leukocytes 40% Lymphocytes 53% Monocytes 5% Eosinophils 2% Hemoglobin 12.0 g/dL (120 g/L) Platelet count 102 ´ 103/µL (102 ´ 109/L) Blood glucose 82 mg/dL (4.5 mmol/L) Electrolytes Normal Blood urea nitrogen 28 mg/dL (10 mmol/L) Creatinine 10.9 mg/dL (964 µmol/L) A chest radiograph shows moderate cardiomegaly with a moderate degree of pulmonary edema. There are no pleural effusions. A 12-lead electrocardiogram is shown in the Figure . Of the following, the MOST likely diagnosis is A. Hunter syndrome (mucopolysaccharidosis type II) B. Hurler syndrome (mucopolysaccharidosis type I) C. McArdle disease D. Pompe disease E. Von Gierke disease

D. Pompe disease Failure to thrive, hypotonia, hepatomegaly, and an electrocardiogram (ECG) showing short PR interval and tall QRS complexes are characteristic for Pompe disease (glycogen storage disease II). Pompe disease is an autosomal recessive lysosomal storage disease caused by mutations in the GAA gene. The GAA gene is responsible for making acid a-glucosidase (also known as acid maltase), which breaks down glycogen. Mutations in the GAA gene prevent acid a-glucosidase from breaking down glycogen effectively, which allows this sugar to build up to toxic levels in lysosomes. This damages organs and tissues throughout the body, particularly the cardiac and skeletal muscles. The severity of the disease and the age at onset are related to the degree of enzyme deficiency. In the classic infantile form of Pompe disease, patients present with cardiomyopathy and muscular hypotonia. Infants usually present with muscle weakness (myopathy), poor muscle tone (hypotonia), an enlarged liver (hepatomegaly), failure to thrive, breathing problems, and cardiomegaly. If untreated, this form of Pompe disease leads to death from heart failure in the first year after birth. The effects of glycogen accumulation are very pronounced in the heart. Lysosomal glycogen accumulation results in a significant amount of cardiac hypertrophy that may begin in utero and can be pronounced on neonatal echocardiography. The electrocardiogram in Pompe disease shows tall QRS complexes, left axis deviation, short PR interval, and T-wave inversion. These conduction abnormalities, in conjunction with the hypertrophic cardiomyopathy, place these patients at high risk for tachyarrhythmia and sudden death, especially in situations of stress such as infection, fever, dehydration, and anesthesia. Enzyme replacement therapy using alglucosidase alfa has been approved by the US Food and Drug Administration for the treatment of Pompe disease. In clinical trials with infantile-onset Pompe disease, this drug has shown to decrease heart size, maintain normal heart function, improve muscle function, tone, and strength, and reduce glycogen accumulation. McArdle disease (glycogen storage disease type V) is caused by mutations in the PYGM gene, leading to a deficiency in glycogen phosphorylase, principally in skeletal muscle. The PYGM gene mutations prevent myophosphorylase from breaking down glycogen effectively in the skeletal muscles, leading to easy fatigability during exercise. Since the skeletal muscle tissue is mainly affected, this disease is referred to as a myopathic type of glycogen storage disease. People with glycogen storage disease V typically experience fatigue, muscle pain, and cramps during the first few minutes of exercise. The discomfort is generally alleviated with rest. If individuals rest after brief exercise and wait for their pain to go away, they can usually resume exercising with little or no discomfort. In contrast to Pompe disease, the presentation is later and there is no cardiac involvement or hepatomegaly. Von Gierke disease (glycogen storage disease type I) is caused by a mutation in the G6PC gene leading to deficiency of glucose 6-phosphatase, the enzyme that removes the phosphoryl group from glucose and permits it to be released from the cell. Von Gierke disease typically manifests during the first year after birth with severe hypoglycemia and hepatomegaly caused by the accumulation of glycogen. Affected children exhibit growth retardation, delayed puberty, lactic acidemia, hyperlipidemia, hyperuricemia, and adults have a high incidence of hepatic adenomas. The patient in the vignette does not present with hypoglycemia and his cardiac involvement makes von Gierke disease unlikely. Hurler and Hunter syndromes are mucopolysaccharidoses. The mucopolysaccharidoses are a group of inherited disorders caused by a lack of lysosomal enzymes involved in the degradation of glycosaminoglycans, or mucopolysaccharides. The clinical features of Hurler syndrome include coarse facies, corneal clouding, mental retardation, hernias, dysostosis multiplex, and hepatosplenomegaly. Children with Hurler syndrome appear normal at birth and develop the characteristic appearance over the first years thereafter. Cardiac involvement is characterized by valvular dysfunction and endocardial fibroelastosis. Hunter syndrome is a rare X-linked recessive disorder caused by deficiency of the lysosomal enzyme iduronate sulfatase. Clinical manifestations include severe airway obstruction, skeletal deformities, cardiomyopathy, and neurologic decline. Death usually occurs in the second decade of life, though some patients with less severe disease have survived into their fifth or sixth decade. Cardiac involvement is characterized by thickening and stiffening of the valve leaflets, commonly leading to mitral and aortic regurgitation and/or stenosis. Cardiomyopathy is less common, but may be associated with an increased risk of cardiac arrhythmia. The infant in the vignette had no facial features of mucopolysaccharidoses. Early onset of cardiomyopathy and absence of other clinical features make it unlikely that he has mucopolysaccharidosis. PREP Pearls Pompe disease (glycogen storage disease II) is an autosomal recessive lysosomal storage disease caused by mutations in the GAA gene. The infantile form of Pompe disease presents with rapidly progressive disease characterized by prominent cardiomegaly, hepatomegaly, weakness, and hypotonia, and if untreated, death resulting from cardiorespiratory failure in the first year. Interference with specialized conducting tissues produces a shortening of the atrioventricular (P-R) interval on the electrocardiogram (ECG). Other ECG findings include tall QRS complexes, left axis deviation, and T-wave inversion. These conduction abnormalities, in conjunction with the hypertrophic cardiomyopathy, place these patients at high risk for tachyarrhythmia and sudden death, especially in situations of stress such as infection, fever, dehydration, and anesthesia.

An 8-year-old girl is referred to you with a 1-year history of shortness of breath with activity that has not been responsive to bronchodilators. A chest x-ray ordered by her pediatrician demonstrated cardiomegaly with mild pulmonary venous congestion, resulting in her referral. Her parents think that her appetite is not as good as it used to be and she has been complaining of stomachaches more frequently than before. Physical examination reveals the following: • Respiratory rate: 20 breaths/min • Pulse rate: 100 beats/min • Blood pressure: 100/60 mm Hg • Oxygen saturation: 98% on pulse oximetry • General: Small but proportional • Neck: + jugular venous distention. • Chest/lungs: No retractions and clear bilaterally • Cardiac: Active precordium; normal S1 and S2; no murmur; + S4 • Abdomen: Liver 3 cm below the right costal margin; spleen not palpable • Extremities: No clubbing, cyanosis, or edema The girl is warm and well perfused. On review of the chest x-ray brought by the parents, you concur with the reading. Electrocardiography shows sinus rhythm. You order an echocardiogram, which is consistent with restrictive cardiomyopathy with preserved systolic function. Of the following, the BEST next step in the management of this patient is to start A. carvedilol B. digoxin C. enalapril D. furosemide

D. furosemide Restrictive cardiomyopathy is a disease of diastolic heart failure that results in elevated left and/or right ventricular filling pressures. This causes symptoms of pulmonary venous and/or systemic venous congestion. This patient has evidence of both based on her physical examination, chest x-ray, and echocardiogram. Diuretics such as furosemide are indicated to relieve the symptoms of venous congestion, whether pulmonary or systemic. There are no data in adults or children that indicate that β-blockers, angiotensin-converting enzyme (ACE) inhibitors, or digoxin relieve symptoms in this disease, nor have these agents been shown to improve outcomes. Patients with restrictive cardiomyopathy may be more dependent on heart rate to maintain cardiac output, therefore β-blockers should not be started without an additional indication. ACE inhibitors reduce afterload and may cause hypotension. Hypotension may be poorly tolerated in patients with diastolic heart failure who have a limited ability to compensate. Digoxin is a weakly positive inotrope that is also capable of slowing the ventricular rate. Because restrictive cardiomyopathy is a disease of diastolic heart failure, digoxin does not have a primary role in its treatment. Therefore none of these agents are recommended unless there are additional factors for which they are primarily indicated, according to the recommendations of the guidelines for the management of pediatric heart failure published by the International Society for Heart and Lung Transplantation. Diuretics have not been shown to improve outcomes either, but they do relieve symptoms. However, it is important not to provide "overdiuresis" to these fragile patients because their diastolic heart failure makes them preload dependent. Dosing must be individualized based on patient response. PREP Pearls Restrictive cardiomyopathy is a disease of diastolic heart failure. Diuretics improve symptoms, but response must be carefully monitored because these patients are preload dependent and may not tolerate overdiuresis. Currently no medical therapies are available, which improve survival in children or adults with restrictive cardiomyopathy.

You are caring for a 7-month-old male infant who presented 1 day earlier with acute respiratory distress and was found to have cardiomegaly on a chest radiograph. Echocardiography confirmed the diagnosis of dilated cardiomyopathy with an ejection fraction of 18% on M-mode measurement. The patient was admitted to the cardiac intensive care unit and initially improved clinically with initiation of a milrinone infusion (0.3 μg/kg per minute) and intravenous diuresis with furosemide. This morning during rounds he appears to be in moderate respiratory distress; the lactate level has increased from 1.3 mmol/L at admission to 3.5 mmol/L; venous saturation drawn from a central venous line situated at the superior vena cava-right atrial junction is 48%, decreased from 63% on admission. Overnight he had 1 episode of nonsustained ventricular tachycardia. He is currently in sinus tachycardia at 180 to 190 beats/min, and his blood pressure has been within normal limits for age. Of the following, the MOST appropriate immediate intervention is to A. initiate an epinephrine infusion B. increase the milrinone infusion to 0.5 μg/kg per minute C. plan placement of a ventricular assist device D. proceed with endotracheal intubation

D. proceed with endotracheal intubation The patient in the vignette has progressed from compensated congestive heart failure secondary to dilated cardiomyopathy to uncompensated congestive heart failure. Despite the initial improvement with milrinone and furosemide, the patient's condition has now acutely worsened. The elevated lactate and low mixed venous saturations both indicate that oxygen delivery to the peripheral tissues is compromised by the poor cardiac function. Without immediate intervention this patient is at risk of progressing to cardiac arrest and death. The fastest and most immediate intervention is endotracheal intubation. This has several beneficial aspects in a patient such as this one. It removes the metabolic demand of the respiratory drive which should improve oxygen delivery to other vital organ systems. In addition, it will lower left ventricular afterload—intrathoracic left ventricular afterload is determined by left ventricular transmural pressure (LVTM). This is calculated by subtracting intrathoracic pressure (PITP) from left ventricular pressure (PLV) such that the equation can be written as follows: LVTM = PLV - PITP LVTM can be reduced by either decreasing PLV or increasing PITP. In situations of shock, decreased PLV would not be tolerated and is likely dangerous. Increasing PITP is easily done via endotracheal intubation and mechanical positive pressure ventilation. Endotracheal intubation should be performed with utmost caution in these patients. Although cardiac output should improve once positive pressure ventilation is established, the period of transition from negative to positive pressure ventilation can be tenuous. Positive pressure ventilation causes decreased venous return to the right atrium, which in turn decreases cardiac output. Volume should be available for administration during intubation if needed. In addition, decreased blood pressure from sedation medications used for intubation can cause further hemodynamic compromise. Resuscitative medications should be immediately available and sedatives should be chosen based on their hemodynamic effects, with cardiodepressant medications such as propofol or midazolam being avoided. An experienced airway manager should perform the intubation given the inherent dangers involved. An epinephrine infusion may be indicated in this patient to augment inotropy and improve cardiac output, but its effect may not be as immediate as intubation and ventilation. In addition, it may expose the patient to higher arrhythmia risk. Increasing the milrinone infusion would also be indicated, however, given the longer half-life of milrinone it would take several hours to see the effect of that intervention. This patient may require mechanical circulatory support such as a ventricular assist device. Further medical interventions should be maximized before proceeding with a ventricular assist device. If the patient's condition progresses to severe hemodynamic compromise that was not responsive to conventional medical interventions then mechanical support with extracorporeal membrane oxygenation could be established emergently and he could move to a ventricular assist device. PREP Pearls Immediate recognition and treatment of decompensated heart failure can be lifesaving. Positive pressure ventilation, once established, improves left ventricular output.

You are called to the bedside of a 3-year-old girl who is recovering in the intensive care unit following a repair of partial anomalous pulmonary venous return. Her operative and postoperative courses have been relatively unremarkable, but the bedside nurse is concerned because the left radial arterial line is reading a blood pressure of 134/52 mm Hg (mean arterial pressure 79 mm Hg). The heart rate is 103 beats/min. The nurse informs you that the patient has received adequate analgesia and sedation and is not exhibiting any signs of discomfort or agitation. You ask the bedside nurse to obtain a cuff blood pressure measurement which reads 93/66 mm Hg (mean arterial pressure 75 mm Hg). Of the following, the BEST next step in your management of this patient is A. administer midazolam 0.1 mg/kg B. administer morphine 0.1 mg/kg C. order a sodium nitroprusside infusion D. reassure the bedside nurse that the arterial line tracing is falsely elevated

D. reassure the bedside nurse that the arterial line tracing is falsely elevated Invasive hemodynamic monitoring involves the use of special pressure tubing and transducers. The basic premise is that a catheter in the arterial system will relay the pressure waves generated to the fluid column present in the pressure tubing. These pressure waves sensed by the fluid column are then translated into a blood pressure tracing by an electronic transducer. The electrical signal is then transmitted to the monitor and amplified and displayed as an analog waveform and digital output. The transducer must be referenced to heart level and zeroed to atmospheric pressure. Absence of either leveling or zeroing will create incorrect blood pressure measurements. A common problem with arterial line tracings is either over or under dampening of the waveform. This can be the result of several issues: air or blood in the pressure tubing as both transmit pressure differently than water thereby created more or less resonance in the pressure measuring system; a soft arterial catheter can cause over-resonance and underdampening ("fling"); and an occlusion can cause under-resonance and overdampening. When under dampened, an arterial line pressure tracing will generally overestimate the systolic blood pressure and underestimate the diastolic blood pressure, but will read the mean arterial pressure accurately. A cuff pressure can reassure you that the reason for the elevated blood pressure measurement is likely an underdampened system, but the true way to confirm its presence it to perform a cuff occlusion pressure. By placing a manual blood pressure cuff on the limb with the arterial line, the manual cuff is inflated until the arterial line is flat. The pressure is slowly released from the manual cuff until the arterial line tracing appears; the pressure on the manual cuff at which this occurs is known as the cuff occlusion pressure and also represents the true systolic blood pressure. It should match with a cuff measurement from another extremity. In our vignette, the patient's arterial line reading has a higher systolic and lower diastolic pressure than the measured cuff pressure, but the mean arterial pressures are very close. If you performed a cuff occlusion pressure on the arm with the radial arterial line, the measurement would be very close to 93 mm Hg reassuring you that the blood pressure reading on the arterial line is falsely elevated. Midazolam and morphine are useful for treating hypertension related to either pain or anxiety, neither of which this patient is displaying. In addition to the nurse's report that the patient is well sedated, the heart rate of 103 beats/min is reassuring that neither pain nor anxiety is likely the primary cause of the blood pressure reading. Sodium nitroprusside solution would be correct if the blood pressure reading from the arterial line was accurate. It is a potent arterial and venous dilator and would be a good first choice to treat postoperative hypertension when it is truly present. PREP Pearls Measure the cuff occlusion pressure to confirm an underdampened arterial line.

A 4-year-old boy without significant past medical history is referred for evaluation of a murmur that was noted for the first time on a recent routine well child visit. Physical examination reveals a well-appearing child in no acute distress and with good pulses and perfusion. His room air oxygen saturation is 98% and the blood pressure in his right upper extremity is 106/64 mm Hg. The precordium is normally active and the first and second heart sounds are normal, but there is a harsh sounding, crescendo-decrescendo grade I-II/VI systolic murmur extending into diastole heard at the bilateral upper sternal borders, left greater than right, and radiating to the left subscapular area, the quality of which is unaffected by maneuvers. There are no clicks, rubs, or gallop rhythms appreciated. Of the following, an echocardiogram obtained in this patient is MOST likely to reveal the diagnosis of A. bicuspid aortic valve B. coarctation of the aorta C. normal anatomy D. muscular ventricular septal defect E. patent ductus arteriosus

E. patent ductus arteriosus A heart murmur is an audible sound of flow through the cardiac and vascular systems, which may be functional due to normal physiologic circumstances, or pathologic and due to flow across an abnormal connection somewhere in the heart or great arteries. The etiology of a given murmur can often be elucidated clinically based on its classification in each of 7 characteristics: timing, shape, location, radiation, intensity, pitch, and quality. Timing refers to when in the cardiac cycle the murmur occurs and its relationship to S1 and S2 leading to a description as either systolic or diastolic. All systolic murmurs terminate prior to or with S2, with their onset occurring either with or immediately after S1. Systolic ejection murmurs follow S1, ending before S2, and are caused by flow across an anatomically or relatively stenotic ventricular outflow tract or semilunar valve. These are crescendo-decrescendo in shape in contrast to the rectangular-shaped holosystolic murmurs. Holosystolic murmurs begin with S1 continuing to the start of S2, and represent atrioventricular (AV) valve regurgitation or flow across a ventricular septal defect. Diastolic murmurs occur between S2 and S1. Murmurs in early diastole beginning at the same time as S2 are usually caused by semilunar valve regurgitation and have a decrescendo shape, while those that start after S2 in mid-diastole are due to turbulent flow across a stenotic AV valve or excess flow across a normal AV valve in the rapid filling phase of the cardiac cycle. Murmurs that occur due to turbulent flow in the vasculature outside of the heart, may occur independent of the cardiac cycle and without specific relationship to systole or diastole, and thus can be heard continuously through all or part of diastole. Continuous murmurs are in the majority of cases pathologic. Location and radiation refer to the point on the chest at which a murmur is best heard and to additional points at which it can also be heard. Aortic and pulmonary murmurs are usually appreciated at the upper right and left sternal borders, respectively, while tricuspid and mitral murmurs are located at the fifth intercostal spaces along the sternal border and in the mid-clavicular line. In general, a murmur will radiate in the direction of the flow of the blood, for example, towards the back in the case of a murmur associated with the pulmonary circulation. The intensity of a systolic murmur is graded I-VI based on how easily audible it is, whether or not there is a palpable thrill, and if it can be heard without the stethoscope fully in contact with the chest wall. Pitch may be low, medium, or high depending on the frequency of the murmur. Terms such as "harsh," "blowing," or "musical" are used to characterize the more subjective qualities of a murmur. A patent ductus arteriosus (PDA) murmur is typically described as being machine-like in quality and occurs continuously as flow through the PDA occurs regardless of timing in the cardiac cycle. However, it is not uncommon to be more pronounced in systole rather than in diastole with a crescendo-decreshendo type shape. These murmurs are often best heard at the upper left sternal border, but may be noted as well in the upper right chest and radiate towards the back and the pulmonary vasculature reflective of the systemic to pulmonary shunt. Murmurs due to a bicuspid aortic valve are classically located at the upper right sternal border. If the valve is stenotic, then the murmur is systolic, while the regurgitant murmur of an incompetent valve will be diastolic in its timing. The murmur of a bicuspid aortic valve typically does not radiate and is not infrequently accompanied by an early systolic click, absent in the vignette, as the valve reaches its point of maximal opening. Another murmur of aortic pathology, that of a coarctation of the aorta, may also have a harsh quality, be heard in the back, and be appreciable in diastole, but would be unlikely to be accompanied by normal blood pressure and good pulses and perfusion in a 4-year-old patient. Muscular ventricular septal defects classically are identified by a pathognomonic murmur described as holosystolic and localized to the left lower sternal border. These murmurs occasionally may radiate throughout the precordium, but unlike in this case, never radiate to the back. Like the murmur of a PDA, a venous hum is continuous and is typically heard in the upper chest. However, unlike a PDA, a venous hum, caused by vibration of the venous walls as they drain the 20% of the cardiac output supplied to the brain, does not radiate to the back and is extinguished by maneuvers such as placing the patient in the supine position, turning the head to one side, or placing a finger over the jugular vein. In this case, the failure of the continuous murmur to be affected by maneuvers and its radiation to the back makes it likely to be pathologic and thus the patient's echocardiogram is likely to reveal entirely normal cardiac structure and function. In this case, the harsh, nearly continuous nature of a murmur heard in the aortic and pulmonary regions of the precordium, but radiating posteriorly towards the pulmonary vasculature suggest that an echocardiogram in this patient is most likely to reveal the presence of a persistent PDA. PREP Pearls Systolic ejection murmurs follow S1, ending before S2, and are caused by flow across an anatomically or relatively stenotic ventricular outflow tract or semilunar valve. Murmurs in early diastole beginning at the same time as S2 are usually caused by semilunar valve regurgitation and have a decrescendo shape, while those that start after S2 in mid-diastole are due to turbulent flow across a stenotic atrioventricular (AV) valve or excess flow across a normal AV valve in the rapid filling phase of the cardiac cycle. Murmurs that occur due to turbulent flow in the vasculature outside of the heart may occur independent of the cardiac cycle and without specific relationship to systole or diastole, and thus can be heard continuously through all or part of diastole. In general, a murmur will radiate in the direction of the flow of the blood.

You are the attending physician for the newborn nursery at a teaching hospital. During morning rounds, the nurse informs you of a 36-hour-old full-term infant who has failed the pulse oximetry screen after the third attempt. The infant has an unremarkable prenatal history, was delivered by spontaneous vaginal delivery, and transitioned well in the newborn nursery except for failing the pulse oximetry screen. Of the following, the MOST appropriate next step in the management is to A. repeat screening within 24 hours B. get a chest x-ray and electrocardiogram C. discharge from the hospital with a follow-up cardiac appointment D. perform arterial blood gas E. perform echocardiography before discharge

E. perform echocardiography before discharge Some congenital heart defects (CHDs) can be detected with fetal echocardiography, but others are not detected during pregnancy. Infants with critical CHDs can appear well in the immediate postnatal period, be discharged home, and present later with life-threatening emergencies. Pulse oximetry screening is a new screening protocol endorsed by the American Academy of Pediatrics and many other national organizations in an attempt to detect these infants in a timely fashion to reduce their morbidity and mortality (Figure). Pulse oximetry screen measures the oxygen level in an infant's upper and lower extremities, and targets lesions such as hypoplastic left heart syndrome, pulmonary atresia, tetralogy of Fallot, truncus arteriosus, tricuspid atresia, total anomalous pulmonary venous return, and transposition of the great arteries. Screening is conducted when an infant is at least 24 hours of age or as close to discharge as possible. This allows neonatal transitional circulatory changes to occur, thereby reducing the false-positive findings and allowing the detection of defects that depend on ductal patency. Failing a screen does not always indicate that the infant has critical CHD. It could also be secondary to other causes like pneumonia or infections; the infant needs further evaluation to account for the low saturations. Every infant undergoes 3 screening attempts before it is determined that the infant has passed or failed, again in an attempt to reduce the false-positives. If an infant fails the third attempt, the next step for the primary physician is to order a "diagnostic quality echocardiogram" as per the protocol as opposed to repeating the screen in 24 hours. Electrocardiography and chest x-ray may be helpful but not sufficient to diagnose the underlying heart defect. Echocardiography is an accurate test to identify critical congenital heart defects, free of complications, widely available, relatively inexpensive, and provides prognostic information that will influence the patient's life. Arterial blood gas is an invasive test and the patient can have complications. Arterial blood gas results have to be analyzed cautiously based on the conditions under which the test is performed and will not provide the final diagnosis. As per the pulse oximetry screening guidelines, the next best step would be to perform echocardiography to rule out a critical congenital heart defect. An infant who fails the pulse oximetry screen after the third attempt cannot be discharged from the hospital without performing echocardiography. An infant who passes the screen is unlikely to have a critical CHD, but it is imperative to state that the screening result does not exclude the existence of a cardiac disorder. Pulse oximetry screen complements a complete history and physical examination and significantly improves the detection of critical CHD, but the physician needs to be aware of the potential for false-positive as well as false-negative results. PREP Pearls Failed pulse oximetry screen in a newborn is an indication for further testing before discharge from the hospital. Pulse oximetry screen should be used in conjunction with history and physical examination. Passing the pulse oximetry screen means the infant is unlikely to have a critical congenital heart defect, but does not exclude a cardiac disorder (false-negative result).

You are caring for a 3-month-old infant who underwent repair of coarctation of the aorta with an extended end-to-end anastomosis 2 days earlier. He continues to receive mechanical ventilation because of suspected underlying pulmonary disease that has slowed his weaning from the ventilator. Because of postoperative hypertension, sodium nitroprusside treatment was initiated in the operating room and continued in the intensive care unit. He currently requires a dose of 6 μg/kg per minute to maintain his blood pressure in the acceptable range. You are called to his bedside because the most recent arterial blood gas demonstrates a pH of 7.15, a partial pressure of carbon dioxide of 34 mm Hg (4.5 kPa), a partial pressure of oxygen of 165 mm Hg (22 kPa), and a lactate level of 6.3 mmol/L. On examination, the infant remains sedated on the ventilator with fraction of inspired oxygen set at 0.4, respiratory rate at 22 breaths/min, and tidal volume at 8 mL/kg. You can feel weak pulses in both femoral arteries, and capillary refill in the lower extremities is 3 seconds. The arterial blood pressure from a right radial arterial line is 77/45 mm Hg (mean arterial pressure, 56 mm Hg) at the same time that a cuff-measured blood pressure in the left leg is 92/40 mm Hg (mean arterial pressure, 57 mmHg). Of the following, the BEST test to help confirm your diagnosis is A. chest radiography B. echocardiography C. serum chemistries D. serum osmolarity E. superior vena cava saturation

E. superior vena cava saturation Sodium nitroprusside (SNP) is a potent arterial and venous vasodilator through direct smooth muscle relaxation. SNP causes the release of nitric oxide which, through a series of steps, increases intracellular cyclic guanosine monophosphate and causes vasodilation. SNP has rapid onset and elimination, is light sensitive, and can be given peripherally. The most common indications for SNP include postoperative hypertension, afterload reduction after cardiac surgery, and hypertensive emergencies. Common side effects from SNP include hypotension, tachycardia, decreased cardiac output from decreased preload, and ventilation-perfusion mismatching. SNP is metabolized as cyanide (CN-), which generally binds to thiosulfate to form thiocyanate (SCN-) which is excreted by the kidney. When thiosulfate levels are low or saturated (such as at high doses of SNP), cyanide toxicity is a risk. This risk is the most feared adverse effect of SNP. CN- causes tissue hypoxia by binding to cytochrome oxidase and inhibiting oxidative phosphorylation. This causes lactic acidosis as cells produce adenosine triphosphate without oxygen. The tissue hypoxia will be present in the face of normal oxygenation (saturation and PaO2). An early clue that the acidosis is secondary to CN- toxicity is an elevated mixed venous saturation. Because of arrested oxidative phosphorylation, oxygen bound to hemoglobin is not being offloaded to cells, and returns to the venous circulation highly saturated. In this case, measuring a superior vena cava saturation (as a surrogate for mixed venous oxygen saturation) will likely reveal a level higher than 90%, raising concern for CN- toxicity. The treatment for CN- should include 3% sodium nitrite and 25% sodium thiosulfate. Certain pharmacies mix their SNP infusions in a solution containing sodium thiosulfate to decrease the risk of CN- toxicity. Any acidosis following repair of coarctation of the aorta should raise suspicion for residual or recurrent coarctation. While pulses are weak and capillary refill is somewhat prolonged, the upper and lower extremity blood pressure measurement are reassuring that residual coarctation is likely not the issue in this patient. If the suspicion for a residual coarctation were more significant, then an echocardiogram would be indicated. When faced with metabolic acidosis, serum chemistries can be very helpful to distinguish the primary problem. The presence of an anion gap should raise concern for a more significant problem beyond hyperchloremia or renal tubular acidosis, and should be rapidly investigated. The patient in the vignette has been identified as having lactic acidosis and will have an elevated anion gap. The serum chemistries would not help determine the primary problem in this case. Serum osmolarity can be helpful when faced with metabolic acidosis that does not fit into one of the usual categories of anion gap acidosis. Measuring an osmolar gap can identify certain alcohol derivatives that are not routinely measured and which can cause metabolic acidosis. The osmolar gap would not be helpful in this situation. The patient in the vignette has pre-existing pulmonary disease, and during any decompensation, it is important to think about other organ systems. His arterial blood measurements, however, are reassuring, and chest radiography would not aid in arriving at the diagnosis for this patient. PREP Pearls Sodium nitroprusside toxicity is dose dependent and can present with normal arterial oxygen saturations. Elevated lactate level and elevated mixed venous saturations are both indicative of possible cyanide toxicity from sodium nitroprusside.

A 15-year-old previously healthy adolescent presents with acute sharp chest pain and shortness of breath. He is thin but of normal height. His heart rate is regular at 120 beats/min, his respiratory rate is 30 breaths/min, and his oxygen saturation is 94% on room air. On physical examination, he has a normal palate and uvula. He has decreased breath sounds on his right side. He has in the past been noted to have frequent bruising, very flexible hands, and his family has noticed that it has always been easy to see his blood vessels through his skin. Of the following, the diagnosis MOST characteristic for these findings is A. hereditary hemorrhagic telangiectasia (HHT) B. Loeys-Dietz syndrome C. Marfan syndrome D. osteogenesis imperfecta E. vascular-type Ehlers-Danlos syndrome

E. vascular-type Ehlers-Danlos syndrome This adolescent's history and physical findings are typical for a spontaneous pneumothorax, ie, a sudden accumulation of air in the pleural space with concomitant partial (or full) lung collapse. Spontaneous pneumothorax is most commonly seen in male adolescents. Pneumothorax can be primary (ie, without underlying abnormality) or can be secondary. In this case, the patient also has a history of easy bruising and translucent skin with easily visible blood vessels. These other features are typical for vascular-type Ehlers-Danlos (E-D) syndrome. This patient has experienced a secondary-type spontaneous pneumothorax as a presenting symptom of E-D syndrome. In this case, the constellation of historical features, physical characteristics, and his spontaneous pneumothorax should lead to further thoracic vascular imaging (echocardiography, magnetic resonance imaging, and/or computed tomography), family screening, and possibly genetic testing for vascular-type E-D syndrome. There are currently 6 types of E-D syndrome identified. Vascular-type E-D type 4 is caused by mutation in COLA31 gene coding for protein used to assemble type III collagen. Vascular-type E-D syndrome has an autosomal dominant inheritance pattern; 50% of E-D cases are sporadic (new mutation in one copy of a patient's COLA31 gene) and 50% are inherited from a parent. Many patients have typical facial features. The major risks of vascular-type E-D syndrome include arterial (especially aortic) rupture/dissection and bowel rupture; because of these risks, vascular-type E-D syndrome is considered the most dangerous form of E-D syndrome. Pregnant females are at especially high risk of arterial or uterine rupture at the time of delivery. Patients diagnosed with vascular-type E-D syndrome should have regular cardiology follow-up with noninvasive imaging of the aorta to assess for aortic dilation and aneurysm. They should avoid contact sports. At this time, prophylactic medical therapy has not been proven to decrease the risk of cardiovascular complications in E-D syndrome. The 2 other most common types of E-D syndrome include hypermobility type and classical type, which are more associated with joint hypermobility, chronic joint pain, and skin extensibility; however, patients with these forms of E-D syndrome do not typically have the potentially severe vascular complications noted previously. Hereditary hemorrhagic telangiectasia (HHT) is an autosomal dominant disorder characterized by vascular malformations and dilation of small blood vessels due to abnormally fragile blood vessel walls. In most (if not all) cases, HHT results from gene mutations in the TGF-β pathway. Patients with HHT may present with easy bruising, but they do not typically have spontaneous pneumothoraces or joint hypermobility as is seen in vascular-type E-D syndrome. Rather, HHT patients will have lung arterial venous malformations, gastrointestinal and mucocutaneous telangiectasias, and pulmonary hypertension. Arterial dilation and dissection are not common findings in HHT. Loeys-Dietz (L-D) syndrome is an autosomal dominant disorder characterized by arterial tortuosity and aneurysms, bifid uvula or cleft palate, and hypertelorism. Loeys-Dietz syndrome is caused by mutations in the transforming growth factor-β signaling pathway. Patients with L-D syndrome are at high risk for aortic dissection at aortic dimensions smaller than what is generally considered at highest risk in cases of Marfan syndrome, as well as aneurysms throughout the arterial system. Loeys-Dietz syndrome patients also can have joint laxity, skin hyperextensibility, pectus, and scoliosis. Findings that distinguish L-D syndrome from Marfan syndrome may include craniosynostosis and club feet. Loeys-Dietz syndrome patients do not typically present with spontaneous pneumothorax. Marfan syndrome is a connective tissue with characteristic cardiovascular findings of aortic root and ascending aorta dilation and dissection, mitral valve prolapse, and mitral regurgitation. Other findings include musculoskeletal abnormalities (overgrowth of bones leading to tall stature and long limbs, craniofacial abnormalities, pectus, scoliosis, joint hyperflexibility), ocular abnormalities (ectopia lentis), and skin, fascia, and lung abnormalities. In more than 90% of Marfan syndrome cases, there is a mutation in fibrillin-I protein leading to abnormal connective tissue, although there are multiple different genes that may cause fibrillin-I mutations. Marfan syndrome is typically inherited in an autosomal dominant pattern. Patients with Marfan syndrome can present as teenagers with spontaneous pneumothorax, but an adolescent with Marfan syndrome would not typically also have the findings of easy bruising and thin skin with prominent vessels seen in this clinical vignette. Osteogenesis imperfecta (OI) is another inheritable connective tissue disorder caused by abnormal collagen synthesis (due to mutations in type I collagen genes). Subjects with OI typically present with bone fragility with frequent fractures and/or osteoporosis; some but not all subtypes of OI have typical "blue sclera." Aortic dilation and mitral regurgitation have been reported rarely in OI. Osteogenesis imperfecta does not typically present with these findings of spontaneous pneumothorax, frequent bruising, hyperflexible hands, or thin skin with easily visible vessels. PREP Pearls The presence of a spontaneous pneumothorax in a child or adolescent should prompt one to look for other evidence of an underlying connective tissue disorder. Vascular complications should be excluded in any child with connective tissue disorder who presents with sudden-onset chest pain.

A 3-year-old boy is being evaluated for a murmur. An echocardiogram demonstrates an enlarged right ventricle. Cardiac magnetic resonance imaging is performed (Video 1, Video 2). Of the following, what is the MOST likely cause of the cardiac chamber enlargement? A.inferior sinus venosus defect B.right pulmonary veins drain to right atrium in isolation C.scimitar syndrome D.superior sinus venosus defect

B.right pulmonary veins drain to right atrium in isolation The findings in this vignette are consistent with partial anomalous pulmonary venous return (PAPVR). The magnetic resonance imaging shows all right-sided pulmonary veins returning to the right atrium, the left veins to the left atrium, and an intact atrial septum. This condition is an uncommon variant of isolated PAPVR. The etiology of isolated PAPVR as seen in this case may occur via 2 embryological processes, malposition of septum primum or anomalous isolated PAPVR, both of which present in the same manner. Malposition of atrial septum primum can occur when the septum secundum is absent (usually in cases of visceral heterotaxy with polysplenia). The septum primum may then be displaced toward the anatomic left atrium. The displacement of the upper border of the septum primum will result in the incorporation of half or all of the pulmonary veins into the right atrium. In many cases, the displaced septum primum will not reach the posterior wall of the left atrium, resulting in a small superior interatrial communication. The appearance of the displaced mobile upper border of the septum primum helps to establish the diagnosis. This case is interesting because the atrial septum is not clearly deviated and there may, in fact, be a septum secundum present. The discussion of malposition of the septum primum in the ninth edition of Moss and Adams' (pp. 887-888) elucidates the following anatomical considerations: "The essential anatomic elements that make possible the malposition of the septum primum toward the anatomic LA [left atrium] include absence of septum secundum and a well-developed septum primum." "In cases where septum primum fuses with the posterior left atrial wall, there is no interatrial communication." "Supporting evidence for the conclusion that the pulmonary veins are connected normally to the atrial wall is that they are located between the right and left SVCs [superior vena cavas] (when two SVCs are present)." The magnetic resonance images in this case appear to demonstrate a septum secundum, and there is no interatrial communication evident. The pulmonary veins seem to connect more rightward than normal, nearly directly underneath the SVC rather than to the left of it. Superior sinus venosus defects represent a deficiency of the common wall between the right SVC and the right upper pulmonary vein or the wall between the right atrium and the right upper and lower pulmonary veins. The result of these deficiencies is the unroofing of the right upper pulmonary vein and its branches into the right SVC (sinus venosus defects of the SVC type) or the unroofing of the right upper and lower pulmonary veins into the right atrium (sinus venosus defect of the right atrial type). Inferior sinus venosus defect is an uncommon defect in which there is a deficiency between the right lower pulmonary veins and the right atrium or at the inferior caval-right atrial junction. According to Banka et al (2011), the diagnosis of inferior sinus venosus defect is based on the following 3 criteria: Posterior or inferior defect confluent with the posterior wall of the atria with no posterior or inferior rim Confluence of the defect with the right pulmonary veins(s) or the inferior vena cava-right atrial junction Presence of a well-developed septum primum covering the fossa ovale without fenestrations Scimitar syndrome is a variant of partial anomalous pulmonary venous connection in which part or all of the right lung is drained by right pulmonary veins that connect anomalously to the inferior vena cava. The anomalous draining vein has an appearance akin to a scimitar, or Turkish sword, on chest radiography or magnetic resonance imaging. The affected lung (or segment) is frequently hypoplastic. Sequestration as well as aortopulmonary collateral vessels may also involve the affected lung. PREP Pearls Partial anomalous pulmonary venous connection should be considered in cases of right-sided heart enlargement. Cardiac magnetic resonance imaging is helpful in determining the anatomy of partial anomalous pulmonary venous return and its variants.

A 14-year-old girl has stroke-like symptoms. She is developmentally delayed and has a seizure disorder that has been controlled with an implanted vagal nerve stimulator. Transthoracic echocardiography is performed. It reveals a normal heart with normal intracardiac anatomy, no interatrial communication, normal aortic root, and no suggestion of thrombus. However, on suprasternal notch imaging, an unexpected finding is detected (Video 1, Video 2, Video 3, and Video 4). What is the BEST explanation of the finding in the lumen of the aorta? A.aortic dissection B.reverberation artifact C.side lobe artifact D.thrombus

B.reverberation artifact Video 1, Video 2, Video 3, and Video 4 reveal a linear structure evident in the lumen of the aorta that appears to bisect it. This structure crosses tissue planes. It causes no disturbance of flow based on color Doppler. Aortic dissection is confirmed when 2 lumina separated by an intimal flap can be visualized within the aorta. Positive criteria include complete obstruction of a false lumen, separation of intimal layers from a thrombus, or shearing of different wall layers during aortic pulsation. Frequently, the tear can be seen when there is disruption of the flap continuity with fluttering of the ruptured intimal borders. The available images do not demonstrate an occluded false lumen, a thrombus, or the appearance of fluttering intimal borders. The blood flow is not disturbed by the linear structure, which would be unexpected in a true dissection. Reverberation artifacts can occur in echocardiography. Within the reflected wave arising at the transducer, part of the energy is converted to electrical energy. However, another part of the wave is simply reflected on the transducer surface and will start propagating away from the transducer as if it were another ultrasound transmission. This secondary transmission will propagate in a way similar to that of the original pulse, which means that it is reflected by the tissue and detected again. These higher-order reflections are called reverberations and give rise to ghost structures in the image. They typically occur when strongly reflecting structures, such as ribs or the pericardium, are present in the image. In this vignette, there are 2 possibilities for this finding. First, the echogenic line appears to perfectly bisect the aorta. This could be a reverberation artifact from the strongly reflecting aortic wall surfaces. A second possibility is that the vagal nerve stimulator wire was positioned in such a way that it reflected signals from the transducer, which were propagated resulting in the appearance of a linear density in the middle of the aortic lumen. Side lobe artifacts can also occur in echocardiography. In the construction of an ultrasound image, the assumption is made that all reflections originate from the region directly in front of the transducer. Although most of the ultrasound energy is centered on an axis in front of the transducer, in practice part of the energy is also directed sideways (ie, directed off-axis). The former part of the ultrasound beam is called the main lobe, whereas the latter is referred to as the side lobe. Because the reflections originating from the side lobes are much smaller in amplitude than the ones coming from the main lobe, they typically can be neglected. However, image artifacts can arise when the main lobe is in an anechoic region (such as inside the left ventricular cavity), causing the relative contribution of the side lobes to become significant. In the typical scenario, a side lobe artifact is seen when the dominant structure is relatively large and anechoic, such as a dilated chamber. There is nothing about the echocardiogram to suggest thrombus. PREP Pearls Reverberation artifacts and side lobe artifacts should be considered in the differential when assessing echocardiographic images.

You are evaluating a term newborn (birth weight, 2.9 kg) with your fellow in the neonatal intensive care unit. There was a prenatal diagnosis of truncus arteriosus. The fellow wants to know the anatomic features that can affect the surgical outcome. In addition to the primary diagnosis (Video ), the echocardiogram also demonstrated a large atrial septal defect, quadricuspid truncal valve with moderate to severe truncal valve insufficiency, and right-sided aortic arch. Of the following, the anatomic feature that has the MOST impact on immediate postoperative outcome in this patient is A.adequacy of the branch pulmonary arteries B.atrial septal defect C.right aortic arch D.truncal valve insufficiency

D.truncal valve insufficiency Truncal valve insufficiency is a significant risk factor for higher mortality as compared to isolated truncus arteriosus repair in large multicenter cohorts. There are multiple repair techniques that can be performed on the truncal valve (if there is severe truncal insufficiency) during primary repair. However, the longer cardiopulmonary bypass time and cross-clamp time that is associated with the combined repair increases the morbidity and mortality of these patients. In most centers (irrespective of the center volumes), severe truncal valve insufficiency prior to the initial repair is associated with poor surgical outcome. As valve repair techniques and postoperative care have improved, recent single-center series have shown good short-term and midterm outcomes, even when truncal valve repair or replacement was performed along with the initial surgery. However, such results are not generalizable. Many series have shown that severe truncal valve insufficiency not only affects immediate surgical outcomes, but also increases the need for truncal valve replacement in late survivors. Significant truncal valve insufficiency in these patients leads to ventricular dilatation and hypertrophy with chronic subendocardial ischemia from inadequate coronary perfusion and ventricular failure. Truncus arteriosus repair includes separating the systemic and pulmonary pathways by closing the ventricular septal defect and placing a conduit from the right ventricle to the pulmonary artery confluence. The echocardiogram images (Video) in this vignette show adequately sized branch pulmonary arteries, hence the right ventricle to pulmonary artery conduit placement alone will not affect the immediate postoperative outcome. However, right ventricular outflow tract obstruction or branch pulmonary artery stenosis/regurgitation is a significant long-term concern in these patients, often requiring reinterventions or reoperations. Right aortic arch with mirror-image brachiocephalic branching is common in truncus arteriosus. Along with right aortic arch, hypoplasia of the aortic arch and interruption can occur frequently and is usually associated with 22q11.2 deletion. Right aortic arch by itself is not a risk factor for poor postoperative outcome; however, many studies have shown that interruption of the aortic arch poses a significantly higher risk in patients undergoing primary repair. This increased risk is secondary to the increased cardiopulmonary bypass time and cross-clamp time associated with the combined repair. The presence of an atrial septal defect is not an indicator of poor surgical outcome in truncus arteriosus repair. Coronary variations are also common in truncus arteriosus. Coronary anomalies like ostial stenosis and higher origin of the coronary arteries also need special attention during the perioperative period, as they have been shown to affect the immediate postoperative outcomes in certain series. PREP Pearls Presence and severity of truncal valve insufficiency is an important determinant in the immediate and long-term surgical outcome. Interrupted aortic arch in association with truncus arteriosus also increases the surgical morbidity in many series.

An echocardiogram has been ordered for a 6-year-old child who fainted on the school playground. The child's left ventricular systolic function and dimensions were normal. No valvular stenosis or significant regurgitation were noted. The left atrial volume was 40 mL/m² (Figure 1 ). Diastolic Doppler parameters obtained included an elevated mitral E/A ratio, and a short mitral E wave deceleration time (Figure 2 ). The pulmonary vein S was attenuated and prominent A wave reversal was seen in the right pulmonary vein. Lateral mitral tissue Doppler imaging e' wave velocity was low (Figure 3 ). Of the following, the findings noted in the child in the vignette are MOST consistent with: A. impaired relaxation B. normal variants C. precursors of systolic dysfunction D. pseudonormalization E. restrictive filling

E. restrictive filling No single isolated Doppler-derived parameter definitively diagnoses diastolic dysfunction in children because these parameters vary with age. However, a left atrial volume of 40 mL/m² is not normal at any age and the number of abnormal parameters described is not normal. Restrictive filling is the most severe form of diastolic dysfunction. The mitral E wave velocity is elevated and the A wave reduced, resulting in an elevated E/A ratio. Mitral E wave deceleration time is short. On pulmonary venous tracings, systolic flow is significantly reduced and A wave reversal is very prominent in the pulmonary veins with atrial contraction. The mitral TDI e′ is significantly reduced. Markedly dilated atria are classic features of restrictive cardiomyopathy. In impaired relaxation, the E/A ratio is low, less than 1, because of a reduced E wave velocity with a greater contribution of atrial contraction to ventricular filling. The mitral E deceleration time is prolonged. On pulmonary venous tracings, the reduced early filling results in reduced diastolic flow in the pulmonary veins with an increase in systolic forward flow. Prominent pulmonary vein A wave reversal is not typical at this stage. A decrease in tissue Doppler imaging (TDI) mitral e′ is typical in diastolic dysfunction, declining further with worsening dysfunction. Abnormal diastolic parameters have not been shown to be precursors of systolic dysfunction in children. Even in children with the worst form of diastolic dysfunction, restrictive cardiomyopathy, although ejection fractions typically are preserved, in some, they do decline over time. Some systolic TDI parameters are abnormal in children with restrictive cardiomyopathy who have preserved ejection fractions. In pseudonormalization, the mitral E/A ratio is normal. As the left ventricle becomes less compliant and stiffer, the filling pressures go up, increasing left atrial and left ventricular pressure. This results in a higher mitral E inflow velocity, "normalizing" the E/A ratio. The mitral deceleration time begins to shorten. On pulmonary vein Doppler, the S wave is diminished with more flow in diastole (D wave) because of the elevated filling pressure, and there is increased pulmonary A wave reversal. As in the other stages of diastolic dysfunction, mitral TDI e′ is reduced. PREP Pearls A significantly dilated left atrium in the absence of valvular pathology is an excellent indicator of significant diastolic dysfunction. Doppler indices of diastolic dysfunction should be assessed in combination because no single parameter is sensitive or specific enough to make a definitive diagnosis in children.

A 15-year-old previously healthy boy presents to the emergency department with worsening shortness of breath. Last week, he had fever, rhinorrhea, and cough. He denies nausea, vomiting, or diarrhea. He has been feeling very fatigued and unable to do his normal activities. His vital signs include a heart rate of 120 beats/min, respiratory rate of 20 breaths/min, blood pressure of 85/40 mm Hg, and oxygen saturation of 97% on room air; he is afebrile. He is alert. He has jugular venous distension. He is in moderate respiratory distress. His lungs have equal air entry, but crackles in the bases. He has regular rhythm with a gallop and a 1/6 holosystolic murmur. His abdomen is soft and mildly tender in the right upper quadrant, with a liver palpable 3 cm below the costal margin. His extremities are cool with 1+ pulses. Of the following, the BEST next step in the management of this patient is to give A. a 20-mL/kg fluid bolus B. broad-spectrum antibiotics C. norepinepherine infusion D. packed red blood cell transfusion E. supplemental oxygen

E. supplemental oxygen The patient in the vignette presents with signs of cardiogenic shock. Given the history of viral symptoms and acute presentation, myocarditis may be the underlying etiology. Systemic perfusion is maintained by the appropriate balance of cardiac output and systemic vascular resistance. Shock is a condition associated with decreased perfusion, resulting in insufficient oxygen delivery to the tissues. Shock has many causes, which are broadly categorized based on the mechanism that leads to decreased perfusion. Because cardiac output is affected by heart rate, preload, contractility, and afterload, diseases affecting any of these can lead to decreased cardiac output and potentially shock. Hypovolemic shock is caused by decreased preload (dehydration, hemorrhage). Distributive shock occurs when vasodilation (systemic inflammation, sepsis, and anaphylaxis) leads to low systemic vascular resistance. Cardiogenic shock is caused by decreased contractility. When myocardial contractility is decreased, the body will compensate using other mechanisms to increase cardiac output. Fluid retention, tachycardia, and vasoconstriction are all attempts to maintain cardiac output. The clinical effects of these compensatory mechanisms are seen in the patient in the vignette as elevated heart rate, jugular venous distension, crackles (pulmonary edema), hepatomegaly, and cool extremities. Patients with hypovolemic shock show signs of volume depletion. Those with distributive shock, or so-called warm shock, are vasodilated and typically have warm extremities despite poor tissue perfusion. The primary goal in treating shock is to improve the delivery of oxygen to the tissues. If possible, this should be done by reversing the underlying cause. Distributive and hypovolemic shock should be approached with aggressive fluid resuscitation. Blood transfusion is indicated if hypovolemia is thought to be related to hemorrhage. In contrast, patients with cardiogenic shock may respond poorly to fluid boluses, with worsening pulmonary edema. Administration of oxygen to patients in shock will improve delivery of oxygen to the tissues and is an important part of the early management of cardiogenic shock cases. A small fluid bolus may be appropriate in patients with suspected cardiogenic shock and minimal signs of fluid overload, but should be given with caution and frequent reassessment. Antibiotics are indicated for septic shock or any suspicion for infection. Norepinepherine may be helpful in patients with distributive shock because of its vasoconstrictive properties, but is contraindicated in patients with cardiogenic shock because it will primarily increase afterload. The use of inotropes such as dobutamine and milrinone may also be indicated. Diuresis for patients with pulmonary edema may help respiratory symptoms, but should be done with caution to avoid excess loss of preload. PREP Pearls Tachycardia, fluid retention, and poor peripheral perfusion are signs of cardiogenic shock. Recognition of cardiogenic shock can guide therapy that is unique to patients with cardiogenic shock compared with those with hypovolemic or distributive shock.

You are evaluating a newborn in the neonatal ICU with a prenatal diagnosis of tetralogy of Fallot (TOF) with hypoplasia of the branch pulmonary arteries. The parents refused prenatal genetic testing. Family history is significant for the diagnosis of congenital heart defect in the paternal grandmother, but the details are unknown to parents. Physical examination of the newborn reveals a small for gestational age baby with a heart rate of 140 beats/min, respiratory rate of 44 breaths/min, blood pressure of 74/46 mm Hg, and pre- and postductal saturations of 92% on room air. The newborn has icterus, broad forehead, deep-set eyes, small-pointed chin, and a bulbous nose. Cardiovascular examination reveals normal first heart sound and single second heart sound, with III/VI high-frequency early systolic murmur along the left upper sternal border radiating to both axillae. Abdominal examination reveals normal liver span without ascites. Echocardiogram confirms the diagnosis of TOF with left aortic arch and normal branching pattern. Initial laboratory results suggest direct and indirect hyperbilirubinemia with elevated liver enzymes. You discuss syndromic TOF with the family and proceed to order karyotyping and fluorescence in situ hybridization analysis. Of the following, the MOST likely diagnosis based on this scenario is A. Alagille syndrome B. DiGeorge syndrome C. Edwards syndrome D. VACTERL association

A. Alagille syndrome Alagille syndrome is an autosomal dominant disorder, defined as the presence of bile duct paucity on liver biopsy in conjunction with 3 of the 5 following features: cholestasis, cardiovascular, ocular, vertebral anomalies, along with the typical facial features described as broad forehead, deep set eyes, and pointed chin. This condition is caused by a single gene mutation (JAG1 gene) in the majority of cases (99%); the rest of the cases are due to a mutation in the NOTCH2 gene. A subset of Alagille patients (3% to 7%) have deletions of chromosome 20p12, which can be detected by fluorescence in situ hybridization (FISH) analysis and karyotyping. Therefore, individuals suspected with Alagille syndrome should undergo a karyotyping and FISH analysis to find the deletion or chromosome rearrangement. The JAG1 gene has been mapped to this region of the chromosome 20p12. If the FISH analysis is negative, JAG1 mutation analysis is clinically available to confirm the diagnosis, which has better detection capability. Thirty percent to 50% of individuals with Alagille syndrome have an inherited mutation that may not be clinically evident in a parent. Ninety percent of the individuals with Alagille have cardiovascular malformations, including peripheral pulmonary hypoplasia, tetralogy of Fallot (TOF), and pulmonary valve (PV) stenosis. These patients can have other features of the syndrome that may adversely affect their neonatal course, including the surgical management. Liver disease is highly variable, presenting with mild symptoms of jaundice and itching to liver failure based on the degree of bile duct paucity. The syndrome also involves, ophthalmologic, orthopedic (butterfly vertebrae), renal, endocrine abnormalities, and typical facies, as described in the vignette. The finding of branch pulmonary hypoplasia, along with TOF and hyperbilirubinemia, should raise clinical suspicion of Alagille syndrome in this patient. The physician should make arrangements for genetic and comprehensive evaluation of other diagnosis associated with this syndrome that may affect the surgical morbidity and long-term prognosis of these patients. DiGeorge syndrome is a microdeletion syndrome (22q11.2 deletion) characterized most commonly by cardiac defects (conotruncal defects, truncus arteriosus, interrupted aortic arch, aortic arch anomalies with ventricular septal defect, TOF with aortopulmonary collaterals or absent PV), neonatal hypocalcemia, immunodeficiency, palatal defects, facial dysmorphism, and learning disabilities. It is helpful to identify the hypoparathyroidism and complete T-cell deficiency that can coexist in a subset of DiGeorge patients during their surgical planning, both pre- and postoperative management. Facial features of this syndrome are mild and difficult to diagnose in a newborn, although have been described with widely set eyes, broad nasal bridge, low set ears, and cleft lip and palate. DiGeorge syndrome is not associated with jaundice or liver abnormalities. Infants with both an intracardiac defect and aortic arch anomalies are more likely to have 22q11.2 deletions, as are a subset of patients with TOF with absent PV and aortopulmonary collaterals. The patient in the vignette does not fit this description. Edwards syndrome (trisomy 18) is a lethal trisomy. Most of the fetal diagnoses end up in miscarriages or early fetal deaths. Fetuses have intrauterine growth restriction and neonates have a low birth weight. Affected individuals can have significant heart, renal, and other system involvement. The dysmorphic features include microcephaly, micrognathia, "rocker bottom" feet, clenched fists with overlapping fingers, and colobomas. The median life span for trisomy 18 is about 3 to 15 days. Trisomy 21 (Down syndrome) has also been associated with TOF. VACTERL association is defined by nonrandom concurrence of a group of congenital malformations: vertebral defects, anal atresia, cardiac defects, tracheoesophageal fistula, renal, and limb anomalies, as the acronym suggests. Clinical diagnosis requires the presence of at least 3 of the component features, in the absence of laboratory evidence of similar genetic syndromes like CHARGE syndrome. Patients with VACTERL association usually do not show facial dysmorphism or neurocognitive impairment. PREP Pearls Genetic testing will help to identify associated abnormalities in syndromic tetralogy of Fallot that will require careful consideration during the surgical planning. The genetic testing can be extended to family members in whom the diagnosis has not been suspected.

A 6-month-old male infant has been hospitalized twice since birth for pneumonia. On this, his second hospitalization for pneumonia, cardiology is consulted because chest x-ray showed a large heart and an infiltrate in the right middle lobe. His mother thinks his motor milestones are delayed compared to his older sister. The family history includes a maternal great uncle who had an enlarged heart and died in childhood. His mother has no known medical problems. The physical examination is notable for small size, tachypnea, and crackles in the right axilla. The point of maximal impulse is laterally and inferiorly displaced with a normal S1 and S2, a I/VI holosystolic murmur at the apex, and an S3. The liver is 3 cm below the right costal margin. Extremities, perfusion, and pulses are normal. Echocardiography shows a dilated left ventricle with an ejection fraction of 35% and moderate mitral regurgitation (Figure). The number and depth of ventricular trabeculations are increased. His complete blood count demonstrates a white blood cell count of 3,200/μL (3.2 × 109/L) with a reduced absolute neutrophil count. The C-reactive protein and erythrocyte sedimentation rate are elevated. Of the following, the MOST likely diagnosis in this patient is: A. Barth syndrome B. Becker muscular dystrophy C. Duchenne muscular dystrophy D. Emery-Dreifuss muscular dystrophy E. myotonic muscular dystrophy

A. Barth syndrome Barth syndrome is an X-linked cardioskeletal mitochondrial myopathy arising from a mutation in the G4.5 gene that encodes for a novel protein called tafazzin. This condition is associated with decreased amounts and altered structure of cardiolipin, the main phospholipid of the inner mitochondrial membrane. This disorder typically presents in infancy with congestive heart failure with variable degrees of cyclic neutropenia and 3-methylglutaconic aciduria. The cardiac manifestations include dilated forms with left ventricular noncompaction/hypertrabeculations, endocardial fibroelastosis (which can also be seen in association with left ventricular noncompaction), and hypertrophic dilated cardiomyopathy. These children are typically small for age before puberty, but may have catch-up growth thereafter. Neutropenia can result in serious bacterial infections and death. Proximal skeletal muscle weakness can lead to motor delays. Mild cognitive impairment occurs in some. Most children survive past infancy. Cardiac transplantation has been successful in such cases. Although most cases of dilated cardiomyopathy (~66%) are "idiopathic," in approximately 30% of children in whom a specific diagnosis is made, approximately 26% are associated with a neuromuscular disorder. Becker muscular dystrophy (BMD) is an X-linked cardioskeletal myopathy that is caused by mutations in the dystrophin gene. Dystrophin is believed to provide structural support for the myocyte and cardiomyocyte sarcolemmal membrane by its linking of actin with the dystrophin-associated protein complex, sarcolemma, and the extracellular matrix of muscle. BMD differs from Duchenne muscular dystrophy (DMD) because of the amount or quality of the expressed dystrophin protein. Patients with BMD remain ambulatory until the age of 15 years or even for many decades. The cardiac manifestations do not always correlate with skeletal muscle progression and may be present sooner or later than the skeletal problems. It does not present in infancy, nor does it have associated neutropenia. Duchenne muscular dystrophy is an X-linked cardioskeletal myopathy that is caused by mutations in the dystrophin gene. Patients with DMD have a much lower amount of dystrophin expressed in skeletal and cardiac muscle than those with BMD. Symptoms occur earlier than in BMD, but later than in most cases of Barth syndrome. Some patients with DMD crawl and walk later than average, but the disease is not typically recognized until about 3 years of age, when it becomes more obvious that they run or jump poorly and cannot keep up with their peers. Most become wheelchair dependent by about 12 years of age. Cardiac symptoms are less obvious because of the overall inability to be physically active. Emery-Dreifuss muscular dystrophy can be inherited as an X-linked syndrome because of a mutation of the STA gene that encodes emerin, a protein of the inner nuclear membrane or a mutation in FHL1, accounting for about 10% of the X-linked form. Autosomal mutations in LMNA gene commonly cause an autosomal dominant transmission, but autosomal recessive mutations also occur. Lamins are also found in the inner nuclear membrane and interact with other proteins, notably emerin, which likely accounts for the overlapping clinical phenotype of the different genotypes. In general, joint contractures appear during the first 2 decades, followed by muscle weakness and wasting. Cardiac involvement usually occurs after the second decade. Cardiac involvement is often manifested by conduction disturbances, ventricular or atrial arrhythmias, with symptoms of syncope or sudden death. The autosomal dominant form is more likely to have ventricular tachycardia and dilated cardiomyopathy than the X-linked form. Hypertrophic cardiomyopathy and 1 case of an adult with restrictive cardiomyopathy have been reported. Myotonic muscular dystrophy (DM1) is an autosomal dominant disease belonging to the group of disorders caused by expansion of a trinucleotide repeat. The mutation occurs in the myotonin protein kinase gene (DMPK) on chromosome 19 (19q13), resulting in a CTG repeat in the 3′ untranslated region of the gene. DM1 is characterized by anticipation, which means there is increasing severity and earlier onset of the disease phenotype in successive generations related to intergenerational expansion of the repeat size. Age at onset is variable (from prenatal to adulthood) with more severe forms seen in infancy. Feeding and swallowing difficulties, generalized hypotonia, and arthrogryposis are the main clinical features in infants with milder forms, without respiratory distress. In those with more severe infantile onset, the cardiac manifestations may include dilated cardiomyopathy (some with left ventricular noncompaction/hypertrabeculations, cardiac rhythm disturbances, and sudden death). Later onset disease (teenage years and later) more commonly manifests with rhythm and conduction disturbances than cardiomyopathy. PREP Pearls When infants present with cardiomyopathy, neuromuscular disorders should be considered in the differential. It is important to look beyond the cardiac data when attempting to make a diagnosis in infants with cardiomyopathy, including complete blood count to look for neutropenia and metabolic studies.

In your role as author of a chapter on echocardiography you are looking for an image showing normal great arteries. A colleague gives you access to her files, which unfortunately are poorly labeled. After looking through many images you find several that might work. Of the following, the image that BEST demonstrates the normal relationship of the great arteries is A. Figure 89.1 B. Figure 89.2 C. Figure 89.3 D. Figure 89.4

A. Figure 89.1 Figure A is the only image with normally related semilunar valves. In the parasternal short-axis view of the base of the heart, the aortic valve is centrally located and the pulmonary valve is leftward and anterior. This view also demonstrates that the semilunar valves are not located on the same plane. The great arteries arise at an angle and cross each other rather than in parallel, which prevents the semi-lunar valves from being shown simultaneously in a short-axis view en face. In Figure A only the aortic valve is seen en face. Figure B is a parasternal long-axis image demonstrating d-transposition of the great arteries. The aortic valve and ascending aorta are anterior to the more posterior pulmonary valve and main pulmonary artery. Figure C also shows d-transposition of the great arteries. In this parasternal short-axis view, the aortic valve is anterior and rightward of the pulmonary valve. Even though this view does not allow differentiation of the semilunar valves, it cannot be from a normal heart. Normally related great arteries cannot be seen in cross-section in the same image plane. Figure D shows a subcostal long-axis view of a patient with parallel great arteries. This is not possible in a heart with normal anatomy. PREP Pearls The aortic valve usually arises posterior and rightward of the pulmonary valve.

You are examining a 6-month-old infant in your clinic for the first time. He was referred to you for cardiac evaluation because a geneticist has made a diagnosis of congenital contractural arachnodactyly (CCA). You examine the infant and find that he does have physical stigmata that could be characteristic of this diagnosis. Of the following, the statement that BEST describes this patient's diagnosis is A. aortic root dilation is an uncommon finding in CCA B. ectopia lentis is a common finding in CCA C. mitral regurgitation is likely to be significant in CCA D. scoliosis is an uncommon finding in CCA

A. aortic root dilation is an uncommon finding in CCA Congenital contractile arachnodactyly (CCA), otherwise known as "Beal syndrome," is a rare autosomal dominant disease caused by a mutation in the fibrillin-2 (FBN2) gene on chromosome 5q23, as opposed to Marfan syndrome, which is usually caused by mutations in the fibrillin-1 (FBN1) gene on chromosome 15q21. Patients with CCA have similar skeletal characteristics as patients with Marfan syndrome, including a tall and asthenic 'marfanoid' body habitus, arachnodactyly, and kyphoscoliosis. The diagnosis of CCA should be contemplated in an infant or young child with contractures of the large joints of the extremities, external ear deformities (particularly crumpled external ears), kyphoscoliosis, and arachnodactyly. The cardiovascular manifestations of CCA are typically not prominent features, as opposed to the common cardiovascular manifestations seen in Marfan syndrome. Indeed, aortic root dilation is an uncommon finding in CCA. If present, aortic root dilation is characteristically mild and not progressive in CCA whereas aortic root dilation and dissection are hallmarks of Marfan syndrome. Mitral regurgitation caused by mitral valve prolapse is the most common cardiac manifestation of CCA, but is still thought to be uncommon in CCA. Other types of congenital heart disease have been described in case reports of patients with CCA but are rare. Ectopia lentis is a rare finding in CCA, but is a common finding in patients with Marfan syndrome, as seen in more than 50% of patients with this condition. Although not common, especially if scoliosis is treated at a young age, restrictive lung disease can occur and can be the cause of death in children with CCA. This lung disease is the result of the severe kyphoscoliosis together with thoracic cage abnormalities such as pectus chest wall abnormalities (pectus carinatum being most prevalent). PREP Pearls Cardiac manifestations are uncommon findings in congenital contractile arachnodactyly (CCA), with aortic root dilation being mild and nonprogressive. The physical stigmata of CCA overlap with those of Marfan syndrome because both are caused by mutations in genes coding for fibrillin.

You receive a phone call from a referring facility regarding a newborn infant with prenatally diagnosed hypoplastic left heart syndrome (mitral stenosis with aortic atresia) who has developed severe respiratory distress, with a respiratory rate of 85 breaths/min and moderate to severe subcostal retractions. The oxygen saturations are now 68%, and an arterial blood gas drawn from the umbilical arterial line demonstrated a pH of 7.17, a partial pressure of carbon dioxide of 33 mm Hg (4.4 kPa), a partial pressure of oxygen of 26 mm Hg (3.4 kPa), a lactate level of 6.7 mmol/L, and a hemoglobin level of 15 g/dL (150 g/L). The infant was delivered via elective cesarean section 2 hours earlier and was initially vigorous with Apgar scores of 7 at 1 minute and 8 at 5 minutes. Umbilical lines were placed and prostaglandins were initiated at 0.05 μg/kg per minute. Initial saturations were 82% on 3 L of nasal cannula flow with 21% oxygen. The neonatologist has intubated the infant, and obtained a chest radiograph (Figure 1). The umbilical arterial line has an excellent pulsatile tracing. An emergent echocardiogram is obtained (Figure 2). The neonatologist requests assistance in managing this case. Of the following, the BEST next management step for this infant should be to A. arrange for immediate transport to the cardiac catheterization laboratory for atrial septostomy B. arrange for immediate transport to the operating room for stage I Norwood palliation C. increase the fraction of inspired oxygen on the ventilator to 1.0 D. increase the prostaglandin infusion to 0.1 µg/kg per minute E. initiate inhaled nitric oxide to treat persistent pulmonary hypertension

A. arrange for immediate transport to the cardiac catheterization laboratory for atrial septostomy Infants born with hypoplastic left heart syndrome (HLHS) and restrictive atrial septum are often extremely ill shortly after delivery. When the restrictive atrial septum is prenatally recognized, immediate intervention can be undertaken after delivery to relieve the obstruction. When untreated, this can cause severe respiratory and hemodynamic compromise very quickly. The infant in the vignette was prenatally diagnosed with HLHS, but the restrictive atrial septum was not recognized. The infant was initially treated conventionally with a prostaglandin infusion and room air nasal cannula flow assuming that patency of the ductus arteriosus would be sufficient to maintain systemic and pulmonary blood flow. The rapid deterioration in the patient could be related to either closing of the patent ductus arteriosus or a restrictive atrial septum. Given that the patient was administered prostaglandins shortly after delivery, and the umbilical arterial line demonstrates adequate blood pressure tracing distal to the ductus arteriosus, the likelihood that this is caused by ductal constriction is low. The chest radiograph demonstrated diffuse interstitial edema with pulmonary venous congestion, raising concern for obstruction to pulmonary venous return, in this case via a restrictive atrial septum. Early and rapid identification via echocardiography and subsequent cardiac catheterization for intervention in these patients can be lifesaving, though often the degree of residual lung disease may prevent further surgical palliation. Increasing the prostaglandin infusion would not help the patient in the vignette. When a patient with HLHS presents with metabolic acidosis, restrictive patent ductus arteriosus with compromised systemic blood flow should be on the differential diagnosis. The patient in the vignette was given a prostaglandin infusion shortly after delivery at 0.05 μg/kg per minute. In addition, the presence of a pulsatile tracing in the umbilical arterial line distal to the ductus arteriosus can be reassuring, though it may not definitively prove that the ductus is patent. Echocardiography is the definitive test to determine patency of the ductus arteriosus, though with the low suspicion that this is the primary problem in this situation it would not be indicated. Increasing the fraction of inspired oxygen on the ventilator to 1.0 may exacerbate this patient's disease. Although severe hypoxemia is the result of a restrictive atrial septum, oxygen is a potent pulmonary vasodilator and could serve to worsen the degree of interstitial pulmonary edema by dilating blood vessels proximal to the outflow obstruction. Therefore, the minimum amount of oxygen possible should be used in this case to maintain adequate saturations. Similarly, the use of inhaled nitric oxide would cause similar complications. Persistent pulmonary hypertension of the newborn is a common cause of hypoxemia and acidosis; the chest radiograph in this case is unusual for persistent pulmonary hypertension. The presence of diffuse edema points to an obstructive process rather than pulmonary hypertension which usually presents with evidence of decreased pulmonary blood flow on chest radiograph. Although immediate surgical intervention with a stage I Norwood procedure in the patient in this vignette can be definitive, given the unknown degree of pulmonary vascular disease, it would be ill-advised. Patients with HLHS and restrictive atrial septum have a significant degree of interstitial lung disease which at baseline will elevate their pulmonary vascular resistance, even after relief of the atrial septal obstruction. In many patients, the degree of pulmonary vascular disease can be prohibitive for future surgical palliation including the Norwood and bidirectional Glenn procedures. Patients with HLHS and restrictive atrial septum require emergent cardiac catheterization to relieve the atrial obstruction, and often require stent placement to maintain patency of the atrial septum. Any potential delay in intervention could be catastrophic in these patients. Even with early catheterization and relief of atrial obstruction, the morbidity and mortality for these patients remain significantly higher than for patients with HLHS and unrestrictive atrial septum. The patient in the vignette developed severe hypoxemic respiratory failure related to a restrictive atrial septum. The treatment for this patient, once the atrial septal restriction is confirmed, is emergent transport to the cardiac catheterization laboratory for atrial septostomy to relieve the pulmonary venous obstruction. PREP Pearls Infants with HLHS with restrictive atrial septum are a medical emergency and require rapid recognition, intervention, and resuscitation immediately following birth. Rapid intervention in the cardiac catheterization laboratory can be life-saving.

A 5-month-old infant was admitted to the intensive care unit for respiratory distress. The intensive care team has ordered an echocardiogram because of the respiratory distress and pulmonary edema on chest x-ray. Her echocardiogram is available for review (Video 1 and Video 2). Of the following, the congenital abnormality that BEST explains the findings on the echocardiogram is A. cor triatriatum sinister B. supravalvar mitral ring C. total anomalous pulmonary venous connection D. unroofed coronary sinus

A. cor triatriatum sinister The echocardiogram in the vignette depicts cor triatriatum sinister, which consists of an accessory chamber that receives all or some of the pulmonary veins. The chamber typically communicates with the left atrium through a narrowed orifice, which may result in obstruction. In the absence of any direct communication with the left atrium, a connection must be established with the right atrium, either via an atrial septal defect or a decompressing vein. The typical appearance on echocardiogram is that of a membrane within the left atrium. The membrane runs parallel to the mitral valve annulus and will connect with the left atrial free wall proximal to the origin of the left atrial appendage and with the atrial septum proximal to the foramen ovale. It is often curvilinear, and when communicating with the left atrium, it can appear like a windsock or funnel. Color Doppler will show turbulent flow across the communication in most instances, though some membranes may have wide communication with the left atrium. There are several anatomic types of cor triatriatum sinister. Classic cor triatriatum sinister has an accessory chamber that receives all of the pulmonary venous return, with no other communications from the accessory chamber. The echocardiogram depicted in Video 3 and Video 4 shows an accessory chamber that communicates with the left atrium and right atrium (via an atrial septal defect). The right atrial connection can also be made via an anomalous venous connection. In some variations, the accessory chamber does not communicate with the left atrium and must have a right atrial connection. Finally, there are types of subtotal cor triatriatum in which only part of the pulmonary venous drainage comes into the accessory chamber. The accessory chamber can communicate with either or both atria. The remaining veins can have normal or abnormal connections. Cor triatriatum dexter is a membrane found in the right atrium, which can cause obstruction of systemic venous return. This is more rare than cor triatriatum sinister. The presentation of symptoms will depend on the anatomic subtype and the extent of obstruction to pulmonary venous return. At the most severe end, infants will present with signs of pulmonary venous obstruction, including pulmonary edema and pulmonary hypertension. Less severe obstruction may present at any age with respiratory distress, failure to thrive, or exertional dyspnea. Cyanosis can be present depending on the connections. Much like cor triatriatum sinister, a supravalvar mitral ring appears as a membrane in the left atrium. The clinical presentation also can be similar, because both obstruct pulmonary venous return. A supravalvar mitral ring can be distinguished from cor triatriatum by the location of the membrane relative to the foramen ovale and left atrial appendage. A supravalvar ring will be distal to both structures, whereas a cor triatriatum membrane is proximal to those structures. The supravalvar ring is often adherent to the mitral valve. Total anomalous pulmonary venous connection or return (TAPVC) can be associated with cor triatriatum. The connection between the pulmonary venous chamber and the left atrium shown in this echocardiogram is not consistent with TAPVC. An unroofed coronary sinus is a communication between the coronary sinus and the left atrium. The communication also allows communication between the left and right atrium through the coronary sinus ostium. The unroofed coronary sinus is often associated with a persistent left superior vena cava. A partially unroofed coronary sinus can involve a defect in any area of the tissue separating the coronary sinus from the left atrium. The coronary sinus is typically dilated, because of either persistence of the left superior vena cava, the left-to-right shunt through the defect, or both. Like a cor triatriatum membrane, the coronary sinus runs parallel to the mitral valve annulus (as it sits in the posterior atrioventricular groove of the mitral valve). The coronary sinus wall, however, will be seen posteriorly and inferiorly and would not be seen in the same plane as the mitral valve from an apical 4-chamber view. The defect in the wall of the coronary sinus is better viewed from a parasternal long-axis view, because the defect is typically in a different plane than that seen with cor triatriatum sinister. Color Doppler also reveals differences between the 2 entities. In a partially unroofed coronary sinus, blood flows from the left atrium, through the coronary sinus defect, into the coronary sinus, then across the coronary sinus ostium to the right atrium. As seen in the patient in the vignette, blood flows from the accessory chamber across the cor triatriatum sinister into the left atrium and across the mitral valve (or across the atrial septal defect to the right atrium). Unroofed coronary sinus and cor triatriatum sinister can be seen in association with one another. PREP Pearls Cor triatriatum sinister may obstruct pulmonary venous return into the left atrium. The anatomic type of cor triatriatum sinister will determine its echocardiographic appearance and the resulting physiology. Key features distinguish cor triatriatum sinister from other abnormal structures in the left atrium.

A 7-year-old boy who received a heart transplant at another institution presents to your emergency department (ED) accompanied by his aunt whom he was visiting while his parents went out of town. He developed a fever, with a temperature of 101°F, vomiting, and diarrhea, which his younger cousin also has. He vomited after taking his medications and his aunt brought him to the ED, but forgot to bring his medications and cannot remember their names. Phone calls to his parents went to voice mail. You examine the patient while you are waiting for a call back from the center where he received his transplant. The patient is sleepy but easily awakens and responds to questions. His heart rate is 110 beats/min, blood pressure is 100/70 mm Hg, and respiratory rate is 20 breaths/min. An examination of his head, eyes, ears, nose, and throat shows mildly coarse facial features, and mildly dry-appearing mucosa with marked gingival hyperplasia. His skin is warm and dry; he has a well-healed midline sternotomy scar; and his hirsutism is notable. The remainder of his examination is essentially normal. Of the following, the drug he is taking that is MOST likely to have caused these physical findings is A. cyclosporine B. mycophenolate mofetil C. prednisone D. sirolimus E. tacrolimus

A. cyclosporine Cyclosporine is a calcineurin inhibitor that was introduced in the early 1980s and led to significant improvement in posttransplant survival. Calcineurin is needed to activate T cells. Activated T cells mediate rejection. Cyclosporine binds to an immunophilin called cyclophilin. This complex in turn inhibits calcineurin, reducing T cell activation. Common cosmetic side effects of cyclosporine include gingival hyperplasia and hirsutism. Coarsening of facial features can also occur. More serious side effects include nephrotoxicity, hypertension, neurotoxicity (tremor, headache, motor disturbances, seizures), lymphoproliferative disorders, and hyperlipidemia. Cyclosporine interacts with multiple other medications that can change the level of cyclosporine or have additive nephrotoxicity. When starting a new drug in addition to cyclosporine a careful search for drug interactions should be performed. Although less commonly used now than tacrolimus, cyclosporine is still used in about 25% to 30% of pediatric patients. Mycophenolate mofetil is an antiproliferative agent that inhibits the de novo pathway of purine synthesis. It is an oral prodrug that is converted to mycophenolic acid. Side effects include leucopenia, nausea, vomiting, diarrhea, anorexia, and abdominal pain. Mycophenolate has largely replaced azathioprine at most pediatric transplant centers because of the suggestion that it may be more efficacious and has less nonspecific bone marrow suppression. Glucocorticoids (prednisone) have broad anti-inflammatory effects on cell-mediated immunity. However, the specific mechanisms of their immunosuppressive effects are myriad and incompletely understood. They do inhibit T cell proliferation and the activation of cytotoxic T cells. Side effects are multiple and include, but are not limited to hypertension, dyslipidemia, diabetes mellitus, obesity, osteopenia/porosis, cataracts, and muscle weakness. Use with cyclosporine may exacerbate hirsutism. Glucocorticoid resistance can develop during chronic glucocorticoid therapy because of the downregulation of glucocorticoid receptor expression, which could make the transplant recipient relatively steroid resistant during a rejection episode. Animal models have suggested that steroids inhibit tolerance induction and therefore could have a deleterious effect on transplant acceptance. In addition, data suggest that their long-term use may increase the risk of transplant coronary vasculopathy. Therefore many centers are trying to minimize or eliminate the use of steroids. Tacrolimus is a calcineurin inhibitor that binds to an immunophilin, FK506 binding protein. This complex in turn inhibits calcineurin, reducing T-cell activation. It is now the most commonly used calcineurin inhibitor in pediatric heart transplant patients, as some studies suggest that it reduces the frequency of rejection. Side effects associated with tacrolimus include nephrotoxicity, diabetes (more than with cyclosporine), hypertension (less than with cyclosporine), lymphoproliferative disorders, and neurotoxicity. Like cyclosporine, tacrolimus interacts with multiple other medications that can change its level or have additive nephrotoxicities. When starting a new drug in addition to tacrolimus a careful search for drug interactions should be performed. Sirolimus is a macrolide antibiotic and has a structure similar to tacrolimus. It binds to FK-binding protein, but this complex acts at a different site than the tacrolimus FK506-binding protein complex. Sirolimus inhibits a kinase, the target of rapamycin which results in the inhibition of clonal expansion of activated T cells. Approximately 9% of pediatric heart transplant patients take this agent. It is used to reduce or eliminate calcineurin inhibitors in patients with significant renal toxicity. It may slow transplant coronary vasculopathy and has been added to other agents in the face of refractory/recurrent rejection. Side effects include hyperlipidemia, hypertriglyceridemia, thrombocytopenia, anemia, leucopenia, noninfectious pneumonitis, and impaired wound healing. PREP Pearls Cyclosporine has more cosmetic side effects than tacrolimus. The most common side effects of calcineurin inhibitors are nephrotoxicity, hypertension, and neurotoxicity. Calcineurin inhibitors are a class of drug that has had the most significant impact on improving long-term survival in heart transplant recipients.

A 6 kilogram, 6-month-old infant with tricuspid atresia status post Blalock-Taussig shunt, returns from the operating room today after undergoing a bidirectional Glenn operation and takedown of the Blalock-Taussig shunt. The patient was transferred from the operating room about 1 hour ago. His vital signs included a heart rate of 125 beats/min with a sinus rhythm, a blood pressure of 80/40 mm Hg, pulmonary arterial pressure of 15 mm Hg, left atrial pressure of 8 mm Hg, and oxygen saturation of 60%. Chest radiograph shows hyperinflated lung fields. Ventilatory parameters are as follows: Synchronized intermittent mandatory ventilation with tidal volume, 70 mL Peak inspiratory pressure, 20 cm H20 Positive end-expiratory pressure (PEEP), 7 cm H20 Mean airway pressure, 14 cm H20 Fraction of inspired oxygen, 100% Arterial blood gases pH, 7.49 PCO2, 27 mm Hg (3.6 kPa) PO2, 28 mm Hg (3.7 kPa) Bicarbonate, 21 mEq/L (mmol/L) Base excess, -1 Hemoglobin, 14 g/dL (140 g/L) Hypoxemia is a concern. Of the following, given the infant's cardiac physiology, the optimal next step in his treatment is to A. decrease ventilator settings to reduce mean airway pressure and PEEP B. return to the operating room for addition of modified Blalock-Taussig shunt C. return to the operating room for revision of Glenn operation D. start inhaled nitric oxide E. transfuse packed red blood cells

A. decrease ventilator settings to reduce mean airway pressure and PEEP The Glenn operation consists of connecting the superior vena cava (SVC) to pulmonary arteries. Thus, the flow from SVC is the only source of pulmonary blood flow. The amount of pulmonary blood flow determines systemic oxygenation. The SVC flow consists primarily of venous return from the brain and upper extremities. Cerebral blood flow is affected by various factors including alkalosis. The patient in the vignette has respiratory alkalosis that will decrease cerebral blood flow, and therefore pulmonary blood flow. Furthermore, high mean airway pressure will increase pulmonary vascular resistance. High positive end-expiratory pressure (PEEP) and high tidal volume (exceeding 10 mL/kg) contribute to high mean airway pressure. Hyperinflation in the chest radiograph is supportive of this assessment. These ventilatory parameters will increase pulmonary vascular resistance, further contributing to decrease in pulmonary blood flow. Therefore, the optimal management steps that should be taken in this situation are reducing PEEP to 5 or 4 cm H2O and decreasing tidal volume while watching arterial blood gases for normalization of pH from the current respiratory alkalosis (Figure). Transfusion of packed red blood cells is not necessary because a hemoglobin of 14 g/dL (140 g/L) is adequate in an infant with cyanotic heart disease. Initiating inhaled nitric oxide is not indicated until the aforementioned changes to ventilator parameters are made to reduce pulmonary vascular resistance. Returning to the operating room for any surgical intervention would be premature until the effect of ventilatory optimization is assessed. PREP Pearls Change in pulmonary vascular resistance has a parabolic relationship with change in functional residual capacity of the lungs. Pulmonary vascular resistance changes are in direct proportion to mean airway pressure. Alkalosis decreases cerebral blood flow. Despite the fact that alkalosis reduces pulmonary vascular resistance, reduction in cerebral blood flow leads to reduced pulmonary blood flow in patients with Glenn operation.

You are evaluating a 22-month-old female child who was recently adopted from China. She has a history of congenital heart defect for which she has been taking furosemide at the orphanage. The past medical details are written in Mandarin, and the family has been instructed to follow up with a pediatric cardiologist after reaching the United States. On examination, she is found to be in no acute distress. Her weight and height are less than the third percentile. Her heart rate is 110 beats/min, respiratory rate is 30 breaths/minute, blood pressure is 80/40 mm Hg, and upper and lower extremity saturations are 100%. On physical examination, mild chest wall retractions are noted. The first heart sound is normal, but the second heart sound is difficult to appreciate. A high-frequency continuous murmur is best audible at the second left intercostal space. A separate holosystolic murmur is also audible at the apex. The liver is 1 cm below the costal margin. Bounding peripheral pulses are noted. Echocardiography was performed (Video 1, Video 2, and Figure 1), and a chest x-ray (Figure 2) obtained. Of the following the BEST next step in the management is: A. device occlusion of the ductus B. hemodynamic catheterization for vasodilator testing C. to add sildenafil to furosemide D. to administer intravenous indomethacin

A. device occlusion of the ductus The child in the vignette has features of compensated CHF due to left to right shunting through a PDA that is unrestrictive to volume. The clinical and echocardiographic features of the child in the vignette do not support severe pulmonary hypertension. The ductal flow is continuous left to right with LA and LV dilation and mitral regurgitation, all indicative of volume overload by the PDA. Therefore, closure of the ductus will be the most beneficial strategy for this patient. If the echocardiogram were to show right-to-left shunt at the PDA, hemodynamic catheterization with pulmonary vasoreactivity testing would be the next best step before considering PDA closure. Sildenafil is not the appropriate treatment because of the left-to-right shunt. Indomethacin is used for nonsurgical closure of the PDA in the neonatal intensive care unit population. The effects of this treatment are best when used in premature infants before 10 days of age. Indomethacin is presently considered the first line of therapy, unless contraindications like renal dysfunction or necrotizing enterocolitis coexist in the neonatal period; however, this is not the first line of management in a 22 month old with longstanding PDA. Device occlusion of the ductus will alleviate the symptoms of CHF and reduce the progression of pulmonary vascular disease in this patient. In a moderate to large ductus, left-to-right shunt can cause left ventricular (LV) dilation and increased LV end-diastolic pressure, thus raising the left atrial (LA) pressure, which results in overt cardiac failure symptoms. These symptoms are often irritability, poor feeding, easy fatigability, and recurrent respiratory infections. Physical signs of a ductus causing congenital heart failure (CHF) include bounding peripheral pulses, hyperdynamic precordium, laterally shifted apical impulse, and a continuous high-frequency murmur of crescendo-decrescendo quality at the second left intercostal space with mid-diastolic rumble at the apex. Some infants also have a murmur of mitral regurgitation secondary to LV volume overload. Some infants may not survive without treatment for heart failure, but some are capable of adequate compensation, whereby they continue to grow at a lower percentile. As the disease progresses, pulmonary vascular resistance increases to an extent with irreversible changes in the pulmonary vasculature. The medial smooth muscles of the small pulmonary arteries develop intimal damage, cellular proliferation, hyalinization, and finally thrombosis and fibrosis, causing pulmonary hypertension. The clinical symptoms of this state would be cyanosis, which is more pronounced in the lower extremities, initially with exertion, and eventually even at rest. The signs would include prominent right ventricular impulse, palpable second heart sound, and a diastolic murmur of pulmonary insufficiency. A holosystolic murmur of tricuspid insufficiency may also be audible if pulmonary hypertension has developed. Two-dimensional echocardiography with Doppler and color flow mapping is the diagnostic modality of choice to detect the presence of the PDA and assess the physiology. Doppler echocardiography can show flow through the ductus arteriosus and the direction of the shunt, thus allowing an estimation of pulmonary artery pressure. Echocardiography also helps to assess the left-sided volume overload or the degree of pulmonary hypertension in a child. PREP Pearls Longstanding patent ductus arteriosus (PDA) can cause pulmonary vascular disease. Patients with compensated congenital heart failure may exhibit minimal symptoms other than failure to thrive. Echocardiography is the imaging modality of choice to evaluate PDA.

A full-term female newborn is evaluated in the neonatal intensive care unit after being diagnosed prenatally with an unbalanced complete atrioventricular septal defect. Echocardiography reveals levocardia and confirms the intracardiac diagnosis with bilateral identical atrial appendages noted. Echocardiography also reveals the presence of abdominal situs inversus with a left-sided liver and a right-sided stomach. Of the following, the embryonic event MOST likely to have occurred in this patient is A. disruption of motile ciliary function B. disruption of sensory ciliary function C. failure of motile cilia to form D. leftward flow of extraembryonic fluid at the left-right organizer generated by motile cilia E. rightward flow of extraembryonic fluid at the left-right organizer generated by motile cilia

A. disruption of motile ciliary function The visible outward symmetry of the human body belies its complex internal asymmetry, the development of which is driven by a ciliary-mediated process during embryonic development. In the normal situation, the midline heart tube forms a rightward (D) loop and the visceral organs assume characteristic positions including the liver on the right and the stomach on the left. When the cellular and molecular processes that lead to this normal asymmetry are disrupted, the heart may form a leftward (L) or midline loop, often with a major structural defect such as an unbalanced atrioventricular septal defect, and the arrangement of the visceral organs may be random or completely inverted. The condition in which there is neither levocardia and normal abdominal situs nor situs inversus totalis, is defined as heterotaxy. In a unique process that is based almost entirely on mechanical forces and occurs outside the embryo itself, the establishment of visceral and cardiac laterality in vertebrates and the breakage of inherent embryonic bilateral symmetry are initiated at the embryonic left-right organizer (LRO) or node late in gastrulation. This is when motile primary cilia generate directional flow of extraembryonic fluid surrounding the node (nodal flow). The axonemal heavy chain left-right dynein localizes to the LRO and drives the counterclockwise movement of the nodal primary cilia, which creates the flow. The leftward-directed flow generated by the motile cilia is detected subsequently by immotile sensory cilia. The immotile cilia in turn transduce that flow into a cascade of downstream asymmetric signals and gene expression. Molecular asymmetry is present before the development of embryonic asymmetry. For example, Pitx2 is an early marker of left-right patterning that is normally induced by nodal signaling on the left side of the embryo and expressed in the left lateral plate mesoderm but inhibited on the right. Nodal flow also leads to increased intracellular calcium in cells at the left side of the node, which contributes to the asymmetric gene expression and morphogenesis (Figure 1). In mice, ciliary defects have been shown to be of 3 types: motility, sensory function, and biogenesis. If the cilia are structurally normal but immotile, the mice demonstrate randomization of left-right asymmetry. In the case of abnormal ciliary sensory function, bilateral symmetry persists. When ciliary biogenesis is disrupted, the cardiac structure is markedly abnormal and there is universal lethality. Multiple candidate genes have been implicated in abnormalities of morphologic left-right development and new ones continue to be identified on an ongoing basis. These include transforming growth factor beta (TGF-β) and polypeptide N-acetylgalactosaminyltransferase 11 (GALNT11). Both increased and decreased signaling of TGF-β cause abnormalities in global left-right axis formation as well as in later stages of looping morphogenesis. The NODAL gene is a member of the TGF-β superfamily and its expression at the left edge of the embryonic node is essential for normal left-right development. It was recently shown that GALNT11, a polypeptide enzyme, modifies the Notch signaling pathway thereby starting a process that affects cilia in a way that ultimately determines organ laterality. GALNT11 O-glycosylates human NOTCH1 peptides in vitro, leading to Notchactivation and signaling. This process modifies the spatial distribution and ratio of the motile and immotile cilia at the LRO. In cases of GALNT11 overexpression and resultant increased Notchexpression, ratio of motile to immotile cilia is decreased, and the clinical picture mimics that of primary ciliary dyskinesia with randomization across the left-right axis. However, in the setting of GALNT11 insufficiency and decreased Notch expression, a greater number of motile cilia are present, leading to bilateral symmetry and isomerism similar to that seen when Pkd2, a ciliary sensor that helps facilitate flow detection, is lost (Figure 2). In addition to the abnormalities of cardiac and visceral asymmetry that are evident in patients with disrupted left-right development, many of these patients have associated defects in other organ systems because of the critical roles played by the genes controlling left-right development in those positions through their effect on cilia. These include the respiratory tract, the kidneys, and the male genitourinary tract. Affected individuals may present with the so-called "Kartagener triad" of situs inversus, chronic sinusitis, and bronchiectasis, with male infertility frequently seen in those patients. The respiratory insufficiency present in patients with abnormal situs is often a complicating factor in patients' clinical course, particularly those with cardiac disease, and can be a major contributor to their disproportionately poor surgical outcomes. Because of the important clinical implications, cardiologists are increasingly expanding their investigations into the genetic basis of the heterotaxy syndromes present in their patients. PREP Pearls The establishment of visceral and cardiac laterality in vertebrates is initiated at the embryonic left-right organizer (LRO) or node where motile primary cilia generate directional flow of extraembryonic fluid surrounding the node (nodal flow). If the cilia are structurally normal but immotile, the mice demonstrate randomization of left-right asymmetry. In the case of abnormal ciliary sensory function, bilateral symmetry persists. In addition to the abnormalities of cardiac and visceral asymmetry that are evident in patients with disrupted left-right development, many of these patients have associated defects in other organ systems because of the critical roles played by the genes controlling left-right development in those positions through their effect on cilia.

A 1-week-old male neonate is admitted to the hospital for stridor and increased work of breathing, which gets worse during feeding. The mother states that the infant stops breathing during the feeds and turns blue. He was born at term without complications and was discharged from the hospital in 2 days. The mother noticed "noisy breathing" after discharge and discussed the same with the pediatrician at the 1-week follow-up visit. The pediatrician admits the infant to your hospital for evaluation of symptoms. On physical examination, the infant has mild nasal flaring and intercostal retractions. He keeps his neck extended in the bed. Auscultation reveals coarse upper airway sounds and wheezing, with normal heart sounds and no murmurs. Cardiac magnetic resonance imaging scans are shown (Figure 1 and Figure 2). Of the following, the MOST likely diagnosis causing this infant's stridor is: A. double aortic arch with a dominant right arch B. left aortic arch with aberrant right subclavian artery and left ductus arteriosus C. right aortic arch with aberrant left subclavian artery and right ductus arteriosus D. right aortic arch with mirror image branching and left ductus arteriosus

A. double aortic arch with a dominant right arch Among the choices in the vignette, the true vascular ring is only the double aortic arch. Double aortic arch usually presents in infancy with stridor, dyspnea, and a barking cough, all of which are worse during feeding. Reflex apnea lasting for seconds can be triggered by feeding as described in the vignette. Symptoms related to esophageal compression are less frequent, but can include feeding difficulties, vomiting, and choking. None of the other choices form a true vascular ring and hence cannot explain the symptoms in this patient. The description of the posture of the infant is also characteristic of this anatomy because arching of the back and extension of the neck minimizes the tracheal narrowing. The clinical presentation of vascular rings can be variable with variable degrees of respiratory symptoms ranging from severe stridor to no symptoms. Vascular rings are rare anomalies and a high index of suspicion is necessary in a neonate who presents with stridor and reflex apnea with feeding. The differential diagnosis also includes common conditions like aspiration, laryngomalacia, tracheomalacia, subglottic stenosis, or pneumonia. Once arch anomalies are suspected as the cause for stridor, appropriate diagnostic studies should be performed to delineate the anatomy and with the goal of surgical planning. Understanding the embryology of aortic arch development is essential to understand the development of vascular rings. The branchial arches develop between the second and seventh weeks of gestation. The branchial apparatus consists of 6 branchial arches that develop in the wall of the foregut. Each of these arches connects the truncus arteriosus of the embryonic heart tube to the paired dorsal aorta. The arches appear and disappear during different times in embryonic life, and are never present simultaneously. The first arches contribute to the external carotid arteries. The second arches involute before the development of the sixth arch and a portion of it remains to form the stapedial and hyoid arteries. The third arches form the common carotid arteries and the proximal portion of the internal carotid arteries. The fourth arch develops into the definitive left aortic arch between the left carotid artery and left subclavian artery (LSCA). The right fourth arch is incorporated into the proximal right subclavian artery (RSCA). The fifth arches regress. The proximal part of sixth arches becomes the branch pulmonary arteries, and the distal portions join the pulmonary vascular tree to the descending aorta via bilateral ductus, with the right regressing completely to leave a left ductus arteriosus. The seventh intersegmental arteries arise from the dorsal aortae and become the LSCA and distal right subclavian artery (RSCA). When the right dorsal aorta regresses, as is the norm, the definitive left aortic arch is formed (Figure 3 , Figure 4 , and Figure 5). Most of the arch anomalies are asymptomatic, but a true vascular ring can cause tracheoesophageal compressive symptoms. A double aortic arch is the most common vascular ring, followed by right aortic arch (RAA) with an aberrant left subclavian artery (ALSCA) and a left ductus/ligamentum. Isolation of subclavian, brachiocephalic, or carotid arteries can cause steal phenomenon from the cerebral circulation (Table). Double Aortic Arch This anomaly persists if there is no break in the hypothetical double arch and both fourth arches and dorsal aorta persist. The ascending aorta bifurcates in front of the trachea with 1 arch coursing to the right and 1 to the left. They may be equal in size or 1 may be hypoplastic. RAA is dominant in 75% of double arches. Double aortic arch usually forms a vascular ring with the associated left ligamentum. Occasionally a segment of an arch may be atretic with a fibrous cord between the carotid and subclavian arteries, which then become difficult to identify as a double arch on imaging studies. Left Aortic Arch Anomalies and Clinical Presentation Based on the arch configuration, the most common anomaly associated with left aortic arch is the ARSCA with a left ductus arteriosus. This anomaly occurs because of a break in the primitive RAA between the right subclavian and right common carotid arteries. Thus ARSCA is the last aortic branch that arises from the arch and travels behind the esophagus to the right upper extremity. There is usually a diverticulum of Kommerell (dilated vascular structure from the remnants of the embryologic fourth aortic arch and constitutes the proximal portion of ARSCA) in this condition. When an ARSCA arises from a dilated diverticulum of Kommerell, it can cause compressive symptoms like dysphagia (dysphagia lusoria), though this entity is not a true vascular ring. Isolation of the subclavian artery is more common with the RAA than the left aortic arch and will be discussed along with the right arch anomalies. RAA Anomalies The RAA develops from dissolution of left dorsal aorta instead of the right. RAA can be classified as follows. 1) RAA with mirror image branching. Regression of the dorsal aorta between the left ductus and descending aorta produces mirror image branching. This anomaly is the mirror image of the conventional branching pattern of the normal left aortic arch. Mirror image branching is usually associated with cyanotic congenital heart diseases and hence further evaluation is necessary in these patients. 2) RAA with aberrant LSCA (ALSCA) arising from diverticulum of Kommerell with right or left ductus arteriosus. The break between left common carotid and LSCA causes type 2 or RAA with ALSCA. Thus ALSCA arises as the last branch from RAA or from the diverticulum of Kommerall and passes behind the esophagus and trachea. This course of ALSCA together with a left ligamentum can form a "complete vascular ring" and cause tracheoesophageal compressive symptoms. In this anomaly, the branching configuration will be left common carotid, right common carotid, and right subclavian arteries, and then the LSCA arising from the Kommerell diverticulum. The presence of a diverticulum at the origin of ALSCA is important to note in imaging because it adds complexity to the surgical correction. If the ligamentum is to the right of the midline in the vignette, it does not form a vascular ring. 3) RAA with isolation of LSCA. RAA with isolated LSCA takes place if the break in the aortic arch is proximal and distal to the origin of the LSCA. RAA with isolation of LSCA is a rare anomaly and LSCA may be connected to the pulmonary artery via ductus or vertebral artery where it causes steal phenomenon. Imaging Considerations Diagnostic imaging in vascular rings should be obtained with the goal of identifying the cause of the symptoms and preoperative planning. There is no universal imaging algorithm, but it should be based on the center's expertise, costs, and available technology. Barium esophagograms are usually helpful to determine whether the vascular ring is the culprit for symptoms but may have limited depiction of vasculature. Echocardiogram allows good depiction of cardiovascular anatomy but many centers require additional imaging for definitive anatomic delineation for surgical planning. Its weakness is poor visualization of the airway and poor windows in certain patients. Both cardiac magnetic resonance imaging (MRI) and computed tomography angiography define the vasculature, airway, and their 3-dimensional relationship with high accuracy and reliability. CTA has the disadvantage of ionizing radiation; MRI may have the disadvantage of longer study duration and the need for sedation, especially in younger patients. PREP Pearls Infantile stridor could be secondary to vascular rings. The most common vascular rings are double aortic arch and right aortic arch with aberrant left subclavian artery with left ductus arteriosus. Accurate imaging of the arch anatomy is critical to surgical planning. The clinical presentation of vascular rings can be variable with variable degrees of respiratory symptoms.

A 4-year-old child with newly diagnosed acute lymphoblastic leukemia (ALL) is referred for prechemotherapy echocardiogram. Figure 1 shows parasternal long axis view. Of the following, the systemic venous anomaly associated with the finding in this image would MOST likely be caused by the persistence of A. left anterior cardinal vein B. left posterior cardinal vein C. left subcardinal vein D. left supracardinal vein E. left vitelline vein

A. left anterior cardinal vein Figure 1 shows an echocardiogram with a dilated coronary sinus. Persistent left superior vena cava (LSVC) draining to coronary sinus is the finding associated with this image. Additional views may confirm that the left-sided vertical vein connects from the left innominate vein, on the left side of the aortic arch and anterior to the left pulmonary artery. Embryologically, LSVC develops from the left anterior cardinal vein and left common cardinal vein, and coronary sinus develops from the left common cardinal vein and left horn of sinus venosus. Therefore, LSVC continues as coronary sinus in the atrioventricular groove posterior to the left atrium, and drains to the right atrium. The LSVC does not communicate with the left atrium unless the coronary sinus is unroofed. In rare instances, apparent LSVC draining to the left atrium is probably secondary to complete unroofing of the coronary sinus or results from persistence of a connection that represents levoatrial cardinal vein. The levoatrial cardinal vein is an embryonic vessel connecting pulmonary venous plexus on the caudal side to the anterior cardinal venous system on the cranial side. The pulmonary venous plexus forms future pulmonary veins, so the levoatrial cardinal vein may connect to the pulmonary vein or left atrium. The levoatrial cardinal vein courses posterior to left pulmonary artery, whereas persistent LSVC courses anterior to left pulmonary artery. Figure 2 shows a schematic diagram of embryologic development of systemic veins of the heart, as viewed from the back. Evaluation of LSVC should confirm the following details: direction of flow in LSVC which is usually toward the coronary sinus presence or absence and size of left innominate vein ("bridging vein" between LSVC and right superior vena cava) presence or absence of right superior vena cava unroofed coronary sinus (partial or complete) presence of any tributaries to the LSVC before it joins the left innominate vein, which may include either left superior intercostal vein or 1 of the left pulmonary veins In the latter situation, partial anomalous pulmonary venous return exists. PREP Pearls Presence of a dilated coronary sinus on echocardiogram should prompt further evaluation for persistent left superior vena cava (LSVC). Evaluation for persistent LSVC includes direction of flow, presence of left innominate vein, tributaries to LSVC such as superior intercostal vein or pulmonary vein, and presence of right SVC. The LSVC represents persistence of left anterior cardinal vein and common cardinal vein.

A 2-day-old full-term male infant is in the neonatal intensive care unit (NICU). He was diagnosed with cleft palate after birth, and has remained in the NICU because of difficulty with feeding. On physical examination, he is noted to have mild tachycardia (heart rate 170 beats/min) and tachypnea (respiratory rate 60 breaths/min). He is alert, and his oxygen saturation is 95% in his right arm and 88% in his left arm and both feet. His 4 extremity blood pressures are as follows: right arm, 75/45 mm Hg; left arm, 50/35 mm Hg; right leg, 52/34 mm Hg; left leg, 49/32 mm Hg. His lungs are clear despite mild tachypnea. His precordium is hyperactive, and he has a regular rate and rhythm with a 2/6 systolic murmur at the left sternal border. His abdomen is soft, with the liver palpable 2 cm below the costal margin. He has a 2+ pulse in his right arm, but decreased pulses in his left arm and legs. His lower extremities are cool with delayed capillary refill. An echocardiogram was obtained (Video 1 and Video 2). Of the following, the embryologic structure that BEST explains the presentation of this patient is regression of A. left fourth arch and right dorsal aorta B. left fourth arch and right sixth arch C. right dorsal aorta and left dorsal aorta D. right sixth arch and right dorsal aorta

A. left fourth arch and right dorsal aorta The infant in the vignette has the clinical presentation of an infant with obstruction of the aortic arch. The location of the obstruction is somewhere between the takeoff of the right subclavian artery and the left subclavian artery, based on the differential saturations and blood pressures in the right and left arms. Obstruction of the arch in this region is most commonly associated with an interrupted aortic arch. The infant has a cleft palate that raises the suspicion for deletion of 22q11. He also shows evidence of a ventricular septal defect (Video 2). These 2 associations are seen most commonly in interrupted aortic arch type B. The embryologic basis for interrupted aortic arch type B is regression of the left fourth aortic arch and the right dorsal aorta. This results in an interruption between the left common carotid artery and the left subclavian artery (Figure ). Three main types of interrupted aortic arch are seen, which are defined as follows: Type A: Interruption distal to the left subclavian artery (or right subclavian artery with right aortic arch) Type B: Interruption between the left subclavian artery and the left common carotid artery (or right common carotid and right subclavian in right arch) Type C: Interruption between the carotid arteries Type B is the most common and is often associated with a ventricular septal defect (often with posterior malalignment). About two-thirds of patients with an interrupted aortic arch type B have 22q11 deletion. The developing embryo has 6 arterial arches and 2 paired dorsal aortae. In the course of development, parts of this system persist while others regress, eventually forming the mature great arteries and head vessels. The normal left aortic arch contains components of the truncus arteriosus and truncoaortic sac, the left fourth arch, the left dorsal aorta between the fourth and sixth arch (which forms the patent ductus arteriosus), and distal to the sixth arch. The innominate artery is formed by the right branch of the truncoaortic sac, the right subclavian is formed by the right fourth arch and the right seventh intersegmental artery, the carotid arteries are formed by the third arches, and the left subclavian artery is formed by the left seventh intersegmental artery. Regression of the right sixth arch and the right dorsal aorta results in the normal left aortic arch. Incorrect persistence or regression of these vessels explains normal and abnormal variants of aortic arch anatomy. Regression of the left fourth arch and the right sixth arch results in a right-sided aortic arch, aberrant left subclavian artery with a left patent ductus arteriosus. This arrangement leads to a vascular ring and is typically seen in isolation. Regression of the left and right dorsal aortae results in an interrupted aortic arch type A. PREP Pearls Interrupted aortic arch type B is an interruption in the portion of the aortic arch between the left common carotid artery and the left subclavian artery in patients who have a left-sided aortic arch. Regression of the left fourth arch and the right dorsal aorta results in this type of interruption.

You are performing a cardiac catheterization on a 2-day-old girl who presented with cyanosis approximately 4 hours after birth. She was delivered at full term with a birthweight of 2,652 g. Initial oximetry revealed saturations in the 60% to 70% range. She was given supplemental oxygen, an intravenous line was placed and prostaglandin E1 was started. Her oxygen saturations increased into the 80% range. She was transferred to your institution where echocardiography was performed. Input Z score Aortic annulus diameter (PLAX) 0.8 2.35 Aortic sinotubular junction 0.79 0.89 Ascending aorta diameter 0.61 -0.7 Transverse aorta diameter 0.6 0.56 Aortic isthmus diameter 0.37 -1.08 MM LV diastolic dimension 2.38 3.54 MM LV diastolic septal thickness 0.5 1.4 MM LV diastolic wall thickness 0.8 7.4 MM LV systolic dimension 2.5 10 MM LV systolic wall thickness 1.3 12.11 Tricuspid annulus diameter (4-ch) 0.8 -1 Proximal right coronary artery 0.12 0.85 Left main coronary artery 0.18 1.38 PLAX = parasternal long axis; MM = M-mode; LV = left ventricular; 4-ch = 4-chamber. Video 1 and Video 2 show angiograms obtained during the catheterization. Of the following, the BEST next step in management is to A. perform balloon pulmonary valvuloplasty B. perform surgical tricuspid valvuloplasty C. place a surgical aortopulmonary shunt D. place a surgical cavopulmonary (Glenn) shunt

A. perform balloon pulmonary valvuloplasty For the patient in the vignette, who has pulmonary atresia with intact ventricular septum, a balloon valvuloplasty is the best next step in management. The tricuspid valve annulus, with a Z score of -1, is not abnormally small. The right ventricular (Video 1) and aortic (Video 2 ) angiograms demonstrate membranous pulmonary atresia with a tripartite right ventricle with normal coronary artery anatomy. There is no evidence of right ventricular-dependent coronary circulation, which is defined as coronary blood flow that is at least partially dependent on retrograde blood flow from the right ventricle. The preferred treatment in the patient with an adequate-sized tricuspid valve without right ventricular-dependent coronary circulation is to decompress the right ventricle. The goal of decompression of the right ventricle is to promote growth of the tricuspid valve and right ventricle. The ultimate goal is to achieve a biventricular repair. The surgical approach to decompression involves a transannular patch and/or pulmonary valvotomy. In patients with membranous pulmonary valve atresia, radiofrequency perforation of the atretic pulmonary valve followed by balloon valvuloplasty is an acceptable alternative to surgery to decompress the right ventricle. An additional procedure may be necessary to provide adequate pulmonary blood flow and oxygenation until the right ventricle growth is adequate. If needed, additional pulmonary blood flow can be provided with either surgical placement of an aortopulmonary shunt, such as a modified Blalock-Thomas-Taussig shunt, or transcatheter stent placement in the patent ductus arteriosus. Placement of a surgical aortopulmonary shunt is not the best option for this patient given the favorable anatomy for biventricular repair. It may be necessary, but the primary therapy should be aimed at decompressing the right ventricle. The right ventricle angiogram demonstrates significant tricuspid valve regurgitation, but there is no evidence of an Ebstein anomaly. The tricuspid valve annulus appears small, but the echocardiographic measurement of the valve annulus is within normal for this infant. There is no role for tricuspid valve repair as the initial therapy in this case. Because the anatomy favors a biventricular repair, a Glenn shunt would not be the preferred choice of treatment. In addition, the infant's age would preclude this surgery until she is older, after the pulmonary vascular resistance had fallen. PREP Pearls Pulmonary valve atresia with intact ventricular septum is associated with a great degree of variability in the size of the right ventricle and tricuspid valve. Patients with a tripartite right ventricle without right ventricular-dependent coronary arteries and mild hypoplasia of the tricuspid valve often can undergo a biventricular repair.

A 3-year-old boy is seen for evaluation of cardiac murmur. He seems to tire easier than his peers but otherwise seems largely asymptomatic. He has a heart rate of 102 beats/min, respiratory rate of 20 breaths/min, and blood pressure of 118/68 mm Hg in the right arm, 102/66 mm Hg in the left arm, 104/64 mm Hg in the right leg, and 104/64 mm Hg in the left leg. A thrill is evident in the suprasternal notch and a grade 4/6 crescendo-decrescendo murmur is evident along the right and left upper sternal borders with radiation to the suprasternal notch. Diastole is quiet. There is no click. Radial and femoral pulses are 2+ bilaterally. He has a long philtrum, flat nasal bridge, and prominent lower lip. He smiles frequently, and his smile is broad and wide. Which of the following chromosomal deletions is MOST likely to be evident in this patient? A.7q11.23 B.8p23 C.18q D.22q11

A.7q11.23 This vignette describes a case of Williams syndrome (sometimes referred to as Williams-Beuren syndrome) with supravalvar aortic stenosis. Williams syndrome is characterized by congenital heart disease and other manifestations including hypercalcemia, skeletal anomalies, renal anomalies, cognitive defects, characteristic facial features (the so-called "elfin facies"), and a social personality. The cardiac manifestations may include supravalvar aortic stenosis and supravalvar pulmonary stenosis. There may be an associated bicuspid aortic valve, but not typically. Coronary artery ostial stenosis can occur and may lead to sudden cardiac death. The supravalvar aortic stenosis (rather than valvar aortic stenosis) does not result in a click. The "Coanda effect" with excess blood flow directed toward the innominate artery may result in elevated blood pressure in the right upper extremity, as evident in this vignette. The supravalvar aortic stenosis worsens with time, while supravalvar pulmonary stenosis may improve with time. Approximately 90% of patients with the clinical diagnosis of Williams syndrome have a deletion at 7q11.23 (https://ghr.nlm.nih.gov/condition/williams-syndrome#genes). This region houses the gene responsible for making elastin, ELN. ELN encodes the protein tropoelastin. Multiple copies of tropoelastin join to form the mature protein elastin. Elastin is the major component of elastic fibers, which are slender bundles of proteins that provide strength and flexibility to connective tissue. As a result of this deletion, people with Williams syndrome exhibit decreased production of elastin. Large blood vessels with abnormal elastic fibers are often thicker and less resilient than normal. These vessels can narrow, increasing the resistance to normal blood flow over time. This process results in the characteristic supravalvar aortic and pulmonary stenosis seen in Williams syndrome. Deletion of 8p23 is associated with atrial or ventricular septal defects, genitourinary anomalies, abnormally formed ears, and minor hand anomalies. Deletion of 18q (also called monosomy 18q syndrome) is associated with atrial and/or ventricular septal defects as well as pulmonary stenosis. Other system anomalies include cleft palate and genitourinary anomalies. The 22q11 deletion syndrome (also known as DiGeorge syndrome or velocardiofacial syndrome) is characterized by hypocalcemia, immunodeficiency, and congenital heart disease, typically tetralogy of Fallot, truncus arteriosus, or ventricular septal defect with interrupted aortic arch type B (https://ghr.nlm.nih.gov/condition/22q112-deletion-syndrome#). PREP Pearls Williams syndrome results from a defect in elastin production, usually caused by a deletion on the long arm of chromosome 7 (7q11.23). Characteristics of Williams syndrome include supravalvar aortic and/or pulmonary stenosis as well as coronary artery ostial stenosis. Supravalvar aortic stenosis typically is progressive, while supravalvar pulmonary stenosis may improve with time. Coronary artery ostial stenosis puts affected patients at risk for sudden cardiac death.

A 7-day-old neonate is admitted to the hospital because of tachypnea and poor feeding that has worsened over the last few days. He has a heart rate of 170 beats/min and respiratory rate of 70 breaths/min. Blood pressure in the right arm is 80/35 mm Hg. Oxygen saturations are 98% in the right hand, 99% in the left hand, 85% in the right foot, and 87% in the left foot. He is in mild respiratory distress. He is not dysmorphic. His head, eyes, ears, nose and throat are unremarkable. He is tachypneic with mild retractions but clear lung fields. His precordium is hyperactive. He has a normal S1. S2 is narrowly split with a loud P2. There are no rubs, clicks or gallops. His abdomen is soft with a liver edge palpable 2 to 3 cm below the costal margin. He has palpable pulses. An echocardiogram is obtained (Video 1 and Video 2). What additional congenital heart defect is MOST likely to be found in this patient? A.aortopulmonary window B.bicuspid aortic valve C.coronary artery fistula D.partial anomalous pulmonary venous return

A.aortopulmonary window The neonate in this vignette has an interrupted aortic arch (IAA) type A; the echocardiographic and oxygen saturation data indicate that the interruption is distal to the left subclavian artery. Interrupted aortic arch type A is associated with aortopulmonary window. Interrupted aortic arch type A is the second most common form of IAA and results from the involution of both dorsal aorta, creating an interruption in the arch distal to the origin of the left subclavian artery (Figure and Video 1, Video 2, and Video 3). The patent ductus arteriosus provides the only source of blood flow to the lower body and its closure leads to shock and death. Just after birth, infants may be asymptomatic, although oxygen saturations will be decreased in the lower extremities, and pulses will be palpable while the ductus arteriosus is large. Oxygen saturations and pulses will diminish as the ductus arteriosus constricts. An aortopulmonary window is a communication between the ascending aorta and main pulmonary artery (Figure and Video 3 and Video 4). It is a rare form of congenital heart disease. Approximately half of the cases are associated with other congenital heart defects, with IAA being the most common association. There is continuous left-to-right shunting, and pulmonary artery pressures remain elevated because of the unrestrictive communication between the aorta and pulmonary artery. As the pulmonary vascular resistance drops, the shunt increases, and the resulting pulmonary overcirculation leads to congestive heart failure. Aside from signs of congestive heart failure, the physical examination will reveal a systolic (more common) or continuous murmur, widened pulse pressure, and bounding pulses. The neonate does not have a systolic click, and although this does not rule out a bicuspid aortic valve, bicuspid aortic valve is not commonly seen with IAA type A. Coronary artery fistula and partial anomalous pulmonary venous return are also uncommon associations with IAA type A. Ventricular septal defects can be seen with IAA type A, but not as commonly as with IAA type B (interruption between left common carotid artery and left subclavian artery). PREP Pearls Interrupted aortic arch type A is an interruption that occurs distal to the origin of the left subclavian artery. Aortopulmonary window is a communication between the aorta and main pulmonary artery. Although both are rare congenital heart defects, interrupted aortic arch type A and aortopulmonary window are often seen in association.

You are called to urgently evaluate a patient in the neonatal intensive care unit. She is a 2-day-old full-term newborn who was referred from a community hospital because of severe, persistent pulmonary hypertension of the newborn. The treatment plan is to place her on venovenous extracorporeal membrane oxygenation, and you are asked to perform an echocardiogram prior to cannulation. Her chest radiograph is shown (Figure). Upon your arrival, the patient is on a conventional ventilator with FIO2 of 1.0, respiratory rate of 45 breaths/min, and positive-end expiratory pressure of 8 cm H2O. The saturations are 68%, and the blood pressure is 49/22 mm Hg by umbilical arterial line. There are clear breath sounds bilaterally, no murmur is present, the liver is palpable 3 cm below the right costal margin, and no abdominal or cranial bruits were auscultated. You perform a rapid echocardiogram that demonstrates a dilated right ventricle with moderately depressed function, a dilated main pulmonary artery, and exclusively right-to-left shunting at the atrial level communication. A transthoracic echocardiogram is shown (Video). Of the following, the BEST next step in the management of this patient is to plan for A.cardiac surgery B.extracorporeal membrane oxygenation cannulation C.high-frequency oscillatory ventilation D.inhaled nitric oxide

A.cardiac surgery Total anomalous pulmonary venous connection (TAPVC) is a defect that involves the embryological development of the splanchnic venous plexus. The common splanchnic plexus initially drains through the paired common cardinal, umbilical, and vitelline veins with no direct connection to the heart. Later in development, the common pulmonary vein invaginates through the left atrial wall and establishes a connection to the heart, after which the primitive cardinal, umbilical, and vitelline veins become unnecessary for pulmonary venous drainage. Total anomalous pulmonary venous connection occurs when there is a failure of the left atrium to link to the pulmonary venous plexus, which results in a retention of the connections to the primitive cardinal and umbilicovitelline drainage systems. There are 4 types of TAPVC: Supracardiac TAPVC (type I) is the most common type (40%-50% of cases), with the pulmonary veins draining via a left vertical vein to the innominate vein. It is frequently unobstructed or mildly obstructed; but, it can become significantly obstructed where the vertical vein passes between the left mainstem bronchus and either the left pulmonary artery or the aorta. Cardiac TAPVC (type II) occurs in 18% to 31% of cases, with the most common site of connection to the coronary sinus. Obstruction is rare. Infracardiac TAPVC (type III) occurs in 13% to 24% of cases and involves a descending vertical vein penetrating the diaphragm and connecting with a vessel of the portal venous system. Obstruction is very common, especially at the point of intersection of the vertical vein and the portal venous system. Mixed TAPVC (type IV) occurs in 5% to 10% of cases and has 2 or more sites of anomalous venous return. The presentation of TAPVC is often similar to that of persistent pulmonary hypertension of the newborn, with severe hypoxemia, hemodynamic compromise, and right ventricular dysfunction. Cardiologists are often called upon to ensure that normal venous return is present prior to extracorporeal membrane oxygenation cannulation. The patient in this vignette has TAPVC type III (infracardiac). The first clue that persistent pulmonary hypertension of the newborn is not the primary problem in this patient, is the chest radiograph. Persistent pulmonary hypertension of the newborn will typically present with oligemia on chest radiography, whereas TAPVC will have a congested appearance. The second clue is the severely dilated and poorly functioning right ventricle with exclusive right-to-left atrial level shunting. Although this finding could be present in persistent pulmonary hypertension of the newborn, the inability to demonstrate pulmonary veins returning to the left atrium is diagnostic. In addition, the presence of blood flow in the hepatic circulation moving away from the heart raises concern for infradiaphragmatic TAPVC with obstruction. Obstructed total anomalous pulmonary venous connection is one of the few true cardiothoracic surgical emergencies; the patient in this vignette should be referred for immediate cardiac surgery to relieve the obstructed pulmonary veins. Extracorporeal membrane oxygenation cannulation may temporize this patient but would not change the clinical course. High-frequency oscillatory ventilation may improve saturations slightly, but would not definitively treat the underlying issue. Inhaled nitric oxide could potentially worsen this patient's condition by dilating pulmonary vessels proximal to the obstructed veins, causing further interstitial pulmonary edema and lower oxygen saturations. Prostaglandins are often started in the setting of cyanotic congenital heart disease, and there is some controversy regarding their use in TAPVC. They have vasodilatory effects on the pulmonary vasculature and may worsen congestive heart failure given the limited egress of blood from the pulmonary veins. However, the presence of a patent ductus arteriosus may provide a "pop-off" for a right ventricle that is severely hypertensive. The general practice in most institutions is to initiate prostaglandins and continue the infusion until a definitive repair is undertaken. PREP Pearls Persistent pulmonary hypertension of the newborn can often have a similar clinical presentation as total anomalous pulmonary venous connection. Obstructed total anomalous pulmonary venous connection is a surgical emergency; surgery should not be delayed while medical intervention is attempted.

A 10-year-old boy with bicuspid aortic valve and progressive aortic stenosis undergoes routine echocardiography that reveals moderate concentric left ventricular hypertrophy with normal systolic function. In this case, aortic stenosis resulting in left ventricular hypertrophy may be associated with an increase in myocyte mass. In this patient left ventricular wall stress is MOST likely: A.decreased B.increased C.independent of left ventricular thickness D.unchanged

A.decreased Compensatory left ventricular hypertrophy secondary to aortic stenosis reduces ventricular systolic wall stress and maintains the systolic function of the left ventricle. When the myocardium faces a hemodynamic burden in the form of aortic stenosis or systemic hypertension, it compensates by augmenting muscle mass to maintain cardiac output. This increased mass is caused by concentric hypertrophy (increase in the ratio of wall thickness/chamber dimension) secondary to myocyte hypertrophy. According to the law of LaPlace, the ventricular wall stress is calculated based on the following formula: (pressure × radius)/(2 × wall thickness) An increase in wall thickness therefore results in a reduction in systolic wall stress. The left ventricular hypertrophy resulting from aortic stenosis reduces the elevated wall stress and helps maintain a normal ejection fraction, even when needing to generate higher systolic pressure. Left ventricular dilation and myocardial lengthening will result in increases in left ventricular systolic wall stress. Velocity of circumferential fiber shortening normalized for heart rate is inversely related to end-systolic wall stress in a linear fashion. Accurate quantification of ventricular wall stress can be performed by echocardiography. This index is a sensitive measure of contractile state that is independent of preload, normalized for heart rate, and incorporates afterload. Ventricular wall stress increases with increase in afterload and ventricular dilation, while the compensatory hypertrophy reduces the ventricular wall stress. PREP Pearls According to the law of LaPlace, the ventricular wall stress is calculated as: (pressure × radius)/(2 × wall thickness). When the myocardium faces increased afterload, it compensates by augmenting muscle mass and ventricular hypertrophy to reduce wall stress and maintain cardiac output.

A 15-year-old girl with no significant medical history presents to the clinic for evaluation of syncopal events. She is a soccer player and reports 2 episodes of sudden-onset syncope during exertion. She describes no prodrome before the syncopal events, and she denies chest pain, exercise intolerance, or lower-extremity edema. Family history is negative for sudden cardiac death. On physical examination, she is well appearing. She has normal cardiac examination findings. Her electrocardiogram is shown in Figure 1. A loss-of-function mutation in a gene coding for which of the following ion channels would be the MOST likely cause of this clinical scenario? A.delayed rectifier potassium channel B.hyperpolarization-activated cyclic nucleotide gated channel C.L-type calcium channel D.voltage-gated sodium channel

A.delayed rectifier potassium channel The patient described in the vignette has long QT syndrome, as evidenced by her episodes of syncope without prodrome during activity and prolonged QTc interval on electrocardiography (ECG). A loss-of-function mutation in a delayed rectifier potassium channel (IKs, IKr) is the only option in the vignette that would cause a prolonged QT interval. In fact, of the more than 15 genes known to cause long QT syndrome, 4 of them (including the 2 most common ones: LQT1 and LQT2) are caused by a loss-of-function mutation in a delayed rectifier potassium channel. Two of the other genes causing long QT syndrome lead to loss of function of the inward rectifier potassium channel (IK1). Together, potassium efflux through these channels during phases 2 and 3 of the action potential lead to repolarization; therefore, loss of function in these channels would delay repolarization, causing a prolonged action potential and QTc interval on ECG. Ion channels are pore-containing transmembrane proteins that allow for movement of charged particles (typically cations) across the lipophilic cell membrane. They typically consist of one or more pore-forming subunits (ie, α subunit) and several secondary subunits (ie, β and γ), which typically alter the expression of the gene, augment channel activity, or assist in gating (opening and closing of the ion channels). Gating can be triggered by voltage, attachments of ligands, or stretch. The opening and closing of these channels are what creates the action potential (Figure 2). Phase 4 is known as the resting potential. Most myocytes will remain in this phase until acted on by the depolarization of an adjacent cell via gap junctions. The resting potential is approximately −90 mV for most myocytes and is maintained by adenosine triphosphate using ion pumps (eg, Na+-K+ exchanger, Na+-Ca2+ exchanger). The exception to this rule, of course, is pacemaker cells, such as the sinus node and atrioventricular node, which have very different action potentials compared with typical myocytes. In these cells, many ion channels contribute to a gradual phase 4 depolarization, such as IKr and L- and T-type calcium channels. However, these cells additionally uniquely express hyperpolarization-activated cyclic nucleotide-gated (HCN) channels. These channels allow influx of both Na+ and K+ (the so-called funny current [If]) and are activated by the hyperpolarization seen at the start of phase 4, leading to gradual depolarization and thus automaticity. Phase 4 is followed by phase 0 or rapid depolarization. This phase occurs when voltage-dependent conformational change takes place in the sodium channel, allowing Na+ to rapidly enter the cell. Phase 1 is a short phase of transient rapid partial repolarization and is largely due to potassium efflux via the transient outward current (Ito). Most sodium channels also close at this time. Phase 2 is the plateau phase. During this phase, there is a balance of outward (K+, via delayed rectifier potassium channels) repolarizing and inward (Ca2+, via the L-type calcium channels) depolarizing currents. Phase 3, then, is rapid repolarization, where the L-type calcium channels close while the delayed rectifier potassium channels remain open, causing the cell to repolarize. Thus, a loss-of-function mutation in the delayed rectifier potassium channel would hinder repolarization, leading to a prolonged QTc interval (eg, LQT1 and LQT2). A gain-of-function mutation would cause a shortened QTc interval (SQT1 and SQT2). A loss-of-function mutation in the hyperpolarization-activated cyclic nucleotide-gated channels would be expected to cause sinus bradycardia and have no effect on the QTc interval. A loss-of-function mutation in the L-type calcium channels would be expected to shorten the QTc interval (by allowing the potassium current to be unopposed during phase 2, eg, SQT5 and SQT6). A gain-of-function mutation in the L-type calcium channel, conversely, leads to a prolonged QTc interval, as would be expected (LQT8, Timothy syndrome). Finally, a loss-of-function mutation in the sodium channel would primarily be expected to slow conduction velocity (which is related to the upstroke in phase 0 of the action potential), which clinically can lead to Brugada syndrome, sick sinus syndrome, and conduction disease. However, a small number of sodium channels remain open throughout the repolarization phase of the action potential (the so-called late sodium current, a depolarizing current). Thus, a loss-of-function mutation in the sodium channel might also be expected to shorten the QTc interval, and a gain-of-function mutation prolongs the QTc interval (LQT3) (Figure 3). PREP Pearls Sodium channel opening leads to rapid depolarization, and their inhibition or loss of function can lead to conduction slowing and Brugada syndrome. Potassium channel opening typically leads to repolarization, and their inhibition or loss of function can lead to increased refractoriness and long QT syndrome. L-type calcium channel opening leads to an inward current opposing repolarization (as well as regulating excitation-contraction coupling), and augmentation of its function (gain of function) leads to long QT syndrome (Timothy syndrome).

A 12-year-old girl was diagnosed with a heart murmur during a routine sports physical examination. She has no significant cardiac, medical or surgical history. Her pediatrician had diagnosed her with a normal heart murmur at 3 years of age. She has been participating in soccer and volleyball and never had any cardiorespiratory symptoms until the past year, when she noticed shortness of breath and occasional midsternal chest pain during soccer practice. She also notes that she tires more easily now, but reports no exertional palpitations, dizziness, or syncope. On physical examination, she is in no distress. Her heart rate is 80 beats/min, her blood pressure is 105/75 mmHg, and her respiratory rate is 18 breaths/min. There is a grade 3/6 low-frequency continuous murmur at the mid-left sternal border. The rest of the physical examination findings were normal. Echocardiography (not provided) showed a dilated left main and left anterior descending coronary artery and evidence of a fistulous communication to the apex of the right ventricle. The patient underwent transcatheter closure of a fistulous communication to the right ventricle with an 8-mm Amplatzer vascular plug 4 (Figure 1 and Figure 2). After closure via device, the patient was administered 20 units/kg/h of intravenous heparin for a total of 24 hours and then transitioned to 81 mg of aspirin once daily and discharged from the hospital. Two days after discharge, she was brought to the emergency department because of severe chest pain radiating to the neck and left arm, shortness of breath at rest, and diaphoresis. Laboratory studies showed a cardiac troponin I level of 30 ng/mL (30 µg/L). She was emergently taken to the catheterization laboratory, where she underwent a selective left anterior descending artery angiogram (Figure 3). Of the following, the BEST management strategy to prevent such a complication is A.enhanced anticoagulation regimen after device closure of coronary artery fistula B.longer inpatient monitoring with daily troponin level measurements, electrocardiography, and echocardiography C.nitroglycerin treatment for 24 hours after device closure of coronary artery fistula D.oral heart failure regimen beginning immediately after device closure of coronary artery fistula

A.enhanced anticoagulation regimen after device closure of coronary artery fistula Coronary artery fistulas (CAFs) are broadly classified into 2 types. In the proximal type, a fistulous communication arises from a proximal main epicardial coronary artery and terminates in a cardiac chamber. The feeding epicardial coronary artery is dilated up to the origin of the fistulous communication, and the remainder of the epicardial artery and its branches are normal in size. In the distal type, the fistulous communication is at the terminal end of the epicardial coronary artery, leading to dilation of the entire length of the coronary artery. Normal coronary artery branches arise from the dilated/aneurysmal epicardial coronary artery. The patient in the vignette has a large distal-type left anterior descending (LAD) CAF communication to the right ventricle (Figure 1). The entire LAD artery is dilated and supplies normal branches to the left ventricle before communicating to the right ventricle at its apex. Surgical patch closure or ligation or, if amenable, transcatheter device closure of such a large CAF is performed at the site of drainage into the right ventricle. After closure, patients with a distal CAF are at increased risk of proximal clot propagation and thromboembolism because of relative stasis of blood in the remaining dilated coronary artery. This patient did well for the first 24 hours when she received adequate anticoagulation therapy with intravenous heparin. However, after 24 hours, the heparin was discontinued and she was transitioned to an oral antiplatelet agent (aspirin) only. Aspirin alone is insufficient to prevent clot formation in large dilated coronaries. This led to thrombosis of the distal LAD artery in this patient owing to proximal clot propagation (Figure 3), causing myocardial ischemia and troponin elevation. Patients with a large distal CAF who also have a dilated epicardial artery should receive anticoagulation therapy for 6 to 12 months after closure to prevent proximal clot propagation and thrombosis of small coronary artery branches arising from the dilated conduit artery. This therapy usually entails either oral warfarin or subcutaneous enoxaparin in addition to aspirin. Enhanced anticoagulation with a transition to warfarin and aspirin is recommended to prevent distal vessel stasis and clot propagation. If intravenous heparin needs to be discontinued before the therapeutic range of the international normalized ratio (2-3) is reached, subcutaneous enoxaparin injections are typically used as a bridge. Some centers use clopidogrel in addition to aspirin for enhanced anticoagulation. Although aspirin may be continued indefinitely, warfarin is stopped after 6 to 12 months once the conduit epicardial artery has remodeled and decreased to a normal size. Nitroglycerin is used to treat angina because it improves coronary blood flow by reversing and inhibiting coronary vasospasm. Nitroglycerin treatment for 24 hours after device closure will not help with prevention of thromboembolic complications as seen in this patient. Longer inpatient monitoring with daily troponins, electrocardiography, and echocardiography will not alter the clinical picture except for possible early recognition of the distal LAD artery occlusion (Figure 3). Patients taking aspirin and therapeutic doses of warfarin do well and are unlikely to develop myocardial infarction and subsequent heart failure. Patients will not benefit from receiving oral heart failure medications immediately after successful device closure of a distal LAD artery fistula to the right ventricle, as long as an adequate anticoagulation regimen is instituted. PREP Pearls Symptomatic large distal-type congenital coronary artery fistulae need to be treated to prevent the risk of angina, myocardial infarction, endocarditis, arrhythmias, aneurysmal dilation, and rupture. Proximal clot propagation following device closure of large distal-type coronary artery fistula can lead to myocardial infarction. Enhanced anticoagulation (dual therapy) is recommended to prevent distal vessel stasis and clot propagation.

A 27-year-old woman has a history of unrepaired large ventricular septal defect complicated by Eisenmenger physiology. Over the last 6 months, she reports increased dyspnea on exertion. She has been taking bosentan 125 mg twice daily for the last 2 years and is compliant. She has had no recent syncope, chest pain, or palpitations. She has a resting heart rate of 90 beats/min, oxygen saturation of 82%, and blood pressure of 115/72 mm Hg. She has a right ventricular heave but no gallops or murmurs. Her lungs are clear. Her liver is not enlarged. She appears cyanotic and has significant digital clubbing throughout. There is no pitting edema. Laboratory data are shown: Laboratory Test Result Hemoglobin 12.0 g/dL (120 g/L) Platelet count 171 × 103/µL (171 × 109/L ) Ferritin (reference range, 9-120 ng/mL [20-270 pmol/L]) 20 ng/mL (45 pmol/L) Creatinine 0.6 mg/dL (46 µmol/L) Uric acid (reference range, 4-8 mg/dL [238-476 µmol/L]) 5.6 mg/dL (333 µmol/L) INR 1.01 Aspartate aminotransferase 22 U/L Alanine aminotransferase 16 U/L Of the following, the BEST next step in her medical management is the addition of A.ferrous sulfate B.furosemide C.sildenafil D.warfarin

A.ferrous sulfate The patient in this vignette has Eisenmenger syndrome. Her resting oxygen saturation of 82% is not atypical for this disease due to right-to-left shunting at the level of the ventricular septal defect. The hemoglobin level should always be viewed in the context of the baseline oxygen saturation. Given her degree of baseline desaturation, her hemoglobin level is expected to be elevated in a range that is supra-normal. Thus, for her, a hemoglobin level of 12 g/dL (120 g/L) is quite anemic and her ferritin is in the low-normal range suggesting iron deficiency. The best therapy at this point is the addition of oral ferrous sulfate, which is a class I treatment recommendation in Adult Congenital Heart Disease (ACHD) management of patients with Eisenmenger physiology with evidence of iron deficiency. Patients with Eisenmenger physiology and anemia tend to have more pronounced symptoms, such as dyspnea on exertion, and some studies suggest worse outcomes. Clinicians should be screening for anemia on a regular (at least annual) basis. For women, taking a menstrual history is important to assess for menorrhagia as a possible cause of anemia. Pulmonary arterial hypertension therapy including bosentan is reasonable in patients with Eisenmenger physiology. It is a class IIa recommendation by the ACHD guidelines and has been associated with improved quality of life and 6-minute walk distances. A mortality benefit has not been shown. This patient is taking bosentan, an endothelin-receptor antagonist, so the addition of a second agent, such as sildenafil, from a different class might be considered, although the anemia should be addressed first. Warfarin is controversial in these patients given the risk of pulmonary hemorrhage; thus, most Eisenmenger patients are not prescribed warfarin unless there is a specific indication. There are no signs or symptoms of heart failure that would be improved by diuretics such as furosemide in this patient. PREP Pearls Anemia in patients with baseline hypoxia is relative and should be judged in the context of baseline oxygen saturation. Treatment of iron deficiency is a class I recommendation in Adult Congenital Heart Disease guidelines for management of Eisenmenger patients.

You are called to the cardiac intensive care unit to evaluate the rhythm of a 16-year-old adolescent boy who is postoperative day 3 after a mechanical mitral valve placement. His surgery was complicated by complete heart block, and temporary atrial and ventricular wires have been placed. His rhythm is shown in Figure 1. Of the following pacemaker changes, which is the next BEST step to improve the patient's rhythm? A.increase the atrial sensing threshold (decrease the sensitivity) B.increase the postventricular atrial refractory period C.increase the upper tracking rate D.increase the ventricular pacing threshold

A.increase the atrial sensing threshold (decrease the sensitivity) The rhythm strip shown in Figure 2 reveals a patient whose pacemaker is set DDD with atrial oversensing. Therefore, assuming the sensed P waves are large enough, the best step to avoid oversensing while maintaining atrioventricular synchrony is increasing the atrial sensing threshold in an attempt to filter out the noise that is being oversensed. When troubleshooting a pacemaker, it is imperative to know the pacing mode. Ideally, this will be known. However, when not known, the electrocardiogram should give clues. In this case, there is clearly atrial and ventricular pacing, so the pacemaker is not in a single-chamber mode (eg, VVI, AAI). Likewise, there is evidence of atrial sensing and inhibition of atrial pacing, so this is not an asynchronous mode (eg, DOO). Finally, there is evidence of atrial tracking (where P waves are clearly seen, there is a paced ventricular event with a stable PR interval, and there is evidence of variation in the paced ventricular rate). This patient is almost certainly in DDD or DDDR mode. Next, based on the mode of pacing, the apparent abnormality in the rhythm must be explained (in this case, the irregularity of the rhythm). One might initially hypothesize that the patient's pacemaker is tracking atrial fibrillation, atrial tachycardia, or premature atrial contractions. Indeed, to completely rule this out, it would be wise to interrogate the pacemaker (in the case of a permanent pacemaker) or perform an atrial electrogram. However, based on the strip, atrial fibrillation is unlikely because normal-appearing and identical P waves are seen at times. A salvo of tracked atrial ectopic tachycardia could account for the 9th and 10th QRS complexes on the strip with the P wave buried in the preceding T wave; however, one would expect to see a P wave before the 6th and 11th QRS complex if an atrial tachycardia were being tracked. Likewise, the PR interval on the 4th complex is too short to be tracked, and no additional P wave is seen ahead of the P wave here, so tracking of atrial tachycardia is unlikely. Simple premature atrial contractions are not the answer because the early beats are not isolated events. Pacemaker-mediated tachycardia is a possibility (tracking a retrograde P wave). However, pacemaker-mediated tachycardia is usually quite regular (as the "circuit" consists of ventriculoatrial conduction, which is typically fairly constant at a given rate, and the sensed atrioventricular delay, also constant). Finally, one might hypothesize upper rate behavior, which causes irregularity of DDD paced rhythms at high sinus rates. However, typically upper rate behavior is regularly irregular with grouped paced beats at the same interval (the upper tracking interval) before a blocked beat ("pacemaker Wenckebach"), which is not the case in this strip. The most likely explanation, then, is atrial oversensing, leading to ventricular "overpacing." This could be confirmed by interrogation of the pacemaker or evaluating the sensing pattern on a temporary pacemaker—in both circumstances looking for evidence of sensing without a corresponding atrial event on electrocardiography. If the sensed true P wave is large enough, increasing the sensing threshold (ie, making the lead less sensitive), will allow the pacemaker to filter out the noise, while still sensing the true P wave. Increasing the postventricular atrial refractory period would be potentially helpful if this were thought to be tracked premature atrial contractions or pacemaker-mediated tachycardia; however, as noted above, these diagnoses are unlikely. Increasing the upper tracking rate would be appropriate if this were upper rate limit activity, which is also unlikely as noted above. Increasing the ventricular threshold would be helpful if there were loss of ventricular capture; however, there is no evidence of such loss in the strip. PREP Pearls With a patient in DDD pacing mode, an irregular ventricular paced rhythm (QRS) should raise suspicion of an atrial tachycardia/atrial fibrillation, tracked premature atrial contractions, upper rate behavior, or atrial oversensing. Close inspection of the underlying atrial rhythm is needed to rule out atrial tachycardia or premature atrial contraction. Upper rate behavior appears as "grouped beats" at a regular interval, followed by a pause ("pacemaker Wenckebach").

You are caring for a 3.5-kg neonate with a diagnosis of dextro-transposition of the great arteries who has returned from the operating room with a closed chest after an arterial switch operation. His initial vital signs are as follows: pulse, 145 beats/min; respiratory rate, 28 breaths/min (on mechanical ventilation); and blood pressure, 60/40 mm Hg. His laboratory results are as follows: Laboratory Test Result Lactic acid 2 mg/dL (0.22 mmol/L) Hemoglobin 14 g/dL (140 g/L) Platelet count (status after 15 mL/kg platelet transfusion after cardiopulmonary bypass) 275 ×103/µL (275 ×109/L) Baseline coagulation: Prothrombin time 36 s International normalized ratio 2.1 Fibrinogen 350 mg/dL (10.29 µmol/L) The neonate's chest tube output was 28 mL and sanguineous the first hour in the intensive care unit, 25 mL and sanguineous the second hour, and 5 mL and sanguineous the third hour. His vital signs are now as follows: pulse, 193 beats/min in sinus rhythm; respiratory rate, 28 breaths/min (on mechanical ventilation); and blood pressure, 50/41 mm Hg. His lactic acid level is now 5.5 mg/dL (0.61 mmol/L), and his hemoglobin level is 8.2 g/dL (82 g/L). Of the following, the BEST next step in the management of this patient is A.notify surgeon for chest exploration B.order an echocardiogram C.start epinephrine infusion D.transfuse fresh frozen plasma

A.notify surgeon for chest exploration Bleeding after cardiac surgery in neonates and infants is associated with increased mechanical ventilation and length of stay in the intensive care unit. Significant early postoperative bleeding has been independently associated with increased surgical mortality. Thus, early recognition of significant postoperative bleeding and correction of the underlying mechanism of bleeding is paramount in caring for this patient population. Postoperative bleeding may be placed into 2 broad etiological categories: coagulopathic and surgical. In the pediatric patient, particularly the neonate and infant (younger than 1 year), the pathophysiology of coagulopathic bleeding after cardiac surgery can be multifold. Neonates, infants, and toddlers undergoing cardiopulmonary bypass often require blood priming of the cardiopulmonary bypass circuit, resulting in a dilutional coagulopathy. Findings of studies in this age group support that coagulation factors are decreased by as much as 56% after cardiopulmonary bypass. Normal coagulation is further encumbered by the decreased activity of platelets after cardiopulmonary bypass. The pathophysiology is complex and multifactorial, and it includes a combination of factors that lead to decreased platelet function related to activation and aggregation of platelets, after contact with the foreign material of the cardiopulmonary bypass circuit, systemic inflammation, adverse effects of heparin on platelets, and hypothermia. Use of fresh frozen plasma can help replace the diluted coagulation factors and platelet transfusion can replace dysfunctional platelets with functional platelets. Studies supporting coagulopathy would include abnormal coagulation studies (partial prothrombin time, prothrombin time, international normalized ratio, fibrinogen, heparin assay, activated clotting time). More recent testing such as thromboelastography with platelet mapping can determine whether or not platelet function is normal. In the setting of normal coagulation studies and normal platelet activity, surgical or anatomical lesions are likely the culprit behind significant bleeding. Postoperative hemorrhage after pediatric cardiac surgery has not been extensively studied. However, it is well accepted that bleeding approaching 10 mL/kg/h in the setting of a normal coagulation profile is likely surgical in nature and needs immediate surgical attention. Chest tubes are generally in place after pediatric cardiac surgery and provide an egress for blood from the thoracic cavity. These tubes can become blocked with clots, resulting in a hemothorax that in turn leads to tamponade physiology. Clotting or malfunction of chest tubes should be suspected and ruled out for any patient who has had moderate to severe bleeding that stops or decreases abruptly but is associated with clinical deterioration, as evidenced by decreased cardiac output and worsening anemia. If active drainage of chest tubes cannot be restored, surgical intervention is required. The patient in the vignette arrives in the intensive care unit with bloody chest tube output of 8 mL/kg the first hour, about 7 mL/kg the second hour, and then close to 1.5 mL/kg the third hour. In the setting of a nearly normal coagulation profile, it is unlikely that bleeding of this magnitude is secondary to coagulopathy. Cardiopulmonary bypass-associated platelet dysfunction may be a contributing factor but, again, is likely not the cause of bleeding of this magnitude given that the patient received a platelet transfusion after undergoing cardiopulmonary bypass. Consequently, surgical bleeding should be suspected. This patient has had a significant drop in hemoglobin from 14 g/dL to 8.2 g/dL in the setting of acutely decreased chest tube output, narrowing pulse pressure, tachycardia, and rising lactic acid level, arousing concern regarding tamponade. Ongoing bleeding in the setting of chest tube malfunction should be suspected. Consequently, given the options, the best next step for this patient is notification of the surgeon for immediate chest exploration. Although an echocardiogram would potentially confirm the diagnosis, time is of the essence in this patient in light of the evidence of impending hemodynamic collapse (narrowing pulse pressure, tachycardia, and rising lactate level). An echocardiogram would likely only delay the definitive surgical intervention and further increase the patient's morbidity and mortality risk. Fresh frozen plasma would likely not alter the course of bleeding in the setting of a relatively normal coagulation profile. Finally, although the patient's blood pressure has dropped significantly, starting epinephrine in the setting of tamponade physiology would exacerbate tachycardia further, decreasing diastolic filling time and further lowering cardiac output. PREP Pearls Pediatric patients, particularly infants and neonates, are at risk of experiencing dilutional coagulopathy after undergoing cardiopulmonary bypass. Postoperative bleeding after cardiac surgery increases morbidity and mortality. Prompt recognition of tamponade physiology and appropriate intervention is key in postoperative treatment.

A 34-year-old man with a history of tricuspid atresia and a classic atriopulmonary Fontan operation has been seen sporadically for follow-up. Today he is seen for a new concern of abdominal swelling and fullness. He is compliant with his medications, which include aspirin (81 mg/day), lisinopril (5 mg/day), and a daily multivitamin. He reports no alcohol or tobacco use. He has a blood pressure of 110/65 mm Hg, heart rate of 60 beats/min, and a resting oxygen saturation of 92%. He has a well-healed sternotomy scar. He has a single S2 and no murmurs. His abdomen is mildly distended, although there is no fluid wave. His liver is palpable 2 cm below his right costal margin. His spleen tip is not palpable. He has chronic venous stasis changes with trace pitting edema in his lower extremities. A routine laboratory evaluation is ordered. Of the following, the laboratory abnormality that is MOST concerning and that should prompt an urgent evaluation is A.α-fetoprotein of 170 ng/mL (reference, < 9 ng/mL) B.γ-glutamyltranspeptidase of 110 U/L(reference, 11-55 U/L) C.INR (international normalized ratio) of 1.5 (reference, 0.9-1.1) D.total bilirubin of 2.5 mg/dL (reference, < 1.2 mg/dL)

A.α-fetoprotein of 170 ng/mL (reference, < 9 ng/mL) Long-term survival following single-ventricle palliation has improved significantly, with many patients surviving into adulthood. Long-term follow-up data suggest that patients who have undergone a classic atriopulmonary Fontan operation have a 25-year survival rate of more than 75%. All Fontan patients face long-term, multisystem complications as a result of their single-ventricle anatomy and systemic venous hypertension, which is the direct result of the Fontan operation. One of the principal organ systems, aside from the heart, that may be affected in Fontan patients is the liver. Liver "disease" is ubiquitous in this population and differs from other types of liver pathology; it has been termed Fontan-associated liver disease (FALD). Expert consensus suggests different screening modalities for the extent of FALD (Suggested Reading 1). One of the most serious but rare complications of FALD is liver cancer, most commonly hepatocellular carcinoma (HCC). Case reports and case series indicate that HCC may occur in the absence of traditional risk factors (eg, hepatitis C). Given the inherent risks associated with FALD, universal screening for hepatitis and vaccination for hepatitis A and B should be considered. About 80% of HCCs secrete α-fetoprotein. An α-fetoprotein level of 170 ng/mL (170 µg/L), which is listed in the response choices, is quite elevated and should prompt urgent evaluation with liver imaging, typically magnetic resonance imaging or computed tomography, to screen for HCC. Early detection is critical for more effective treatment and improved survival. Fontan-associated liver disease is commonly associated with mild elevation in liver biochemical markers. Of these, elevation in γ-glutamyltranspeptidase is one of the most common, seen in 40% to 60% of outpatient adults with a Fontan operation, and is not inherently a cause for concern. Elevated total serum bilirubin occurs in 25% to 40% of patients and is primarily unconjugated. Levels typically do not exceed 3 mg/dL (51.3 µmol/L) unless there is other underlying liver pathology. The INR (international normalized ratio) is a useful screen for synthetic liver function. It may be mildly elevated in patients with FALD. PREP Pearls Liver "disease" following the Fontan operation is ubiquitous; the spectrum of liver problems in this population is often referred to as Fontan-associated liver disease. Liver cancer, especially hepatocellular carcinoma, has been reported in adult survivors of single-ventricle palliation. α-Fetoprotein is commonly secreted by hepatocellular carcinoma, and if significantly elevated should prompt urgent evaluation.

A 15-year-old previously healthy adolescent boy has episodes of nonexertional chest pain. He is otherwise asymptomatic with no palpitations, dyspnea, or syncope. He is the starting quarterback for his high school varsity football team. He wishes to continue to play football. He has a negative family history for cardiac conditions and sudden death. His physical examination findings are normal with only reproducible chest pain on palpation. Electrocardiography is performed (Figure ). He has a normal echocardiogram. Of the following, the BEST next step in evaluating him to determine if he can return to play is A.cardiac computed tomography to evaluate his coronary arteries B.exercise stress test C.genetic testing to evaluate for long QT syndrome D.no further evaluation is needed, he is cleared to return to sports

B.exercise stress test The electrocardiogram from the patient in this vignette demonstrates ventricular pre-excitation with a short PR, wide QRS, and the presence of a delta wave. Patients with pre-excitation are at risk for developing 2 distinct types of arrhythmias, supraventricular tachycardia and atrial fibrillation (AF). The risk of sudden death in patients with pre-excitation is secondary to having AF that conducts rapidly through the accessory pathway, which then triggers ventricular fibrillation. The conduction properties of the pathway determine if AF can lead to ventricular fibrillation. Pathways with very rapid antegrade conduction properties place the individual at risk. The pathway is considered high risk if in AF the shortest pre-excited R-R interval is less than 250 milliseconds. Risk stratification for patients with pre-excitation can be performed noninvasively and invasively. Noninvasive risk stratification can be performed with an exercise stress test (correct response). If sudden loss of pre-excitation in one beat occurs during an exercise stress test, it correlates with a low-risk accessory pathway. This sudden loss of pre-excitation demonstrates that the accessory pathway does not conduct well antegrade and thus cannot conduct AF rapidly to lead to ventricular fibrillation. Loss of pre-excitation cannot be gradual to be considered low risk. If there is a gradual loss of pre-excitation this could be secondary to improved conduction through the AV node secondary to increased catecholamines. Gradual loss of pre-excitation has more to do with the AV node response to exercise and does not mean that the accessory pathway cannot conduct rapidly during AF. If there is no sudden loss of pre-excitation in one beat during an exercise stress test, then invasive electrophysiology studies can be performed to further risk stratify the patient. This can be performed with either transesophageal or transvenous catheters in the electrophysiology laboratory. During the study, AF is triggered and the shortest pre-excited R-R interval is measured. The patient's chest pain is not characteristic of ischemic chest pain that would be seen with anomalous coronary arteries. The fact that the chest pain is reproducible on examination makes the chest pain more consistent with costochondritis and not ischemia. Negative delta waves seen in pre-excitation can at times give the appearance of a q wave. This can be seen in the inferior leads (II, III, and aVF in the included electrocardiogram). Although 10% to 20% of patients with pre-excitation can have associated congenital heart disease, anomalous coronaries are not a common finding. Therefore, cardiac computed tomography to evaluate coronary anatomy is not indicated. The classically associated congenital lesions with pre-excitation are Ebstein anomaly, congenitally corrected transposition of the great arteries, and hypertrophic cardiomyopathy. Although the QTc can be prolonged when a patient has underlying pre-excitation, one cannot reliably measure a QTc when depolarization is abnormal. This is the case when there is underlying pre-excitation, bundle branch block, or ventricular paced rhythm. The usual presentation of patients with long QT syndrome tends to be pre-syncope, syncope, palpitations, or family history of sudden death or long QT syndrome. Because this patient does not have any of the above symptoms and no family history of sudden death, genetic testing for long QT syndrome is not indicated. Given the risk of sudden death in patients with pre-excitation it would not be prudent to clear this patient to return to sports prior to performing a risk stratification of his accessory pathway. PREP Pearls Classically associated heart defects seen in patient with pre-excitation are Ebstein anomaly, congenitally corrected transposition of the great arteries, and hypertrophic cardiomyopathy. Risk stratification should be performed in patients with pre-excitation prior to clearance for competitive sports. Sudden death in patients with pre-excitation occurs from atrial fibrillation with rapid conduction to the ventricles via the accessory pathway leading to ventricular fibrillation. Sudden loss of pre-excitation in one beat demonstrates that the accessory pathway does not conduct well antegrade and thus cannot conduct atrial fibrillation rapidly to lead to ventricular fibrillation.

A 10-year-old boy with tricuspid atresia and ventricular septal defect has undergone a fenestrated Fontan procedure with an extracardiac conduit at 4 years and moved to Denver, CO, with his family 2 weeks ago. He presents to the emergency department with dyspnea. He is noted to have a heart rate of 90 beats/min, a respiratory rate of 20 breaths/min, a blood pressure of 98/52 mm Hg, and a saturation of 85% on room air. He appears comfortable, and has mild jugular venous distension, clear lung fields, mild hepatomegaly, and no peripheral edema. The remainder of his examination is unremarkable. His parents are concerned about the lower saturation, as his typical saturation is around 93%. They understand that the change in altitude will require some adjustment, but want to know what to expect. Of the following, the physiologic parameter MOST likely to increase in the boy in the vignette in the next few weeks is A. heart rate B. hematocrit C. oxygen saturation D. partial pressure of oxygen in the blood E. respiratory rate

B. hematocrit Travel to a high altitude leads to changes in cardiac and respiratory status. Over time, the body will acclimatize to the change in altitude, unless the individual develops high altitude illnesses. The response to changes in altitude in an individual with single ventricle physiology is similar to that seen in individuals with normal hearts, however they may have more complications. During the first few weeks at high altitude, subacute adaptive changes occur. Erythropoietin levels increase in response to hypoxemia, leading to increased hematocrit. The steepest increase occurs within the first week, with continued increase until the body is fully adapted to the high altitude by around a month. Production of 2,3-disphosphoglycerate also increases and causes a rightward shift in the hemoglobin dissociation curve. This decreases hemoglobin's affinity to oxygen, lowering the oxygen saturation at a given partial pressure of oxygen (Pao2) and increasing oxygen delivery in the tissues (Figure ). As oxygen delivery improves, cardiac output normalizes and heart rate decreases, though it will remain elevated compared to baseline. Respiratory rate also decreases with adaptation. Longer periods at high altitude result in chronic adaptive changes that vary among ethnic groups. Infants born at high altitude will have delayed transition from fetal physiology. The normal cardiopulmonary adaptations to altitude may be limited in individuals with underlying cardiopulmonary disease. The effects of high altitude are mainly related to the decreased Pao2 seen with the decreased barometric pressure at high altitude. Most of the physiologic responses are designed to improve oxygen delivery to the tissues despite the decrease in Pao2. Acute changes are secondary to increased cardiac output and ventilation. The heart rate and respiratory rate both increase. Stroke volume and systolic ventricular function have not been found to change in response to altitude changes, so increases in cardiac output are mostly dependent on higher heart rates. This may be limited in patients with single ventricle physiology who often have chronotropic incompetence. The response of blood pressure is highly variable among individuals. Typically pulmonary vasoconstriction occurs in response to hypoxia, leading to increased pulmonary artery pressures. This response may have added consequences in patients with single ventricle physiology because of increases in Fontan pressures. Increased pulmonary vascular resistance is also more likely to have a negative effect on cardiac output in patients with single ventricle physiology. Oxygen saturation will be lower in all individuals because of the low Pao2 and varies depending on the altitude (approximately 90%-95% at 1,500 m and 85%-90% at 3,500 m), but will be lower in most patients with single ventricle physiology who have a lower starting point. Increased pulmonary vascular resistance in a fenestrated Fontan is likely to lead to more right to left shunting, and thus a lower saturation. Individuals in the acute phase of altitude adaptation will typically feel breathless and have a lower exercise tolerance. Nausea, headache, and lightheadedness may indicate the development of acute mountain sickness. The speed of ascent, final altitude, and genetic adaptations all influence the severity of symptoms and the development of acute mountain illness or more severe altitude illnesses. Patients with single ventricle physiology are at higher risk for the development of altitude illnesses. PREP Pearls Acute changes in cardiopulmonary function occur within the first few days at high altitude as a response to low partial pressure of oxygen from low barometric pressure. Increases in hematocrit and shifts in the hemoglobin dissociation curve occur within the first few weeks to improve oxygen delivery to the tissues.

A 10-year-old boy with a history of congenital bicuspid aortic valve and severe aortic insufficiency underwent a Ross procedure. On postoperative day 3, he was noted to be tachycardic, and the following electrocardiogram (ECG) was obtained (Figure ). In order to determine the mechanism of the tachycardia, you review the telemetry. Which feature, if seen on telemetry review, would MOST STRONGLY suggest atrial tachycardia as a mechanism as opposed to pathway mediated tachycardia (AV reciprocating tachycardia)? A.intermittent bundle branch block associated with longer cycle length B.episodes of second degree AV block C.spontaneous termination ending in an atrial event D.sudden onset and termination

B.episodes of second degree AV block The electrocardiogram (ECG) in the vignette shows supraventricular tachycardia (SVT) with a stable RR interval, 1:1 atrioventricular (AV) relationship, and a long PR interval. The differential diagnosis includes atrial tachycardia (ectopic, microreentrant, or macroreentrant), AV reciprocating tachycardia (AVRT), or atypical AV nodal reentrant tachycardia (AVNRT). Of the features mentioned in the vignette, frequent second degree AV block would most strongly support a diagnosis of atrial tachycardia over AVRT. Second degree AV block frequently occurs in patients with atrial tachycardia as the atrial cycle length is often shorter than that of the refractory period of the AV node. The AV node itself is not part of the circuit, therefore the tachycardia can continue in the setting of second degree AV block. In contrast, any second degree AV block that occurred in the setting of AVRT would result in termination of the tachycardia, as the AV node is an integral part of the circuit. Second degree AV block can, on occasion, occur in AVNRT without terminating the tachycardia, though this occurs less commonly. SVT occurs in approximately 1 in 500 persons with structurally normal hearts. Of this number, approximately 10% are caused by an atrial tachycardia. This can be further subdivided by mechanism into reentrant (atrial flutter/ macroreentrant or microreentrant) and automatic (i.e. ectopic). In postoperative cardiac patients, both SVT and the proportion due to atrial tachycardia are significantly higher. Differentiation is important because treatment and natural history are different for AVRT versus AVNRT versus atrial tachycardia. Prolongation of the tachycardia cycle length (and VA time) with intermittent bundle branch block strongly suggests AVRT using an accessory pathway ipsilateral to the site of bundle branch block. Spontaneous termination ending in an atrial event (i.e., P wave) would strongly suggest AVRT or AVNRT over an atrial tachycardia. In AVRT (and most of the time for AVNRT), one would expect a block in the AV node to result in termination of the tachycardia because the AV node participates in the circuit. However, in an atrial tachycardia, AV block would not be expected to lead to termination of the tachycardia because the AV node is not involved in the circuit (or ectopic focus), but would simply cause second-degree AV block. Thus, for an atrial tachycardia to end with an atrial event, there would need to be simultaneous termination of the tachycardia and unrelated AV block, which would be extremely unlikely (especially when an atrial tachycardia displays 1:1 AV conduction, as seen here). AVRT, AVNRT, and atrial tachycardias can all have sudden onset and termination and is therefore not suggestive of atrial tachycardia over AVRT. Ectopic atrial tachycardias frequently display "warm-up/cool-down" onset and termination rather than sudden onset or termination. In summary, differentiation of the SVT mechanism can be very important for treatment decisions and expected outcomes. If second-degree heart block is seen without affecting the atrial rate, atrial tachycardia should be diagnosed with few exceptions. Alternatively, termination with an atrial event almost completely excludes atrial tachycardia (in the setting of typically 1:1 AV ratio). Other methods that differentiate AVRT from atrial tachycardia include tachycardia cycle length lengthening with bundle branch block (suggesting AVRT) and a warmup and cool down period (suggesting an ectopic focus). PREP Pearls Second-degree atrioventricular block in the setting of supraventricular tachycardia almost always indicates atrial tachycardia (vs occasionally atrioventricular nodal reentrant tachycardia ). Termination of a 1:1 supraventricular tachycardia with an atrial event almost always rules out atrial tachycardia. Atrial ectopic tachycardia frequently has "warm-up/cool down" onset and termination.

You are evaluating a 26 week old fetus who was referred by Maternal Fetal Medicine (MFM) after a cleft palate and thickened nuchal translucency were noted. No other abnormal findings were seen by MFM. They thought the 4 chamber view was normal, but they were unable to see the outflow tracts or image the aortic arch well. On fetal echo, you note: S,D,S (atrial situs soitus, D-looping and normally related great vessels. There is a conoventricular type VSD that appears to be posteriorly maligned with an aortic valve Z score of -2.1. The imaging of the aortic arch is difficult. In the sagittal view, the ascending aorta is prominent and appears to have a straight course with 2 proximal branching vessels (Figure 1a and Figure 1b). There appears to be lack of continuity between the ascending and descending aorta. There is a large right to left ductus arteriosus. You then attempt the 3 vessel-tracheal view. The aortic arch and ductal arch are to the left of the spine and trachea; however, continuity of the transverse aortic arch towards the descending aorta cannot be demonstrated. You discuss your findings with the parents. The parents refuse genetic evaluation but want to know if there is a risk of genetic abnormality. Of the following, based on the findings, the MOST likely genetic association in this fetus is A. 5p15 deletion B. 22q11 deletion C. monosomy X D. trisomy 13

B. 22q11 deletion The fetus in the vignette is at increased risk for a congenital heart defect. Both cleft palate and a thickened nuchal translucency may be markers for the presence of a heart defect and warrant fetal echocardiography. In this scenario, the posterior malaligned ventricular septal defect (VSD) and "straight" course of the ascending aortic arch with the inability to show arch continuity are markers for a possible interruption of the aortic arch. The more advanced (and at times, difficult to obtain) 3-vessel tracheal view shows the transverse duct and aortic arch view, which is normally V-shaped (Figure 2 and Figure 3). This finding was noted in the fetus, indicating that this is a left aortic arch (the ductus and aorta to the left of the trachea and spine). Figure 4 shows a 3 vessel view in the normal fetus (A) and in the fetus with IAA (B). The fetus showed lack of aortic arch continuity in the sagittal aortic arch view. Normally, the fetal arch in the sagittal view is candy cane-shaped and not straight. This lack of continuity is a sign of an interrupted aortic arch (IAA). This is most likely an IAA type B because the first 2 head and neck vessels are seen and no transposition is seen. In a fetus, an IAA with a VSD should raise suspicions of IAA type B and 22q11 deletion. Cleft palate and IAA type B are associated with 22q11.2 deletion (DiGeorge syndrome, velocardiofacial syndrome). It is the most common chromosomal deletion syndrome and occurs in 1 in 6,000 births. Approximately 5% to 10% are familial and a parent may be found to have it retrospectively. Only 5% to 10% inherit the 22q11 deletion from a parent, whereas about 90% to 95% of cases have a de novo deletion of 22q11. The deletion follows standard mendelian inheritance, so individuals carrying the deletion 22q11 have a 50% chance of passing it to their offspring. The clinical phenotype is variable. The most common cardiac defects include IAA type B, tetralogy of Fallot (TOF), truncus arteriosus, VSD, and isolated arch anomalies. Less commonly, pulmonary stenosis, atrial septal defect, heterotaxy syndromes, and hypoplastic left heart syndrome may be seen. In addition to cardiac defects, common features include feeding problems, palate anomalies, hypocalcemia, behavioral issues, and developmental disorders with delayed learning. Some studies in the literature suggest that as many as 50% to 68% of patients with IAA type B have a 22q11 deletion. In addition to IAA type B, 22q11 deletion is associated with an increased risk of TOF (15%), truncus arteriosus (35%), an isolated aortic arch anomaly (24%), and conoventricular VSDs (10%). The Celoria and Patton classification of IAA includes type A, in which an interruption occurs distal to the origin of the left subclavian artery in 30% to 37% of cases. Type B IAA is the most common type, with disruption occurring between the left common carotid and left subclavian (62%-70%). In type C, the rarest type, the interruption occurs between the right and left common carotids, and is found in less than 1% of cases. Type A may also be associated with aorticopulmonary defects (aortopulmonary windows) or transposition of the great arteries. Type B is associated with a VSD in 80% to 90% of the cases. Although Turner syndrome (monosomy X) is associated with bicuspid aortic valve, coarctation, and aortopathy, interruption is very rare. This fetus had no documentation of lymphedema or a horseshoe kidney. Cleft palate is rare in Turner syndrome. 5p15 deletion is often called "cri du chat syndrome" (cat like cry). This condition is associated with an increased risk of congenital heart defects with VSDs, atrial septal defect, patent ductus arteriosus, and TOF being the most common. IAA is not reported. Trisomy 13 and trisomy 18 may present with multiple forms of complex defects, including VSDs, atrial septal defects, TOF, and polyvalvar dysplasia. Cleft lip and plate are common in trisomy 13 but uncommon in trisomy 18. IAA is uncommon in trisomy 13 and trisomy 18. In summary, the fetus in the vignette had a posterior malaligned VSD, a suspected IAA type B, and a cleft plate. This should raise concerns for 22q11.2 deletion. Testing for the 22q11 deletion should be considered, in addition to a fetal karyotype, when a cardiac defect is detected in utero by fetal echocardiography, especially if conotruncal defects or IAA is suspected. Counseling about the other associated features, especially learning/developmental/behavioral issues, is an important part of fetal counseling. PREP Pearls A posterior malaligned ventricular septal defect and an aortic arch anomaly should make you suspicious of 22q11.2 deletion. Conotruncal defects diagnosed prenatally may be associated with 22q11 deletion. 22q11 deletion may be associated with hypocalcemia. Lack of aortic arch continuity on fetal echocardiography may be a subtle marker of interrupted aortic arch.

A 5-year-old girl is referred for cardiology evaluation for a murmur. The murmur was noticed at her last 2 health supervision visits, and she was referred this year because the murmur seemed louder. Her mother notes that although she is generally asymptomatic, she does occasionally complain that her heart is beating fast. She is active and is growing well. On physical examination, her vital signs are normal. She has mild jugular venous pulsations. Her lung fields are clear with equal air entry. Her precordium is hyperactive, but there is no thrill. Her S1 and S2 are normal, there is also a prominent S3 present, but no rub. There is a low-frequency 3/6 blowing holosystolic murmur at the left midsternal border with a mid-diastolic murmur. Her abdomen is soft and her liver is palpable 2 cm below the costal margin. Her extremities are warm and well perfused with equal pulses. You review her electrocardiogram (Figure 1) and order an echocardiogram. Of the following, the congenital heart defect MOST likely to be found on her echocardiogram is A. congenitally corrected transposition (l-TGA) with ventricular septal defect B. Ebstein anomaly with tricuspid regurgitation C. primum atrial septal defect with cleft mitral valve D. secundum atrial septal defect

B. Ebstein anomaly with tricuspid regurgitation Conduction and electrocardiographic abnormalities are seen in many congenital heart defects. The electrocardiogram can aid in diagnosis, and in many congenital heart defects, is crucial to evaluate for associated electrophysiologic abnormalities. The patient described in the vignette has a physical examination consistent with severe tricuspid regurgitation and an electrocardiogram showing pre-excitation. Ebstein anomaly is the most likely congenital heart defect to be found in this patient. Ebstein anomaly is caused by the failed delamination of the tricuspid valve leaflets from the myocardium. This results in varying levels of valvar dysfunction. The physiology is variable depending on the severity and associated anomalies. Electrocardiographic abnormalities are common in Ebstein anomaly. Wolff-Parkinson-White syndrome is seen in 20% of patients. The characteristic delta wave and short PR interval are displayed on this patient's electrocardiogram (Figure 1). Some patients can have concealed pathways. This makes the risk for atrioventricular re-entrant tachycardias very high. Patients can also develop intra-atrial reentrant tachycardias because of atrial dilation. Patients frequently need electrophysiology studies with or without ablation of the substrate for arrhythmias. Other electrocardiographic abnormalities are often seen. Wide P waves are common because of delayed intra-atrial conduction and right atrial dilation. Right ventricular conduction abnormalities can be seen, including RSR′ pattern and right bundle branch block. These are typically absent in patients with pre-excitation. Congenitally corrected transposition of the great arteries (l-TGA) can also be associated with pre-excitation, as well as Ebstein anomaly of the tricuspid valve. However, the patient in the vignette has physical examination findings that are more consistent with regurgitation of the atrioventricular valve receiving the systemic venous return. She does not have physical examination findings of a ventricular septal defect. Because of the L-looping in patients with l-TGV, the conduction system is displaced. This leads to right-to-left activation of the ventricular septum. Surface electrocardiography shows absent Q waves in the lateral precordial leads with Q waves in the right precordial leads (Figure 2). Knowledge of the abnormal course of the conduction system is important when performing a surgical procedure, particularly ventricular septal defect closure, because the bundle branches are more easily damaged with placement of the patch. The abnormal pathway of the conduction system also places patients at risk for atrioventricular block; this occurs in one-third of patients. If not present at birth, the risk is 2% per year. Primum atrial septal defects with cleft mitral valve or partial atrioventricular septal defects are typically associated with a left axis deviation on the electrocardiogram (Figure 3). Depending on the degree of mitral regurgitation and shunt across the atrial septal defect, tall or wide P waves may occur because of atrial enlargement, right ventricular hypertrophy, or RSR′ pattern in V1 because of right ventricular enlargement. RSR′ pattern in V1 is also seen in secundum atrial septal defects in which right ventricular dilation occurs (Figure 4). PREP Pearls Ebstein anomaly is frequently associated with Wolff-Parkinson-White syndrome. Conduction system abnormalities are common in congenital heart defects. Patients with congenitally corrected transposition of the great arteries are at risk for Wolff-Parkinson-White syndrome and heart block.

The parents of a 6-year-old patient of yours who has a bicuspid aortic valve tell you that the boy's mother is now 20 weeks pregnant. The parents do not have any known heart disease, but have never been seen by a cardiologist or undergone any diagnostic testing. You tell them that there is still much research being done in risk assessment for different forms of congenital heart disease, but that you can make some general statements. Of the following, the statement that MOST accurately describes the risk of congenital heart disease in this fetus is that A. The fetus has an approximately 2 in 1,000 (0.2%) risk of having a cardiac malformation B. The fetus has at least a 5% to 10% risk of having a bicuspid aortic valve C. The fetus is at increased risk for Turner syndrome D. The risk of the fetus having a bicuspid aortic valve is higher if the fetus is a female

B. The fetus has at least a 5% to 10% risk of having a bicuspid aortic valve Historically, the incidence of congenital heart disease has been reported as 8 per 1,000 live births, but according to the 2007 American Heart Association position statement on the genetic basis of congenital heart disease (CHD), the incidence of CHD may be as high as 50 per 1,000 live births. Bicuspid aortic valve has been reported to be the most common adult form of CHD, with a prevalence rate of 0.5% to 2% of the adult population. Despite this high disease prevalence, our understanding of the genetic inheritance risk of CHD in general, and bicuspid aortic valve in particular, is still evolving. However, some genetic factors in bicuspid aortic valve and the genetics underlying other forms of CHD are well established. For example, familial clustering of patients with bicuspid aortic valve has been observed for quite some time, but only recently has it been shown that familial bicuspid aortic valve syndrome can be inherited as an autosomal dominant condition with variable penetrance. This heritable nature of bicuspid aortic valve was reported by Huntington et al, who found that 9% of first-degree relatives of subjects with congenital bicuspid aortic valve also have a bicuspid aortic valve on screening echocardiography. Lewin et al found that the relative risk of bicuspid aortic valve in parents and siblings of patients with left-sided obstructive lesions was 5:1 compared with an estimated risk of 1% in the general population. Cripe et al similarly investigated the history and echocardiographic findings of first-degree relatives of 50 probands with a bicuspid aortic valve. They found that 24% of first-degree relatives had bicuspid aortic valve, and 31% had either bicuspid aortic valve or other forms of congenital cardiac malformations, specifically other left-sided obstructive lesions. This and other studies are consistent with variable penetrance of an autosomal dominant inheritance. Thus, the risk that the fetus in this vignette has a bicuspid aortic valve is at least 5% to 10%, but may be even higher. Because of the increased risk, echocardiographic screening has been recommended for all first-degree relatives of a patient with bicuspid aortic valve. Most studies of the genetics of bicuspid aortic valve and other left-sided obstructive lesions have demonstrated a male predominance. Finally, although bicuspid aortic valve is more common in patients with Turner syndrome and has been reported to be associated with a deletion in the short arm of the X chromosome, this has not been shown to be associated with familial bicuspid aortic valve syndrome. PREP Pearls The incidence of all forms of congenital heart disease is likely significantly higher than the commonly quoted value of 8 per 1,000 live births. The risk of bicuspid aortic valve in a sibling or other first-degree relative with bicuspid aortic valve is most likely at least 5% to 10%. The genetic makeup of familial bicuspid aortic inheritance has not been definitively shown, but may be autosomal dominant with variable penetrance.

A 10-day old, 3.2-kg infant with the diagnosis of tricuspid atresia and severe pulmonary stenosis is in the intensive care unit 1 day after placement of a palliative 4.0-mm modified Blalock-Taussig shunt. The infant develops acute desaturation to 54% and is not improving with mechanical ventilation with 100% oxygen. You are called to the bedside to evaluate the patient. The infant's vital signs include a heart rate of 165 beats/min, respiratory rate of 35 breaths/min via the ventilator, oxygen saturation of 54%, and blood pressure of 96/58 mm Hg by femoral arterial line. You quickly measure her arterial blood gas, which demonstrates a pH of 7.28, a partial pressure of carbon dioxide of 57 mm Hg (7.5 kPa), and a partial pressure of oxygen of 24 mm Hg (3.2 kPa). A hemoglobin level measured 2 hours before this event was 14.5 g/dL (145 g/L). On cardiac auscultation, you hear a high-frequency systolic ejection murmur that is somewhat obscured by the mechanical ventilation. Chest rise is good with mechanical breaths, and breath sounds are audible throughout both lung fields. Echocardiography is performed (Figure). Of the following, the BEST intervention for this patient should be to A. administer a dose of epinephrine B. administer heparin 100 U/kg C. initiate inhaled nitric oxide D. order a blood transfusion E. increase positive end-expiratory pressure

B. administer heparin 100 U/kg Infants often develop hypoxia after the placement of a modified Blalock-Taussig (BT) shunt, with multiple etiologic factors contributing to the hypoxia. At the bedside, the patient's vital signs, physical examination, laboratory measures, and other technology can help the clinician narrow down the diagnosis. The infant in the vignette has recently undergone placement of a modified BT shunt and is having an episode of acute desaturation in the intensive care unit that is not responding to oxygen administration. Often, multiple interventions are undertaken in parallel with diagnostic tests. In this case, recognition that acute shunt thrombosis is the etiology is key to rapid directed therapy to treat the thrombosis. Thus the best next intervention for this patient is to administer heparin. Blood pressure is an important marker to follow after placement of a modified BT shunt. Although flow through the shunt is driven by the systolic blood pressure, the diastolic blood pressure is often low because of diastolic runoff through the modified BT shunt. Shunt size can contribute to the degree of diastolic runoff. The patient in the vignette had a larger (4.0 mm) shunt placed. The diastolic pressure therefore would be expected to be lower; the fact that the diastolic pressure is 58 mm Hg in the setting of a higher systolic blood pressure raises concern for shunt occlusion because of thrombosis or mechanical postoperative obstruction. Definitive treatment for shunt occlusion is either surgical or interventional, but immediate interventions should include a large dose of heparin to try and prevent further thrombus formation. In addition, the maintenance of supra-physiologic blood pressure levels can often force blood flow through a narrowed BT shunt, though at the risk of cerebral hemorrhage in a neonate such as the patient in the vignette. Once shunt occlusion has been identified as the primary cause, rapid steps must be taken to address the occlusion, which may include immediate surgical intervention, immediate cardiac catheterization, or extracorporeal support until definitive intervention can be undertaken. Despite the desaturation, this patient's heart rate is stable and the systolic blood pressure is elevated for age. Administration of a dose of epinephrine would increase both the heart rate and blood pressure. This could possibly lead to a transient increase in saturation by increasing antegrade blood flow through the native pulmonary artery with severe stenosis. However, this measure would not provide definitive treatment to reverse the underlying process. Pulmonary hypertension is common after congenital heart surgery and often requires treatment with medication. In situations such as this one, inhaled nitric oxide may be started empirically while diagnostic tests are under way. Although this patient is at risk for pulmonary hypertension, there is no evidence that this acute event is a pulmonary hypertensive crisis. In addition, patients with modified BT shunts can often overcome some degree of pulmonary hypertension because their pulmonary blood flow is driven by the systemic systolic blood pressure. Diagnosis of pulmonary hypertension in these patients is ideally made via cardiac catheterization and should be considered when conventional diagnostic means fail to provide an explanation for the hypoxia. Should other interventions fail, and the modified BT shunt is proven to be patent, then treatment with pulmonary vasodilators such as inhaled nitric oxide would be indicated. Any lung pathology can increase the pulmonary vascular resistance and decrease blood flow to the lungs through a modified BT shunt. This could include atelectasis, pleural effusions, pneumothorax, and pneumonia. Although this patient is at risk for all the aforementioned etiologies, given the audible breath sounds on examination throughout the lung fields, the likelihood of lung etiology severe enough to cause significant desaturations is low. Increasing the positive end-expiratory pressure (PEEP) while useful for treating hypoxia would not address the underlying issue in the vignette. Although increasing PEEP can be useful in treating hypoxia, it is unlikely to increase the saturations in this patient. Patients with cyanotic congenital heart disease often depend on higher hemoglobin levels to maintain oxygen-carrying capacity. Based on the equation for oxygen-carrying capacity [CaO2 = (Hemoglobin) × (Oxygen saturation) × (1.36) + (PaO2) × (0.003)], hemoglobin is an important factor in oxygen-carrying capacity in the setting of lower oxygen saturations. Although the optimal hemoglobin level has not been identified yet, a hemoglobin level above 13 g/dL (130 g/L) is generally sufficient in infants with cyanotic congenital heart disease. The patient in the vignette had a hemoglobin level of 14.5 g/dL (145 g/L) 2 hours before this event and there is no indication that a bleeding event has occurred. PREP Pearls Clinical interventions to treat hypoxia in a patient after placing a BT shunt depend on the etiology of the hypoxia. Rapid identification and treatment of modified Blalock-Taussig (BT) shunt occlusion is important. The etiology of wide pulse pressure after placement of a BT shunt is diastolic runoff through the shunt.

You are called to the neonatal intensive care unit (NICU) to evaluate a male neonate who was born via spontaneous vaginal delivery within the hour. His mother is visiting from out of town and told the obstetrics team that she was due about 2 weeks from now and that the baby's heart was enlarged on ultrasonography a few months ago. The neonatologist tells you that immediately after delivery the baby had acute respiratory distress and bradycardia, requiring intubation and a short episode of chest compressions. He was brought to the NICU intubated on room air with oxygen saturations in the low 80 range. His physical examination reveals a grade III/VI holosystolic murmur heard best along the left sternal border radiating throughout the precordium. An electrocardiogram is shown (Figure ). You prepare to perform an echocardiogram. Of the following, the anatomic feature that is MOST likely to be evident on that study is A. anterior and superior malalignment of the conal septum B. apical displacement of the septal leaflet of the tricuspid valve C. great arteries arising in parallel D. single outflow tract supporting systemic, pulmonary, and coronary circulations

B. apical displacement of the septal leaflet of the tricuspid valve The clinical situation described in the vignette is consistent with a diagnosis of Ebstein's anomaly as demonstrated on the baby's echocardiogram (Video 1, Video 2, Video 3). The fetus was diagnosed with cardiomegaly in utero and the subsequent presentation in the delivery room characterized by respiratory insufficiency, marked desaturation, and a murmur typical of tricuspid regurgitation are all highly suggestive of that defect. Ebstein's anomaly is a rare form of congenital heart disease (CHD) occurring in less than 1% of all affected individuals. It is characterized by apical displacement of the functional annulus of the tricuspid valve leaflets leading to atrialization of a portion of the right ventricle, which often becomes thinned and dilated; adherence of the septal and posterior leaflets to the underlying myocardium due to a failure of delamination; a large, redundant, often fenestrated anterior leaflet; and dilation of the true tricuspid valve annulus. Depending on the degree of pathology of the tricuspid valve leaflets, affected individuals may have only mild regurgitation of the valve, but alternatively may have severe regurgitation with resultant massive dilation of the right atrium, and if an atrial communication is present, right-to-left shunting, leading to cyanosis that can be severe. The echocardiograms shown in Video 1, Video 2, and Video 3 represents an extreme example of this disease consistent with both the description of cardiomegaly in fetal life, as well as of a very sick neonate. In an anatomically normal heart, there is up to 8 mm/m2 apical offsetting of the tricuspid valve annulus in relation to the mitral valve annulus. In patients with Ebstein's anomaly, this offsetting can be minimally increased but may be so profound as to result in functional absence of the body of the right ventricle. The patient shown in Video 1 has a functional annulus of the tricuspid valve that is so far displaced into the true right ventricle that there is limited contractile ventricle, but instead a significant component of atrialized right ventricle that has become markedly dilated as a result of the severe tricuspid regurgitation produced by the abnormalities of the leaflets themselves. In this case, the finding of mid-gestation cardiomegaly is consistent with that massive dilation of the right atrium evident on the patient's echocardiogram and itself has likely led to the patient's neonatal respiratory distress, as the cardiomegaly in utero resulted in some degree of pulmonary hypoplasia. In addition, as in the majority of patients with Ebstein's anomaly, this patient has an interatrial communication which, in this case, transmits a right-to-left shunt leading to the room air oxygen saturations in the 80% range described in the vignette. Newborns with tetralogy of Fallot, whose intracardiac anatomy is typified by anterior and superior malalignment of the conal septum, rarely present in severe distress in the delivery room absent associated anomalies or comorbidities. Cardiomegaly pre- or postnatally is an uncommon finding. In these patients, desaturation may be present if the limitation to pulmonary blood flow is significant, however, the presence of the patent ductus arteriosus early will mitigate that. Physical examination is unlikely to reveal a murmur consistent with severe atrioventricular valve regurgitation. Transposition of the great arteries, characterized by the great arteries arising in parallel, would also be unlikely to result in patients having cardiomegaly prenatally. In addition, such patients often do not have immediate profound cyanosis, as long as the atrial septum is nonrestrictive. A single outflow tract supplying all 3 circulations (systemic, pulmonary, and coronary) is consistent with truncus arteriosus. Newborns with this disease may have a clinical presentation similar to that of a neonate with Ebstein's anomaly, including respiratory distress and cardiomegaly. In patients with truncus arteriosus, the desaturation is typically not as profound as that described in the vignette and they will usually not have murmurs consistent with severe atrioventricular valve regurgitation. PREP Pearls The presentation of an acutely ill newborn characterized by respiratory insufficiency, desaturation, and a holosystolic murmur should prompt consideration of the diagnosis of Ebstein's anomaly. Ebstein's anomaly is defined by apical displacement of the functional annulus of the tricuspid valve leaflets, leading to atrialization of a portion of the right ventricle, adherence of the septal and posterior leaflets to the underlying myocardium due to a failure of delamination, and a large, sail-like anterior leaflet.

A 10-year-old boy was diagnosed with a dilated cardiomyopathy of unknown etiology 3 years earlier after presenting with acute decompensated heart failure. He was readmitted for worsening heart failure manifested by volume overload, but without hemodynamic compromise. He was receiving appropriate doses of enalapril and carvedilol. After a good diuretic response to intravenous (IV) furosemide he was transitioned to oral diuretics. He and his family wish to go home. He is now taking oral furosemide 20 mg 3 times daily and oral spironolactone 12.5 mg twice daily. His urine output has declined in the last 24 hours. On examination this morning, his weight is 25 kg (up 0.5 kg), pulse rate is 110 beats/min, respiratory rate is 32 breaths/min, and blood pressure is 95/65 mm Hg. He is able to answer questions without effort. He is warm, with a capillary refill time of less than 3 seconds. His lungs are clear with no use of accessory muscles. His point of maximal impulse is laterally and inferiorly displaced; he has a normal S1 and narrowly split S2; a II/VI holosystolic murmur that is loudest at the apex to the axilla; and an S3. His liver is 5 cm below the right costal margin. The right upper quadrant is mildly tender. He displays no clubbing, cyanosis, or edema. Your assessment is that he is not ready to be discharged. Of the following, the BEST next medication to add for the boy in the vignette is: A. bumetanide B. chlorothiazide C. ethacrynic acid D. low-dose dopamine E. nesiritide

B. chlorothiazide Chlorothiazide is a thiazide diuretic. The thiazide diuretics inhibit the reabsorption of sodium and chloride in the distal convoluted tubule. The sodium-chloride (Na+/Cl-) cotransporter in this region is insensitive to loop diuretics. However, nephron hypertrophy can occur in the distal convoluted tubule because of loop diuretic exposure, resulting in a reduced ability to reabsorb sodium thus contributing to diuretic resistance. Thiazides block the nephron sites at which hypertrophy occurs, which helps to overcome loop diuretic resistance. The duration of action varies considerably with the thiazide chosen, but as a class of drugs is usually longer than the loop diuretics. The duration of action of chlorothiazide is 6 to 12 hours. The addition of a thiazide to this patient's regimen is an example of sequential nephron blockade with the use of 3 diuretics that act at different sites in the nephron to enhance diuresis (Figure). Furosemide acting at the ascending loop of Henle, chlorothiazide at the distal convoluted tubule, and spironolactone which blocks the mineralocorticoid receptor further along in the distal convoluted tubule and collecting ducts and is potassium sparing. To enhance their synergistic effect, a thiazide and loop diuretic should be given together. The addition of a thiazide to loop diuretics can cause overdiuresis and severe electrolyte disturbances that require close monitoring. Like furosemide, bumetanide is a loop diuretic. The loop diuretics act in the ascending limb of the loop of Henle, inhibiting the Na+/K+/2Cl- cotransporter involved in the transport of chloride across the lining cells in the ascending limb of the loop of Henle. Because it has a very similar site and mechanism of action as furosemide it would be more appropriate to increase the frequency of the furosemide rather than adding an additional loop diuretic to this regimen. Although not a response choice it would not be wrong to increase the frequency of furosemide to every 6 hours because furosemide's duration of action is 4 to 5 hours. Higher doses of furosemide could also be used. However, a declining response to a loop diuretic regimen may be the result of loop diuretic resistance. There are a number of reasons for this. Use of loop diuretics can result in hypertrophy and hyperplasia of cells in the distal convoluted tubules because of increased exposure to sodium. The hypertrophy and hyperplasia result in the ability to reabsorb more sodium resulting in a decreased effect of loop diuretics. Dopamine is an endogenous catecholamine with differing effects at different doses. At low doses (<5 μ/kg/min) its effects are primarily on dopamine receptors in the renal and mesenteric beds, resulting in vasodilation in these beds. It increases renal blood flow out of proportion to changes in cardiac output at low doses and may enhance urine output. Stimulation of dopamine agonist receptors (DA1) on the renal tubular cells have also been shown to oppose the effects of antidiuretic hormone. However, studies in adult populations have not shown significant diuretic benefit from low-dose dopamine. This strategy, if chosen, should be reserved for patients in whom other diuretic strategies have been exhausted or considered in the setting of hypotension. Nesiritide is a recombinant human B-type natriuretic peptide. Natriuretic peptides cause afferent arteriolar dilation and efferent arteriolar constriction, leading to an increase in the glomerular filtration rate. Limited data in children suggested safety and possible benefit in terms of urine output, central venous pressure, and/or improved neurohumoral profile. After promising larger adult studies, subsequent studies in adults suggested relative safety but no benefit in patients experiencing heart failure. One meta-analysis in adults suggested higher mortality and renal dysfunction associated with its use. The use of nesiritide in adults has a IIb level of recommendation as an adjuvant to diuretic therapy for the relief of dyspnea, level of evidence A. In the most recent International Society for Heart and Lung Transplantation guidelines for the management of pediatric heart failure, nesiritide was not recommended for routine use in acute heart failure, but "may be considered in select situations when other interventions have failed to lower the central venous pressure," class IIb recommendation, level of evidence C. Therefore it would not be recommended for the boy in the vignette. PREP Pearls Sequential nephron blockade can enhance diuresis. This may allow the use of lower doses of each agent. Electrolyte disturbances are increasingly likely with the combined use of loop and thiazide diuretics. In select complicated cases, dopamine or nesiritide may be used to try to enhance diuresis after other maximal diuretic therapies fail.

A 6-month-old infant with aortic stenosis is referred to you for cardiac catheterization. He is asymptomatic. His electrocardiogram meets voltage criteria for left ventricular hypertrophy, but is otherwise normal. His echocardiogram is available for review (Figure 1). His LV function is normal. The Doppler velocity across the left ventricular outflow tract is 3.5 m/s. Simultaneous left ventricular and ascending aortic pressures are obtained in the catheterization laboratory (Figure 2). Of the following, the BEST treatment recommended for the infant in the vignette is: A. balloon aortic valvuloplasty B. continued outpatient follow-up C. initiation of diuretic therapy D. surgical aortoplasty E. surgical resection of a subaortic membrane

B. continued outpatient follow-up The echocardiogram demonstrates isolated aortic valve stenosis. The infant in the vignette should continue to be followed up on an outpatient basis. The peak systolic pressure gradient in this asymptomatic child with aortic valve stenosis is not high enough to warrant intervention. The American Heart Association scientific statement on indications for intervention in pediatric cardiac disease recommends that intervention be provided for a catheter-measured peak gradient greater than 50 mm Hg (class 1 recommendation). This patient has a catheter-measured peak gradient of less than 25 mm Hg, which is not severe enough to warrant intervention. Symptoms consistent with angina or syncope, ischemic ST-T wave changes on electrocardiography, or depressed left ventricular systolic pressure would be reasons to consider intervention at lower gradients. If the mean gradient measured during nonsedated echocardiography is greater than 50 mm Hg, intervention may be reasonable in an asymptomatic patient with a catheter-derived peak gradient less than 50 mm Hg. Balloon aortic valvuloplasty is not appropriate for an asymptomatic child with this degree of stenosis. There is no benefit to starting diuretics for the patient in the vignette. Catheter intervention is widely regarded as first-line therapy for children with valvar aortic stenosis severe enough to warrant therapy. Even in centers where surgical therapy is the treatment of choice, surgical aortoplasty is not indicated for the same reasons as balloon aortic valvuloplasty. The echocardiogram does not demonstrate a subaortic membrane, therefore surgical resection for this condition is not indicated for the infant in the vignette. PREP Pearls Class I indications for balloon aortic valvuloplasty in Pediatric patients include: Regardless of the gradient: in neonates with isolated critical valvar AS who are ductal dependent or in children with isolated valvar AS who have depressed left ventricular systolic function. In children with a resting peak systolic valve gradient measured by catheterization greater than 50 mmHg. In children with a resting peak systolic valve gradient measured by catheterization greater than 40 mmHg if there are symptoms of angina, syncope or ischemic ST-T

A 2-week-old full-term infant had a murmur that was heard in the newborn nursery and persisted at her 2-week check-up. She is seen in your office within a week of referral. She is thriving without symptoms, and her vital signs are normal, including 4 extremity blood pressures. Her blood pressure in the right arm was 82/50 mm Hg. Other notable examination findings were a mildly increased right ventricular impulse, a normal S1 and S2 with a long III/VI systolic ejection murmur loudest toward the left upper sternal border, nearly obscuring A2 and radiating to the back. There is an early systolic click, but no diastolic sounds. On echocardiography, the pulmonary valve is echo bright with doming leaflets and a dilated main pulmonary artery (Figure 1 ). The right ventricle is mildly hypertrophied with otherwise normal size and function. The peak velocity is 3.02 m/s with an estimated peak pressure gradient of 36.4 mm Hg (Figure 2). You recommend that the family make a follow-up visit to the office, as long as the infant remains asymptomatic. Of the following, the BEST follow-up recommendation for the infant in the vignette is an echocardiogram: A. in 1 week B. in 6 weeks C. at 9 months of age D. at 18 months of age E. at 24 months of age

B. in 6 weeks Mild pulmonary valve stenosis is generally defined as a pressure gradient less than 30 to 35 mm Hg with a right ventricular pressure less than half systemic. The course of mild pulmonary valve stenosis is almost always benign, without the need for intervention, when diagnosed after infancy. However, in young infants, stenosis may progress significantly in the first 6 months after birth. Some of this is likely because of the fall in pulmonary vascular resistance "unmasking" a more significant gradient, but others do appear to have worsening anatomic obstruction. At around 6 weeks of age, the decrease in pulmonary vascular resistance is likely to be complete or nearly complete. The full degree of stenosis will likely have been unmasked by that time, allowing for a more definitive assessment with further follow-up and/or intervention if necessary to be determined at that time. Follow-up in 1 week may not allow enough time for the fall in pulmonary vascular resistance to be complete, therefore the degree of pulmonary stenosis may still be masked at that time. The most rapid progression of pulmonary valve stenosis often occurs in the first 6 months after birth. By waiting until 9 months of age for the next follow-up, the infant may develop a severe degree of stenosis that would have warranted an earlier intervention. Although the degree of stenosis may still be progressing at age 18 months, waiting this long may result in an infant with a significant degree of stenosis that warranted earlier intervention. However, around age 2 years, the rate of growth in children slows. It is uncommon for pulmonary stenosis to progress thereafter, until they reach the next stage of rapid growth (puberty), when stenosis is again likely to worsen. Waiting to see the infant until age 2 years would likely allow for determination of the most significant degree of stenosis, by then the child may have developed a much more hypertrophied ventricle and infundibular stenosis. This will take longer to resolve after intervention. PREP Pearls Pulmonary stenosis is more likely to be progressive in infancy and at other times of rapid somatic growth. More frequent evaluation is needed during periods of rapid growth

A 10-year-old girl who received a heart transplant comes to the emergency department with a 4-day history of cough with onset of fever up to 102°F today. Her medication list is currently unavailable. Her examination findings are notable for left upper lobe crackles. A complete blood count (CBC) is obtained and reveals the following: White blood cells 2,500/μL (2.5 × 109/L) Hemoglobin 11.9 g/dL (119 g/L) Hematocrit 35% Platelets 200 × 103/μL ( 200 × 109/L) The differential is pending. Of the following, the immunosuppressant drug she may be taking that is MOST likely to cause the hematologic abnormality on her CBC is A. cyclosporine B. mycophenolate mofetil C. prednisone D. tacrolimus

B. mycophenolate mofetil Mycophenolate mofetil (MMF) is a prodrug that is rapidly metabolized to the active drug mycophenolic acid (MPA). MPA is a selective, noncompetitive, reversible inhibitor of inosine monophosphate dehydrogenase, which is important in the de novo pathway of guanine nucleotide synthesis. B and T lymphocytes are highly dependent on that pathway for cell proliferation. MPA selectively inhibits lymphocyte proliferation and function. The major side effects of MPA are leukopenia, pure red cell aplasia, diarrhea, and vomiting as well as an increased incidence of some infections. Cyclosporine and tacrolimus are both calcineurin inhibitors. Both of these agents inhibit T-cell activation through calcineurin inhibition. Cyclosporine binds to an immunophilin called cyclophilin. Tacrolimus binds to an immunophilin called FK-506. Either 1 of these complexes interacts with calcineurin, blocking its phosphatase activity. Calcineurin-catalyzed dephosphorylation is required in activated T cells to induce a number of cytokine genes that promote T-cell growth and differentiation. Cyclosporine side effects include nephrotoxicity, hypertension, tremors, hirsutism, gingival hyperplasia, and hyperlipidemia. Hematologic toxicities are uncommon. Thrombotic microangiopathy has been reported. Tacrolimus side effects include nephrotoxicity, neurotoxicity (tremor, headache, motor disturbance, seizures), gastrointestinal complaints, hypomagnesemia, hypertension, hyperkalemia, hyperglycemia, and diabetes. Hematologic toxicities are uncommon. Pure red cell aplasia has been reported. Both cyclosporine and tacrolimus increase the risk of infection and post-transplant lymphoproliferative disorders. Prednisone is a glucocorticoid that affects the number, distribution, and function of all types of leukocytes, including T and B cells, granulocytes, macrophages, and monocytes by multiple mechanisms and is the least specific of the immunosuppressants. Glucocorticoids bind to specific receptor proteins in target tissues and cells regulating the expression of corticosteroid-responsive genes, changing the levels and array of proteins synthesized by the target tissues/cells. Glucocorticoids suppress formation of proinflammatory cytokines, inhibit T cells from making IL-2 and proliferating, and inhibit the activation of cytotoxic T lymphocytes. Prednisone has multiple side effects, including electrolyte abnormalities, hypertension, hyperglycemia, increased susceptibility to infection, osteoporosis, myopathy, poor growth, cataracts, "moon" facies, and "buffalo hump." PREP Pearls Of the most commonly used immunosuppressants in the patient with a heart transplant today, mycophenolate mofetil is the most likely agent to cause leukopenia. Leukopenia may result in the need, at least temporarily, to discontinue mycophenolate. One should decrease or discontinue MMF for white blood cell counts of less than 3,000 to 4,500/μL (3.0-4.5 × 109/L), thresholds vary by center.

A 4-year-old boy is seen at a follow-up visit 2 weeks after surgical closure of a restrictive ventricular septal defect that had been associated with aortic valve prolapse. The mother reports that the surgery and postoperative course were quite uneventful. She is concerned because he has had fevers up to 40°C in the past 3 to 4 days. He has complained of chest pain and his energy level has been lower. His heart rate is approximately 130 beats/min and his blood pressure is 90/75 mm Hg. His examination is remarkable for a III/VI holosystolic murmur along the left lower sternal border. Echocardiography is performed. Of the following, based on the history and examination findings, the echocardiographic finding MOST likely associated with the change in the patient's recent clinical presentation is A. aortic valve vegetation B. pericardial effusion C. tricuspid valve regurgitation D. ventricular septal defect patch leak

B. pericardial effusion Postpericardiotomy syndrome is a febrile illness that can occur in any patient who has undergone surgery involving the pericardium. However, it most commonly occurs in patients older than 3 years of age who have undergone atrial septal defect, ventricular septal defect, or tetralogy of Fallot repair. In this latter group of patients, postpericardiotomy syndrome can occur at rates as high as 30%, and symptoms including fever, chest pain, malaise, and shortness of breath typically occur approximately 10 days after surgery. The underlying pathophysiology of postpericardiotomy syndrome remains a debate, but the process is considered to be a result of an autoimmune inflammatory trigger. The patient in the vignette is 2 weeks post-ventricular septal defect and has signs and symptoms consistent with postpericardiotomy syndrome. However, some of the signs and symptoms could potentially overlap with endocarditis. Morris et al published data showing the cumulative incidence of endocarditis and in the repair of the 12 more common congenital heart defects. Approximately 3,800 patients were included in this cohort. Twenty-five years after surgery, the cumulative incidence of infective endocarditis in patients with ventricular septal defects was 3.5%. Consequently, although the patient in the vignette could have infective endocarditis, postpericardiotomy syndrome is much more likely. A holosystolic murmur could be explained by a residual ventricular septal defect patch leak. However, such a finding on echocardiography would not explain the findings of fever, tachycardia, and narrowed pulse pressure. The murmur could also be that of tricuspid regurgitation. However, in this patient who likely has an outlet or supracristal ventricular septal defect, the surgical approach to repair is unlikely to leave tricuspid regurgitation. A pericardial effusion on echocardiography would be most consistent with the diagnosis of postpericardiotomy syndrome and would best fit with the other signs and symptoms seen in the patient in the vignette. Furthermore, the patient has evidence of tamponade physiology with tachycardia and a narrowed pulse pressure. An aortic valve vegetation would be consistent with endocarditis and would explain the patient's fevers. However, one would expect to hear a diastolic murmur related to aortic insufficiency. Although endocarditis is a possibility, the diagnosis would be much less likely than postpericardiotomy syndrome. Consequently, aortic valve vegetation is not the best answer. PREP Pearls Postperiocardiotomy syndrome should be suspected in children within 6 weeks after cardiac surgery. Fever in association with tachycardia and narrow pulse pressure precede tamponade. A pericardial friction rub could mimic holosystolic heart murmurs when the effusion is small.

You are reviewing the echocardiogram of a 4-year-old patient who has a moderate-sized, superiorly located secundum atrial septal defect with "adequate atrial septal defect margins" and normal pulmonary venous return. You are asked to give your opinion about transcatheter device closure of this defect. Specific systemic venous findings appear in echocardiographic images (Media 1, Media 2, Media 3, Media 4). The finding that would be MOST likely preclude a standard transcatheter atrial septal defect device closure technique is A.Media 1 - suprasternal view B.Media 2 - subcostal view C.Media 3 - subcostal view D.Media 4 - suprasternal view **Please note media 1 and 4 would not load. Corresponding pictures are labeled 9.2 and 9.3 for the two available images**

B.Media 2 - subcostal view Because transcatheter closure of a secundum atrial septal defect (ASD) is usually performed through the femoral vein and inferior vena cava (IVC), the presence of an interrupted IVC can significantly affect the placement of an ASD device through a femoral approach. Interrupted IVC is the result of absence of the hepatic IVC segment with azygous continuation into the right or left superior vena cava (SVC) (or rarely to bilateral SVCs). Interrupted IVC may occur as an isolated defect, but is much more common in the presence of heterotaxy syndrome with left atrial isomerism (86%). An interrupted IVC can be demonstrated echocardiographically by the absence of an IVC draining into the right atrium, together with the presence of a large venous vessel whose flow is directed superiorly, as seen in the subcostal imaging in media 2. There have been case reports, including reports by Lowry et al and Ozbarlas et al, that have described alternative techniques for transcatheter ASD closure in the presence of an interrupted IVC, including through transhepatic, jugular, and even transazygous approaches. The systemic veins develop embryologically from the connection and involution of the 3 paired venous systems, the cardinal, umbilical, and vitelline veins, which drain to the ipsilateral horns of the sinus venosus. Congenital abnormalities of the systemic veins develop during the first 8 weeks of gestation as consequences of abnormal drainage or failed involution. Such abnormalities may occur as isolated defects, but are more common in the presence of other congenital cardiac defects, and are especially common in patients with heterotaxy syndrome. Systemic venous abnormalities include abnormalities of superior veins, including right- or left-sided superior vena cava and innominate vein, or of inferior venous drainage, including interruption of the IVC or abnormal IVC drainage to the left atrium. Although these abnormalities do not usually lead to any clinical symptoms, systemic venous abnormalities can affect cardiac interventional procedural or surgical planning and success. Thus, in all patients referred for either transcatheter or surgical repair of congenital heart disease, care should be taken to delineate systemic venous anatomy, including the presence/absence of the SVCs (right- and/or left-sided), innominate vein, and IVC before intervention. Echocardiography is well suited to diagnose congenital abnormalities of systemic venous drainage in infants and young children, although alternative imaging such as magnetic resonance imaging or computed tomography may be required for anatomic confirmation, or to assess for congenital systemic venous abnormalities in older or larger patients with suboptimal echocardiographic windows. Abnormalities of superior systemic venous drainage typically have little impact on transcatheter ASD device placement. However, should a patient require surgical ASD closure, abnormalities of the superior systemic veins are important to recognize as venous cannulation site(s) may need to be modified. A persistent left SVC usually occurs in the presence of a normal right SVC (ie, bilateral SVCs), and is likely the result of failed involution of the left anterior and common cardinal veins. A persistent left SVC is common in patients with other congenital cardiac abnormalities, occurring in 3% to 34% of these patients, depending on the specific form of congenital heart disease. A persistent left SVC is rare as an isolated finding, occurring in approximately 0.5% of subjects in the absence of other congenital cardiac abnormalities. In over 90% of patients with a persistent left SVC, the persistent left SVC drains through a dilated coronary sinus to the right atrium, as seen in media 3. The presence of a dilated coronary sinus in the context of other underlying cardiac abnormalities may have an impact on blood flow patterns, for example, by impairing flow from the left atrium to the left ventricle. In the remainder of cases, a left SVC drains directly to the left atrium through a partially or completely unroofed coronary sinus; this is most common in cases of heterotaxy syndrome. Rarely, there can be an isolated left SVC, either to a dilated coronary sinus or directly to the left atrium, with absent right SVC. In addition to identifying the presence or absence of right- and left-sided SVC(s) and of the IVC, the presence and course of the innominate vein should be assessed prior to interventional or surgical procedures. Normally, there is a left innominate vein that courses anterior to the aorta to drain the left jugular vein to the right SVC to the right atrium. As noted above, the innominate vein can serve as a bridging vein when there are both right- and left-sided SVCs, as seen in media 1, or there may be an absent innominate vein when there are bilateral SVCs. In the presence of bilateral SVCs, the presence or absence of a bridging vein is particularly important in patients who are undergoing staged single-ventricle palliation procedures, as one must determine whether bilateral cavopulmonary anastomoses must be performed (when a bridging vein is absent) or whether one of the SVCs can be surgically ligated (when a bridging vein is present). Finally, there can be a retroaortic left innominate vein that drains the left jugular vein to the right SVC normally, but courses behind and underneath the aorta, as seen in media 4. A retroaortic left innominate vein is more common in patients with right ventricular outflow tract obstruction lesions such as tetralogy of Fallot. Although of no clinical significance, care must be taken so as not to confuse a retroaortic innominate vein with a pulmonary artery or with a persistent left SVC on imaging studies. PREP Pearls Although systemic venous abnormalities may not cause clinical findings, they can influence interventional catheterization or surgical procedures. Anomalies of the systemic veins rarely occur as isolated lesions. They are more common in association with other forms of congenital heart disease, and are present in about 90% of patients with heterotaxy syndrome.

A 24-year-old man with a history of hypoplastic left heart syndrome has undergone a Fontan revision. On postoperative day 1, he suddenly became tachycardic, with a heart rate of 110 beats/min. He is well sedated and intubated. He has a blood pressure of 108/65 mm Hg, temperature of 37.2°C, and respiratory rate of 18 breaths/min. His oxygen saturation is 93% on 0.21 fraction of inspired oxygen. He is breathing comfortably with the ventilator. His pulses are 1+ in his distal extremities, and his capillary refill time is 2 seconds. The cerebral oxygenation, according to near infrared spectroscopy, has dropped by about 5 points. An electrocardiogram (Figure 1) and atrial electrogram (from a temporary atrial pacing wire) (Figure 2) are shown. Of the following, the BEST next step is A.adenosine administration B.atrial overdrive pacing C.direct current cardioversion D.evaluation of complete blood cell count

B.atrial overdrive pacing The patient in this vignette has developed an atrial tachycardia. Although the initial electrocardiogram (Figure 1 ) did not provide diagnostic certainty, the atrial electrogram clearly documents an atrial rate of approximately 230 beats/min with a much slower ventricular rate, proving an atrially driven tachycardia (Figure 3 ). Although the mechanism could be automatic or reentrant, a reentrant mechanism is suggested by the stable atrial rate, sudden onset, and patient characteristics (ie, an adult survivor of the Fontan procedure). No additional maneuvers are necessary for diagnosis. Treatment should be initiated. Atrial overdrive pacing is the best next step for the treatment of the patient in this vignette. Overdrive atrial pacing is frequently successful in terminating reentrant tachycardias and can be performed using a transesophageal lead, transvenous lead, or epicardial lead (when available). There is a 71% to 81% rate of successful termination of atrial flutter/intraatrial reentrant tachycardia using a transesophageal electrode in pediatric or adult congenital patients. Because the atrial electrogram was easily obtained, and the patient has atrial pacing wires in place, the atrial wires should be used in this circumstance. Pacing protocols typically involve bursts of pacing starting at approximately 20 milliseconds faster than the clinical tachycardia, gradually increasing the rate and length of pacing bursts until the tachycardia is terminated (or the rate is felt to be "too fast"). Decremental pacing (gradually increasing the pacing rate while pacing until tachycardia termination is seen) and pacing drive trains (pacing faster than the tachycardia cycle length) followed by a premature beat ("extra stimulus") can also be used. To successfully terminate a reentrant atrial tachycardia, the pacing rate must be 10% to 20% faster than the atrial rate, rather than the ventricular rate. In general, the risks of overdrive pacing are low, although atrial fibrillation may ensue, especially with faster pacing rates. With esophageal pacing there is a small risk of esophageal damage. Neither adenosine nor vagal maneuvers would be expected to terminate the tachycardia in this case, as its circuit does not involve the atrioventricular node. These maneuvers might "unmask flutter waves" aiding in the diagnosis; however, in this case the diagnosis is already clear based on the results of the atrial wire study. Evaluation of a complete blood cell count would be a reasonable action if sinus tachycardia was suspected; however, the atrial rate alone (230 beats/min) excludes sinus tachycardia in an adult. Direct current cardioversion would likely be successful in terminating the tachycardia and would be the treatment of choice in an unstable patient (or perhaps in a patient without temporary wires). However, direct current cardioversion is more painful to the patient and has greater risks, such as dislodgement of support structures. The patient in this vignette has stable hemodynamics and temporary wires in place; therefore, overdrive pacing is the treatment of choice. PREP Pearls Overdrive pacing is a safe and effective way to terminate reentrant tachycardias in stable patients, especially in the postoperative setting and with temporary wires in place. The greatest risk associated with overdrive atrial pacing is induction of atrial fibrillation, especially when pacing at fast rates (approximately 200 milliseconds or less). Neither adenosine nor vagal maneuvers are expected to terminate intraatrial reentrant tachycardia/atrial flutter.

A 5-year-old girl is seen for evaluation of a murmur. She is asymptomatic. A maternal aunt has "a problem with one of her valves," but otherwise the family history is unrevealing. An electrocardiogram shows normal sinus rhythm, a broad, biphasic P wave, and possible left ventricular hypertrophy. A high-pitched, S1-coincident, pansystolic murmur near the apex is auscultated. Her parents are asked further questions about her medical history. They report that she was a colicky infant but did not have any hospitalizations or prolonged or unexplained febrile illnesses. Findings from echocardiography are shown (Video 1, Video 2, Video 3). What is the MOST LIKELY explanation for these findings? A.chordal elongation B.congenital valve malformation C.papillary muscle dysfunction D.valve perforation

B.congenital valve malformation Echocardiography demonstrates moderate to severe mitral regurgitation (MR) secondary to a cleft mitral valve for the patient in this vignette. The clinical presentation of MR is highly variable and depends on the underlying cause and presence of any associated lesions. Patients can be asymptomatic with a murmur or may have signs and symptoms of congestive heart failure early in life. The characteristic murmur is high pitched, S1 coincident, and pansystolic, and it is best heard at the left lower sternal border and apex and may radiate to the left axilla or back. If there is significantly increased diastolic flow across the mitral valve, a flow rumble may be appreciated. When severe, there could be a prominent P2 or a narrowly split or single S2 caused by the development of pulmonary hypertension. Midsystolic clicks may be heard if there is mitral valve prolapse. Electrocardiography is nonspecific. It may be normal when regurgitation is mild. Left atrial enlargement may be seen when there is more severe and chronic disease or if there is associated mitral stenosis. Echocardiography can be used to elucidate the underlying mechanism of MR. Globally, rheumatic mitral valve disease is a common cause, but it is infrequently seen in the United States. Acute rheumatic MR is associated with chordal elongation, mitral valve prolapse, and annular dilation. Chronic rheumatic MR is associated with chordal shortening, incomplete valve coaptation, and varying amounts of mitral stenosis. Mitral valve prolapse by echocardiogram is defined as systolic protrusion of the body of the leaflet beyond the annular plane, usually identified in parasternal long axis views. Congenital mitral valve anomalies are generally rare but tend to predominate in pediatric surgical series for repair of MR. These anomalies include isolated anomalies of the leaflets, tension apparatus, and papillary muscles as well as combination lesions with phenotypic variability in degree of regurgitation and/or stenosis. An isolated cleft in the anterior mitral valve leaflet is recognized by division of the anterior leaflet such that the normally bifoliate or "fish-mouth" mitral valve appears trifoliate or triangular. This cleft is directed towards the aortic outflow, as opposed to the residual zone of apposition in atrioventricular septal defects, which points towards the ventricular septum. The space between the cleft components produces the substrate for regurgitation. Repair consists of closing the cleft with sutures. Papillary muscle dysfunction may be secondary to ischemic insult, as can be seen in anomalous origin of the left coronary artery from the pulmonary artery (ALCAPA). Echocardiography may reveal echobright fibrotic changes of the papillary muscles and left ventricular endocardium, with or without mitral valve prolapse. Patients typically present in infancy with symptoms of angina (irritability, poor feeding), congestive heart failure, and an abnormal electrocardiogram. Mitral valve perforation or chordal rupture resulting in significant MR could be a consequence of endocarditis. These patients are frequently quite ill, with symptoms of heart failure and/or embolic phenomena, in addition to fever and bacteremia. PREP Pearls The murmur of mitral regurgitation is S1 coincident, high pitched, and holosystolic (sometimes described as blowing) at or near the apex. Echocardiography will confirm the diagnosis of mitral regurgitation and elucidate the underlying mechanism, which will help inform treatment strategy. Isolated mitral valve cleft is an anomaly of the anterior mitral valve leaflet and one cause of congenital mitral regurgitation.

A 12-month-old boy with Williams syndrome has been followed for moderate supravalvar aortic stenosis and mild branch pulmonary artery stenosis. His last echocardiogram was limited, so he is scheduled to undergo testing with general anesthesia, including echocardiography and computed tomography angiography of the chest and abdomen. The anesthesia team intubates the patient to ensure a stable airway and induces with inhaled anesthetic. About 10 minutes following induction, the patient develops ST segment changes and then acutely becomes bradycardic and hypotensive followed by pulseless electrical activity. Cardiopulmonary resuscitation is initiated, and after 15 minutes of resuscitation, he regains spontaneous circulation and is transferred to the intensive care unit. Electrocardiography is performed after arrival in the intensive care unit (Figure). The physiologic change caused by anesthesia and/or positive pressure ventilation that MOST likely precipitated the cardiac arrest is A.decreased heart rate B.decreased systemic vascular resistance C.increased intrathoracic pressure D.increased pulmonary vascular resistance

B.decreased systemic vascular resistance The patient in this vignette went into cardiac arrest while under general anesthesia and had evidence of myocardial ischemia on an electrocardiogram following the arrest (Figure ). The physiologic change that most likely precipitated the episode of cardiac arrest is decreased systemic vascular resistance. Anesthetic agents often lead to decreases in systemic vascular resistance, as well as other physiologic changes, which are poorly tolerated in patients with supravalvar aortic stenosis, as it contributes to decreased coronary perfusion. Supravalvar aortic stenosis is caused by a discrete area of narrowing at the level of the sinotubular junction or long segment narrowing of the ascending aorta. This type of left-sided heart obstruction is most commonly seen in patients with Williams syndrome or other types of elastin arteriopathies. Supravalvar aortic stenosis puts patients at risk for ischemia through multiple mechanisms. If obstruction is significant, it leads to the development of left ventricular hypertrophy, which increases myocardial oxygen demand. Oxygen delivery to the myocardium via coronary blood flow can be limited by multiple mechanisms. The abnormal elastin content of the aorta leads to decreased distensibility and elastic recoil, which limits coronary blood flow. The coronary artery tissue is also abnormal and may limit flow and autoregulation. The thickened tissue causing the supravalvar narrowing can also cause obstruction of the coronary ostia by adhering to the wall of the sinus. Ischemia may be subendocardial or transmural. At baseline, patients are able to maintain the coronary perfusion needed to meet the myocardial oxygen demand, but that balance is tenuous and any decrease in coronary perfusion or increase in demand may be poorly tolerated. The effects of anesthetic agents can lead to physiologic changes that disrupt this balance. Most anesthetic agents decrease systemic vascular resistance, which decreases coronary perfusion pressure and can lead to ischemia. Some agents can also depress myocardial function, while other agents can lead to tachycardia and increased myocardial oxygen demand. Intake restrictions prior to general anesthesia may also decrease preload. Care should be taken to avoid hypotension by maintaining preload, limiting agents that decrease systemic vascular resistance or depress myocardial function. Because of the high risk for ischemia and cardiac arrest during general anesthesia, patients with Williams syndrome and/or supravalvar aortic stenosis should receive care from anesthesiologists experienced in caring for similar patients, including availability of extracorporeal membrane oxygenation backup. Decreased heart rate may be seen in patients under general anesthesia, but this is less likely to lead to decreased coronary perfusion unless there is severe bradycardia. Some decrease in heart rate may actually decrease myocardial oxygen demand. Increased intrathoracic pressure is a result of positive pressure ventilation. It may decrease preload, but will also decrease afterload, leading to some decrease in myocardial oxygen demand. Increased pulmonary vascular resistance is unlikely to lead to the ischemic changes in this patient. PREP Pearls Supravalvar aortic stenosis is associated with abnormalities in coronary artery flow. Decreased systemic vascular resistance or increased myocardial oxygen demand can lead to ischemia in patients with supravalvar aortic stenosis Anesthetic agents can lead to physiologic changes that can cause ischemia in patients with supravalvar aortic stenosis.

You are called to the emergency department to consult on the case of a 4-year-old girl who has dilated cardiomyopathy with severely depressed systolic function. She has previously been followed at an outside institution, having had multiple admissions there. She has had several days of vomiting and diarrhea with decreased appetite. Her medications include carvedilol, digoxin, enalapril, furosemide, and spironolactone. On examination, she is irritable, but responsive. Her height is in the 40th percentile; her weight is in the 15th percentile. She has a heart rate of 120 beats/min, a respiratory rate of 36 breaths/min, and blood pressure of 85/60 mm Hg. Her lungs are clear. The cardiac examination reveals a laterally and inferiorly displaced point of maximal impulse. On auscultation, the rhythm is irregular. There is a normal S1 and a physiologically split S2. No murmurs are heard. There is an S3 at the apex. The liver is at the right costal margin. The extremities are lukewarm, with a capillary refill time of 3 seconds, with no clubbing, cyanosis, or edema. The cardiac monitor demonstrates frequent premature ventricular contractions. Her laboratory results are as follows: Laboratory Test Result Sodium 125 mEq/L (125 mmol/L) Potassium 2.8 mEq/L (2.8 mmol/L) Chloride 87 mEq/L (87 mmol/L) Calcium 9 mg/dL (2.25 mmol/L) Blood urea nitrogen 35 mg/dL (12.5 mmol/L) Creatinine 1.0 mg/dL (88.4 µmol/L) Of the following, the drug imparting the MOST immediate toxicity risk is A.carvedilol B.digoxin C.enalapril D.spironolactone

B.digoxin In the setting of hypokalemia, renal dysfunction, spironolactone, and carvedilol, the drug imparting the greatest immediate risk as a result of its toxic effects is digoxin. Hypokalemia increases the risk of digoxin-induced arrhythmias. The most classic is atrial tachycardia in association with atrioventricular block; however, premature ventricular contractions and ventricular tachycardia (particularly bidirectional ventricular tachycardia) may occur, as well as isolated variable degrees of atrioventricular block. Additional electrolyte disturbances that increase the risk of digoxin-mediated arrhythmias include hypomagnesemia and hypercalcemia. Hypoxia and acidosis also increase the risk. Spironolactone, carvedilol (in children), and renal insufficiency increase digoxin levels and, hence, the potential for digoxin toxicity. Therefore, when digoxin is used in this setting, monitoring digoxin levels is suggested. Current recommended levels for digoxin to minimize toxicity are 0.5 to 1.0 ng/mL in women and children and lower in men, 0.5 to 0.8 ng/mL. Previous "therapeutic levels" were 1 to 2 ng/mL, but the risk of adverse events at these levels has exceeded any benefit in studies involving adults. The primary reason to monitor levels is to be sure they are not high, not to aim for a specific "therapeutic" level. Digoxin is a sodium/potassium-ATPase inhibitor that inhibits the sodium pump in myocytes. When the sodium pump is inhibited, sodium temporarily increases within the cells, near the sarcolemma, resulting in a calcium influx via the sodium-calcium exchange mechanism; this enhances myocardial contractility but increases arrhythmia risk. Digoxin also activates the parasympathetic system, resulting in slowed atrioventricular conduction and prolongation of the atrioventricular refractory period. Conversely, digoxin inhibits the sympathetic system, as well as renin release in the kidney owing to decreased activity of the kidney sodium pump. Digoxin has a long serum half-life of approximately 1.5 days. Digoxin elimination is predominantly renally mediated; thus, with renal impairment, excretion decreases and levels rise. Based on the most recent adult heart failure guidelines, the use of digoxin is being recommended much less frequently in heart failure management and in rate control of tachyarrhythmias owing to its narrow therapeutic-to-toxic window and its potential for life-threatening side effects in these patients, who often have electrolyte abnormalities and renal injury. In studies in adults, it has not been shown to reduce mortality. It reduces recurrent hospitalizations in adult heart failure patients and is used in some patients with the goal of reducing symptoms and hospital admissions. The 2014 Guidelines for the Management of Pediatric Heart Failure from the International Society of Heart & Lung Transplantation do not recommend the use of digoxin in asymptomatic children who have a reduced ejection fraction (class I, evidence level C). The 2014 pediatric heart failure guidelines also state that digoxin can be considered for reducing symptoms in children with a low ejection fraction, with target levels of 0.5 to 0.9 ng/L, with careful monitoring of patients who are also receiving carvedilol or amiodarone and patients who have or are at risk of experiencing renal impairment (Class IIa, evidence level C). However, studies by Brown et al. (2016) and Oster et al. (2016) reported a reduced risk of interstage mortality in single-ventricle patients receiving digoxin. Therefore, the pediatric guidelines for digoxin use may undergo further evolution. Carvedilol would not impose an immediate risk, as it would not be expected to cause electrolyte abnormalities and its β-blocking effects could be helpful in the setting of ventricular ectopy. The concern regarding spironolactone at this time would be its ability to increase the potassium level, but this patient's potassium level is low, so its risk would not be immediate. Both spironolactone and carvedilol can increase digoxin levels, but the drug with the most toxic effect would be the digoxin itself. Enalapril could worsen renal function and elevate the potassium level in this scenario, but the potassium level is low at this time. Worsening renal function could further elevate the digoxin level, but this would not result in the most acute toxic effect. PREP Pearls Digoxin is a sodium/potassium-ATP inhibitor whose action results in increased calcium influx enhancing myocardial contractility. It also activates the parasympathetic nervous system and inhibits the sympathetic nervous system and renin release in the kidney. Digoxin has a narrow therapeutic window between beneficial and toxic effects. Hypokalemia, hypomagnesemia, hypercalcemia, hypoxia, and acidosis as well as renal dysfunction increase the risk of digoxin toxicity, resulting in arrhythmias and conduction disturbances.

A 1-month-old infant is noted to be tachycardic with an irregular rhythm at the her 1 month well-child visit. She was born by spontaneous vaginal delivery at term with no complications. She is feeding well, taking 3 to 4 oz of standard formula every 3 hours without tiring, respiratory distress, or sweating. There have been no concerns about color change, irritability, or lethargy. The patient's weight is 3.9 kg (birth weight, 3.2 kg). Her temperature is 36.9°C, heart rate is 200 beats/min and irregular, blood pressure is 71/42 mm Hg, and respiratory rate is 36 breaths/min. Examination findings are normal with the exception of the irregular rhythm. The electrocardiogram is shown in Figure 1 . Echocardiography (not provided) reveals a structurally normal heart with mildly depressed left ventricular function (shortening fraction, 27%; ejection fraction, 51%) with no distinct wall motion abnormalities. Of the following, what is the BEST medical management for this patient? A.adenosine B.digoxin C.direct current cardioversion D.verapamil

B.digoxin The patient described in this vignette has multifocal (or chaotic) atrial tachycardia. The electrocardiogram demonstrates a multifocal atrial tachycardia with intermittent aberrant conduction. Of the choices given, the best initial treatment option for this patient is digoxin. The use of digoxin may have some effect on the atrial rate itself but at a minimum should provide ventricular rate control. Given the patient's mildly depressed function, digoxin may enhance the recovery of function, primarily by its ventricular rate control. Although it does have mild inotropic properties, clinically in heart failure patients digoxin does not actually improve function. Multifocal atrial tachycardia can be diagnosed by surface electrocardiography according to the following classic criteria: At least 3 distinct P-wave morphologic findings (Figure 2) Varying PP, PR, and RR intervals (ie, irregular) Isoelectric baseline between P waves (ruling out atrial fibrillation) Tachycardia Multifocal atrial tachycardia is a rare arrhythmia in the pediatric population, accounting for less than 1% of pediatric supraventricular tachycardia or an overall incidence of approximately 0.02%. In adults, it is commonly associated with pulmonary or cardiac disease, and treatment is directed at the underlying comorbidity and rate control. In pediatrics, it is seen almost exclusively in neonates and infants. When present, it is often associated with acute respiratory illnesses (such as respiratory syncytial virus) or structural heart disease, although it may also be seen in otherwise healthy infants. Many patients are asymptomatic, although some have signs or symptoms suggestive of cardiovascular compromise, such as irritability or tachypnea. Depressed cardiac function (tachycardia-induced cardiomyopathy) is seen in approximately 25%. The exact mechanism of the arrhythmia is unclear but is either enhanced automaticity from multiple sites (or perhaps one site with several exit points) or triggered activity. The rhythm is generally self-limited (especially in structurally normal hearts), with patients typically outgrowing the condition by 6 to 12 months. In asymptomatic patients with structurally normal hearts and normal cardiac function, some physicians may advise no therapy; however, most would recommend treatment with an atrioventricular nodal blocker, such as digoxin (as in this vignette) or a β-blocker (with caution if ventricular function is depressed). Calcium channel blockers are used frequently in adults, although they should not be used in infants. If more complete control is desired, amiodarone, flecainide, and propafenone can be effective. Direct current cardioversion is ineffective given the mechanism of the tachycardia (triggered vs automatic; not reentrant). Adenosine is likewise ineffective because the tachycardia is not dependent on atrioventricular node conduction (although its use may allow more accurate evaluation of different P-wave morphologic findings). PREP Pearls Multifocal (or chaotic) atrial tachycardia is a rare arrhythmia in pediatrics, seen almost exclusively in neonates and infants, at times with respiratory illness or structural heart disease. Multifocal atrial tachycardia is diagnosed with at least 3 distinct P-wave morphologic findings; varying PP, PR, and RR intervals; an isoelectric baseline between P waves; and tachycardia. Multifocal atrial tachycardia is generally self-limited, resolving after 6 to 12 months. Initial treatment is typically with nodal blocking agents, such as digoxin or propranolol, although more complete control can be obtained with amiodarone, flecainide, or propafenone. Direct current cardioversion and adenosine are ineffective therapies for multifocal atrial tachycardia, although adenosine may aid in the diagnosis, by blocking the AV node and thus allowing recognition.

You are one of the non coordinating institutional principal investigators for a multicenter randomized placebo-controlled study. The study will evaluate the efficacy of drug X as compared to metformin in lowering serum glucose and hemoglobin A1C in patients with diabetes mellitus. At Thanksgiving, you learn from your brother-in-law that his company has invested millions of dollars in the development of drug X. You own no shares or interest in this company. He tells you that if drug X is a success, his company stands to triple, if not quadruple, its investment. You wish him much success with the endeavor but do not disclose that you are participating in the clinical trial to evaluate the drug's efficacy. Of the following, the BEST next step regarding your role in the study is to A.assign the role of principal investigator to another colleague B.disclose your role to the conflict of interest committee and continue as principal investigator C.disclose your role to the conflict of interest committee and step down as principal investigator D.do nothing and continue as principal investigator

B.disclose your role to the conflict of interest committee and continue as principal investigator A conflict of interest arises when a secondary interest provides incentive for an investigator to make a determination or judgment regarding a study outcome that is informed by the incentive versus the actual data. The presence of a conflict of interest, however, does not implicitly indicate the presence of wrongdoing on the part of the investigator. Each institution has some form of a conflict of interest office or review committee. When a conflict is realized, the first step is disclosure to that entity. The disclosure should include any of the following: compensation, equity interest, licensing agreement or royalties, and travel. Furthermore, it is the investigator's responsibility to disclose when the conflict arises. Annual updates should be made, but any new conflict should be reported within 30 days of its occurrence. The office then reviews the case to determine if the conflict is material or related. For research that is sponsored by the National Institutes of Health, material interest is defined as follows: Aggregate remuneration from a profit or nonprofit entity of at least $5,000 Equity in a publicly traded entity of at least $5,000 Holding equity interest in a non-publicly traded entity Travel reimbursed or paid for on behalf of an investigator other than by a government agency or an institution of higher education For research not sponsored by the National Institutes of Health: Aggregate enumeration from an entity of at least $10,000 Ownership in a publicly traded entity of at least $10,000 equity or greater than 5% Holding an ownership interest or equity in a non-publicly traded entity The office then determines if the material interest is related. In this process, answers to the following questions must be assessed: Are the activities of the entity related to the research? Could the research directly benefit the entity? Does the entity sponsor the research? Does the entity own or license intellectual property studied in the research? Finally, the committee must determine the ability of the individual to affect the research in areas including, but not limited to, study design, enrollment, data collection, data analysis, and reporting of data. Once these factors have been considered, a risk assessment can be made by the committee to determine a management plan based on the level of risk. For the case presented, the principal investigator (PI) has discovered that a family member has material and related interest in the research she is conducting. Her first step must be disclosure. As the PI, she is in a position to influence the study design, enrollment, data collection, data analysis, and reporting of data, placing her in a moderate- to high-risk conflict-of-interest situation that will require moderate to aggressive management by the conflict of interest office. Because the conflict is indirect, and because the PI holds no material interest herself and is not the coordinating PI for the multicenter trial, it is unlikely that she will need to excuse herself from the study nor would she need to assign the role of PI to another colleague. This would be true as long as she remains within the guidelines of the management plan. To do nothing and continue as PI, because her brother-in law's relationship to the company is coincidental but nonconsequential, would not follow guidelines. Consequently, the best action is for her to disclose her role to the conflict of interest committee and continue as PI. PREP Pearls Any perceived conflict of interest requires disclosure. Conflict of interest does not equate to wrongdoing. Conflict of interest does not always require removal of an individual from a research study.

You are called to the emergency department to evaluate a 13-year-old, 32.5-kg adolescent boy with chest pain. Approximately 1 month ago he was diagnosed with ulcerative colitis for which he has been on a steroid taper, and 2 weeks prior to admission he had influenza which was treated with supportive care. His steroid taper was weaned 3 days prior to admission, and the next day he woke with left-sided chest pain that has persisted. The morning of admission he was febrile and seen in the emergency department of another hospital, where an electrocardiogram (Figure 1) was obtained prior to transport to your institution. On arrival, he appeared nontoxic, and his physical examination findings were unremarkable with the exception of chest pain aggravated by recumbency. His current medications are: 100 mg Mesalamine twice a day (started 2 weeks prior to his steroid wean), 40 mg pantoprazole once a day, and 5 mg of prednisone twice a day. Laboratory Test Result Reference Range C-reactive protein, high sensitivity 976 mg/L (9,290 nmol/L) 1-30 mg/L (9.52-286 nmol/L) Erythrocyte sedimentation rate 120 mm/h 0-20 mm/h Platelet count 628 × 103/µL (628 × 109/L) 140-465 × 103/µL (140-165 × 109/L) Troponin T 0.46 ng/mL (0.46 µg/L) < 0.01 ng/mL (< 0.01 µg/L) Of the following, what is the BEST next step in this patient's medical management? A.administer 30 mg ketorolac intravenously every 6 hours B.discontinue 100 mg mesalamine orally twice a day C.increase prednisone to 1 mg/kg/d D.initiate 500 mg sulfasalazine orally every 8 hours

B.discontinue 100 mg mesalamine orally twice a day The sinus tachycardia, diffuse ST segment elevations, and PR segment depression on the electrocardiogram together with the elevated acute phase reactants and troponin levels are consistent with a diagnosis of acute myopericarditis. Inflammatory bowel disease is associated with the presence of nongastrointestinal illnesses, particularly during flares of the underlying disorder. Patients with inflammatory bowel disease, particularly patients with ulcerative colitis, develop myocarditis or pericarditis during the course of their disease. The causes are multifactorial and include diffuse inflammatory response, viral agents, and drug reactions. Drug reactions, particularly to 5-aminosalicylic acid (5-ASA) compounds, such as mesalamine and sulfasalazine, occur when administered orally or rectally. The specific mechanism by which these medications result in myocardial and particularly pericardial inflammation has not been determined; but, several mechanisms have been postulated, including direct toxicity, allergic reaction, humoral immune response, and cell-mediated hypersensitivity. Pericarditis associated with 5-ASA is typically seen within a few weeks after the initiation of therapy, as occurred in the patient in this vignette. Improvement in clinical symptoms of pericarditis following cessation of 5-ASA compounds has been widely reported, as have recurrence and progression of disease with a change in therapy to alternative 5-ASA compounds. Thus, the change in medical management most likely to result in resolution of this patient's myopericarditis is the discontinuation of mesalamine. Using sulfasalazine as an alternative therapy should be avoided because it may result in relapse and even the development of more hemodynamically significant disease. Administration of ketorolac would not be appropriate for this patient because the use of nonsteroidal anti-inflammatory drugs should at a minimum be done guardedly and in many cases is contraindicated in the setting of inflammatory bowel disease. Low-dose prednisone (0.2-0.5 mg/kg/d) is as efficacious as high-dose steroids in the treatment of pericarditis, while reducing the risk of serious steroid side effects including cushingoid syndrome, recurrence rate, and disease-related hospitalizations. Patients treated with higher-dose steroids also have lower event-free survival (with events including cardiac tamponade and the development of constrictive pericarditis). Therefore, maintaining this patient's prednisone dose at 5 mg twice a day (0.3 mg/kg/d) would be appropriate, rather than increasing it to 1 mg/kg/d. PREP Pearls Patients with inflammatory bowel disease, particularly those with ulcerative colitis, may develop myocarditis or pericarditis during the course of their disease. The use of 5-aminosalicylic acid compounds in the treatment of inflammatory bowel disease may be associated with the development of pericarditis or myocarditis, typically within a few weeks of initiation of therapy. Low-dose prednisone is as efficacious as high-dose steroids in the the treatment of pericarditis while reducing the risk of serious steroid side effects, including Cushing syndrome, recurrence rate, and disease-related hospitalizations.

The high-risk obstetrician has diagnosed ectopia cordis with a small omphalocele with intestinal contents in a fetus at 23 weeks of gestation. In addition to the ectopia cordis, fetal ultrasonography also shows a midline supraumbilical abdominal wall defect, a defect of the lower part of the sternum, a deficiency of the anterior diaphragm, and an intracardiac defect. Of the following, the MOST likely intracardiac defect associated with ectopia cordis is A.double inlet left ventricle B.double outlet right ventricle C.Ebstein's anomaly D.hypoplastic left heart syndrome

B.double outlet right ventricle The fetus in this vignette has ectopia cordis, diaphragmatic deficiency without hernia, and a small omphalocele. Of the choices given, ectopia cordis is most commonly associated with conotruncal defects such as double outlet right ventricle. Approximately 95% of newborns with ectopia cordis have associated intracardiac anomalies, with a ventricular septal defect being the most common in all series. The other common cardiac defects include: atrial septal defects; conotruncal defects, such as double outlet right ventricle, tetralogy of Fallot, and pulmonary atresia; and ventricular diverticulum. Less common intracardiac defects include: atrioventricular septal defect, single ventricle, transposition of the great vessels, mitral valve anomalies, double inlet left ventricle, Ebstein's anomaly, and hypoplastic left ventricle. Ectopia cordis can also be associated with pentalogy of Cantrell, which consists of a defect in the supraumbilical abdominal wall, a defect of the lower sternum, deficiency of the diaphragm, a defect in the diaphragmatic pericardium, and ectopia cordis with intracardiac defect. Some morphologists consider this syndrome to be a thoracoabdominal type of complete ectopia cordis. The associated noncardiac defects in patients with ectopia cordis include midline defects such as cleft palate, cleft lip, absence of corpus callosum, diastasis recti, diaphragmatic hernia, and omphalocele. Ectopia cordis is lethal if surgical intervention is not feasible. It is a rare congenital anomaly with an incidence of 5 to 7 per million live births. Depending on the location of the heart and the extent of the body wall defects, it can be classified as cervical, thoracic, thoracoabdominal, or abdominal. The cervical type of ectopia cordis, in which the heart is retained in its embryonic position in the neck, has almost no reported survivors. Approximately 90% of infants with ectopia cordis die; however, with advances in fetal imaging and surgical techniques, and a multidisciplinary team approach, there are increasing numbers of case reports of survivors. Thoracoabdominal defects generally have better prognosis with staged surgical repair. In complete ectopia cordis, there is a deficiency of the structures formed from the somatic mesoderm including the sternum, parietal pericardium, diaphragm, and anterior abdominal wall, in addition to the intracardiac defects. Partial types of ectopia cordis can present with a sternal cleft without a complete deficiency and may have intact pericardial layers. Partial ectopia cordis has a better prognosis. The surgical repair and outcome of patients with ectopia cordis depends on the variability and extent of the intracardiac and extracardiac defects. The initial repair includes separation of the peritoneal and pericardial cavities, coverage of the midline defect by prosthetic reconstruction of the chest wall, and repair of the associated abdominal defects. In complete ectopia, the heart can be eventually covered by mobilization of a latissimus dorsi free flap and skin autotransplant. After appropriate growth of the thoracic cavity and lungs, the intracardiac anomalies are repaired at a later stage, and the heart is returned to the thoracic cavity. A multidisciplinary approach is key to the successful repair of this complex defect. PREP Pearls Approximately 95% of newborns with ectopia cordis have associated intracardiac anomalies. Common defects in ectopia cordis include: ventricular septal defects; atrial septal defects; conotruncal defects, such as double outlet right ventricle, tetralogy of Fallot, and pulmonary atresia; and ventricular diverticulum.

A 19-year-old woman is seen for routine outpatient follow-up. You have known her since birth when she was diagnosed with double inlet left ventricle. As a neonate, she underwent a Damus-Kaye-Stansel operation with a Blalock-Taussig shunt. Her most recent surgery was at age 4 years when she underwent a lateral-tunnel fenestrated Fontan operation. She has been maintained on aspirin (81 mg daily) as well as lisinopril (10 mg daily) since her Fontan operation. She desires to become pregnant and presents for prenatal counseling. She has a heart rate of 100 beats/min and a resting oxygen saturation of 92%. She has a well-healed midline sternotomy, a grade 2/6 systolic ejection murmur at the base, and a single S2. There is subtle digital clubbing but the remainder of her examination findings are normal. Electrocardiography (Figure 1) and echocardiography were performed (Video 1 Video 2 , Video 3). Of the following findings, which would be MOST important to address prior to her becoming pregnant? A.bulboventricular foramen restriction B.electrocardiogram findings C.resting oxygen saturation D.ventricular function

B.electrocardiogram findings Pregnancy in patients with a functionally univentricular heart who have had a Fontan operation is complicated and fraught with potential risks for both mother and fetus. There is still debate in the literature and adult congenital heart disease community about the appropriateness of pregnancy in this group, yet many patients do wish to become pregnant and certainly there is a need for a thoughtful discussion and consideration of risks prior to pregnancy. From a structural and historical standpoint, there are a number of factors that place a woman at higher risk for an adverse outcome. In the CARPREG (Cardiac Disease in Pregnancy) study by Siu et al, there were 4 primary risk factors for maternal cardiac complications: prior cardiac event or arrhythmia, NYHA (New York Heart Association) class II symptoms (www.heart.org) or cyanosis (saturation less than 90%), left heart obstruction, or systemic ventricular dysfunction. Having any one of these factors alone prior to pregnancy was equal to a 27% risk of maternal cardiac complications. In this vignette, the patient has evidence of intra-atrial reentrant tachycardia with 2:1 conduction as seen on the electrocardiogram (Figure 2). It would be imperative to address this prior to considering pregnancy. Her mild baseline cyanosis does not meet the cutoff commonly cited for concern. There is some evidence of turbulence at the bulboventricular foramen or interventricular communication as evidenced in Video 3, although the echocardiogram shows no significant obstruction with peak velocity of 2 m/s (16 mm Hg). A cutoff of 30 mm Hg was used in the CARPREG study as a peak gradient for significant ventricular outflow obstruction. She has normal ventricular function as is evident by Video 1. In counseling a patient with complex congenital heart disease who is considering pregnancy, it is important to address current medication type, use, and cautions and/or contraindications in pregnancy. In this case, angiotensin-converting enzyme inhibitors including lisinopril are contraindicated and the recommendation is to avoid this class of drugs as they have been associated with teratogenicity. It is particularly important to have this conversation prior to pregnancy, because medications may cause issues within the first trimester of pregnancy, potentially before some women realize they are pregnant. Aspirin is considered safe to continue during pregnancy. It is used regularly in women with preeclampsia, and in this patient, would be important to avoid clotting given the prothrombotic state of pregnancy. Finally, it would be important to have a discussion in regards to the genetics of congenital heart disease and risk of fetal transmission. It would be critical for a patient such as the patient in this vignette to be followed jointly by an adult congenital heart disease cardiologist and a maternal-fetal medicine specialist. PREP Pearls Counseling a patient with congenital heart disease about pregnancy risks requires a review of current medications as well as consideration of known structural or historical elements associated with complications. Counseling should begin with a review of medications to identify potential teratogens in pregnancy. In an adult Fontan patient with a heart rate of 100 beats/min or greater, it is important to rule out intra-atrial reentrant tachycardia.

A 17-year-old adolescent girl with type 1 diabetes mellitus is brought to your cardiology office by her mother. The patient's insulin is managed by her family practice physician on a sliding scale. She has not seen an endocrinologist. She is overweight and admits to eating poorly and often skips checking her blood glucose. Her last hemoglobin A1C level was 11.5%. Her mother and younger sister are thin and have a normal lipid profile. Her father is obese and has severe type 2 diabetes mellitus; he has had 2 coronary stents placed at 40 and 42 years of age. The patient's family practitioner placed her on the CHILD-2-TG (Cardiovascular Health Integrated Lifestyle Diet-2-TG) diet 6 months ago for a low-density lipoprotein cholesterol level of 168 mg/dL (4.4 mmol/L) and triglyceride level of 398 mg/dL (4.5 mmol/L). You obtain 2 fasting lipid profiles 3 weeks apart, and you average the results: Laboratory Test Average Result Total cholesterol 322 mg/dL (8.3 mmol/L) Low-density lipoprotein cholesterol 152 mg/dL (3.9 mmol/L) Triglyceride 1,020 mg/dL (11.5 mmol/L) High-density lipoprotein cholesterol 22 mg/dL (0.6 mmol/L) Of the following, the BEST next step in the management of this patient is to begin A.a bile acid sequestrant B.fenofibrate C.gemfibrozil and pravastatin D.omega-3 fish oil, 2 g/d

B.fenofibrate The patient in this vignette is at significant risk for cardiovascular disease because of familial risk and type 1 diabetes mellitus. The first step in approaching this patient is reviewing her lipid panel. The Table shows the normal and abnormal cholesterol levels for children and adolescents. This patient has a high total cholesterol level, high low-density lipoprotein (LDL) level, high triglyceride level, and low high-density lipoprotein (HDL) level. The most common cause of low levels of HDL cholesterol is associated with insulin resistance. Insulin resistance results in an influx of free fatty acids to the liver, overproduction of very low-density lipoprotein with an elevated triglyceride level and increased small LDL particles. In patients with diabetes, lowering the LDL cholesterol level is the main goal in order to decrease cardiovascular risk in adulthood. This patient's LDL cholesterol level of 152 mg/dL (3.9 mmol/L) is misleading. The Friedewald formula cannot be used in this patient because it underestimates the LDL cholesterol level when the triglyceride level exceeds 400 mg/dL (4.5 mmol/L). Therefore, her LDL value is not accurate and may actually be higher. Her calculated non-HDL cholesterol level of 290 mg/dL (7.5 mmol/L) is also high. Of greatest concern is her triglyceride level of greater than 1,000 mg/dL (11.3 mmol/L). She is at significantly increased risk of pancreatitis and therefore the best next management step is to target her triglyceride level first. Initiation of fenofibrate would be the optimal choice and will lower her triglyceride level by 20% to 50%. The 2 fibrates currently available in the United States are fenofibrate and gemfibrozil. Myopathy is the main concern with fibrates. As a monotherapy, fibrates have a 5-fold higher risk of myopathy than statins. Therefore, muscle enzyme levels must be closely monitored. The use of gemfibrozil together with a statin is contraindicated because there is a 33-fold higher risk of myopathy as compared to using fenofibrate alone. Gemfibrozil increases the statin level and thus increases myopathy risk. If this patient's triglyceride level was to read less than 500 mg/dL (5.7 mmol/L) and her LDL was normal, 2 grams per day of fish oil might be a good choice as a first-line therapy. A bile acid sequestrant is contraindicated because marked exacerbations of hypertriglyceridemia may occur with its use. In diabetic patients, hyperglycemia increases the risk of arteriosclerosis by raising levels of cholesterol and enriched apolipoprotein B. While type 1 diabetes mellitus often presents in childhood, studies show 8% to 50% of new adolescent diabetics are now diagnosed with type 2 diabetes mellitus. Early identification of these at-risk patients is critical since 40% of all diabetic patients aged 35 years and older are diagnosed with cardiovascular disease. In summary, this patient's triglyceride level of greater than 1,000 mg/dL (11.3 mmol/L) places her at high risk for pancreatitis. Therefore, fenofibrate is the best management option. Once her triglyceride level is less than 400 mg/dL (4.5 mmol/L), her LDL cholesterol level may be calculated accurately. Her LDL and HDL cholesterol goals to reduce cardiovascular risk can then be addressed. Referring her to an endocrinologist for better control of her blood glucose level, as well as ensuring she does not have hypothyroidism, will also help to decrease the triglyceride level. PREP Pearls Triglyceride levels greater than 400 mg/dL (4.5 mmol/L) may result in underestimation of low-density lipoprotein levels. Triglyceride levels greater than 1,000 mg/dL (11.3 mmol/L) put patients at an increased risk of pancreatitis. The combination of gemfibrozil and a statin increases the risk of rhabdomyolysis. Fish oil is a valuable adjunct in the treatment of moderately elevated triglyceride levels.

A 22-year-old woman underwent repair of a primum atrial septal defect and cleft mitral valve when she was 3 years of age. She has had residual mitral regurgitation that has been audible since she was 12 years of age. She comes to the emergency department today with a 6-week history of myalgias, fatigue, and fevers as high as 38°C. About 1 month ago, she saw her primary care physician who thought her throat looked red and prescribed an antibiotic. She seemed to get better for 1 or 2 weeks, but now has recurrent fevers, still does not feel well, and has developed tender red spots on the pad of her left long finger and on the pad of her fourth right toe. She also noticed some discoloration on her fingernails. Her family history includes a sister with systemic lupus erythematosus. The patient looks tired. She has a temperature of 38°C, respiratory rate of 18 breaths/min, blood pressure of 110/75 mm Hg, and heart rate of 110 beats/min. The rest of her examination reveals: Hyperdynamic precordium Normal S1 and S2 Regurgitant 3/6 murmur at the apex to axilla Silent diastole The liver edge is not palpable, but the spleen is 2 cm below the left costal margin. Extremities show no clubbing cyanosis or edema. Many of the nail beds on her fingers have dark streaks toward the tips (figure 1). An erythematous tender lesion can be seen on the left long finger (figure 2) and fourth right toe. She also has scattered petechiae on her feet. Laboratory data are shown: Laboratory Test Result White blood cell count 35,000/μL (35 × 109/L) Hemoglobin 10 g/dL (100 g/L) Hematocrit 30% (0.30) Platelet count 160 × 103/μL (160 × 109/L) C-reactive protein 65 mg/L (619 nmol/L) Erythrocyte sedimentation rate 140 mm/h Rheumatoid factor Positive Antinuclear antibodies Negative Blood cultures and echocardiography findings are pending. Of the following, the MOST likely diagnosis is A.acute myelogenous leukemia B.infectious endocarditis C.Libman-Sacks endocarditis D.systemic rheumatoid arthritis

B.infectious endocarditis Infectious endocarditis is the most likely diagnosis for the patient in this vignette, based on her history of repaired congenital heart disease with a residual lesion, the symptom history, and physical examination and laboratory findings. The diagnosis cannot be verified until the results of blood cultures and echocardiography are available. Infectious endocarditis is the most common cause of Osler nodes and Janeway lesions, but they may also occur in Libman-Sacks endocarditis, which is a noninfectious endocarditis associated with systemic lupus erythematosus. This patient's history and negative antinuclear antibody profile make Libman-Sacks endocarditis less likely. The skin manifestations described in this vignette would not be expected in acute myelogenous leukemia. Rheumatoid nodules, as seen in rheumatoid arthritis, are typically larger and not painful. They are usually associated with the affected joints. Skin manifestations of endocarditis include splinter (periungual) hemorrhages (Figure 1) in the nail bed, Osler nodes (Figure 2), and Janeway lesions (Figure 3). There has been some controversy as to whether Osler nodes represent vasculitis or infectious microemboli from the vegetation. Osler nodes have been reported in sepsis without endocarditis and distal to infected intravascular grafts. Janeway lesions are a result of microemboli from the intracardiac vegetation. Skin manifestations of endocarditis were common in the preantibiotic era, occurring in 40% to 90% of cases, but current literature reports an incidence of 3% to 5%. They are infrequent in children and considered minor criteria in the modified Duke criteria (Suggested Reading 3). PREP Pearls Skin manifestations are uncommon in pediatric endocarditis. Infectious endocarditis is the most common cause of Osler nodes and Janeway lesions.

A 12-year-old girl is seen in cardiology clinic to re-establish care. She underwent device closure of a large patent ductus arteriosus at 6 months of age. She has no concerns and has not had any significant medical issues since her last visit 5 years ago. She has a grade 2/6 systolic ejection murmur that is loudest at the left upper sternal border and the left side of the back. Echocardiography is performed (Video 1, Video 2, Video 3). What is the MOST likely explanation for the murmur? A.descending aortic stenosis B.left pulmonary artery stenosis C.physiologic pulmonary flow murmur D.residual patent ductus arteriosus

B.left pulmonary artery stenosis The most likely explanation for the murmur in this patient is device-associated left pulmonary artery stenosis. Left pulmonary artery stenosis has been reported with multiple devices used for patent ductus arteriosus (PDA) occlusion. Patent ductus arteriosus occlusion devices often extend into the pulmonary artery. The proximity of the pulmonary arterial end of the PDA to the left pulmonary artery origin may result in a portion of the device partially occluding the left pulmonary artery origin. This complication is more likely to occur in small patients with large PDAs. The stenosis is usually mild and unlikely to be significant enough to warrant intervention. Spontaneous improvement over time has been noted. Descending aortic stenosis has also been reported; however, the echocardiographic data are not compatible with that complication (Video 3). Residual shunts after device placement are rare. Residual shunts 1 year after device placement have been reported in the 2% to 3% range. The data provided in this vignette (Video 1 and Video 2) do not support a residual shunt. A physiologic pulmonary flow murmur would not be expected to radiate to the back at this age. PREP Pearls Complications after percutaneous patent ductus arteriosus device occlusion are rare. Left pulmonary artery stenosis and aortic obstruction have been reported after patent ductus arteriosus device closure. Left pulmonary artery stenosis, if present, after patent ductus arteriosus device closure is usually mild.

An 11-year-old boy has palpitations and fatigue. He reports no chest pain or syncope and has no history of medication or illicit drug use. Electrocardiography (Figure 1) is performed. The MOST likely mechanism of his tachycardia is A.orthodromic supraventricular tachycardia B.permanent junctional reciprocating tachycardia C.supraventricular tachycardia secondary to Wolff-Parkinson-White syndrome D.typical atrioventricular nodal reentrant tachycardia

B.permanent junctional reciprocating tachycardia The electrocardiogram (Figure 2) of the patient in this vignette demonstrates a narrow complex long RP tachycardia. Narrow complex tachycardia can be subdivided depending on the RP relationship. To determine the RP relationship, a line is drawn at the midpoint of the R-R interval. A P wave that falls in the first half of the R-R interval indicates a short RP tachycardia, and a P wave that falls in the second half indicates a long RP tachycardia. In the electrocardiogram provided, there is a long RP tachycardia because the P wave falls into the second half of the R-R interval (ie, the RP interval is greater than the PR interval). The differential diagnosis for long RP tachycardia includes sinus tachycardia, atypical atrioventricular nodal reentry tachycardia, atrial ectopic tachycardia, and permanent junctional reciprocating tachycardia. Permanent junctional reciprocating tachycardia is a narrow complex re-entrant tachycardia that is caused by an accessory pathway with decremental conduction. The circuit in these patients is down the atrioventricular node and up the decrementally conducting accessory pathway. The fact that the pathway has decremental conduction leads to the longer conduction from ventricle back to the atria and thus the longer RP. The classic electrocardiographic findings in permanent junctional reciprocating tachycardia are deeply negative P waves in the inferior leads (II, III, and aVF). Patients with permanent junctional reciprocating tachycardia tend to have incessant tachycardia and usually present with signs and symptoms of heart failure secondary to tachycardia-induced cardiomyopathy. Permanent junctional reciprocating tachycardia can be difficult to control with antiarrhythmics and can often require an ablation as definitive therapy. Orthodromic supraventricular tachycardia, typical atrioventricular nodal reentry tachycardia, and supraventricular tachycardia seen in patients with Wolff-Parkinson-White syndrome are short RP tachycardias. In concealed accessory pathways and manifest accessory pathways, the circuit is down the atrioventricular node and up the accessory pathway. Given that the pathways in these scenarios do not have decremental properties, the RP relationship is short. In typical atrioventricular nodal reentry tachycardia, the circuit is down the slow pathway and up the fast pathway leading to a short RP tachycardia. By contrast, in atypical atrioventricular nodal reentry tachycardia, the circuit is down the fast pathway and up the slow pathway and thus is a long RP tachycardia. PREP Pearls Differential diagnosis for long RP tachycardia includes sinus tachycardia, atypical atrioventricular nodal reentry tachycardia, atrial ectopic tachycardia, and permanent junctional reciprocating tachycardia. Deeply negative P waves in inferior leads are classic finding in permanent junctional reciprocating tachycardia. Narrow complex tachycardia can be subdivided by the RP relationship.

You are evaluating an infant with intestinal malrotation. You explain to the cardiology fellow that this condition can be associated with abnormalities of atrial situs. The fellow asks you about the cardiac anatomic features that would help identify normal atria. Of the following, the MOST defining feature of the morphologic right atrium is A.a connection to the superior vena cava B.the crista terminalis C.the flap valve of the foramen ovale D.a narrow-based appendage

B.the crista terminalis The morphologic right and left atria are defined by their anatomic features and not by their relative positions. Determination of atrial morphology is important in identifying atrial situs. Abnormalities in atrial situs may lead to a diagnosis of heterotaxy syndrome, prompting the evaluation of multiple organ systems for associated anomalies. Understanding the atrial anatomy is critical for interventional cardiologists and electrophysiologists. The morphologic right atrium has features that distinguish it from the morphologic left atrium. The crista terminalis is a muscle bar that divides the sinus venosus portion of the right atrium from the right atrial appendage. This structure does not exist in the left atrium, and is thus an identifying feature of the right atrium. The right atrial appendage is broad-based and triangular in shape. In the normal heart, it is anterior relative to the left atrial appendage. The left atrial appendage is narrow-based or fingerlike. Anatomy of the atrial appendages can be determined with echocardiography or other types of cardiac imaging. Identifying the appendages is often the easiest way to identify the atrial morphology, but in the case of atrial situs ambiguous or atrial dilation, determination may not be possible. The sinus venosus portion of the right atrium is smooth-walled and ends at the crista terminalis. Distal to the crista terminalis, the right atrial free wall is trabeculated with pectinate muscles that extend to the valve annulus. The left atrium is smooth-walled; the only trabeculations are within the left atrial appendage. The venous connections to the atria are not reliable indicators of atrial morphology. Although the right-sided superior vena cava drains to the right atrium in most cases, a left-sided superior vena cava can drain to the left atrium. The inferior vena cava can be interrupted, having no direct connection to either atrium. The suprahepatic portion of the inferior vena cava and coronary sinus drainage, when present, identifies the morphologic right atrium. The pulmonary venous return, although usually draining to the left atrium, can drain directly to the right atrium or various other structures in anomalous pulmonary venous return. The atrial septal features can also help to identify atrial morphology. The limbus of the fossa ovalis is a feature of the right atrial septal surface. The flap of the foramen ovale is found on the left atrial septal surface. PREP Pearls The crista terminalis, pectinate muscles, and a broad-based appendage are defining features of a morphologic right atrium. A narrow-based appendage and smooth-walled surface are features of the morphologic left atrium. Venous connections are not defining features of atrial morphology.

A 16 yo female with idiopathic dilated cardiomyopathy diagnosed 4 years ago returns for follow up having missed 2 prior appointments. Her last ejection fraction on echocardiogram was 42%. Today she is complaining of increased shortness of breath on exertion, nausea, decreased appetite and a 5 pound weight gain over the last month or so. She is sleeping on 3 pillows at night. She reports she usually remembers to take her medicine, but can't say how often she forgets. Her medications include enalapril, carvedilol, furosemide and spironolactone. On physical exam is notable for: height 5'4", weight 180 lb, BMI 31. HR 108 bpm, RR 24, BP 108/72. Neck: + JVD, Chest/Lungs: no retractions. Decreased breath sounds posteriorly, right greater than left. Cardiac: PMI difficult to find. Distant heart sounds. Normal S1 and narrowly split S2. + S3 and S4. No murmurs heard. Abdomen: Striae present. Liver edge difficult to palpate. By percussion ~ 4-5 cm below the RCM. Extremities: 1+ pitting edema. Laboratory data: ECG demonstrates sinus rhythm CXR demonstrates cardiomegaly with small bilateral pleural effusions right greater than left with mild interstitial edema. Echocardiogram: The left ventricular end diastolic dimension is 7.4 cm; end systolic dimension is 6.0 cm with a shortening fraction of 19% and an ejection fraction of 30%. BNP (brain natriuretic peptide): 50 (normal < 100 PG/ml), creatinine 1.5 (normal 0.12-1.06 mg/DL), Hgb 8.2 (13-16 G/DL), HCT 28.8 (37-49%) The most likely cause of the BNP value is: A. Anemia B. Female gender C. Obesity D. Renal insufficiency

C. Obesity Obesity is a common cause of unexpectedly low BNP levels in patients in heart failure. The definitive mechanism/mechanisms for this are unknown, but in part postulated to result from visceral fat expansion which can increase the clearance of active natriuretic peptides due to increased expression of clearance receptors on adipocytes. However additional cardiac endocrine factors are thought to be involved. Answers A, B and D are incorrect because they are associated with elevated BNP levels. Anemia and other high output states, including sepsis, hyperthyroidism and cirrhosis can all cause an elevated BNP resulting from ventricular wall stress, in the absence of an acute heart failure picture. Renal dysfunction contributes to elevation of the BNP due to decreased clearance. After puberty healthy females have statistically higher BNPs as a group than prepubertal girls or boys (pre or post puberty), but in the pediatric age range, remain much less than 100 PG/ml normally. Healthy children beyond early infancy have a BNP less than 41. Of note both newborns and older adults have higher BNPs than the generally accepted norm. PREP Pearls Obesity is a common cause of unexpectedly low BNP Normal BNP levels are higher in post pubertal females than prepubertal females and male children pre or post puberty, but remain within normal values in healthy females in the pediatric age range Renal dysfunction and high output states including anemia, sepsis, hyperthyroidism and cirrhosis increase BNP levels

A 12-month-old boy with trisomy 21 who has had surgical repair of an atrioventricular septal defect underwent cardiac catheterization for assessment of pulmonary hypertension. He has been asymptomatic. He is on thickened feeds because of aspiration. His only medication is polyethylene glycol. He has a heart rate of 134 beats/min, respiratory rate of 26 breaths/min, blood pressure of 100/65 mm Hg, and pulse oximetry reading of 92% in room air. No murmurs are noted. His most recent echocardiogram showed a small secundum atrial septal defect with predominantly left-to-right shunting and no residual ventricular septal defect. There is mild left-sided atrioventricular valve regurgitation without evidence of stenosis. There is mild right-sided atrioventricular valve regurgitation with a peak instantaneous velocity of 3.5 m/s. The right ventricle is mildly dilated with mildly decreased function. The ventricular septum is flattened. Left ventricular function is normal. Catheterization data are shown: Location Oxygen Saturation (%) Pressure (mm Hg) Superior vena cava 71 8 (mean) Right atrium 68 8 (mean) Right ventricle 67 46/10 Main pulmonary artery 72 45/19, 27 Right pulmonary artery 72 43/18, 26 Left pulmonary artery 70 41/18, 25 Right pulmonary capillary wedge — 10 Left pulmonary capillary wedge — 16 Left atrium 93 9 (mean) Left ventricle 92 75/11 Femoral artery 91 79/41, 50 Which of the following angiograms BEST represents the diagnosis? A. Figure 1. Choice Figure 1 B. Figure 2. Choice Figure 2 C. Figure 3. Choice Figure 3 D. Figure 4. Choice Figure 4

C. Figure 3. Choice Figure 3 The boy in this vignette has left lower pulmonary vein stenosis, which is demonstrated in Figure 3. Pulmonary vein stenosis may be a diffuse process with uniformly hypoplastic pulmonary veins. Alternatively, it may be localized at the junction of the vein with the left atrium. In this case, the stenosis is localized (Figure 5). Pulmonary vein stenosis should be considered in patients with unexplained pulmonary hypertension. Localized pulmonary vein stenosis may be the only diagnosis, but it has been associated with other cardiac lesions. When associated with other anomalies, left-sided obstructive lesions are most common, however other lesions including atrioventricular septal defects have been identified. In this vignette, the elevated left pulmonary capillary wedge pressure is consistent with a diagnosis of unilateral pulmonary vein stenosis. Despite the presence of an atrial septal defect, the oximetry data does not support a large atrial level shunt as an explanation for the pulmonary hypertension. Figure 1, Figure 2, and Figure 4 show lesions that are associated with pulmonary hypertension but are not consistent with the data in this vignette. Figure 1 demonstrates severe left atrioventricular valve regurgitation after surgical repair of an atrioventricular septal defect. Figure 2 demonstrates partial anomalous pulmonary venous return of the left upper pulmonary vein to the innominate vein. Figure 4 is a left ventricular angiogram in an unrepaired atrioventricular septal defect. PREP Pearls Pulmonary vein stenosis is a rare cause of pulmonary hypertension. Pulmonary vein stenosis can be an isolated diagnosis or associated with other cardiac anomalies, most commonly left-sided obstructive lesions, however it can also be seen in atrioventricular septal defects.

You are caring for a 5-month-old infant with hypoplastic left heart syndrome in the intensive care unit 6 hours after she underwent a superior cavopulmonary anastomosis. She remains pharmacologically sedated and is receiving mechanical ventilation with synchronized intermittent mandatory ventilation using pressure-regulated volume control with the following settings: a fraction of inspired oxygen of 1.0; tidal volume of 8 mL/kg; positive end-expiratory pressure (PEEP) of 5 cm H2O; respiratory rate of 18 breaths/min; and pressure support of 5 cm H2O. Her oxygen saturations have been persistently 66% to 68%, and her most recent arterial blood gas results are as follows: pH, 7.48; partial pressure of carbon dioxide, 30 mm Hg (4.2 kPa); partial pressure of oxygen, 26 mm Hg (3.4 kPa); and base deficit, -1. A chest radiograph demonstrates a small right-sided pleural effusion but otherwise clear lung fields bilaterally. Her central venous pressure measured by way of an internal jugular venous catheter is 13 mm Hg and her hemoglobin level is 13.4 g/dL (134 g/L). Her urine output has been 1.5 mL/kg per hour since her return from the operating room. A bedside echocardiogram demonstrates normal single right ventricular function with trace tricuspid valve regurgitation, and mild neo-aortic valve insufficiency. Of the following, the MOST appropriate immediate intervention to improve her oxygen saturation would be A. 10 mL/kg crystalloid fluid bolus B. 15 mL/kg of packed red blood cells C. decreasing her ventilator rate to 12 breaths/min D. increasing the PEEP to 10 cm H2O E. initiating inhaled nitric oxide

C. decreasing her ventilator rate to 12 breaths/min Permissive hypercapnia has been shown to be helpful in patients with low saturations after superior cavopulmonary anastomosis. The response of the cerebral vasculature to hypercarbia is vasodilation. With increased cerebral blood flow, venous return via the superior vena cava increases. This increases venous blood flow into the pulmonary arteries and potentially increases oxygen saturation levels. This patient's partial pressure of carbon dioxide (Pco2) at 30 mm Hg (4.2 kPa) is low, in the setting of an alkalotic pH. The goal in these patients would be to normalize both pH and PCO2. By lowering the ventilator rate, her Pco2 will rise and indirectly increase blood flow to her pulmonary circulation. In addition, weaning of the ventilator rate will facilitate more rapid extubation and transition to spontaneous breathing, which would also improve her oxygen saturation levels. Following cavopulmonary anastomosis, blood flow to the pulmonary circulation is passive and dependent on central venous pressure. Without the driving force of the right ventricle, the influence of volume status on pulmonary blood flow is exaggerated compared with healthy patients. The central venous pressure and urine output are both good indicators of intravascular volume status. This patient's central venous monitoring catheter is located in the internal jugular vein immediately proximal to the cavopulmonary anastomosis, thereby accurately measuring the driving pressure of her pulmonary circulation. A central venous pressure of 13 cm H2O is generally indicative of adequate intravascular volume status. Without a left atrial pressure the transpulmonary gradient cannot be determined, but the presence of normal ventricular function on the echocardiogram and adequate urine output both point to adequate intravascular volume status. Anemia is a common cause of hypoxia in single ventricle patients. Lacking the ability to fully saturate their hemoglobin with oxygen, these patients are dependent on higher baseline hemoglobin levels to maintain oxygen delivery. A low hemoglobin level will cause lower oxygen saturations and decreased delivery of oxygen. This patient's hemoglobin level is 13.4 g/dL (134 g/L), which should be adequate. The presence of good urine output and the absence of metabolic acidosis in her arterial blood gases are both evidence that oxygen delivery to her cells is adequate at this time. Giving her a transfusion of 15 mL/kg of packed red blood cells would provide her with slightly increased oxygen-carrying capacity, but would not increase her oxygen saturation significantly. The influence of pulmonary pathology such as atelectasis or pleural effusions on pulmonary blood flow in patients with cavopulmonary anastomosis is more exaggerated than in patients with biventricular physiology. In addition, positive pressure ventilation adversely affects pulmonary blood flow whereas spontaneous negative pressure ventilation improves pulmonary blood flow. The negative intrathoracic pressure during spontaneous breathing draws blood from the central venous system into the thoracic cavity which helps drive pulmonary blood flow. This patient's chest radiograph does not demonstrate any atelectasis. If atelectasis were present then maneuvers to recruit the atelectatic segment of lung such as increased tidal volume or small incremental increases in PEEP may improve pulmonary blood flow. The presence of a small right pleural effusion is inconsequential in this clinical scenario. A large pleural effusion would warrant drainage because the increase in pulmonary vascular resistance can affect pulmonary blood flow. Increasing PEEP to 10 cm H2O may decrease the oxygen saturation in this patient, because the increased positive intrathoracic pressure would decrease flow through the cavopulmonary anastomosis. Elevated pulmonary vascular resistance caused by pulmonary arterial hypertension can have devastating effects on single ventricle patients with cavopulmonary anastomoses. Clues to its presence after surgery would include persistent hypoxia and elevated central venous pressure with normal left atrial pressure. Cardiac catheterization to directly measure the pulmonary vascular resistance would be the test of choice in that situation. The presence of normal central venous pressure is indicative of a normal transpulmonary gradient. Initiation of inhaled nitric oxide may transiently improve the oxygen saturations, but would not treat the underlying issue in this patient, which is not pulmonary hypertension. PREP Pearls When faced with hypoxia after cavopulmonary anastomosis, the common causes of decreased saturations should be considered and therapy should be directed to the most likely cause.

A newborn girl with a fetal diagnosis of congenital heart disease is born at 36 weeks. At 32 weeks' gestation, she started to develop hydrops, which worsened and necessitated early delivery. Initial echocardiography shows dextrocardia, an interrupted inferior vena cava, persistence of the left superior vena cava, and a normal right superior vena cava which receives flow from a prominent azygous vein. The pulmonary veins return to the left side of a common atrium. There are 2 separate atrioventricular valves without significant regurgitation and normal ventricular function. The outflow tracts are unobstructed. Of the following, the BEST test that explains the development of prenatal course and neonatal prognosis is A. abdominal ultrasound B. blood smear C. electrocardiogram D. nasal epithelial brushing E. upper gastrointestinal series

C. electrocardiogram Heterotaxy syndrome is a disorder of laterality that can lead to anomalies in multiple systems, including congenital heart defects. Nearly any type of congenital heart defect can be seen in heterotaxy syndrome, but some lesions are more commonly seen in right or left atrial isomerism. Recognition of heterotaxy syndrome should lead to evaluation for the associated extracardiac abnormalities. The heart disease described in the vignette is likely to be associated with the left atrial isomerism form of heterotaxy syndrome. An interrupted inferior vena cava is present in most (60%-96%) patients with left atrial isomerism or polysplenia syndrome. It is rare in patients with right atrial isomerism or asplenia syndrome. Patients with right atrial isomerism have a high incidence of total anomalous pulmonary venous return. They often have unbalanced atrioventricular septal defects and/or double outlet right ventricle, and are more likely than those with left atrial isomerism to require single ventricle palliation. Patients with left atrial isomerism typically have less severe congenital heart defects. Atrioventricular canal defects are also seen in left atrial isomerism, but are more often balanced. Electrocardiography should be performed quickly, particularly when the heart rate is low, because of the implications for management and prognosis. Bradycardia and complete heart block are noted in 15% to 50% of patients with left atrial isomerism, but is rarely seen in right atrial isomerism. Bradycardia is a risk factor for fetal hydrops and perinatal death. One study showed a 79% survival to birth and a 63% survival to 1 year of age in patients with a fetal diagnosis of heterotaxy who have bradycardia or heart block. Not all patients with left atrial isomerism will have bradycardia or heart block. A nonsinus atrial rhythm can be seen in patients with left and right atrial isomerism. Patients with right atrial isomerism may also have a wandering pacemaker, indicating multiple sinus nodes. One of the most common findings in left atrial isomerism is polysplenia, or less frequently, asplenia. On abdominal ultrasonography, the presence or absence of a spleen or polysplenia should be identified. Polysplenia rarely results in normal splenic function. A blood smear demonstrating Howell-Jolly bodies indicates abnormal splenic function but this can be a normal finding in newborns. Patients with heterotaxy are more susceptible to infection, particularly from encapsulated organisms. They should receive antibiotic prophylaxis and be vaccinated appropriately. A nasal epithelial brushing can be used to assess ciliary function. Patients with complete situs inversus can have primary ciliary dyskinesia, leading to chronic respiratory problems from poor airway clearance. Patients with heterotaxy should be evaluated for ciliary dysfunction. The presence of ciliary function does not have as strong an influence on neonatal mortality as heart block. An upper gastrointestinal (UGI) series or other imaging should be routinely performed in infants with heterotaxy syndrome. There is a high rate of malrotation (60%-70% in right atrial isomerism, lower in left atrial isomerism). Debate exists as to the best approach for these patients. Some advocate prophylactic Ladd operation to prevent a midgut volvulus. Abdominal ultrasonography may also help in identifying biliary atresia, which is found in about 10% of patients with left atrial isomerism. PREP Pearls Fetal and neonatal bradycardia are common in patients with left atrial isomerism and are associated with a high mortality rate. Heterotaxy syndrome affects multiple organ systems, and may include congenital heart defects, abnormal splenic function, malrotation, and lung abnormalities. The associated defects seen in patients with heterotaxy are different in left versus right atrial isomerism.

You are called to the emergency department (ED) to evaluate an 11-month-old female infant who presented with 3 days of vomiting and lethargy. Her mother brought her in today because her activity level and responsiveness have significantly decreased. When the patient was placed on the monitor, the ED physician became concerned when there appeared to be an arrhythmia and obtained an emergent 12-lead electrocardiogram (Figure 1). You arrive to find the patient cool peripherally with difficult-to-palpate pulses and capillary refill of 5 to 6 seconds. During endotracheal intubation by the ED staff, the patient has a cardiac arrest and requires resuscitation for 5 minutes before restoration of a perfusing rhythm. Following the arrest, the vital signs include a heart rate of 190 beats/min, a noninvasive blood pressure of 62/38 mm Hg, and pulse oximetry of 99% on a fraction of inspired oxygen of 1.0 via manual ventilation. You view the chest radiograph (Figure 2) and perform rapid echocardiography (Figure 3). The child is moved urgently to the cardiac intensive care unit. Of the following, the NEXT best intervention should be to: A. initiate an epinephrine infusion B. initiate a normal saline bolus infusion C. initiate a milrinone infusion D. plan for emergent extracorporeal membrane oxygenation cannulation

C. initiate a milrinone infusion Patients with acute fulminant myocarditis often present with cardiogenic shock and cardiovascular collapse. They require immediate hemodynamic support and frequently require mechanical circulatory support to survive. In spite of the severity of their instability at presentation, when appropriate support is provided they tend to have a good recovery. Acute myocarditis on the other hand, tends to have a more indolent presentation with less hemodynamic compromise. Recovery of heart function after acute myocarditis is less likely when compared with acute fulminant myocarditis. The patient in the vignette presents with typical nonspecific symptoms that are often confused with a viral respiratory infection. The absence of cardiomegaly and pulmonary edema on chest x-ray (Figure 2), while typical of acute fulminant myocarditis, can often confuse the clinical picture further. In this case, the presence of a wide complex tachycardia is an important clue that the primary pathology is myocardial in origin (Figure 2). The hemodynamic instability and cardiac arrest on intubation are also typical of the presentation of acute fulminant myocarditis. This patient requires immediate hemodynamic support. In most institutions, the most rapid means available is extracorporeal membrane oxygenation (ECMO) cannulation. This provides both pulmonary and respiratory support, and allows time for myocardial recovery. Many patients with acute fulminant myocarditis who are treated with ECMO will be successfully decannulated and often have complete recovery of cardiac function. Some patients will require long-term medical treatment for myocardial dysfunction, and others may require consideration for cardiac transplantation. The need for long-term medical support and/or cardiac transplantation is more common in acute myocarditis. Both epinephrine and milrinone infusions could be potentially helpful in this situation; however, given the hemodynamic instability, the use of mechanical circulatory support is lifesaving in this case. A normal saline bolus would not be indicated and could potentially cause further harm because of the degree of myocardial dysfunction. Intravenous immunoglobulin (IVIG) is used in varying degrees to treat patients with myocarditis. The hope is that IVIG will mitigate the body's immune response which is primarily responsible for myocardial cell damage. The disadvantages of IVIG treatment include volume load, potential allergic reactions, and blood product exposure. The medical literature is not definitive regarding the benefits of IVIG in the treatment of myocarditis, though it is commonly used and well tolerated in most patients. PREP Pearls Patients with acute fulminant myocarditis often require urgent or emergent mechanical circulatory support to survive. Once support is established, survival and myocardial recovery tends to be good in acute fulminant myocarditis.

A 4-month-old male infant presents to the emergency department with daily fever of 2 weeks' duration (Tmax 40°C). He was initially seen 48 hours after the onset of fever and diagnosed with pneumonia. His fever persists daily despite treatment with amoxicillin for the past 10 days. He is irritable and mottled with neck stiffness but has no other focal findings on examination. Notable laboratory findings include a platelet count of 1,240 × 103/μL (1,240 × 109/L) and an erythrocyte sedimentation rate of 115 mm/h. A cardiology consultation is requested and findings on transthoracic echocardiography are shown in Figure 1 and Figure 2. The left anterior descending artery measured 5 mm (Z score +15.7) and the right coronary artery measured 4 mm (Z score +8.3). The patient began treatment with intravenous immunoglobulin (IVIG) and high-dose aspirin. He was given a second dose of IVIG for persistent fever which then resolved. Of the following, the MOST appropriate long-term therapeutic regimen for this patient is A. clopidogrel bisulfate B. high-dose aspirin C. low-dose aspirin and warfarin or low-molecular-weight heparin D. low-dose aspirin, warfarin or low-molecular-weight heparin, and propranolol E. methylprednisolone followed by low-dose aspirin

C. low-dose aspirin and warfarin or low-molecular-weight heparin The infant in the vignette presents with fever but without any of the classic diagnostic criteria of Kawasaki disease, as commonly seen in the youngest affected patients. However, the presence of coronary aneurysms on echocardiogram makes it a diagnosis of atypical Kawasaki disease. In this case, both the right and left systems are affected with large to giant aneurysms, stratifying the patient to a level IV risk according to the 2004 American Heart Association recommendations. The recommendations for long-term care of patients with Kawasaki disease are based on the degree of coronary involvement, and include pharmacologic therapy, physical activity, follow-up, and diagnostic as well as invasive testing (Table). Intravenous immunoglobulin (IVIG) and high-dose aspirin are the mainstays of acute therapy in patients with Kawasaki disease, with a significant reduction in the incidence of coronary aneurysms seen in patients who receive dual therapy versus those who receive aspirin alone. Currently IVIG is typically given as a 1-time dose of 2 g/kg. Although different regimens have been used in the past, meta-analyses have revealed that higher doses given as single infusions are the most efficacious. Its exact mechanism of action is unclear as is its efficacy after day 10 of illness. IVIG is most useful within the first 7 days of illness with the vast majority of patients who receive it defervescing and showing symptom resolution within 2 to 3 days of treatment. Patients who fail to respond to IVIG or those who have recurrent symptoms can be retreated with up to a total of 3 courses. High-dose aspirin (80-100 mg/kg per day) is given in the acute phase until patients are afebrile for 48 hours to suppress inflammation. High doses are required at this point in the course of illness because aspirin is malabsorbed in this highly inflammatory state. Although some retrospective data have suggested a benefit of steroid therapy for Kawasaki disease, to date no large prospective randomized trial has demonstrated any efficacy of adding steroids to conventional primary therapy. Newburger et al showed that compared with conventional therapy, patients receiving IV methylprednisolone had similar coronary dimensions at week 1 and week 5 after randomization. Although those receiving steroids had a somewhat shorter initial hospitalization, with a lower erythrocyte sedimentation rate and tendency toward a lower C-reactive protein at week 1, both groups had similar days in hospital, days of fever, rates of retreatment with IVIG, and numbers of adverse events. They concluded that the data did not support the addition of a single pulsed dose of IV methylprednisolone to conventional IVIG therapy. Antiplatelet therapy is a critical element in the long-term treatment of patients with coronary aneurysms as a result of Kawasaki disease because of the persistence of platelet activation in the convalescent and chronic phases. Low doses of aspirin (3-5 mg/kg per day) inhibit platelet aggregation and are the mainstay of long-term therapy for patients with any persistent coronary abnormalities after Kawasaki disease. In patients with more severe and extensive disease, there may be some benefit to adding other antiplatelet agents such as clopidogrel bisulfate to further suppress platelet activation via antagonism of adenosine diphosphate-mediated activation. Long-term antiplatelet therapy should be combined with anticoagulation in patients with a level IV risk for Kawasaki disease such as the one described in the vignette; patients with 1 or more large or giant coronary artery aneurysms (typically defined as 8 mm or greater though this does not take into account patient body surface area); or multiple or complex aneurysms in the same coronary artery without evidence of obstruction. Although guidelines for the use of Z scores have not yet been established in the care of these patients, the absolute size of the aneurysms, the high Z scores, and the bilateral nature of the disease in the vignette are consistent with a level IV risk. Such patients have a high risk of thrombosis formation because of abnormal flow conditions, including low velocity flow through and stasis in the aneurysm. These conditions may be aggravated over time by endothelial remodeling processes, the development of stenosis associated with the aneurysms, and derangements in the clotting cascade because of the presence of chronic thrombus. Anticoagulation can be achieved with either warfarin or low-molecular-weight heparin, but the latter has the disadvantage of requiring twice-daily injections. In patients with documented coronary artery obstruction confirmed on angiography, treatment with beta-blockers should be considered to reduce myocardial oxygen consumption. PREP Pearls Long-term antiplatelet therapy should be combined with anticoagulation in those with risk level IV Kawasaki disease—patients with 1 or more large or giant coronary artery aneurysm, or multiple or complex aneurysms in the same coronary artery without evidence of obstruction—to address the platelet, endothelial, and humoral clotting factors that put these patients at increased risk for thrombosis formation. Intravenous immunoglobulin and high-dose aspirin are the mainstays of acute therapy in patients with Kawasaki disease. There are no prospective randomized data to date supporting the addition of steroids to conventional therapy. Antiplatelet therapy is a critical element for patients with coronary aneurysms as a result of Kawasaki disease because of the persistence of platelet activation in the convalescent and chronic phases.

You are asked to evaluate a 30-hour-old female neonate who was transferred to the neonatal intensive care unit after failing her screening test for critical congenital heart defects. She was born via normal spontaneous vaginal delivery at 35 weeks of gestation after an unremarkable pregnancy to healthy parents who have no other children and whose family history is unremarkable. They declined aneuploidy screening and any other genetic testing during the pregnancy. Her cardiac examination findings include a continuous murmur heard at the bilateral upper sternal borders radiating to the back. The femoral pulses are decreased and there is no blood pressure gradient between the upper and lower extremities. General medical examination reveals a small, 1- to 2-cm, oval, reddish-purple lesion on the left upper aspect of her neck. Echocardiography demonstrates the findings shown in the Figure. Of the following, the ADDITIONAL diagnostic test that is most likely to yield an abnormal result is A. abdominal ultrasonography B. bone scan of the upper extremities C. magnetic resonance imaging/arteriography of the brain D. voiding cystourethrography

C. magnetic resonance imaging/arteriography of the brain PHACES syndrome is an association of posterior fossa and other structural brain abnormalities, cervical/facial hemangiomas, cervical and cerebrovascular arterial malformations, cardiac defects, and eye abnormalities with sternal defects and/or a supraumbilical raphe sometimes included. The diagnosis, which is more commonly seen in girls, typically can be made by physical examination together with an eye examination. Diagnostic testing is used as an adjunct to assess for the features not manifest on physical examination. The syndrome is recognized to be heterogeneous with 1 or more of the typical findings often absent in affected individuals. The patient described in the vignette has a cervical-facial hemangioma. As is the case with this patient, the hemangiomas may be quite small in early infancy but can grow quite rapidly in the first 6 to 18 months after birth and even within the first few weeks, typically become large and potentially disfiguring, and may disrupt vision, hearing, and/or breathing depending on their location. The finding of continuous forward flow in diastole and a double-envelope Doppler pattern on the patient's echocardiogram indicates the presence of a coarctation of the aorta. Together, these anomalies are suggestive of PHACES syndrome. The infant in the vignette should undergo magnetic resonance imaging of the brain to look for the intracranial anomalies that are characteristic of the syndrome. Although abdominal pathology such as an omphalocele has been reported with PHACES syndrome, this should be evident on physical examination, making an abdominal ultrasound unnecessary. Neither genitourinary malformations nor skeletal involvement other than of the sternum is typical of PHACES syndrome; therefore neither a bone scan of the upper extremities nor a voiding cystourethrogram would be indicated. The typical intracranial abnormalities found in PHACES syndrome are Dandy-Walker malformation, cerebellar atrophy, and agenesis of the vermis. Anomalies of the cerebral vasculature are common and if unrecognized and/or untreated, can lead to potentially devastating neurologic complications. Eye anomalies include microphthalmos, optic nerve hypoplasia, and congenital cataracts. Although any form of congenital heart disease can be seen in the setting of PHACES syndrome and affects between one-quarter and one-third of patients with this syndrome, the most frequently described cardiac manifestations involve aortic arch obstruction/coarctation of the aorta. Unlike typical isolated coarctation of the aorta, however, patients with PHACES syndrome usually have associated complexities of the aortic arch and head vessels such as aberrant subclavian arteries, which can make recognition of the arch obstruction challenging. In the patient in the vignette, the presence of palpable femoral pulses and the absence of a blood pressure gradient do not rule out the possibility of such an associated vascular anomaly. In addition, these patients often do not have the accompanying bicuspid aortic valve seen in many patients with typical coarctation. The etiology of PHACES syndrome is thought to be a disruption in normal vascular development early in embryonic life. Although the exact cause is not known, lesions of the neural crest have been implicated because certain crest cells are known to contribute to the development of the great vessels as well as the head and neck vessels while others are involved with the formation of the facial vasculature. The coarctation of the aorta in these patients is usually complex and not isolated. Therefore its presence is more consistent with a vascular accident and disruption in the normal activity of the neural crest cells than with either the hemodynamic or ectopic ductal tissue theories that usually describe the embryologic origins of typical coarctation of the aorta. In the former theory, it is hypothesized that abnormalities in ductal flow and angulation result in augmented right-to-left ductal shunting and resultant underdevelopment of the aortic isthmus. Alternatively, in the ectopic ductal tissue theory it has been postulated that abnormal extension of ductal tissue into the lumen of the aorta creates a posterior shelf, which in turn results in aortic obstruction when ductal closure occurs. In addition, the lesions seen in patients with PHACES syndrome often demonstrate laterality and occur on the same side of the body. This suggests that they are all intimately related and that 1 or more of them may be the cause of others and therefore the coarctation likely does not arise because of a separate mechanism. The presence of head and neck hemangiomas together with congenital heart disease should prompt consideration of PHACES syndrome. Therefore diagnostic testing should be performed to facilitate evaluation for the other features of the syndrome. The aortic arch abnormalities present in PHACES syndrome are more likely caused by disruptions of neural crest cells than either disruptions in prenatal aortic arch flow or abnormalities of ductal tissue. PREP Pearls The presence of head and neck hemangiomas together with congenital heart disease should prompt consideration of PHACES syndrome. Coarctation of the aorta is one of the congenital heart diseases most commonly seen in PHACES syndrome and may be associated with complexities of the arterial anatomy. The aortic arch abnormalities present in PHACES syndrome may occur because of lesions of the neural crest rather than ectopic ductal tissue and/or abnormal preductal flow as in typical coarctation of the aorta.

A 3-week-old, ex-35 week premature female newborn remains in the neonatal intensive care unit as she advances towards full oral feeding. The care team noted that her feeding ability has not progressed over the past few days and she was becoming fussy with feeds. She was more tachypneic than previously (respiratory rates in the 70 breaths/min range) and they noted a new murmur on her physical examination. In the past 24 hours, her extremities are cooler, her blood pressure is lower, and she is more tachycardic. An electrocardiogram (Figure) is obtained. The team orders an echocardiogram (Video 1, Video 2, Video 3). Of the following, the intervention that would MOST likely worsen the baby's clinical status is A. dopamine infusion B. normal saline bolus C. oxygen by nasal cannula D. transfusion of packed red blood cell

C. oxygen by nasal cannula The baby in the vignette has anomalous left coronary artery from the pulmonary artery (ALCAPA). The administration of oxygen may potentially worsen her status, as it will decrease the pulmonary vascular resistance, increase coronary artery steal, and lead to further ischemia and myocardial dysfunction. A rare congenital heart defect that can present in several different ways, ALCAPA most commonly presents in infants and is also known as Bland-White-Garland syndrome. Fetuses and newborns rarely display any symptoms, as the elevated pulmonary vascular resistance (PVR) and pulmonary artery pressure (PAP) allows for appropriate anterograde coronary blood flow from the pulmonary artery. After birth, the oxygen content in the blood supplied by the left coronary artery will be lower than normal, but more importantly, the PVR decreases and PAP pressures become too low to provide anterograde blood flow to the myocardium. There is reversal of flow in the coronary artery with retrograde flow into the pulmonary artery (Video 1, Video 4). The myocardium that is supplied by the left coronary artery (the interventricular septum, left ventricle, and left ventricular papillary muscles) develop ischemia. Ischemia initially happens during feeding or other times of exertion, but eventually there will be areas of myocardial infarction. The papillary muscles are particularly vulnerable and their infarction results in dysfunction and regurgitation of the mitral valve (Video 3). Infarction in the left ventricular myocardium leads to cardiac dysfunction and dilation. Video 2 shows the dilated left ventricle with poor function and echogenic papillary muscles. This constellation of pathology results in the clinical signs of irritability (particularly with feeds), poor feeding, diaphoresis, tachypnea, and failure to thrive, as is seen in the baby in the vignette. In response to insufficient coronary blood flow and infarction, collateral blood vessels between the right and left coronary arteries enlarge and proliferate. The right coronary dilates to support the extra blood flow. The low PAP will steal some of this coronary blood flow, however, the collateralization is sometimes enough to maintain cardiac function and limit symptoms. Patients with adequate collateralization may have little or no symptoms in infancy. Collaterals continue to grow and ventricular function is adequate. These patients may present later in childhood due to a continuous murmur caused by the high volume collateral blood flow. Others may present with sudden cardiac death, typically during exercise. Some may develop worsening congestive heart failure with time and others may be diagnosed due to an incidental finding. Patients in whom the PAP does not decrease after birth may also present with minimal symptoms. These patients may have other forms of congenital heart disease, such as a large ventricular septal defect or patent ductus arteriosus, that result in elevated PAP. This allows the left coronary artery to maintain anterograde coronary blood flow, and although it is deoxygenated blood, the oxygen content is usually adequate to avoid ischemia. The direction of blood flow may make detection via echocardiography more challenging. Acute decreases in PAP occurring with surgical or percutaneous correction of the ventricular septal defect or patent ductus arteriosus will result in acute ischemia and infarction of the left ventricular myocardium if the coronary anomaly is not recognized and addressed. In some patients with ALCAPA, the PAP fall normally after birth, but then become elevated secondary to severe mitral regurgitation or cardiac dysfunction and the resulting left atrial hypertension. The baby in the vignette is exhibiting early signs of congestive heart failure. Her echocardiogram shows a dilated left ventricle with poor function, which could easily be mistaken for myocarditis or dilated cardiomyopathy, but should always raise suspicion for ALCAPA. The presence of mitral regurgitation and echogenic papillary muscles further suggest ALCAPA. The finding of retrograde flow in the coronary artery confirms this diagnosis. If peripheral or organ perfusion is compromised, dopamine or another inotrope would be appropriate and unlikely to cause harm. A normal saline bolus may be warranted if the baby is not receiving adequate intake and shows signs of dehydration. Care should be taken in fluid resuscitation of patients in cardiogenic shock, as it may worsen pulmonary edema, however, euvolemia should be maintained. Transfusion of packed red blood cells may be helpful if the baby has reached her nadir, as it will increase oxygen carrying capacity. These interventions may be appropriate in some patients with ALCAPA and are less likely to result in harm than giving supplemental oxygen, which may increase coronary artery steal by decreasing pulmonary vascular resistance. Supportive measures should be used only as a way of stabilizing the infant for surgical repair, which should not be delayed. PREP Pearls Anomalous left coronary artery from the pulmonary artery (ALCAPA) presents in infants as irritability and difficulty with feeding and signs of congestive heart failure. Coronary artery steal resulting in ischemia and infarction leads to cardiac dysfunction in ALCAPA. Oxygen and other drugs that decrease pulmonary vascular resistance can worsen coronary artery steal in ALCAPA.

You are seeing a 2-day-old infant with a known diagnosis of double-outlet right ventricle, d-malposition of the great vessels, and subpulmonic ventricular septal defect, who is in the neonatal intensive care unit because of worsening oxygen saturations over the last 12 hours. When you arrive, the infant is sedated and receiving a fraction of inspired oxygen of 1.0 through the ventilator. The infant is receiving a prostaglandin infusion at a rate of 0.03 μg/kg per minute. On examination, the infant appears dusky and cyanotic. Pulses are easily palpated in the femoral arteries and arterial blood pressure measured with an umbilical arterial catheter is 62/34 mm Hg. The oxygen saturation with a probe on the right hand is 62%. You remember looking at the echocardiographic images the day before, which demonstrated a large ventricular septal defect with a left-to-right shunt and no outflow tract obstruction or coarctation of the aorta. You ask the bedside nurse to switch the pulse oximetry probe to the right foot, and the saturation there measures 77%. Of the following, the MOST appropriate next step in the management of this infant is A. increasing the prostaglandin dose to 0.05 µg/kg per minute B. initiating inhaled nitric oxide C. performing balloon atrial septostomy D. placing a modified Blalock-Taussig shunt E. planning for extracorporeal membrane oxygenation

C. performing balloon atrial septostomy Double-outlet right ventricle with subpulmonic ventricular septal defect (VSD) and d-malposition of the great vessels (Taussig-Bing anomaly) can be challenging to manage in the preoperative period. Even in the presence of a large VSD, mixing of systemic and pulmonary venous blood can be limited at the ventricular level. Streaming of ventricular blood to the closest outflow tract (right ventricular blood to the aorta, and left ventricular blood to the pulmonary artery) can prevent adequate mixing and create refractory cyanosis. This lesion is often associated with coarctation of the aorta and close attention should be paid on initial imaging. Patients with Taussig-Bing anomaly will demonstrate reverse differential cyanosis with higher measured oxygen saturations in the lower extremities. This is attributed to desaturated systemic venous blood from the right ventricle being ejected out of the ascending aorta to perfuse the head and neck vessels, while saturated pulmonary venous blood is ejected from the left ventricle to the pulmonary artery and through the patent ductus arteriosus to the descending aorta. Simultaneous measurement of upper and lower extremity saturations (absent an aberrant right subclavian artery) will usually yield higher saturations in the lower extremities. To create adequate mixing of systemic and pulmonary venous blood, these patients often require a large atrial level communication. When not present at birth this can be accomplished via a balloon atrial septostomy. Once the atrial septum is opened, cyanosis will often improve because of adequate mixing, and the patient can be weaned off the prostaglandins. Timing for surgical intervention after atrial septostomy often depends on several factors, including patient weight, persistent cyanosis, and risk for pulmonary overcirculation. The patient in the vignette is demonstrating reverse differential cyanosis that is refractory to oxygen therapy and would therefore benefit from an urgent atrial septostomy to improve mixing of systemic and pulmonary venous return. A modified Blalock-Taussig shunt would not necessarily help this patient because his pulmonary blood flow is already increased. The shunt would carry more deoxygenated systemic blood to the lungs which, upon returning to the left atrium and ventricle, would return to the lungs via the pulmonary artery. In addition, with shunt placement, the ductus would be ligated, leading to cyanosis in the descending aorta as well. Inhaled nitric oxide would increase pulmonary blood flow without increasing oxygen delivery to the head and neck vessels (preductal) in the absence of atrial level mixing. This patient's ductus arteriosus is widely patent. The presence of higher saturations in the lower extremities confirms that oxygenated blood from the left ventricle is being delivered via the main pulmonary artery through the ductus. In addition, the low diastolic pressure is secondary to diastolic runoff through the ductus. Increasing the dose of prostaglandins will not make any appreciable difference in the patient and will likely cause hypotension. Extracorporeal membrane oxygenation (ECMO) is a treatment for persistent neonatal hypoxia but would not be indicated in this patient. It would temporarily treat the cyanosis, but would not provide definitive treatment for the problem creating the cyanosis. The only indication for ECMO in such patients would be severe hemodynamic instability secondary to refractory hypoxia, which would allow the patient's condition to be stabilized enough to treat the underlying problem. PREP Pearls In patients with physiology concurrent with transposition of the great arteries, severe cyanosis may persist even in the presence of a sizable ventricular septal defect (VSD) because of streaming. An unrestrictive atrial communication is needed to increase mixing and improve saturations. In patients with double-outlet right ventricle, d-malposition of the great arteries, and subpulmonic VSD, there is increased pulmonary blood flow, and significant desaturation should prompt evaluation for decreased mixing.

A 12-year-old girl comes to your office for evaluation of a heart murmur. Her history is unremarkable, except for frequent nosebleeds and a vague history of shortness of breath with exertion. No family history is available. Her vital signs show a heart rate of 86 beats/min, respiratory rate of 18 breaths/min, and blood pressure of 95/60 mm Hg. Although she is not obviously cyanotic, her oxygen saturation is 93% on room air in both upper and lower extremities. She is well appearing and has had normal growth (25th percentile for both height and weight) and development. Her examination is normal except for a functional-sounding heart murmur. Her electrocardiogram is unremarkable. A transthoracic echocardiogram was performed, was of excellent quality, and showed no abnormalities. Her hemoglobin measured 16 g/dL (160 g/L). Of the following, the BEST next diagnostic test to evaluate this patient would be A. cardiac catheterization B. exercise stress test C. saline contrast echocardiography D. transesophageal echocardiographycardiography

C. saline contrast echocardiography The patient in this vignette has a history of epistaxis and mild dyspnea, as well as evidence of mild cyanosis and polycythemia in the context of normal intracardiac anatomy. These are typical presenting findings in an adolescent with hereditary hemorrhagic telangiectasia (HHT). Hereditary hemorrhagic telangiectasia, formerly known as Osler-Rendu-Weber syndrome, is an autosomal dominant disorder of vascular dysplasia that may include mucocutaneous telangiectasias, as well as visceral arteriovenous malformations in the pulmonary, hepatic, and cerebral systems. Epistaxis due to telangiectasias of the nasal mucosa is the most common presenting symptom in children and adolescents with HHT; other mucocutaneous telangiectasias such as those of the skin and lips are often not apparent until adulthood. In HHT, pulmonary arteriovenous malformations (AVMs) are a source of intrapulmonary shunting that can result in mild cyanosis and polycythemia. Although these pulmonary AVMs are often present in children, they may not be obvious clinically, as in the case in the vignette. From the list of diagnostic tests, a saline contrast echocardiogram would be the next diagnostic test of choice to detect pulmonary AVMs when HHT is suspected. Cardiac catheterization could then be used to confirm and potentially embolize pulmonary AVMs. Exercise stress testing or transesophageal echocardiography would not lead directly to a diagnosis of HHT or pulmonary AVMs. Saline contrast echocardiography involves injection of agitated saline into a peripheral intravenous line. Agitated saline results in the presence of short-lived air micro-bubbles in the systemic circulation that can be seen as echogenic "cavitation effect" in the right-sided heart structures. In the normal circulation, these micro-bubbles diffuse into the lungs through the pulmonary capillary circulation and are therefore not visible in the left-sided heart structures. However, when either an intracardiac or intrapulmonary shunt is present, the micro-bubbles will also be seen in the left side of the heart because the agitated saline micro-bubbles can bypass the normal pulmonary capillary circulation. Saline contrast echocardiography often can directly demonstrate the location that the micro-bubbles enter the left side of the heart, whether this is from the right atrium into the left atrium (in the case of an intracardiac atrial shunt) or from the pulmonary veins in cases of pulmonary AVMs (Video 1). Another approach to interpret saline contrast echocardiography is to assess the timing of the appearance of the micro-bubbles in the left side of the heart: when micro-bubbles appear early in the left side of the heart (ie, within 3 heart beats of the micro-bubbles being seen in the right side of the heart) this signifies an intracardiac shunt, while later appearance in the left side of the heart (after 3-5 beats) is consistent with an intrapulmonary shunt. Saline contrast echocardiography has been shown to have high sensitivity in detecting intrapulmonary shunting, and has an excellent safety profile. Therefore, contrast echocardiography with saline injection is generally considered to be the diagnostic test of choice to assess for pulmonary AVMs in a child with unexplained cyanosis and a normal high-quality transthoracic echocardiogram. Alternately, high-resolution computed tomographic angiography can be performed in patients in whom there is a high suspicion for HHT. Aside from the mucocutaneous telangiectasia and pulmonary AVMs, less common presenting findings in children with HHT could be related to cerebral AVMs such as high-output cardiac failure or neurologic symptoms such as strokes and seizures. Pulmonary AVMs in patients with HHT can also lead to neurological and systemic complications due to paradoxical embolization, as well as hemoptysis, hemothorax, and pulmonary hypertension. Gastrointestinal bleeding from gastrointestinal mucosal telangiectasias (which can lead to iron-deficiency anemia) and hepatic abnormalities are uncommon in children, but are often found in adults with HHT. In addition to its use in screening patients for intrapulmonary shunting and pulmonary AVMs in cases of suspected HHT, saline contrast echocardiography can also detect intrapulmonary shunting or pulmonary AVMs as a source for desaturation in patients with single-ventricle physiology status-post partial or complete cavopulmonary shunt (ie, Glenn or Fontan palliation) or in patients with suspected hepatopulmonary syndrome. Other indications for saline contrast echocardiography in children and adults with congenital heart disease may include assessment for intracardiac shunting in patients with suboptimal echocardiographic windows, for example, to diagnose a patent foramen ovale in a child or young adult who presents with a stroke or other sudden-onset neurologic abnormalities, or to assess for residual intracardiac shunt in a patient status post congenital heart disease repair. Saline contrast echocardiography by injection of agitated saline into a left upper extremity peripheral intravenous line can also be used to diagnose a persistent left-sided superior vena cava (SVC) draining to the coronary sinus with an absent innominate vein (by visualization of micro-bubbles in the coronary sinus to the right atrium), or in the rare case of a left SVC draining directly to the left atrium (by visualization of micro-bubbles in the left atrium without opacification of the right heart). As in adults, saline contrast echocardiography has also been used in children with pulmonary hypertension or cardiomyopathy and suboptimal echocardiographic windows to optimize endocardial edge detection on 2D or Doppler echocardiography. Contrast echocardiography using agitated saline is thought to be very safe, although there have been rare case reports of transient ischemic attack and even stroke due to paradoxical right-to-left shunting. When performing saline contrast echocardiography, care should therefore be taken to inject agitated saline only and not larger air bubbles. In lieu of using agitated saline, contrast echocardiography can instead be performed using so-called "second-generation" intravascular contrast agents. A number of different contrast agents are currently available. With each of these agents, very small gas bubbles are encased in some type of biochemical shell. This enables opacification of left heart structures as well as right side of the heart structures because the contrast micro-bubbles persist over longer time and are also not filtered by the pulmonary circulation. However, these contrast agents are currently not approved for use in children and are also contraindicated in subjects with suspected or confirmed right-to-left shunting. PREP Pearls Saline contrast echocardiogram can be used to detect intrapulmonary shunts (pulmonary arteriovenous malformations) in cases of suspected hereditary hemorrhagic telangiectasia (HHT) and in subjects status post Glenn or Fontan palliation for single-ventricle physiology. Hereditary hemorrhagic telangiectasia should be considered in children presenting with frequent epistaxis and occult mild cyanosis, with or without a positive family history of HHT. Contrast echocardiography using "second-generation" agents is currently not approved for use in children, and is contraindicated in subjects with suspected or confirmed right-to-left shunting.

A 6-year-old girl is referred to the cardiology office for evaluation of a murmur. Echocardiography is performed, and the following structure is noted (Figure). Of the following, the embryological origin of the identified structure is MOST likely the A. bulbus cordis B. crista terminalis C. sinus venosus valve D. septum primum E. septum secundum

C. sinus venosus valve The Figure demonstrates the Eustachian valve. One key aspect to note is that it is located anterior to the inferior vena cava. This subcostal sagittal view can be deceiving, particularly if the Eustachian valve is large, causing it to be mistaken for the atrial septum. A large Eustachian valve may be misinterpreted as the inferior aspect of an atrial septal defect thus leading to errors during transcatheter or surgical closure of atrial septal defects. The Eustachian valve is the embryological remnant of the inferior portion of the fetal sinus venosus valve. The inferior right sinus valve is responsible for directing blood across the foramen ovale in fetal life toward the left atrium. Lack of normal regression of the right valve of the sinus venosus can result in abnormal septation of the right atrium by a large membrane called cor triatriatum dexter. Extensive fenestration of the right sinus valve can result in a weblike structure called a Chiari network. The bulbus cordis later gives rise in part to the right ventricle and the conotruncus not the Eustachian valve. The crista terminalis is a ridge of tissue in the superior aspect of the right atrium that divides the smooth posterior right atrium (sinus venosus) from the more muscular part of the atrium (pectinate muscles). The septum primum in the subcostal sagittal view is located posterior to the inferior vena cava not anteriorly as the image shows. The septum secundum, which forms much of the atrial septum, originates superiorly and grows inferiorly to help form the atrial septum. In this view, it would be located superiorly. PREP Pearls The Eustachian valve is the embryological remnant of the inferior portion of the fetal venous sinus valve, and it functions in fetal life to direct blood across the foramen ovale toward the left atrium. A prominent Eustachian valve can be mistaken for the inferior portion of the atrial septum, leading to the false impression of a secundum atrial septal defect. In assessing atrial membranes seen from the subcostal sagittal view, it is important to note where in relation to the inferior vena cava the membrane is located.

You are seeing a 4-month-old female infant with trisomy 21 in your clinic for the first time. The family just moved to the area. She was born at 36 weeks' gestation and had a birth weight of 1.8 kg. After a 6-week hospital stay, she was discharged on nasogastric tube feeds, ranitidine, and furosemide. Her parents report that she breathes fast, but otherwise is doing well. Her physical examination findings are notable for a weight of 3.5 kg, a heart rate of 152 beats/min, a respiratory rate of 45 breaths/min, and a blood pressure of 70/44 mm Hg in the right arm. She has normal pulses without brachiofemoral delay. Her oxygen saturation on room air is 91%. She appears comfortable. She has a grade 1/6 systolic murmur loudest at the left upper sternal border and bilaterally in the back. Her liver edge is about 2.5 cm below the right costal margin. Echocardiography is performed (Video 1, Video 2, Video 3, Figure). The BEST estimate of the pulmonary artery systolic pressure is A.25 mm Hg B.40 mm Hg C.75 mm Hg D.95 mm Hg **Note that videos could not be loaded or saved**

C.75 mm Hg A 4-month-old infant with a large complete atrioventricular septal defect without obstruction to pulmonary outflow should have a systolic pulmonary artery pressure that is at or near the systemic level. The echocardiographic data demonstrate a large complete atrioventricular septal defect with large atrial and ventricular components. There is also a small patent ductus arteriosus (Video 3) with low-velocity bidirectional flow. No other abnormalities are present. The flow is left-to-right during diastole and right-to-left during systole (Figure). The peak velocity of flow through the patent ductus arteriosus is low in both directions. The systolic systemic blood pressure obtained by physical examination was 70 mm Hg, so the estimated pulmonary artery pressure would be essentially the same. The infant in this vignette has signs consistent with congestive heart failure, including tachypnea, tachycardia, and hepatomegaly. The pulmonary flow to systemic flow ratio (Qp:Qs) will be elevated, and pulmonary vascular resistance is likely not high enough to preclude repair. Although infants with trisomy 21 are at an increased risk of developing pulmonary vascular obstructive disease, a 4-month-old patient with this clinical presentation is a good candidate for complete surgical repair without further testing. Older patients or patients without obvious symptoms and signs of congestive heart failure would benefit from cardiac catheterization to help assess operability. PREP Pearls Pulmonary artery pressure is near the systemic level in the presence of a complete atrioventricular septal defect without obstruction to pulmonary blood flow.

You are evaluating a 5-day-old male neonate who was referred to you for a murmur. He was born at home and seen at his pediatrician's office 2 days after birth. His parents report that he has been doing well. They have not noted any tachypnea, cyanosis, or poor feeding. He is in no acute distress and is alert. He has a heart rate of 145 beats/min, respiratory rate of 40 breaths/min, blood pressure of 70/40 mm Hg in upper and lower extremities, and oxygen saturation of 85% on room air. He has no retractions and clear lung fields. He has a mildly increased right ventricular impulse. S1 is normal and S2 is single. He has a grade 3/6 harsh systolic ejection murmur at the left upper sternal border. His liver is not palpable. His peripheral pulses are 2+ and equal and his extremities are well perfused. An echocardiogram is obtained (Video 1 and Video 2). Of the following, the syndrome MOST likely to be found in this patient is A.CHARGE syndrome B.DiGeorge syndrome C.Down syndrome D.heterotaxy syndrome

C.Down syndrome The echocardiogram of the patient in this vignette demonstrates a complete balanced atrioventricular septal defect (AVSD), as well as tetralogy of Fallot (TOF). These defects are common in patients with trisomy 21, or Down syndrome, and 80% of patients with this combination also have trisomy 21. Congenital heart defects are seen in about one-half of patients with trisomy 21. Of those patients with trisomy 21 with congenital heart defects, 40% have an AVSD and 5% have TOF. Conversely, 75% of all patients with an AVSD and 5% of patients with TOF have trisomy 21. Patients with the combination of AVSD and TOF almost always have trisomy 21. The anatomy in these patients consists of the major anatomical features of both defects. There is a common atrioventricular valve with both inlet ventricular septal defects and primum atrial septal defect. The ventricular septal defect extends to the outlet septum, and the conal or outlet septum deviates anteriorly. The anterior bridging leaflet in most patients with AVSD/TOF is typically free floating, with no division and no attachments to the septum, which is classified as Rastelli type C. DiGeorge syndrome is caused by a deletion in 22q11.2. It is commonly associated with congenital heart defects, which are seen in approximately 75% to 80% of patients with 22q11.2 deletion. Tetralogy of Fallot, interrupted aortic arch type B, truncus arteriosus, and other conotruncal and arch defects are the most common types of congenital heart defects associated with 22q11.2 deletion. Although other defects can be seen in these patients, AVSDs are uncommon. CHARGE syndrome is caused by CHD7 mutation in most cases. The clinical findings of coloboma, heart defects, atresia choanae (also known as choanal atresia), growth retardation, genital abnormalities, and ear abnormalities are most commonly seen in these patients. Congenital heart defects are seen in 75% to 85% of patients with CHARGE syndrome. Conotruncal defects, including TOF, are the most common. Arch abnormalities, ventricular septal defects, atrial septal defects, and AVSDs can be found as well. Heterotaxy syndrome is a disorder of laterality. Multiple genetic mutations have been associated with familial heterotaxy syndrome, but gene abnormalities have not been identified in most cases. Atrioventricular septal defects are often seen in patients with heterotaxy syndrome. They are more often unbalanced AVSDs and are associated with other cardiac abnormalities. Tetralogy of Fallot is not commonly seen in patients with heterotaxy syndrome. PREP Pearls Trisomy 21, or Down syndrome, is associated with congenital heart defects in 50% of patients. Atrioventricular septal defects and tetralogy of Fallot are among the most common congenital heart defects seen in patients with trisomy 21. The combination of atrioventricular septal defect with tetralogy of Fallot is almost always seen in association with trisomy 21. In patients with atrioventricular septal defects and tetralogy of Fallot, the atrioventricular valve morphology is usually classified as Rastelli type C.

A 3-month-old infant with severe supravalvar aortic stenosis is brought to the operating room for repair. Just after induction of anesthesia, he has a heart rate of 130 beats/min, a blood pressure of 70/40 mm Hg, and oxygen saturation of 100%. His rhythm on continuous electrocardiogram monitoring appears to be sinus, and there are no ST segment changes noted. Surgery is performed via median sternotomy. He is cannulated and placed on cardiopulmonary bypass with cardioplegic arrest and hypothermia. The surgeon repairs the supravalvar narrowing by using the 3-patch technique. During the procedure, the coronary artery ostia are inspected and probed. There is no ostial stenosis or web. Following repair, the patient is successfully separated from cardiopulmonary bypass. Postoperative transesophageal echocardiogram is performed (Video ). Approximately 2 hours later, continuous electrocardiogram monitoring begins to show ST segment elevation in leads II and III, and the patient has a heart rate of 160 beats/min, blood pressure of 75/45 mm of Hg, and oxygen saturation of 99%. Of the following, the MOST likely cause of ST segment elevation in this patient is A.air embolism to coronary artery B.coronary artery vasospasm C.inadequate systemic blood pressure D.residual supravalvar aortic stenosis

C.inadequate systemic blood pressure he most likely cause of ST segment elevation in this patient is inadequate systemic blood pressure leading to inadequate coronary perfusion and ischemia. This can quickly lead to cardiac arrest, and interventions are needed to increase the systemic blood pressure. Supravalvar aortic stenosis (SVAS) can lead to abnormalities in coronary artery perfusion. It is also associated with abnormalities of the coronary arteries, such as ostial stenosis, webs, coronary artery stenosis, or ectasia. The supravalvar ridge may also cause obstruction to coronary flow. Coronary arteries in patients with significant obstruction may also have thickening in the walls of the arteries leading to increased resistance. Prior to repair, the supravalvar narrowing leads to obstruction to the normal retrograde diastolic flow that should perfuse the coronary arteries. In association with the thickened walls of the coronary arteries, limited coronary perfusion may occur during diastole. The resistance to forward flow at the sinotubular junction causes a high pressure in the aortic root during systole and diastole, which provides better driving pressure for coronary perfusion in the already thickened coronaries. When the SVAS is relieved, even when there is relief of webs that cause obstruction to retrograde diastolic flow, the diastolic pressures may not be high enough to achieve adequate flow into these abnormal vessels. As a result, patients may require higher blood pressures after surgery than were needed preoperatively, unless the coronary arteries are unroofed. Air embolism and coronary artery vasospasm may also lead to inadequate coronary perfusion and ischemia. The patient did have instrumentation of the coronary arteries that could stimulate vasospasm, and air embolism can be seen in association with bypass surgery. Both were more likely to have happened earlier in the immediate intraoperative course, soon after separation from bypass. Residual SVAS is less likely to cause inadequate coronary perfusion, given the physiology described above. Severe residual obstruction may generate a high enough afterload to result in subendocardial ischemia if myocardial oxygen demand exceeds oxygen delivery. Contemporary surgical techniques are generally effective in relieving SVAS. PREP Pearls Supravalvar aortic stenosis is frequently associated with abnormalities in coronary arteries and coronary perfusion. Increased systemic pressures may be needed after relief of supravalvar aortic stenosis to maintain adequate coronary perfusion.

A 2-day-old term neonate was noted to have decreased urine output and poor feeding. He was born by cesarean delivery after prolonged rupture of membranes and failure to progress. His birth weight was 4.02 kg and Apgar scores were 8 and 9. The heart rate is 161 beats/min with a respiratory rate of 58 breaths/min. Blood pressures are 73/35 mm Hg (right arm), 70/32 mm Hg (left arm), 40/28 mm Hg (right leg), and 42/29 mm Hg (left leg). A systolic murmur is present. The liver is palpable 2 cm below the right costal margin. Pulses are 2+ in the arms and 1+ in the groin. Oxygen saturation is 92% in the right arm and 81% in the leg. He is started on prostaglandin E1 and transferred for cardiac evaluation. Echocardiography is performed (Video 1 and Video 2). The BEST surgical management of this patient is A.coarctation repair B.interrupted aortic arch repair C.interrupted aortic arch repair with closure of aortopulmonary window D.interrupted aortic arch repair with ventricular septal defect closure

C.interrupted aortic arch repair with closure of aortopulmonary window The echocardiogram clips demonstrate an aortopulmonary window with interrupted aortic arch type A (Figure 1 and Figure 2 ). The best surgical management of this patient is repair of both defects. Aortopulmonary window is a rare defect with a prevalence of 0.2% of patients with congenital heart disease. It is a failure of septation between the aortic and pulmonary trunks. It is often associated (50%) with other heart defects including interrupted aortic arch. The Society of Cardiothoracic Surgeons classifies aortopulmonary windows based on their location. Type 1 is located proximal in relation to the semilunar valves, type 2 is distal, and type 3 is total and includes the largest defects. The fourth type is indeterminate; it is neither closer to the semilunar valves nor the branch pulmonary arteries. Interrupted aortic arch is the lack of connection between the ascending and descending aorta. It is classified by location of the interruption. Type A is distal to the left subclavian artery, type B is between the left carotid artery and the left subclavian artery, and type C is between the innominate artery and the left carotid artery. Type B interruptions are the most common (50%-70%), followed by type A (30%-45%) and then type C (< 5%). Type A interruptions have the strongest association with the presence of an aortopulmonary window. Type B interrupted aortic arch and aortopulmonary window have also been reported but in much smaller numbers. Type B interruptions are more likely to be associated with a ventricular septal defect and 22q11.2 deletion. Aortopulmonary windows can also be classified as simple in the absence of associated cardiac defects and complex in the presence of an associated defect. Simple aortopulmonary windows present as a left-to-right shunt lesion usually in the first weeks after birth. The defect is typically large enough to result in pulmonary hypertension with signs and symptoms of congestive heart disease. Aortopulmonary window with interrupted aortic arch typically presents as a ductal-dependent systemic blood flow lesion. In addition to differential cyanosis and pulses, metabolic acidosis and shock may be present if the ductus arteriosus becomes inadequate. Echocardiography is diagnostic and usually provides adequate anatomic definition for surgical planning. It is easy to overlook the aortopulmonary window unless careful interrogation in multiple views is performed. If an aortopulmonary window is identified, a complete assessment of the cardiac anatomy is mandatory to identify any possible associated lesions. Conversely, if an interrupted aortic arch, particularly type A, is diagnosed, careful examination looking for an aortopulmonary window is required. If echocardiography is inadequate, magnetic resonance imaging or computed tomography is recommended. Cardiac catheterization is usually not required. PREP Pearls Interrupted aortic arch type A is associated with aortopulmonary window.

A 14-year-old adolescent girl comes to your office for evaluation of syncope. Last week, she passed out after soccer practice. She had been feeling well during practice and was able to keep up with her teammates and participate in all of the drills. Her exercise tolerance was no different than usual. The team did stretches and cool down for about 30 minutes on the field, then went back to the locker room. When the patient stood up after getting something out of the bottom of her locker, she felt lightheaded, her vision went black, and she passed out. Her teammates caught her as she fell, and they reported that she was unconscious for less than 30 seconds. When she awoke, she still felt a little lightheaded, but after drinking some water and resting for 15 minutes, she felt better. She does not recall any other symptoms preceding or during the episode, including any palpitations. Her medical history is unremarkable. She had one previous episode of syncope that occurred while in church 6 months ago. The event was otherwise similar to last week's event. Her mother is not aware of anyone in the family who has congenital heart disease, arrhythmias, sudden death, seizures, or syncope. She is on no medications or supplements. Findings of the physical examination are normal. An electrocardiogram is performed (Figure). The BEST next step in the treatment of this patient is to order A.a 24-hour Holter monitor B.an echocardiogram C.no further testing D.tilt-table testing

C.no further testing Syncope is common, and cardiologists are frequently consulted to determine its etiology and guide its management. Although cardiac syncope is rare in the pediatric population, syncope due to cardiac etiologies is often serious and may require intervention. Careful evaluation is needed to differentiate patients who have cardiac syncope from patients who have other types of syncope. The symptoms and event reported by the patient in this vignette represent classic neurocardiogenic syncope (NCS) or vasovagal syncope. When patients have classic symptoms, with no concerning features in the history, normal physical examination findings, and a normal electrocardiogram, further evaluation is rarely warranted. Neurocardiogenic syncope is the most common etiology of syncope, with more than one-third of individuals having had at least one episode by the time they reach their 60s. Adolescence is a common time for a first episode. Various triggers can lead to venodilation and venous pooling, with a subsequent decrease in cardiac output and insufficient cerebral blood flow. The decreased blood pressure is noted by baroreceptors, triggering a sympathetic response that increases heart rate and blood pressure. In NCS there is a vagal response that inhibits the sympathetic effects and instead leads to more venodilation and vasodilation and lower heart rate. There are numerous triggers for NCS, including position changes, prolonged standing or sitting, pain, exposure to medical situations, micturition, vagal stimulation, hairbrushing, and many others. The typical symptoms of NCS include a prodromal period of lightheadedness, nausea, flushing, diaphoresis, and blurring or blackout of vision that are followed by loss of tone and consciousness. The loss of tone and consciousness are brief (< 2 minutes) and self-resolving, as the vagal stimulation decreases and cardiac output returns to normal. Individuals may still feel tired or lightheaded for some time after regaining consciousness, and repeat episodes with position changes are possible. The evaluation of syncope should first include a detailed history of the event, including the activity and symptoms preceding the syncopal event, what happened when the patient passed out, and how the patient recovered from the event. A history of syncopal events or other cardiac symptoms should be determined. Family history of channelopathies, arrhythmias, syncope, sudden death, or seizures should also be obtained. Physical examination and electrocardiography should be performed. Further testing should be based on these findings. Further testing is not indicated if syncope occurs with the classic NCS prodromal symptoms and meets the following criteria: Syncope does not occur during exercise or as a result of a startlement or a loud noise. The cardiac examination is normal. The family history is negative for sudden death or inherited arrhythmias. There are no electrocardiographic abnormalities. Echocardiographic monitoring and/or long-term electrocardiographic monitoring (Holter or event monitors) yield little in this situation and should be used only if the above conditions are not met or if there are other factors indicating that there is a possible cardiac cause for syncope. Further testing may also be indicated if symptoms are recurrent and frequent or refractory to treatment. Tilt-table testing is not routinely done in the evaluation of NCS. A positive tilt-table test result does not confirm that the syncopal episode was NCS, only that the patient is at risk of experiencing NCS. It may be useful in limited situations when the diagnosis is unclear. Medical management for NCS may be indicated in patients with frequent episodes. Adequate fluid and salt intake, counterpressure maneuvers, and regular exercise are first-line treatments. Medications used to treat NCS are unproven, but they may be helpful in some patients. Fludrocortisone and midodrine have shown some benefit, although there are few randomized trials that support their use in children. PREP Pearls Neurocardiogenic syncope is the most common type of syncope in children. Evaluation of syncope should focus on patient and family history, physical examination, and electrocardiography, with additional testing being performed on the basis of these findings. A diagnosis of neurocardiogenic syncope can be made on the basis of the typical prodromal symptoms and triggers, the absence of concerning physical examination or family history findings, and normal findings on an electrocardiogram.

A 6-month-old infant with a history of an arrhythmia treated with the maximum dose of propranolol is brought to the emergency department for increased fussiness and irritability. His heart rate is 253 beats/min. Electrocardiography is performed, and the patient receives adenosine 0.1 mg/kg intravenously (Figure 1). The family has been compliant with medications with no missed doses. Echocardiography shows a structurally normal heart with normal function. Of the following, the BEST next pharmaceutical option in the management of this patient is A.digoxin B.esmolol C.sotalol D.verapamil

C.sotalol The electrocardiogram demonstrates a short RP narrow complex tachycardia. The retrograde P waves (Figure 2, red arrows) follow each QRS and fall in the first half of the R-R interval, thus making it a short RP tachycardia. After the administration of adenosine, the tachycardia terminates, and the beats following tachycardia (Figure 2, blue arrow) demonstrate a Wolff-Parkinson-White (WPW) pattern (short PR, delta wave, and wide QRS). In patients with WPW syndrome, atrioventricular nodal blocking agents like digoxin and calcium channel blockers (verapamil) are contraindicated. These agents can enhance accessory pathway conduction and increase the risk of sudden death in patients with WPW syndrome. Sudden death occurs during atrial fibrillation that conducts rapidly down the accessory pathway causing ventricular fibrillation. Sotalol is a class III antiarrhythmic with some weak β-blocker properties that is used in the management of supraventricular and ventricular arrhythmias in children. Class III antiarrhythmics block potassium channels during phase 3 of the action potential and thus prolong repolarization. The increase in the effective refractory period of the cells leads to decreased likelihood that early activation will lead to an new action potential, preventing reentrant arrhythmias. The prolonged action potential and increased refractory period are manifested in the surface electrocardiogram by prolongation of the QTc interval. For this reason, class III antiarrhythmics are contraindicated in patients with long QT syndrome. Esmolol is an intravenous β-blocker. It can be used in the management of supraventricular tachycardia, but in this clinical scenario the patient had already failed to respond to a β-blocker. Also, since the arrhythmia was terminated, an oral antiarrhythmic would be preferable over an intravenous one. PREP Pearls Atrioventricular nodal blocking agents, such as digoxin and calcium channel blockers, are contraindicated in patients with pre-excitation. Sotalol is a class III antiarrhythmic used in the management of supraventricular and ventricular arrhythmias in children. Electrocardiography should be performed in patients receiving a class III antiarrhythmic to monitor for QTc prolongation.

A 15-year-old girl is brought to your clinic because she has frequent panic attacks and palpitations, which appear to be increasing. The patient was healthy until 8 months ago when she began complaining of palpitations. Then 6 months ago she began experiencing brief periods of a rapid heart rate, sweatiness, mild nausea, and anxiety. These episodes usually lasted 5 to 20 minutes and appeared to resolve with rest. The last few episodes have lasted longer and the last one resulted in syncope at school. She recently has lost weight. Her mother has tried valerian root to calm her. She is not taking any other medications or supplements. She does have a cup of coffee in the morning. She has no family history of sudden or early cardiac death. On physical examination, she is a tall, thin girl with mild anxiety. Her vital signs are as follows: Height: 165.1 cm (67.4%) Weight: 45 kg (17.4%) Blood pressure: 148/60 mm Hg Heart rate: 144 beats/min Respiratory rate: 30 breaths/min Temperature: 37.2ºC Oxygen saturation: 98% on pulse oximetry She has no rash. Findings on a head, eyes, ears, nose, and throat examination are normal, with the exception of mildly injected eyes. The thyroid is mildly, diffusely enlarged. Her lungs are clear to auscultation. On cardiovascular examination, she has tachycardia with an irregular rhythm. She has a grade I/VI vibratory systolic ejection murmur heard best at the lower left sternal border with radiation to the left ventricular outflow tract. The murmur disappears on standing. Her pulses are full without lag. She has a scaphoid abdomen with no organomegaly. She has mild tremor when she extends her arms. A screening echocardiogram showed no structural heart disease. Of the following, the electrocardiogram that is MOST consistent with the patient's underlying diagnosis is A. Figure 60.1 B. Figure 60.2 C. Figure 60.3 D Figure 60.4

D Figure 60.4 The patient in the vignette presents with the classic symptoms of hyperthyroidism, most likely Graves disease, with concomitant atrial fibrillation. Symptoms include fatigue, tachycardia, tremor in the arms or hands, difficulty sleeping, diarrhea, weight loss despite a good appetite, and sweating. The patient displayed all of these symptoms including a widened pulse pressure and an enlarged thyroid on physical examination. Symptoms are often dismissed as panic attacks unless the physician recognizes and tests for a thyroid disorder. Graves disease is the most common cause of hyperthyroidism. Hashimoto thyroiditis may also cause cardiac tachyarrhythmias. It is not clear why Graves disease develops in most people, though it is more common in certain families. This disorder is most common in women between the ages of 20 and 40 years, but can occur at any age in children, teens, and adults. Women are at higher risk. In some cases, patients may develop eye problems (Graves ophthalmopathy or orbitopathy), which cause dry, irritated, or red eyes, and in severe cases, may cause double vision. The patient in the vignette did not have proptosis. In its most severe form, individuals with Graves ophthalmopathy can develop inflammation of the optic nerves, which can result in loss of vision. The girl in the vignette complained of palpitations and episodes of tachycardia. The heart rate was irregular and varied with conduction. When conducted at higher rates, patients often complain of lightheadedness and/or syncope. This girl's electrocardiogram shows atrial fibrillation. Atrial fibrillation is less common than atrial flutter in children. It is usually at a rate of 350 to 600 beats/min with an irregular ventricular response and normal QRS complexes. Atrial fibrillation occurs in approximately 13% of all people with hyperthyroidism. It is estimated that hyperthyroidism accounts for 5% of cases of atrial fibrillation. Atrial fibrillation can be seen in pericarditis, pulmonary emboli, Ebstein anomaly, and dilated cardiomyopathy. It can also be seen in other cardiac defects and after cardiac surgery. The mechanism is felt to be related to the stretching of the atria. Other differential diagnoses of atrial fibrillation in a child or teen with a structurally normal heart include drugs that stimulate the heart. These include caffeine, theophylline, attention-deficit disorder medications, and even excessive consumption of energy drinks. Sleep apnea, especially in adults, has been reported to be associated with atrial fibrillation. Patients with atrial fibrillation who are overweight or have a history of snoring or excessive sleepiness during the daytime may benefit from a sleep study. It is of interest that this patient also had a widened pulse pressure and systolic hypertension. A widened pulse pressure is characteristic of hyperthyroidism. In hyperthyroidism, systemic vascular resistance decreases, and blood volume and perfusion in peripheral tissues increase. Hyperthyroidism is associated with increased vascularity and some suggest that T3 may increase capillary density via increased angiogenesis. It is not clear how hyperthyroidism leads to a predisposition to atrial fibrillation. Although Wolff-Parkinson-White (WPW) syndrome (option A) may be associated with supraventricular tachycardia (SVT) and atrial fibrillation, this patient's history, vital signs, and physical examination are classic for Graves disease. Torsades de pointes (option B) is associated with congenital or acquired long QTc syndrome. Her family history was negative for sudden death and she was not taking any QTc lengthening medications. SVT (option C) can present with palpitations, tachycardia, and syncope, but the patient in the vignette had an irregular rhythm and her history and physical examination included the classic findings of Graves disease. In summary, elevation of heart rate, myocardial contractility, stroke volume, myocardial oxygen consumption, systolic blood pressure, and reduction in systemic vascular resistance and diastolic blood pressure often can be seen in hyperthyroidism and even in cases of subclinical hyperthyroidism. The most common cardiac complications are arrhythmias (mainly atrial fibrillation), heart failure (usually in adults), and hypertension. The practitioner should consider Graves disease in the differential diagnosis in patients presenting with atrial fibrillation. A thyroid panel will help to rule out Graves disease; however, obtaining thyroid peroxidase and thyroglobulin antibodies will also help to rule out Hashimoto thyroiditis. PREP Pearls Hyperthyroidism may cause tachyarrhythmias, and so patients must be tested for it. Hyperthyroidism may be associated with sinus tachycardia and widened pulse pressure. Examining the thyroid gland is important in patients referred for tachyarrhythmias.

A 19-year-old woman with a history of mitral valve prolapse and moderate mitral regurgitation is admitted with a 1-week history of daily fevers to 104°F, chills, and myalgias. She was treated with oseltamivir without effect and discontinued therapy because of vomiting. Two days before admission, she noted the onset of red painful lesions on her feet, which then appeared on her hands the day before admission. Blood cultures are sent and empiric antibiotic therapy is started. Transthoracic echocardiography reveals a 6-mm oscillating mass on the anterior leaflet of the mitral valve. Over the next 48 hours she develops severe unrelenting headaches. Magnetic resonance imaging of the brain demonstrates multiple areas of hyperintensity with gray-white signal change. Of the following, the MOST likely infectious cause of the underlying illness in this patient is A. Enterococcus species B. Haemophilus species C. Pseudomonas species D. Staphylococcus species E. Streptococcus species

D. Staphylococcus species The patient in the vignette has a clinical course and findings that are most consistent with definite infective endocarditis (IE) complicated by a mycotic aneurysm. Her presentation includes 1 major (evidence of endocardial involvement on echocardiography) and 4 minor Duke criteria: a predisposing heart condition (mitral valve prolapse), fever, vascular phenomenon (mycotic aneurysm), and immunologic phenomenon (Osler nodes). The Duke criteria for the diagnosis of IE were originally proposed in 1994 and revised in 2000 (Table). In the revision, IE was defined as definite in the presence of the following criteria: 1 pathologic, 2 major clinical, 1 major and 3 minor clinical, or 3 minor. Major criteria include positive blood cultures; evidence of endocardial involvement; a positive echocardiogram with oscillating intracardiac mass, abscess, or new partial dehiscence of a prosthetic valve; or new valvular regurgitation. Minor criteria include predisposition, fever, vascular phenomena, immunologic phenomena, and microbiologic evidence not reaching the level required to meet the definition of major critieria. For the patient in the vignette, the most likely cause of endocarditis is Staphylococcus species. S aureus is recognized as the most common cause of IE in the developed world and primarily involves the left side of the heart when the endocarditis is not associated with intravenous drug abuse. Both coagulase-positive and coagulase-negative species of Staphylococcus have been implicated as causes of IE and can affect both native and prosthetic valves. However, coagulase-positive strains such as S aureus are more typically associated with infections of native valves and negative strains affect prosthetic valves. The incidence of systemic embolization in IE has been reported to occur in as many as half of all cases, with almost two-thirds of those involving the central nervous system (CNS), usually in the distribution of the middle cerebral artery. Higher mortality is associated with endocarditis complicated by CNS involvement compared with cases without CNS involvement. CNS complications are most commonly seen in patients infected with Staphylococcus species but other frequent causes include Streptococcus and Haemophilus, Aggregatibacter (previously Actinobacillus), Cardiobacterium, Eikenella corrodens, and Kingella organisms. The time course of CNS embolization in IE varies depending on the causative organism. Staphylococcus species typically lead to early embolization, before the initiation of antibiotics or within the first week of therapy, whereas other causative agents such as Streptococcus viridans are associated with a more indolent course, as is partially treated endocarditis. Successful early control of infection is associated with a marked decrease in the rate of CNS embolization. The risk of CNS embolization is also affected by the site of endocarditis; the risk is highest with mitral valve disease, particularly that of the anterior leaflet, and to some extent, with the presence of larger vegetations, especially if those vegetations are caused by streptococcal species. The rate of CNS embolization in staphylococcal IE is high regardless of the size of the vegetation. Mycotic aneurysms occur in up to 15% of patients with IE, most often in the intracranial circulation, particularly the distal aspects of the middle cerebral artery territory and at arterial branching points, but can also be seen in the visceral arterial circulation as well as that of the extremities. They are typically caused by bacteria-induced weakening of the blood vessel wall, which occurs with spread of the infection and inflammatory response from septic emboli outward from the intraluminal space through the vessel wall. Clinical presentation of mycotic aneurysms can include headaches, altered mental status, seizures, or focal neurologic deficits. The presence of mycotic aneurysms portends a particularly dangerous course, with an overall mortality rate of more than 50%, and nearly 80% in patients in whom the aneurysms rupture. Although Streptococcus species are more common causes of mycotic aneurysms, in the patient in the vignette, Staphylococcus is a more likely cause of the endocarditis given the infection of a native left-sided heart valve, systemic embolization in the face of relatively small vegetation, and the early appearance of the mycotic aneurysm. PREP Pearls In the 2000 revision of the Duke criteria, infective endocarditis (IE) was defined as definite in the presence of a single pathologic criterion, 2 major clinical criteria, 1 major and 3 minor clinical criteria, or 3 minor criteria. Staphylococcus species typically lead to early embolization, before the initiation of antibiotics or within the first week of therapy, whereas other causative agents are associated with a more indolent course. The risk of central nervous system (CNS) embolization is also affected by the site of endocarditis, with mitral valve disease, particularly that of the anterior leaflet, carrying the highest risk. Also involved to some extent, is the presence of larger vegetations, especially if those vegetations are caused by streptococcal species. The rate of CNS embolization in staphylococcal IE is high regardless of the size of the vegetation. S aureus is recognized as the most common cause of IE in the developed world and primarily involves the left side of the heart when the endocarditis is not associated with intravenous drug abuse.

A 13-month-old male infant with a medical history significant for Noonan syndrome presents to the emergency department with increased work of breathing and poor feeding for 24 hours. He is taking propranolol 2 mg/kg per day, and receiving bottle feeding complemented with G-tube feeding for a history of aspiration. His mother states that he may have aspirated during an attempt at bottle feeding the previous day. Physical examination shows an infant with temperature of 101°F, heart rate of 158 beats/minute, respiratory rate of 64 breaths/minute with retractions, and saturation of 92% on room air. His upper extremity blood pressure is 80/48 mm Hg and capillary refill is 3 to 4 seconds. On physical examination, he is alert but anxious. Chest auscultation reveals diminished air entry to the right chest. Cardiovascular examination reveals tachycardia, forceful apical impulse, and 3/6 harsh ejection systolic murmur, best audible at the left mid- to upper sternal border. The abdomen is soft, and the liver is 1 cm below the costal margin. An echocardiogram is obtained (Video 1 and Video 2). Of the following, the BEST next step in the management is to A. administer intravenous (IV) furosemide B. begin IV ceftriaxone after drawing a blood culture specimen C. initiate albuterol nebulizer treatment D. initiate IV fluid bolus E. initiate IV verapamil

D. initiate IV fluid bolus The patient in the vignette has a known diagnosis of Noonan syndrome, and 20% to 30% of patients with Noonan syndrome have hypertrophic cardiomyopathy (Video 1 and Video 2). Infants with hypertrophic obstructive cardiomyopathy pose a significant challenge to the physician during an acute respiratory illness. In hypertrophic obstructive cardiomyopathy, the obstruction to the left ventricular outflow is dynamic, varying with loading conditions and ventricle contractility. Fever and poor feeding associated with the acute respiratory illness reduce the intravascular volume, increase the dynamic obstruction, and lower the cardiac output. The associated tachycardia also impairs the diastolic filling, thus exacerbating the clinical situation. The infant in the vignette presents with respiratory distress in the setting of dynamic systemic outflow obstruction. The mainstay of therapy in such patients is intravascular volume repletion and rate control with beta-blocker therapy. If prompt intervention is not achieved, these patients can decline to cardiopulmonary arrest because of their minimal cardiac reserve. Hence initiating fluid bolus would be the correct choice for the patient in the vignette. Although albuterol nebulizer therapy is frequently used in the pediatric emergency department (ED) setting for acute respiratory illness, it should be strongly discouraged in hypertrophic obstructive cardiomyopathy because of the tachycardia and worsening of dynamic obstruction. Workup for pneumonia and sepsis is warranted once the patient is stabilized in the ED. Intravenous furosemide can cause further volume depletion and should be discouraged. Verapamil is a calcium channel blocker that would not be helpful in the above-described pathophysiology and is contraindicated in infants. PREP Pearls Infants with hypertrophic obstructive cardiomyopathy have minimal cardiopulmonary reserve and need vigilant evaluation. Intravascular volume repletion and rate control are the mainstay of therapy. Albuterol nebulizer therapy should be discouraged in hypertrophic obstructive cardiomyopathy.

A full-term 2-month-old infant presents to the emergency department with increased work of breathing and "noisy breathing." His parents first noticed the noisy breathing soon after he came home after delivery and state that it has been getting progressively worse. He is still feeding well, though he has increased work of breathing while feeding. His medical history is otherwise unremarkable. On physical examination, he is sleeping, but in moderate distress with nasal flaring and inspiratory and expiratory stridor. His lung fields are clear and his cardiac examination findings are normal. You suspect a vascular ring and obtain a computed tomography angiogram (Figure 1). Of the following, the abnormal embryologic process that BEST explains this patient's symptoms is A. involution of the left fourth arch B. involution of the right fourth arch C. persistence of the fifth arch D. persistence of the right dorsal aorta

D. persistence of the right dorsal aorta The patient in the vignette displays symptoms of upper airway obstruction. These symptoms are caused by airway compression by the vascular structures. The computed tomography (CT) angiogram demonstrates a double aortic arch. Double aortic arch is the most common vascular ring and stridor is the most common presentation in infancy. The normal left aortic arch and its branches develop from the 6 pairs of vessels (arches) that connect the truncoaortic sac with the dorsal aortae. Some portions of these vessels persist, while others disappear. The totipotential arch has been used to understand which embryologic structures contribute to the end-result, be it normal or abnormal. The embryologic structures that are part of the totipotential arch include the truncoaortic sac, the third, fourth, and sixth arches, the seventh intersegmental arteries, and dorsal aortae (Figure 2). The normal left aortic arch with left ductus arteriosus (which becomes the ligamentum arteriosus after it closes) results from the regression of the right sixth arch and the right dorsal aorta (Figure 2). Double aortic arch is the result of persistence of the right dorsal aorta. The right fourth aortic arch persists, but unlike in the normal left arch where it becomes a part of the right subclavian artery, it continues to form an arch connecting the truncoaortic sac and the dorsal aorta. The result is much like the totipotential arch (in double aortic arch, the right sixth arch typically disappears) with an arch on either side of the trachea. One arch, usually the right, is dominant. Some patients may have atretic portions of one of the arches. The area of atresia will determine how the head vessel branching appears. When both arches are patent, there are 4 separate head vessels (4-vessel sign), 2 from each arch. If there is an atretic portion, there may appear to be 2 branches arising together, much like an innominate artery, making the double arch harder to identify. The involution of the left fourth aortic arch (in association with the normal involution of the right dorsal aorta and right sixth arch) results in aortic arch interruption type B. The left sixth arch persists as normal, and provides blood flow to the descending aorta via the ductus arteriosus. When this occurs, there is no connection between the ascending and descending aortae and the division is between the left common carotid artery and the left subclavian artery. This is a ductal-dependent lesion and does not result in a vascular ring. The involution of the right fourth arch leads to a left aortic arch with an aberrant right subclavian artery. The right subclavian artery is formed from the right dorsal aorta and the seventh intersegmental artery. This does not compose a vascular ring, though the retroesophageal course of the artery can cause compression of the esophagus, and rarely, cause symptoms of dysphagia. The right and left fifth arches normally regress and compose no part of the normal aortic arch. In rare instances, the fifth arch persists and forms a connection between the ascending and descending aortae on the same side of the trachea as the normal transverse arch. It does not form a vascular ring and becomes clinically significant only if there is associated interruption in the normal transverse arch. PREP Pearls Double aortic arch results from persistence of the right dorsal aorta. Double aortic arch forms a vascular ring that causes upper airway and esophageal obstruction. Understanding the embryology of the aortic arch helps to explain the arch anomalies and vascular rings.

You arrive in the catheterization laboratory to view the catheterization results of a 7-year-old boy with coarctation of the aorta and bicuspid aortic valve. He underwent neonatal coarctation repair with resection and end-to-end anastomosis. You referred him for a catheterization after his echocardiogram demonstrated a high flow velocity in the left ventricular outflow tract. There was no aortic insufficiency. The right heart catheterization data were unremarkable. You review the pressure tracing obtained by pulling the end-hole catheter from the left ventricle to the descending aorta (Figure). Of the following, the BEST next step in the management is to A. perform balloon aortic angioplasty B. perform balloon aortic valvuloplasty C. place a stent in the left ventricular outflow tract D. refer for surgery E. treat with propranolol

D. refer for surgery Subaortic stenosis is often associated with left-sided obstructive lesions such as coarctation of the aorta. Cardiac catheterization may be useful to delineate the severity of an individual stenosis. Surgery is the standard therapy for subaortic stenosis. For the boy in the vignette, the severe obstruction (peak systolic gradient >50 mm Hg) is an indication for surgical resection of the subaortic membrane. The pressure tracing shown in the vignette demonstrates a pullback from the left ventricle to the descending aorta (Figure). The tracing is recorded as an end-hole catheter is moved from the left ventricle to the descending aorta during a pullback maneuver. The pullback technique is useful for identifying a discrete stenosis. An end-hole catheter is required to ensure that the tracing does not represent pressures from both sides of the stenosis. The recorded pressures demonstrate a significant obstruction in the left ventricle because the systolic pressure drops before the tracing changes to an aortic tracing. The systolic pressure does not change as the tracing loses a ventricular waveform and assumes an aortic waveform. The systolic pressure is the same in the ascending and descending aorta. Therefore the boy in the vignette does not have more distal obstruction. Aortic valve stenosis, supravalvar aortic stenosis, and recurrent coarctation of the aorta are not present. Discrete subvalvar aortic stenosis caused by membranous or fibromuscular obstruction is the most likely explanation for the hemodynamic data. Subvalvar aortic stenosis is commonly associated with other left-sided obstructive lesions, including coarctation of the aorta and aortic valve abnormalities. Subaortic stenosis is rare in neonates and infants. When it occurs in the setting of other left-sided obstructive lesions it usually is not detected until years after the initial diagnosis. Although balloon dilation has been reported for this condition, subaortic stenosis may recur and surgical intervention is the recommended treatment for this lesion. Balloon aortic valvuloplasty is not indicated because there is no significant aortic valvular stenosis. Placing a stent in the left ventricular outflow tract is not recommended. Medical management with propranolol is not beneficial or indicated. The boy in the vignette has a fixed, not dynamic obstruction. The catheterization data do not support the presence of recurrent coarctation or aortic valve stenosis. PREP Pearls Pressure pullback with end-hole catheter accurately identifies the location of stenosis. Subvalvar aortic stenosis is often associated with other lesions.

A 7-year-old boy was transported by ambulance to the emergency department (ED). He had been playing at recess when the teacher saw him fall. He did not hit his head or lose consciousness. When she went over to check on him he was having trouble getting up because his left side was not moving well and would not support him. In the ED, the only additional information obtained was a history of asthma since age 5 years. He often coughed and got short of breath with exercise despite inhalers. He has also been seen for wheezing-associated lower respiratory illnesses. His father had asthma as a child, which he outgrew. He had no family history of cardiac disease, except for some maternal great aunts and uncles who had coronary stents and bypass surgery in their 60s and 70s. Head computed tomography performed at the ED showed a right-sided stroke. An electrocardiogram was also obtained (Figure 1) and you were asked to evaluate the child for possible cardiac cause for his stroke. On examination, he is found to be afebrile. His height is at the 25th percentile and weight is at the 15th percentile. His heart rate is 106 beats/min, respiratory rate is 16 breaths/min, and blood pressure is 100/60 mm Hg. His examination is notable for clear lung fields, mildly increased right ventricular impulse, normal S1, and narrowly split S2, with no murmurs. There is an S4. The liver is 4 cm below the right costal margin. No peripheral edema is noted. Of the following, the MOST likely underlying diagnosis is: A. arrhythmogenic right ventricular cardiomyopathy B. idiopathic pulmonary arterial hypertension C. myocarditis D. restrictive cardiomyopathy

D. restrictive cardiomyopathy Restrictive cardiomyopathy (RCM) is an uncommon form of cardiomyopathy that is a disease of diastolic dysfunction. The average age at diagnosis is 6 years and appears to affect boys and girls in equal numbers. Respiratory complaints, including shortness of breath with exercise or exercise intolerance, are common and these children frequently have a history of "asthma" or "lower respiratory tract infections." Because of left-sided diastolic dysfunction, left atrial hypertension and pulmonary venous congestion result in respiratory tract symptoms. Pulmonary hypertension may develop and manifest on physical examination with an increased right ventricular impulse and an abnormal second heart sound with narrow or absent splitting and a loud P2, depending on the severity of the pulmonary hypertension. The diastolic dysfunction is almost always biventricular, therefore signs and symptoms of systemic venous congestion are also common with hepatomegaly, ascites, and at times, peripheral edema. Patients often see a pulmonologist or gastroenterologist before the correct diagnosis is made. Most patients have abnormal physical examination findings at the time of presentation, and electrocardiograms (ECGs) are abnormal about 98% of the time at presentation. This patient demonstrates typical findings of biatrial enlargement, ST-T wave abnormalities, as well as biventricular hypertrophy. Evidence of right and/or left ventricular hypertrophy is also common. Thromboembolic events have been reported to occur in about 20% of patients with RCM, and can be 1 of the first symptoms/signs. The exact cause is not clear but is likely stagnant flow in the dilated atria, leading to thrombus formation. Some patients develop atrial arrhythmias, including atrial fibrillation, which could also be a contributing factor. The onset of arrhythmogenic right ventricular cardiomyopathy is rare before the age of 10 years, with the mean age at diagnosis being 30 years. It typically presents with an arrhythmia or aborted sudden death. The most common presenting complaint is palpitations. Dizziness and syncope are also common complaints. Classic ECG (Figure 2) findings include an Epsilon wave (seen at the end of the S wave/beginning of the T wave in V1) or localized prolongation (>110 msec) of the QRS complex in V1-V3 and inverted T waves in V2 and V3 in patients older than 12 years and in the absence of right bundle branch block. Aborted sudden death may result in neurologic impairment, but an initial strokelike picture would be uncommon. Idiopathic pulmonary hypertension may also present with shortness of breath, fatigue, or syncope. Other clinical symptoms may include seizures, hemoptysis, chest pain, dizziness, and in advanced disease, symptoms of right heart failure. Physical findings include a single loud and sometimes palpable P2. Murmurs of pulmonary or tricuspid regurgitation may be heard. If right heart failure has developed, there may be a gallop, hepatomegaly, and peripheral edema. The ECG often demonstrates evidence of right ventricular hypertrophy and upright T waves in the right chest leads (Figure 3). Presenting symptoms may be syncope or hypoxic seizures, but an initial strokelike picture would be uncommon. Myocarditis may occur at any age, but typically presents with overt heart failure. However, the disease has a broad spectrum ranging from subclinical to cardiogenic shock at presentation. Chest pain or discomfort, shortness of breath, decreased exercise tolerance, and fatigue may be presenting symptoms. On ECG, sinus tachycardia is common with or without low-voltage QRS complexes and/or inverted T waves. Figure 4 is an ECG from a patient with myocarditis demonstrating sinus rhythm with left atrial enlargement, low-voltage QRS complexes throughout, and diffuse ST-T wave changes. PREP Pearls The signs and symptoms of restrictive cardiomyopathy (RCM) often mimic problems in other systems such as pulmonary or gastrointestinal. Almost all children with RCM will have abnormal findings on physical examination and electrocardiography at presentation.

You are asked to evaluate a 5-year-old boy who has been admitted for an asthma exacerbation and in whom a murmur has been noted apparently for the first time. Besides his asthma, he has no other significant medical history. In questioning his parents, you learn that he has no symptoms referable to the cardiovascular system. A physical examination reveals a thin child with a dynamic precordium. There is a normal S1 and a widely and fixed split S2. There is a grade III/VI crescendo-decrescendo systolic murmur heard best at the upper left sternal border but widely through the precordium, radiating to the neck bilaterally and into the axillae and the back. His pulses are strong and equivalent throughout all 4 extremities. There are no other abnormal findings. Of the following, the MOST likely diagnosis in this patient is A. aortic valve stenosis B. muscular ventricular septal defect C. pulmonary valve stenosis D. secundum atrial septal defect

D. secundum atrial septal defect The physical examination findings of the patient in the vignette are typical of those of a patient with a secundum atrial septal defect. In that setting, the left-to-right atrial level shunt results in a volume load on the right side of the heart, increased right ventricular stroke volume, and an expansion in the flow being carried across the pulmonary valve, which itself is usually normal. This increased volume, in turn, leads to a dynamic precordium and a murmur of relative pulmonary stenosis appreciated as a systolic ejection murmur with a radiation pattern in the pulmonary arterial distribution, characteristically to the axillae and into the back. The increased flow across the pulmonary valve also causes its closing to be relatively delayed resulting in an increasingly split second heart sound as the normal separation of P2 from A2 is augmented. This widened splitting is also often fixed, as the normal inspiratory exaggeration associated with it is lost. In this case, while the quality of the murmur could be consistent with that of valvar pulmonary stenosis, and less likely that of valvar aortic stenosis given the description of its radiation pattern, the absence of a systolic click on physical examination makes the presence of valve pathology less probable. A muscular ventricular septal defect would be expected to have a well-localized holosystolic murmur without associated findings such as a dynamic precordium or fixed splitting of S2. PREP Pearls An atrial septal defect results in a volume load on the right heart with increased flow across an often normal pulmonary valve. The increased flow across the pulmonary valve results in a murmur of relative pulmonary stenosis, as well as widened fixed splitting of S2. Pathology of the aortic and pulmonary valves is usually associated with the presence of a systolic click.

You receive a call from a community hospital about a 2-day-old full-term male neonate with cyanosis and respiratory distress. He had an uncomplicated prenatal course and delivery. He was with his mother in the mother-baby unit until this morning when he was noted to have difficulty feeding and had tachypnea. He developed progressive respiratory distress and tachypnea. His current vital signs show a heart rate of 177 beats/min, respiratory rate of 60 breaths/min, blood pressure of 62/40 mm Hg, oxygen saturation of 70% on continuous positive airway pressure of 5 cm H2O, and 100% oxygen. He is cyanotic appearing and in moderate respiratory distress. There are subcostal retractions and tachypnea. His abdomen is soft and his liver is palpable 2 to 3 cm below the costal margin. His pulses are 1+ and symmetric and his capillary refill is about 3 seconds. The chest radiograph is reported to show a normal cardiac silhouette with bilateral pulmonary edema and increased pulmonary vascular markings. Brief echocardiography is performed and transmitted for you to view (Video 1 , Video 2, Video 3). Of the following, the BEST next step in the management of this patient is to A. start diuretics B. start epinephrine infusion C. start nitric oxide D. transfer for emergent surgery

D. transfer for emergent surgery The clinical description and echocardiogram in the vignette are consistent with infracardiac total anomalous pulmonary venous connection (TAPVC) with obstruction. Total anomalous pulmonary venous connection is a cardiac defect in which the pulmonary venous return is to a systemic vein or the right side of the heart. The return of the pulmonary veins is classified as supracardiac (45%-50%), cardiac (15%-20%), infracardiac (20%-25%), or mixed (10%). Depending on the type, infants may present with cyanosis and/or respiratory distress (persistent pulmonary hypertension of the newborn) or with failure to thrive, congestive heart failure, and minimal cyanosis. The presentation depends on whether the pulmonary venous return is obstructed. Obstruction can occur at any level along the pulmonary venous pathway. Most patients with infracardiac TAPVC have obstruction, but it can be present in any form of TAPVC. In infants with TAPVC, the level of cyanosis depends on the presence of obstruction. When there is no obstruction and pulmonary vascular resistance is low, the Qp to Qs ratio will be high, leading to saturations in the 90s range. With obstruction, the Qp to Qs ratio is lower. The obstruction will lead to pulmonary capillary hypertension. This causes elevations in pulmonary artery pressures and pulmonary edema. Since pulmonary edema can be quite severe, there is increased V-Q mismatch and the pulmonary venous return will not be fully oxygenated. The neonate in the vignette has cyanosis with an oxygen saturation of 70%. The differential diagnosis includes congenital heart defects with ductal-dependent pulmonary blood flow (pulmonary atresia) and other cyanotic lesions with mixing of pulmonary and systemic venous return (single ventricle defects, Ebstein anomaly, truncus arteriosus, TAPVC). The profound cyanosis is typical of ductal-dependent pulmonary blood flow, but the presence of pulmonary edema and increased pulmonary vascular markings rule this out. Infants with hypoplastic left heart syndrome may become profoundly cyanotic when the ductus closes and there is minimal systemic blood flow, even though hypotension and other signs of shock may be expected. The echocardiogram shows right-to-left shunting across the atrial septal defect, which is not present in hypoplastic left heart syndrome, where a left-to-right shunt is necessary. Patients with TAPVC, however, must have a right-to-left shunt. The diagnosis of TAPVC in the vignette is also supported by the echocardiographic image showing blood flow traveling inferiorly, below the diaphragm, and into the liver. This is abnormal and characteristic of infracardiac TAPVC. The turbulence seen at the distal part of the vessel, in association with the clinical signs, is suggestive of TAPVC with obstruction. Total anomalous pulmonary venous connection with obstruction is a surgical emergency. Both nitric oxide and prostaglandins will increase pulmonary blood flow, and as the egress for pulmonary blood flow is obstructed, pulmonary venous pressure will be increased and pulmonary edema will worsen. Diuresis may be indicated, but only as a way to improve pulmonary edema perioperatively. Epinephrine may be indicated in patients with hypotension, but will be minimally effective in increasing cardiac output. PREP Pearls Total anomalous pulmonary venous connection (TAPVC) should be suspected in infants with profound cyanosis, respiratory distress, and pulmonary edema on chest radiograph. Total anomalous pulmonary venous connection with obstruction is typically a surgical emergency. Prostaglandins may worsen pulmonary edema in patients with obstructed TAPVC.

A 12-year-old boy is referred to you for evaluation after a first syncopal episode in gym class. His foster parents report that he is being evaluated for a sensorineural hearing loss. His family history is unavailable. He has a blood pressure of 109/64 mm Hg, heart rate of 88 beats/min, respiratory rate of 18 breaths/min, weight of 40 kg (50th percentile), and height of 139 cm (10th percentile). He has black-brown macules on his forehead, cheeks, chin, neck, and upper chest. He has mild hypertelorism and a high-arched palate. A moderate pectus excavatum is noted. A grade 2/6 systolic ejection murmur is heard at the lower left sternal border and radiates to the left ventricular outflow tract while sitting. The murmur increases in intensity on standing. He has normal pulses. Left ventricular hypertrophy with strain is seen on electrocardiography. Of the following, the syndrome that is MOST likely associated with this child's findings is A.Carney complex B.Costello syndrome C.neurofibromatosis type 1 D.Noonan syndrome with multiple lentigines

D.Noonan syndrome with multiple lentigines The patient in this vignette has classic findings of Noonan syndrome with multiple lentigines. He has a history of syncope and sensorineural hearing loss. He is short and has multiple lentigines and café au lait spots, a pectus excavatum, a left ventricular outflow tract murmur (that increases on standing), and an abnormal electrocardiogram showing left ventricular hypertrophy with strain. Lentigines are seen mainly on the face, neck, and upper chest but spare the mucosa. They may be preceded by café au lait spots. Noonan syndrome with multiple lentigines (NSML) was formerly called LEOPARD syndrome, which is a mnemonic for the major manifestations of this disorder (lentigines, electrocardiogram abnormalities, ocular hypertelorism, pulmonic stenosis, abnormal genitalia, growth retardation, and sensorineural deafness). In patients with Noonan syndrome with multiple lentigines, hypertrophic cardiomyopathy is the most common cardiac abnormality and is detected in up to 80% of the patients with a cardiac defect (but was not included in the original Leopard mnemonic). Hypertrophic cardiomyopathy in these patients is usually asymmetric and progressive and commonly involves the interventricular septum. Significant left ventricular outflow tract obstruction is seen in up to 40% of the cases. Pulmonary stenosis is seen in approximately 20% to 25% of patients. Additional cardiac abnormalities reported include aortic valve abnormalities (mild aortic regurgitation, discrete subaortic stenosis, or aortic valve dysplasia), mitral valve prolapse, coronary abnormalities, apical aneurysm of the left ventricle, left ventricular noncompaction, isolated left ventricular enlargement, atrioventricular septal defects, and ventricular septal defects. Other abnormalities may include chest wall abnormalities and mild learning disorders. Recent genetic research has led to an identification of a new group of genetic syndromes called the Ras/mitogen-activated protein kinase (MAPK) pathway disorders or RASopathies. These syndromes are also called neuro-cardio-facial-cutaneous syndromes. The Ras/MAPK pathway protein mutations have been associated with Noonan syndrome, gingival fibromatosis type 1, neurofibromatosis type 1 (NF1), capillary malformation-arteriovenous malformation, Costello syndrome, autoimmune lymphoproliferative disorder, cardiofaciocutaneous syndrome, and Legius syndrome. The underlying genetic mechanism in these syndromes may result in an overlap of phenotypic features with characteristic facial anomalies, cardiac defects, cutaneous and ocular abnormalities, growth deficits, varying degrees of neurocognitive impairment, and an increased risk of cancer. Approximately 85% of patients with Noonan syndrome with multiple lentigines have mutations in PTPN11. Costello syndrome is associated with a large mouth, small wart-like growths that may develop around the nose and mouth, poor growth, delayed intellectual development, hypotonia, and increased risk of certain tumors. The most common noncancerous tumors associated with this condition are papillomas. The facial features of the patient in this vignette are not consistent with Costello syndrome. However, these individuals may also have hypertrophic cardiomyopathy and should be monitored appropriately. Patients with Carney complex have multiple lentigines, myxomas, and multiple endocrine and nonendocrine tumors. Patients previously diagnosed with LAMB syndrome (lentigines, atrial myxomas, myxoid neurofibromas, and ephelide) or NAME syndrome (nevi, atrial myxoma, blue nevi) are now classified as having Carney complex. These patients may have blue nevi and lentigines most commonly on the face, especially in perioral (vermillion borders of the lips) and periocular (lacrimal caruncle and inner and outer canthi) areas. They may have both cutaneous myxomas (30%-55% of patients) and/or cardiac myxomas (20%-40% of patients). Cardiac myxomas may be recurrent and lead to embolic strokes and heart failure. The facial features, lentigines distribution, cardiac features, and other physical examination findings in the patient in this vignette are not consistent with Carney complex. However, both hypertrophic cardiomyopathy and cardiac myxomas may present with syncope. A mid-diastolic murmur and low-pitched tumor plop are characteristic findings of a myxoma, but symptoms and presentation may vary with myxoma location. Any patients with cardiac myxomas (especially recurrent) with cutaneous manifestations should be referred for a genetic evaluation to rule out Carney complex. Neurofibromatosis type 1 is associated with multiple café au lait spots and axillary, underarm, and groin freckling. In adolescence and adulthood, these patients may develop neurofibromas. Patients with NF1 may have a vasculopathy that produces vascular stenoses, aneurysms, pseudoaneurysms, rupture, or fistula formation and even in rare instances moyamoya disease. Neurofibromatosis type 1 vasculopathy is usually recognized in childhood or early adulthood. Most patients with NF1 and vasculopathy have multiple affected vessels, with the renal arteries being the most common site. Abdominal aortic coarctation may be seen in association with renal artery stenosis. The incidence of congenital heart defects is approximately 2%, with pulmonic stenosis being the most common. There are also rare reports of conotruncal and left heart obstructive defects (aortic stenosis or coarctation). Monitoring for thoracic and abdominal coarctation and hypertension is important in patients with NF1. PREP Pearls Hypertrophic cardiomyopathy is the most common cardiac abnormality associated with Noonan syndrome with multiple lentigines. Carney complex is associated with cardiac myxomas. Patients with neurofibromatosis type 1 may develop abdominal coarctation and/or renal artery stenosis.

A 14-year-old adolescent girl with no significant medical history developed the sudden onset of tachypalpitations approximately 30 minutes ago and was brought to the emergency department. She reports mild shortness of breath, but no chest pain, dizziness, or syncope. She appears well. She has a heart rate of 240 beats/min, respiratory rate 15 breaths/min, blood pressure of 104/68 mm Hg, temperature of 36.8°C, and oxygen saturation of 98% on room air. There are no carotid bruits. She is breathing comfortably, and her lungs are clear to auscultation. No murmurs are appreciated. Capillary refill time is 2 seconds, and radial and pedal pulses are 1+. Electrocardiography is performed (Figure). Of the following, the BEST next step is to A.apply a bag of ice to the forehead and bridge of nose for 20 to 30 seconds B.apply constant pressure over the right carotid sinus for 5 to 10 seconds C.apply constant pressure with a cold washcloth over both eyes for 20 to 30 seconds D.ask the patient to expire forcefully without letting any air out of her nose or mouth for 15 seconds

D.ask the patient to expire forcefully without letting any air out of her nose or mouth for 15 seconds The patient in this vignette has hemodynamically stable supraventricular tachycardia (SVT). Therefore, prior to attempting to convert with adenosine, it is reasonable to attempt vagal maneuvers. Vagal maneuvers are techniques to transiently increase the parasympathetic activation of the heart, which affects both the sinus node (leading to sinus slowing) and the atrioventricular (AV) node (leading to conduction slowing and an increased AV node refractory period). If the AV node refractory period is sufficiently lengthened so that it leads to second-degree AV block in the setting of the most common types of pediatric SVT (AV reciprocating tachycardia or AV node reentry tachycardia), the SVT is terminated, as the circuit has been "broken." Vagal maneuvers can also be useful in the setting of rapidly conducted and regular atrial flutter by "unmasking flutter waves," when the AV node is blocked. Multiple vagal maneuvers have been described, including the Valsalva and modified Valsalva maneuvers, carotid sinus massage, cold water immersion ("ice to the face," eliciting the "diving reflex"), gagging, and rectal stimulation. A randomized controlled trial comparing the modified Valsalva maneuver (forced expiration against a closed glottis in semirecumbent position for 15 seconds followed by supine positioning and passive leg raise) to the standard Valsalva maneuver (forced expiration against a closed glottis in semirecumbent position alone) found that the modified Valsalva maneuver was more effective in termination of SVT in adult patients (43% vs 17%). Application of ice to the forehead and bridge of the nose for 20 to 30 seconds is frequently effective in the termination of SVT in neonates and infants, whose diving reflex is well preserved. However, this method is less effective in older children and adults. The mouth and nose must not be obstructed during this maneuver. The application of ocular pressure may be effective in terminating SVT; however, it is now considered contraindicated because of the risk of retinal detachment. Applying pressure to the carotid sinus for 5 to 10 seconds can also be effective, although general consensus is that this maneuver should not be used in pediatric patients because of the theoretical risk of carotid artery occlusion and lack of systematic evaluation of its effectiveness and risks in this population. At a minimum, this maneuver should never be performed bilaterally or in patients with carotid bruits. In summary, vagal maneuvers can be trialed in patients with hemodynamically stable SVT. In neonates and infants, application of ice to the forehead/bridge of the nose, rectal stimulation with a thermometer, or gagging are safe and frequently effective. In older children, a Valsalva or modified Valsalva maneuver (which may be more effective) should be used. PREP Pearls A randomized controlled trial performed in adults found that a passive leg raise after Valsalva maneuver in a semirecumbent position is more effective in termination of supraventricular tachycardia than a Valsalva maneuver alone. The proper technique for "ice to the face" is application of a bag of ice to the forehead and bridge of nose for 20 to 30 seconds, without applying ocular pressure or obstructing the mouth and nose. This is frequently effective in infants and neonates but less effective in older children. Carotid massage is not typically recommended in pediatric patients and should never be done bilaterally or in patients with a carotid bruit.

An infant returns to the intensive care unit for recovery after surgical repair of his cardiac defect. The Figure is a representative image of the preoperative transthoracic echocardiogram. Of the following, the MOST common surgical approach used to close the defect is A.left apical ventriculotomy B. right apical ventriculotomy C.right atrial incision D.supravalvar pulmonary incision

D.supravalvar pulmonary incision There are 4 major categories of ventricular septal defects (VSDs) based on their location in the ventricular septum: (1) perimembranous (conoventricular), (2) inlet (atrioventricular canal), (3) muscular, and (4) subarterial (supracristal, conoseptal, outlet). These types of VSDs can readily be distinguished with echocardiography. In addition, the surgical approach to repair each type of VSD may be different and should be recognized because the subsequent complications related to the repair may be different. A supravalvar pulmonary incision would be the best approach for the patient in this vignette. The Figure shows a classic parasternal short axis view looking at the right ventricular outflow tract in relationship to the aorta. This view is often used to identify and distinguish the types of VSDs. A subarterial (supracristal, conoseptal, or outlet) VSD can be identified in the image. Consequently, a common surgical approach to access the VSD is via an incision in the pulmonary artery (transarterial) just above the pulmonary valve. The other common options depend on the type of VSD. A right atrial incision is used if the defect is an inlet or a perimembranous VSD or in some muscular VSDs (midmuscular). An anterior right ventriculotomy approach is used to access muscular VSDs below the moderator band and anterior to the septal band. An apical left ventriculotomy is often used to access a posterior apical muscular VSD. PREP Pearls The surgical approach to repairs of ventricular septal defects varies based on the anatomic location of the defect.

You have been asked to evaluate a 6-month-old infant in the pediatric intensive care unit. She was born at 25 weeks' gestation and has never been discharged from the hospital. She was transferred from the neonatal intensive care unit following tracheostomy placement because of severe chronic respiratory failure from bronchopulmonary dysplasia. Over the course of her hospitalization, she has had serial echocardiograms to evaluate the presence of pulmonary hypertension. The most recent study, 2 weeks before tracheostomy placement, demonstrated moderate to severe ventricular septal flattening and mild tricuspid regurgitation with a peak velocity of 4.2 m/s. In addition, there was physiologic pulmonary insufficiency with an end-diastolic velocity of 2.5 m/s, and there was unobstructed pulmonary venous return. Her blood pressure at the time of the study was 70/45 mm Hg. Based on these findings, your colleague initiated therapy with sildenafil to treat pulmonary hypertension. The pediatric intensive care unit staff has asked for an evaluation because the patient's saturations decrease from the mid-90% range to approximately 80% to 85%, proximal to the administration of the sildenafil doses. They have checked the ventilator, performed multiple chest radiographs, and even performed a bronchoscopy, but none of these approaches have resulted in a significant change in saturations. Echocardiography performed yesterday shows improvement in septal flattening and a decrease in tricuspid regurgitation and pulmonary insufficiency velocities. Despite those findings, there is concern from the pediatric intensive care unit staff that the pulmonary hypertension may have worsened. Of the following, the BEST next step is to A.add a second oral pulmonary vasodilator B.initiate inhaled nitric oxide C.initiate milrinone infusion D.maintain current pulmonary hypertension therapy

D.maintain current pulmonary hypertension therapy Pulmonary hypertension is a common and well-recognized result of extreme prematurity complicated by bronchopulmonary dysplasia (BPD). Premature infants with BPD should be serially screened for pulmonary hypertension, and any clinical concern should prompt echocardiographic evaluation. Although the pathophysiology of BPD-related pulmonary hypertension differs from idiopathic pulmonary hypertension, the evaluation and treatment algorithms are similar. Echocardiographic findings that raise concern for the presence of pulmonary hypertension include: ventricular septal flattening, indicating right ventricular pressure overload; right ventricular enlargement or hypertrophy; right ventricular dysfunction; tricuspid regurgitation velocity greater than 2.5 to 3 m/s; and pulmonary insufficiency gradient greater than 2 m/s. When clinical and echocardiographic evidence of pulmonary hypertension is present, it is recommended that cardiac catheterization be performed to document the degree of pulmonary hypertension and the degree of vasoreactivity of the pulmonary vascular bed prior to the initiation of pulmonary vasodilator therapy, unless the patient is too sick or there are other factors limiting the ability to perform a catheterization, as in the patient in the vignette. The underlying pulmonary disease must be assessed and treated (eg, tracheostomy, inhaled corticosteroids, bronchodilators, and oxygen therapy as indicated) as well as any cardiovascular lesions if present. If there is no improvement then treatment with pulmonary hypertension medications is recommended. Sildenafil, a phosphodiesterase-5 inhibitor, is a reasonable next step for pediatric pulmonary hypertension, although its use is not approved by the FDA for children in the US. Studies of the efficacy of sildenafil in premature infants with BPD-associated pulmonary hypertension are limited, but its use is recommended in the most recent pediatric pulmonary hypertension guidelines. The patient in this vignette has BPD-associated pulmonary hypertension and has been treated with a pulmonary intervention (tracheostomy) and a pulmonary vasodilator (sildenafil). Based on echocardiographic results, she has had a positive response to these treatments with improved measures of pulmonary hypertension. The presence of desaturations after the sildenafil doses, however, has raised concern from the treatment team that her pulmonary hypertension may be clinically worsening. While unnerving, these desaturations are common in patients with BPD-associated pulmonary hypertension because of ventilation/perfusion (V/Q) mismatch. Sildenafil is given systemically and may cause dilation of pulmonary arterioles that are not yet receiving adequate ventilation, thereby creating an intrapulmonary shunt, which would be manifest by hypoxia. This hypoxia typically occurs within 30 to 60 minutes of dose delivery and is self-limited. Careful observation is an appropriate course of action, as the hypoxia typically improves over a short time period as ventilation catches up to perfusion and V/Q mismatching is decreased. Adding a second agent, such as bosentan, is often warranted in BPD-associated pulmonary hypertension, but would not be indicated yet, in this case. The echocardiogram has shown improvement, and the hypoxia can be explained by factors not directly resulting from worsening pulmonary hypertension. Initiation of inhaled nitric oxide can be very effective to treat pulmonary hypertension in the short term, but for a patient with likely long term needs is not preferred. Milrinone, a phosphodiesterase-3 inhibitor, can be effective in treating pulmonary hypertension in the short term, but is not indicated in this patient with normal cardiac function and already on sildenafil. PREP Pearls Pulmonary hypertension is a common complication of bronchopulmonary dysplasia and often requires multiple interventions and medications. Sildenafil is the first-line medical treatment of pulmonary hypertension associated with bronchopulmonary dysplasia. Hypoxia associated with treatment of pulmonary hypertension may not always reflect worsening pulmonary hypertension.

A 14-year-old previously healthy adolescent girl is brought to the emergency department for fever and left-sided chest pain that she has been experiencing for 5 days. Her pediatrician saw her 2 days ago and diagnosed a viral illness. Today, she has persistent fever, shortness of breath, and sharp left-sided chest pain that worsens on inspiration. She appears anxious, leaning forward in the bed. She has a temperature of 38.6°C, heart rate of 120 beats/min, respiratory rate of 40 breaths/min, and blood pressure of 85/70 mm Hg. She has jugular venous distension, distant heart sounds, and oxygen saturations of 95% on room air. An echocardiogram is obtained (Video 1, Video 2, Figure). Of the following, the BEST next step in the management of this patient is to A.initiate colchicine and ibuprofen B.initiate prednisone C.perform emergent pericardial window D.perform emergent pericardiocentesis

D.perform emergent pericardiocentesis The clinical description and echocardiographic findings of the patient in this vignette are consistent with acute pericarditis with a large pericardial effusion progressing to life-threatening cardiac tamponade, which requires emergent pericardiocentesis. The most common presenting symptom in acute pericarditis is chest pain. The pain in pericarditis is usually described as a sharp precordial pain that worsens on inspiration and is relieved by sitting up and leaning forward (tripod position). A pericardial friction rub may be audible, a finding that is highly specific for the diagnosis of acute pericarditis. The pericardial friction rub is a triphasic, superficial, scratchy sound best heard with the diaphragm of the stethoscope at the lower sternal border in the leaning-forward position. Absence of a friction rub does not exclude pericarditis. Large effusions may be less likely to have a rub. Pericarditis may result in a pericardial effusion that can progress to tamponade, as in this vignette. Normally during inspiration, the venous return to the right side of the heart increases because of the reduced intrathoracic pressure. However, in tamponade physiology, an increase in venous return to the right side does not allow the right ventricular free wall to expand because of the increased pericardial pressure from the pericardial fluid. The increased right ventricular volume causes the interventricular septum to bulge to the left. The septal position and the underfilling of the left ventricle reduce its output (consequent to ventricular interdependence). The decreased left ventricular stroke volume can be seen as a drop in systolic blood pressure of more than 10 mm Hg during inspiration. This phenomenon is termed "pulsus paradoxus" and is a sign of imminent cardiovascular collapse. The clinical findings of hypotension, elevated jugular venous pressure with jugular venous distension, distant heart sounds (Beck's triad), and pulsus paradoxus signal the need for emergent intervention in a patient with pericardial effusion. The echocardiographic findings of tamponade include late diastolic right atrial inversion and early diastolic right ventricular collapse (Video 1, Video 3), both of which occur because the intrapericardial pressure exceeds the right atrial and right ventricular diastolic pressure. Respiratory variation in transvalvular flow can be measured via pulsed-wave Doppler echocardiography across the mitral and tricuspid valve. When the difference between the highest and the lowest velocity is more than 25% across the mitral valve or 40% across the tricuspid valve, it is considered indicative of pulsus paradoxus and tamponade physiology (Figure). Inferior vena cava dilation and swinging motion of the heart are additional echocardiographic features of tamponade. When tamponade physiology is noted on clinical examination and confirmed by echocardiography, the next step in the management should be emergent pericardiocentesis. A pericardial window would not be indicated at this point as this is the patient's first presentation with a large effusion. Pericardial windows are performed for recurrent symptomatic effusions that cannot be controlled with medical therapy, and furthermore surgery would take longer to organize. The management of a pericardial effusion without evidence of tamponade physiology depends on etiology and the clinical presentation. Pericarditis results from inflammation of the pericardium.The etiology of acute pericarditis can be infectious (viral disease including HIV, bacterial infections [especially mycoplasma in developing countries], fungal infections, rickettsial infections), inflammatory, metabolic, drug-related, radiation-related, traumatic, postoperative, or idiopathic. If a treatable underlying cause is identified, management should be directed toward the underlying etiology. Although nonsteroidal anti-inflammatory drugs (NSAIDs) are the mainstay of therapy for acute viral pericarditis, recent studies have shown that a combination of an NSAID with colchicine decreased the symptoms and recurrences and was better tolerated than NSAIDs alone. Steroids are primarily indicated for patients with connective-tissue disorders/immune-mediated pericarditis; patients who do not respond to NSAIDs and colchicine; and patients for whom NSAIDs are contraindicated. In most other cases, steroids should be avoided because they are associated with a risk of recurrence and are contraindicated for many types of infections. The treatment for cardiac tamponade is drainage of the pericardial fluid without delay, especially in unstable patients. PREP Pearls Cardiac tamponade is an acute life-threatening complication of pericarditis. Beck's triad (hypotension, elevated jugular venous pressure with jugular venous distension, distant heart sounds) and pulsus paradoxus are indicative of hemodynamic compromise in pericardial effusion. Steroids should be restricted to patients with specific indications.

A 17-year-old female adolescent has recurrent syncopal episodes with exertion. She is evaluated in your clinic. Her physical examination findings are normal. She is taking no medications or supplements. Electrocardiography is performed (Figure). Echocardiography demonstrates a structurally normal heart with normal function and no intracardiac shunting. She is started on nadolol and asked to increase her fluid and salt intake. She continues to have recurrent exertional syncope despite maximizing her nadolol dose and good compliance. Of the following, the BEST next step in her treatment is to A.change nadolol to amiodarone B.change nadolol to sotalol C.perform an electrophysiology study and ablation D.place a transvenous implantable cardioverter-defibrillator

D.place a transvenous implantable cardioverter-defibrillator The electrocardiogram of the patient in this vignette demonstrates a prolonged QT interval consistent with long QT syndrome (LQTS). This syndrome affects approximately 1 in 2,500 individuals and predisposes them to having ventricular arrhythmias, particularly torsades de pointes. At least 16 mutations cause different types of LQTS, with LQTS type 1 being the most common. A mutation in KCNQ1 predisposes a person to having LQTS type 1. Individuals with LQTS type 1 are at risk for developing arrhythmias in particular situations, including exercise (particularly swimming). Patients with LQTS type 2 tend to have events when hearing sudden loud noises, experiencing extreme emotions, or being startled, whereas patients with LQTS type 3 tend to have events during sleep. First-line management of patients with LQTS includes β-blocker therapy, with nadolol being one of the more commonly used drugs, and lifestyle modifications. β-Blockers significantly reduce mortality in patients with LQTS. In patients with recurrent syncope despite maximal β-blocker therapy, the addition of an implantable cardioverter-defibrillator (ICD) to β-blocker therapy should be considered to prevent sudden death (class IIa). Other management options in patients with recurrent syncope despite maximal medical therapy include left cardiac sympathetic denervation. Implantable cardioverter-defibrillator placement is very effective in the prevention of sudden death but it also has associated risks. Acute risks during implantation include bleeding, infection, perforation, lead dislodgement, pneumothorax, and hemothorax. After placement of an ICD, programing should be carefully tailored to the patient to minimize the risk of inappropriate shocks. Inappropriate shocks account for the highest percentage of long-term complications in patients with an ICD and have been linked to the development of post-traumatic stress disorder. Prior to placing a transvenous ICD, cardiac imaging should be performed to evaluate for intracardiac shunting. Presence of intracardiac shunting is a relative contraindication for placement of transvenous ICD secondary to the increased risk of strokes. Amiodarone and sotalol are both class III antiarrhythmics that prolong the QT interval by prolonging cardiac depolarization. These medications are contraindicated in patients with LQTS. Patients with LQTS are at risk for arrhythmias secondary to abnormal cardiac channels, and there is no substrate that is amenable to ablation. PREP Pearls In patients with long QT syndrome and recurrent syncope despite maximal medical therapy, the placement of an implantable cardioverter-defibrillator should be considered. Inappropriate shocks are one of the most common long-term complications in patients with an implantable cardioverter-defibrillator. Intracardiac shunting is a relative contraindication for the placement of a transvenous implantable cardioverter-defibrillator secondary to the increased risk of strokes. Class III antiarrhythmics prolong the QT interval and are contraindicated in patients with long QT syndrome.

A 9-month-old male infant with chronic heart failure is transferred to the cardiac intensive care unit from the stepdown unit for initiation of a milrinone infusion. He has required total parenteral nutrition for the last 10 days because of feeding intolerance related to heart failure. Venous blood gases and serum chemistry are evaluated on his admission to the intensive care unit. Laboratory data are shown: Laboratory Test Result pH 7.31 PCO2 44 mm Hg PO2 39 mm Hg Base deficit −4 mEq/L (−4 mmol/L) Sodium 144 mEq/L (144 mmol/L) Potassium 3.3 mEq/L (3.3 mmol/L) Chloride 114 mEq/L (114 mmol/L) CO2 24 mEq/L (24 mmol/L) Lactic acid 0.9 mmol/L (8.11 mg/dL) Of the following, the BEST next step in managing the patient's metabolic acidosis is to A.administer intravenous furosemide B.administer sodium bicarbonate C.intubate the patient D.reduce chloride in the total parenteral nutrition

D.reduce chloride in the total parenteral nutrition Metabolic acidosis is a common problem in cardiac patients and is often a cause for alarm. It is divided into 2 general categories, defined by the presence or absence of an anion gap. The anion gap is conventionally measured by subtracting the sum of the serum chloride and carbon dioxide levels from the serum sodium level. An anion gap of 12 mEq/L or less is considered normal, with some references reporting up to 16 as normal. This gap represents the difference between physiologic cations and anions that are found in serum but not measured in standard chemistries. The presence of an anion gap greater than 12 mEq/L indicates the presence of nonphysiologic anions that produce a metabolic acidosis. The causes of metabolic acidosis with an anion gap can be remembered by the mnemonic MUDPILES: methanol, uremia, diabetic ketoacidosis, paraldehyde, isoniazid, lactic acidosis, ethanol/ethylene glycol, and salicylates. An anion gap acidosis in a patient who has heart failure should raise concern for lactic acidosis. Tissue hypoperfusion from decreased oxygen delivery related to poor heart function is the primary cause of lactic acidosis in this population. If the lactic acid level is normal in the presence of an anion gap, other causes of acidosis should be investigated. Metabolic acidosis with a normal anion gap is typically caused by losses of serum bicarbonate (as seen in acute diarrheal illnesses or renal tubular acidosis) or excess of serum chloride. Patients with acute diarrheal illnesses will have ongoing volume losses and a metabolic acidosis related to bicarbonate losses in the stool. Renal tubular acidosis can be evaluated by measuring a urine pH and electrolytes to determine whether there is inappropriate bicarbonate excretion in the presence of metabolic acidosis. Metabolic acidosis resulting from excess serum chloride is often iatrogenic in nature. It can occur after large volumes of sodium chloride are administered during fluid resuscitation or in the treatment of cerebral edema. It also can be caused by high levels of chloride in parenteral nutrition. The patient in this vignette has a metabolic acidosis (pH 7.31 and base deficit of −4 mEq/L). The lactic acid level and the anion gap (144 − 114 − 24 = 6) are normal, which eliminates the causes of increased anion gap metabolic acidosis. The chloride level is 114 mEq/L (114 mmol/L), which is elevated and the most likely cause of the metabolic acidosis. The parenteral nutrition that the patient has required for the preceding 10 days is elevating the serum chloride level. The most definitive treatment would be to decrease the chloride level in the total parenteral nutrition and use a buffer such as sodium acetate instead. Over the course of 1 to 2 days, the chloride level will typically return to normal and the metabolic acidosis will resolve. Furosemide is a potential treatment for hyperchloremia, as loop diuretics cause chloride losses from the distal loop of Henle. However, this treatment is not definitive because the elevated chloride levels would persist if the total parenteral nutrition formula was not changed. Intubating the patient would not be indicated, as there is no respiratory acidosis and it does not appear that the metabolic acidosis is caused by poor oxygen delivery and increased metabolic demand, both of which could potentially be helped by intubation and mechanical ventilation. Sodium bicarbonate is often used to treat metabolic acidosis, especially in the critically ill patient, but it does not address the underlying cause of the acidosis. In the presence of hyperchloremic metabolic acidosis, treatment with sodium bicarbonate will often have negligible effects on the acidosis, because it elevates the serum sodium level and, thus, causes parallel elevation in the chloride level and perpetuates the acidosis. PREP Pearls Determination of the anion gap is crucial in the evaluation and treatment of metabolic acidosis. The causes of metabolic acidosis with an anion gap can be remembered by the mnemonic MUDPILES: methanol, uremia, diabetic ketoacidosis, paraldehyde, isoniazid, lactic acidosis, ethanol/ethylene glycol, and salicylates. Hyperchloremic metabolic acidosis is often iatrogenic.

You are called for a consultation on a 6-week-old black infant who was brought to the emergency department for constipation, but whose oxygen saturation was incidentally found to be 65% on room air. You have very little history from the family, except that she was diagnosed as a newborn with a "type of heart problem that would require several surgeries" and that her oxygen saturation was 90% on hospital discharge at 1 week of age. Her family also has a copy of her newborn electrocardiogram (Figure 1) and one still clip from her newborn echocardiogram (Figure 2). Unfortunately, she missed her 2-week follow-up visit with her cardiologist at a different institution due to maternal illness. Her parents had not noticed any recent color change, and they report that she has not had any invasive diagnostic tests or surgical intervention. The infant appears well developed and well nourished. She weighs 5 kg. She is comfortable and has mild tachypnea. Her blood pressure is 75/50 mm Hg in the right and left arms and 80/50 mmHg in her right leg. She has equal and normal breath sounds bilaterally. A brief cardiac examination reveals a quiet precordium with a soft high-frequency systolic murmur. She has no hepatomegaly, and her distal pulses and perfusion are normal. Of the following, the anatomy that BEST explains this child's clinical appearance is A.restrictive atrial septal defect in the context of tricuspid atresia with dextro-transposition of the great arteries B.restrictive atrial septal defect in the context of tricuspid atresia with normally related great arteries C.restrictive ventricular septal defect in the context of tricuspid atresia with dextro-transposition of the great arteries D.restrictive ventricular septal defect in the context of tricuspid atresia with normally related great arteries

D.restrictive ventricular septal defect in the context of tricuspid atresia with normally related great arteries The infant in this vignette, although otherwise well developed and well appearing, has congenital heart disease characterized by significant desaturation. Her electrocardiogram (Figure 1) reveals a normal sinus rhythm with left superior axis deviation. The deep S in V1 suggests prominent left sided forces. Left axis deviation, in addition to right atrial enlargement and prominent left sided forces (not all shown on this electrocardiogram) are the classic findings in infants with tricuspid atresia. Figure 2 represents an apical 4-chamber echocardiographic image demonstrating an atretic tricuspid valve with a hypoplastic right ventricle; the restrictive ventricular septal defect (VSD) can be seen in a parasternal short-axis transthoracic echocardiography image clip (Video). The infant in this vignette has classic features of tricuspid atresia with normally related great arteries and a VSD that has rapidly become restrictive during early infancy (desaturations and a high-frequency, usually holosystolic murmur), thereby leading to decreased pulmonary blood flow while systemic outflow from the left ventricle to the aorta is preserved. This infant probably had a balanced circulation at birth in the context of an unrestrictive VSD, and the VSD restriction has probably evolved over the past several weeks. Because the infant is dark skinned, the progressive cyanosis may be difficult to appreciate by family members as well as clinicians, in an otherwise well-appearing and asymptomatic infant. By contrast, in tricuspid atresia with dextro-transposition (D-transposition) of the great arteries, the development of a restrictive VSD can lead to decreased systemic cardiac output because the flow to the aorta becomes diminished; these patients can present with cardiogenic shock and desaturation, as opposed to the infant in this vignette who has cyanosis with clinical evidence of normal cardiac output. Tricuspid atresia is an uncommon form of congenital heart disease based on its prevalence, but is the third most common form of cyanotic congenital heart disease in newborns. In tricuspid atresia, there is no direct communication between the right atrium and the right ventricle, which is usually hypoplastic to varying degrees. As the parents of this infant were told, patients with tricuspid atresia will typically require staged open-heart surgical procedures culminating in a Fontan palliation because of their functional "single-ventricle" physiology. Tricuspid atresia has been divided into 3 categories based on the great artery relationship and the presence of additional complex lesions. Type I corresponds to tricuspid atresia with normally related great arteries so that pulmonary flow is dependent on flow through the hypoplastic right ventricle (or a left-to-right patent ductus arteriosus [PDA]), and type II corresponds to tricuspid atresia with D-transposition of the great arteries so that systemic aortic flow is dependent on flow through the hypoplastic right ventricle (or a right-to-left PDA). Tricuspid atresia types I and II are further subdivided based on the presence of a VSD and/or degree of pulmonary stenosis or atresia. Type III corresponds to tricuspid atresia with D-transposition of the great arteries in the presence of other associated complex cardiac abnormalities. The type of tricuspid atresia (ie, presence and size of the VSD, great artery relationship, and degree of pulmonary stenosis or pulmonary atresia) will determine the clinical presentation, ranging from "balanced" physiology with proportional and adequate systemic and pulmonary blood flow, to pulmonary overcirculation, or to cyanosis related to limited pulmonary blood flow. The location of the VSD in tricuspid atresia is often in the muscular portion of the ventricular septum, and is particularly susceptible to the development of VSD restriction. Obligate right-to-left atrial shunting is present in all cases of tricuspid atresia. However, a restrictive atrial septal defect is an uncommon occurrence in tricuspid atresia. When it does occur, patients will present with marked hepatomegaly and can develop both decreased systemic and pulmonary blood flow, warranting immediate intervention. Because the infant in this vignette does not have hepatomegaly or signs of decreased systemic blood flow, there is no evidence of a restrictive atrial septal defect. Patients with tricuspid atresia and D-transposition of the great arteries could present with decreased systemic outflow in addition to cyanosis when a PDA closes if systemic output were dependent on PDA flow. This would also be unusual in an otherwise well-appearing 6-week-old infant, whereas tricuspid atresia with evolving VSD restriction in the context of normally related great arteries would be more likely at the age of 6 weeks. PREP Pearls Tricuspid atresia with normally related great arteries and a restrictive ventricular septal defect can present as marked cyanosis in an otherwise asymptomatic young infant. Tricuspid atresia with a restrictive atrial septal defect is uncommon, typically presenting with marked hepatomegaly and decreased systemic output.

You are seeing a 6-year-old boy whose father was recently diagnosed with hypertrophic cardiomyopathy following a syncopal episode. His father had ventricular tachycardia detected by Holter monitoring and now has an implantable cardioverter defibrillator. The boy's parents report that he gets more easily winded than other children when playing hard and that he coughs when winded. He has had no other symptoms. The only other pertinent family history is that his father had asthma as a child. The results of the boy's physical examination are normal. Electrocardiography demonstrates a normal sinus rhythm and possible left ventricular hypertrophy. Of the following, the BEST noninvasive imaging modality for further evaluation at this time is A.cardiac magnetic resonance imaging with gadolinium enhancement B.cardiac magnetic resonance imaging without gadolinium enhancement C.transthoracic echocardiography with stress imaging D.transthoracic echocardiography without stress imaging

D.transthoracic echocardiography without stress imaging The initial noninvasive imaging test of choice for screening of patients of any age in whom there is concern for hypertrophic cardiomyopathy or other conditions/cardiomyopathies that can lead to heart failure is transthoracic echocardiography (TTE) without stress imaging. In most instances, a complete TTE will provide all of the needed information on cardiac morphology and systolic and diastolic function to diagnose a cardiomyopathy and differentiate between the types of cardiomyopathies (eg, hypertrophic, dilated, restrictive, and left ventricular noncompaction). The diagnosis of arrhythmogenic right ventricular cardiomyopathy may be more challenging by TTE, and cardiac magnetic resonance (CMR) imaging may be helpful as an adjunct. The addition of stress testing is not initially indicated and may be contraindicated in some circumstances that TTE could identify. The addition of stress to echocardiography may be useful to bring out exercise-induced gradients on a subsequent evaluation if this is a clinical question. However, stress testing is not recommended in patients with hypertrophic cardiomyopathy with a peak instantaneous Doppler gradient of 50 mm Hg or greater. Cardiac magnetic resonance imaging is indicated if the TTE is inconclusive. In addition, if there is concern about right ventricular function or abnormalities, CMR imaging is better at assessing the right ventricle than echocardiography. The addition of gadolinium to CMR imaging can be useful to look for evidence of scarring, particularly in hypertrophic cardiomyopathy, as an additional method of risk stratification, although the data supporting this practice in children are limited. Cardiac magnetic resonance imaging may be helpful in differentiating etiologies of secondary cardiomyopathies, as different patterns may be seen in Fabry disease, amyloidosis, and LAMP2 cardiomyopathies in hypertrophic cardiomyopathy or an appearance suggesting myocarditis in dilated cardiomyopathy. Cardiac computed tomography delineates the coronary arteries better than echocardiography, but it is generally not needed for the diagnosis and follow-up of cardiomyopathies unless there is a specific question about the coronary origins, ostia, or course. If there is a question of coarctation or other point of obstruction in the thoracic aorta that may be contributing to left ventricular hypertrophy or dilation and dysfunction, and the aorta is not well seen by echocardiography, computed tomography angiography or CMR imaging would be indicated. In children, radionuclide angiography (eg, multigated acquisition scan) has become virtually obsolete, partly because of the radiation exposure. PREP Pearls The screening test of choice for the diagnosis of dilated cardiomyopathy, hypertrophic cardiomyopathy, or restrictive cardiomyopathy is transthoracic echocardiography. Cardiac magnetic resonance imaging is indicated if the echocardiogram is inconclusive or if additional clinical questions arise, as it may be useful for underlying diagnostic purposes and for risk stratification when gadolinium is used, particularly in hypertrophic cardiomyopathy. Cardiac computed tomography is generally not necessary in the assessment of cardiomyopathies unless there are unanswered questions in regards to the coronary artery origins and course, or suspicion of aortic anomalies causing secondary effects on the heart, and visualization is inadequate by echocardiography.

A 15-year-old previously healthy adolescent boy experiences an episode of palpitations during soccer practice. Electrocardiography is performed (Figure 1). Following conversion to sinus rhythm, the patient is referred for an electrophysiology study and possible ablation. During the electrophysiology study, the following arrhythmia is induced (Figure 2). Of the following, the MOST likely mechanism of his tachycardia is supraventricular tachycardia secondary to A.ectopic atrial tachycardia B.left-sided concealed accessory pathway C.right-sided concealed accessory pathway D.typical atrioventricular node reentry tachycardia

D.typical atrioventricular node reentry tachycardia The electrocardiogram (Figure 1) of the patient in this vignette demonstrates a narrow complex tachycardia with a short RP relationship. The RP relationship is determined by drawing a line at the midpoint of the R-R interval. If the P wave falls in the first half of the R-R interval, it is a short RP tachycardia. If the P wave falls in the second half, it is a long RP tachycardia. The differential diagnosis for a short RP tachycardia includes accessory pathway-mediated reentrant supraventricular tachycardia, typical atrioventricular node reentry tachycardia, and junctional tachycardia. The intracardiac electrocardiogram (Figure 2 and Figure 3) demonstrates a narrow complex tachycardia with 2:1 atrioventricular conduction. On the beats with conduction down to the ventricle, you can see the classic H-A-V pattern that is characteristic of typical atrioventricular node reentry tachycardia. On the beats with no conduction down to the ventricle, you can see a His signal before the A with no V signal. In accessory pathway-mediated supraventricular tachycardia, the atrium, atrioventricular node (His), and ventricle are all obligatory parts of the circuit. You cannot have an accessory pathway-mediated reentrant tachycardia when any of these parts of the circuit are not involved. The fact that there are more A's than V's rules out an accessory pathway-mediated reentrant tachycardia. Although you can see ectopic atrial tachycardias with variable atrioventricular conduction or 2:1 atrioventricular conduction, ectopic atrial tachycardia has a long RP relationship. If this tachycardia was atrially driven (as in ectopic atrial tachycardia), the block to the ventricles is occurring following the His activation, implying that the block is infra-hisian in nature. Infra-hisian block is typically seen in patients with atrioventricular node disease, which would be uncommon in a previously healthy teenager. PREP Pearls Narrow complex tachycardias should be classified by their RP relationship into short RP versus long RP tachycardias The differential diagnosis for short RP tachycardias includes accessory pathway-mediated reentrant supraventricular tachycardia, typical atrioventricular node reentry tachycardia, and junctional tachycardia. Typical atrioventricular node reentry tachycardia has a classic H-A-V pattern in intracardiac electrograms.

A 14-year-old adolescent girl with severe mitral valve regurgitation, moderate mitral valve stenosis, and severely decreased left ventricular ejection fraction (28% ejection fraction by 2-dimensional echocardiography) undergoes mitral valve replacement. She returns to the cardiac intensive care unit (ICU) intubated and on a study drug, levosimendan, which was initiated at 0.1 µg/kg/min in the operating room just prior to coming off cardiopulmonary bypass. Of the following, the use of levosimendan in this patient will MOST likely result in A.decreased mortality and decreased ICU length of stay B.decreased mortality and unchanged ICU length of stay C.unchanged mortality and decreased ICU length of stay D.unchanged mortality and unchanged ICU length of stay

D.unchanged mortality and unchanged ICU length of stay Calcium channel sensitizers, also known as inodilators, increase myocardial contractility by sensitizing the contractile apparatus to calcium, resulting in increased contractile force without actually increasing intracellular calcium. The 3 primary drugs in this class are levosimendan, pimobendan, and omecamtiv. Levosimendan works primarily by binding to troponin C, increasing its affinity for calcium and subsequently stabilizing the molecule, resulting in increased inotropy. It also causes vasodilatory effects on arteries and veins by opening ATP-sensitive K+ channels and other K+ channels, resulting in hyperpolarization and relaxation of vascular smooth muscle. Pimobendan improves inotropy through a similar mechanism of calcium sensitization as levosimendan in combination with phosphodiesterase inhibition. Omecamtiv, a cardiac myosin activator agent, has been more recently discovered and facilitates actin-myosin cross-bridge formation increasing the number of myosin heads involved in force generation and stimulates myosin ATPase. Of these 3 drugs, levosimendan has been most widely used and studied, predominantly outside of the United States. These agents are not currently approved for use by the Food and Drug Administration. Levosimendan is available for human trials in the United States. Because this class of drugs has the ability to sensitize the myocardial contractile apparatus to calcium without increasing intracellular calcium, as opposed to milrinone or other drugs that act primarily via increase of cyclic AMP, there are a number of proposed benefits. These drugs are thought to be less arrhythmogenic since intracellular calcium homeostasis is not disturbed. Myocardial contractility is increased without increase in oxygen consumption. Increased intracellular calcium has been associated with accelerated adverse myocardial remodeling and apoptosis, which is avoided with this class of drugs. Because of these and other proposed benefits, calcium sensitizers have been extensively studied. Levosimendan is the most widely used and studied of the 3 drugs for use in decompensated acute or chronic heart failure. Outcomes of such studies have not been as robust as the proposed benefits would suggest. In May 2017, Landoni et al published a multicenter, randomized, double-blind, placebo-controlled trial involving adult patients in whom perioperative hemodynamic support was indicated after cardiac surgery. Patients were randomly assigned to levosimendan or placebo for up to 48 hours or until discharge from the intensive care unit (ICU). The primary outcome was 30-day mortality, with secondary outcomes being duration of ICU length of stay, mechanical ventilation, and overall hospital stay. After enrollment of 506 patients, the trial was stopped due to futility after interim analysis showed no difference in primary or secondary outcomes. Earlier studies have suggested an improvement in mortality but larger more robust studies since 2014 have failed to reproduce these results. While less data is available in the pediatric population, an August 2017 Cochrane database systematic review examined the utility of prophylactic levosimendan for prevention of low cardiac output syndrome and mortality in pediatric patients undergoing surgery for congenital heart disease. Five randomized controlled trials including 212 patients (all younger than 5 years) met criteria for analysis. The review revealed no significant difference in low cardiac output syndrome, mortality, or ICU length of stay in any of the patients treated with levosimendan. Thus, based on current data, despite the promising pharmacological profile of levosimendan and other calcium sensitizers, their use has not been advised or widely accepted in the United States because there has been no data to support clear benefits of their use in adult or pediatric patients after cardiac surgery. PREP Pearls Calcium sensitizers increase contractile force by increasing sensitivity of the myocardial contractile apparatus to calcium without increasing intracellular calcium. Recent studies have not shown a benefit in mortality, intensive care unit length of stay, or prevention of low cardiac output syndrome in patients treated with calcium sensitizers post operatively.

A 3-day-old male neonate with an unbalanced atrioventricular septal defect and pulmonary atresia underwent placement of a 3.5-mm modified Blalock-Taussig shunt today and returned from the operating room a few hours ago. You are with the on-call fellow at the patient's bedside. The patient has a heart rate of 160 beats/min, blood pressure of 60/35 mm Hg, and oxygen saturation of 68% on positive-pressure ventilation with a fraction of inspired oxygen of 0.5. The hematocrit is 45%. You discuss the general medical and surgical principles that increase pulmonary blood flow in patients with shunts, as well as the applicable hemodynamic principles. Of the following, the theoretical intervention that will BEST lead to the largest increase in pulmonary blood flow is A.increasing the blood pressure to 70/40 mm Hg B.replacing the 3.5-cm length shunt with a 4-cm length shunt C.transfusing packed red blood cells to a hematocrit of 50% D.upsizing to a 4-mm diameter shunt

D.upsizing to a 4-mm diameter shunt The intervention that will lead to the greatest increase in pulmonary blood flow would be increasing the diameter of the shunt from 3.5 to 4 mm. The flow of fluid through a tube can be described by Poiseuille's law, which describes how the pressure difference across the tube and the resistance to flow through the tube affects flow of the fluid through the tube: [see attached image for the formula] The law applies to the laminar and steady flow of a Newtonian fluid through a system, so this equation does not perfectly predict the flow of blood through blood vessels, but the general relationships still apply. An increase in the diameter of a shunt from 3.5 to 4 mm is an increase in the radius of about 14%, but will lead to an increase in flow of about 70%, based on Poiseuille's equation. Pressure difference and flow are directly related, so an increase in pressure difference of 10 mm Hg in this patient, or an increase of about 17%, will result in about a 17% increase in flow. Viscosity and length are inversely proportional to flow, so an increase in either will result in a decrease in flow. Resistance is most sensitive to changes in the radius of the vessel; thus, this has the largest effect on flow through that vessel. PREP Pearls Flow through blood vessels is influenced by pressure and resistance. Poiseuille's equation describes how pressure, vessel diameter and length, and viscosity affect flow. Small changes in vessel diameter can have significant effects on the flow through vessels.

A 2-year-old girl with transposition of the great arteries and usual coronaries, who has undergone repair with an arterial switch and Lecompte maneuver, presents to the emergency department (ED) with increased work of breathing and purple lips. She had been sitting while playing and was noted to be moaning and very pale. At her last cardiology evaluation 9 months ago, she had no symptoms, a normal electrocardiogram, and normal right ventricle pressures and left ventricle function on echocardiogram. While in the ED, she develops a wide complex tachycardia, proceeds to cardiac arrest, and is emergently placed on extracorporeal membrane oxygenation. An aortic root angiogram shows no flow to the left coronary artery, which could not be engaged for selective imaging. In the operating room, the coronary origins appear patent on probing with no ostial stenosis. Of the following, the BEST possible intervention to resolve the obstruction is A. applying local thrombolysis B. bypass graft surgery C. coronary re-implantation D. intraoperative coronary stenting E. manipulating the main pulmonary artery position

E. manipulating the main pulmonary artery position For the girl in the vignette, manipulating the main pulmonary artery position would be the best intervention to resolve the obstruction. The arterial switch results in the main pulmonary artery and proximal branch pulmonary arteries draping anterior to the aorta. This could result in compression of the left coronary artery as it traverses between the great vessels anteriorly, resulting in coronary obstruction either early or late after the arterial switch. Intervention to the coronary artery itself then would not benefit the patient. To resolve the LCA compression, tacking the pulmonary artery to the sternum to pull it off the aorta or manipulating the PA position is required. Late coronary complications after an arterial switch have been reported in 8% to 10% of cases. Although some anatomic substrates (ie, intramural course) have a higher risk for coronary complications, the case in the vignette expands the differential away from internal coronary abnormalities and to consider external coronary compression. A number of direct coronary interventions may be helpful in the setting of ostial stenosis, but the probe patency points away from coronary intervention or toward something happening further down the coronary. It has been reported that a more anterior or a higher reimplantation of the left coronary artery increases the risk for coronary obstruction from compression. PREP Pearls Consider coronary artery compression by the pulmonary artery as an early or late complication after the arterial switch when faced with coronary obstruction. Anterior and higher coronary reimplantations may be at higher risk for compression. There are no guidelines for surveillance coronary imaging after the arterial switch and practices are site specific.

A 4-day-old full-term infant with hypoplastic left heart syndrome returns from the operating room after undergoing stage I palliation with aortic arch reconstruction and a 3.5-mm Blalock-Taussig shunt. The patient is intubated and sedated, and his muscles are still relaxed. On physical examination, the heart rate is 170 beats/min, respiratory rate on ventilator is 25 breaths/min, and blood pressure is 55/25 mm Hg, with a mean arterial pressure of 35 mm Hg. The patient is warm with a brisk capillary refill and bounding pulses. Oxygen saturation is 70% on 21% fraction of inspired oxygen. The initial arterial blood gas findings are as follows: pH, 7.32; partial pressure of carbon dioxide, 54 mm Hg (7.2 kPa); partial pressure of oxygen (PaO2), 32 mm Hg (4.7 kPa); bicarbonate, 28 mEq/L (28 mmol/L); base excess, +0.8. A venous saturation measurement from a right internal jugular central venous catheter is 50% and lactate is 1.8 mmol/L. The postoperative chest x-ray reveals that all lines and tubes are in good position and the lung fields are clear without infiltrates. Inotropic support includes milrinone at 0.75 μg/kg per minute and epinephrine at 0.02 μg/kg per minute. Of the following, the therapeutic intervention MOST likely to immediately increase PaO2 and blood pressure is A. decrease milrinone to 0.25 µg/kg per minute B. increase epinephrine to 0.05 µg/kg per minute C. start inhaled nitric oxide at 20 ppm D. start dopamine at 5 µg/kg per minute E. start vasopressin at 0.02 mU/kg per minute

E. start vasopressin at 0.02 mU/kg per minute In a patient with single ventricle physiology immediately after stage I palliation with a right ventricular-to-pulmonary shunt, pulmonary vascular resistance and systemic vascular resistance govern the balance of pulmonary versus systemic blood flow (Qp:Qs). The goal is to maintain a balanced circulation with an equal distribution of blood flow between the pulmonary and systemic vascular beds. Manipulation in pulmonary or systemic vascular resistance will alter this balance. Based on the information provided in the vignette, the appropriate clinical inferences need to be made regarding the patient's Qp:Qs as well as his overall cardiac output (ie, low, normal, high). In calculating the patient's Qp:Qs, one must first determine the systemic venous saturation (50%, as provided in the vignette), systemic arterial saturations (70%, as provided in the vignette), pulmonary artery saturation, and pulmonary venous saturation. Because the patient has had a Blalock-Taussig shunt as a part of the stage I Norwood palliation, the only source of pulmonary blood flow is the shunt, which receives blood flow from the systemic circulation. Consequently, the saturation in the pulmonary artery is the same as that in the systemic artery, 70%. Finally, in this patient who is receiving a fraction of inspired oxygen of 21% and has normal chest x-ray findings, one can assume that pulmonary veins are normally saturated at close to 100%. With this information, Qp:Qs can be calculated as follows: Qp = (Systemic artery saturation - Systemic vein saturation) / Qs = (Pulmonary vein saturation - Pulmonary artery saturation) Qp = (70- 50) = 20/ Qs = (100-70) = 30 Qp:Qs = 0.67:1 Next, one must make a determination regarding the patient's cardiac output. On examination, the patient is warm and well perfused, with a brisk capillary refill and bounding pulses suggestive of peripheral vasodilation. In addition, the venous saturation is 50%, with a systemic saturation of 70% yielding an arteriovenous saturation difference of 20, which represents a normal or increased cardiac output. This information is further corroborated by the fact that in spite of hypotension and low systemic saturation, the patient's lactate concentration is relatively low at 1.8 mmol/L (16.2 mg/dL). Thus the patient has normal pulmonary vascular resistance but his systemic vascular resistance is low, likely secondary to vasodilation. This vasodilation may be caused by rapid rewarming after cardiopulmonary bypass or peripheral vasodilators such as low-dose epinephrine and milrinone. Starting vasopressin treatment at 0.02 mU/kg per minute is the best option for the infant in the vignette. Vasopressin is a neurohypophysial hormone that directly stimulates smooth muscle V1 receptors, resulting in vasoconstriction with a half-life of 10 to 20 minutes. The patient in the vignette has low systemic vascular resistance. Starting vasopressin will lead to a steady state of infusion in 15 to 20 minutes, with subsequent vasoconstriction and increased systemic vascular resistance. Increasing systemic vascular resistance would result in increased pulmonary blood flow through the Blalock-Taussig shunt and re-equilibration of the Qp:Qs closer to 1:1 from 0.67:1. One would expect to see these changes exhibited clinically by a fairly immediate rise in oxygen saturation, partial pressure of oxygen (PaO2), and blood pressure. Although decreasing the milrinone dose would likely increase systemic vascular resistance and improve the Qp:Qs balance, the half-life of milrinone in infants is on average 2 hours. Consequently, the effects of decreasing the milrinone dose would not be realized for at least 2 to 4 hours. Increasing epinephrine to 0.05 μg/kg per minute would likely increase blood pressure if the main pathophysiology included decreased cardiac output, which is not the case in the vignette. In addition, at doses of 0.05 μg/kg per minute, the beta-agonist effects of epinephrine predominate, which would likely not increase systemic vascular resistance. At higher doses when the alpha-agonist effects predominate, systemic vascular resistance would likely be increased. Inhaled nitric oxide decreases pulmonary vascular resistance. In the patient in the vignette, whose pulmonary vascular resistance is expected to be near normal, addition of inhaled nitric oxide would not likely further alter hemodynamics in the effect desired. Like epinephrine, dopamine has a dose-responsive receptor activation. At 5 μg/kg per minute beta receptor activation predominates, which would positively augment cardiac output. This patient has normal to high cardiac output, so dopamine would not address the underlying pathophysiology. PREP Pearls In cases with low oxygen saturation accompanied by evidence of normal cardiac output, consider low systemic vascular resistance as the underlying pathophysiology. In cases with increased oxygen saturation accompanied by poor perfusion/cardiac output after stage I palliation, consider decreased pulmonary vascular resistance/pulmonary over circulation as the underlying pathophysiology.

A 7-week-old infant presents to the emergency department (ED) with tachypnea, cough, retractions, and decreased oral intake that has been worsening over the last 4 days. His temperature has felt high to his parents. He is afebrile. His 4-year-old sibling came home from day care with a fever, runny nose, and cough earlier in the week. In retrospect, the parents think that the infant has been breathing faster and not eating quite as well in the last 2 to 3 weeks, with some increase in diaphoresis. He has been fussy off and on, and worsened in the last few days. The ED obtained a chest radiograph that showed hyperinflation with cardiomegaly, vascular congestion, and left lower lobe atelectasis. A rapid nasal winter viral panel is pending when you see the patient. The only other history of note is that a maternal great uncle had a heart murmur and died before school age, but his mother has no other information. The patient's birthweight was 3.4 kg, and his current weight is 3.8 kg. On physical examination, the infant is fussy and has tachypnea with subcostal retractions. His heart rate is 170 beats/min and respiratory rate is 72 breaths/min. Four extremity blood pressures are normal for age. An examination of the head, eyes, ears, nose and throat finds acyanotic mucosa, and intermittent nasal flaring. His chest is symmetric with subcostal retractions; lungs display scattered rhonchi throughout. An active precordium is noted with a laterally and inferiorly displaced point of maximal impulse. S1 was normal; splitting of S2 was difficult to discern; and II-III/VI holosystolic murmur was loudest at the lower left sternal border. A diastolic rumble was noted. No rubs or clicks were evident. The liver was 3 cm below the right costal margin. The spleen was not palpable. Extremities were warm. No clubbing, cyanosis, or edema were found. Neurologic examination shows the infant to be fussy, and moves all extremities with normal tone. Of the following, the MOST likely cardiac diagnosis for the infant in the vignette is: A. anomalous left coronary artery from the pulmonary artery B. familial dilated cardiomyopathy with mitral regurgitation C. myocarditis with mitral regurgitation D. truncus arteriosus E. ventricular septal defect

E. ventricular septal defect Ventricular septal defects (VSDs) reportedly constituted 20% to 30% of congenital heart disease, occurring in 1.35 to 3.5 per 1,000 live births. After the advent of echocardiography, the recognition of VSDs increased to 5 to 50 per 1,000 live births. Because VSDs are significantly more common than the other pathologies listed, this is the most likely cause of this infant's heart failure. The majority of children hospitalized with heart failure have some form of congenital heart disease, with most of these presenting before age 1 year. The murmur noted in the patient in the vignette is consistent with a large VSD. Anomalous left coronary artery from the pulmonary artery (ALCAPA) is rare, occurring in approximately 1 in 300,000 infants. It typically presents in early infancy when the pulmonary vascular resistance falls, resulting in diminished prograde flow into the left coronary as well as perfusion with desaturated blood with resultant ischemia. The ischemia often affects the mitral valve papillary muscles with resultant mitral valve dysfunction and regurgitation. Infants develop signs and symptoms of heart failure, with irritability and diaphoresis exacerbated by eating (angina episodes). Overall dilated cardiomyopathy in childhood has an incidence of slightly more than 1 in 100,000 children, with approximately 50% presenting in the first year after birth. Most present with signs and symptoms of heart failure. About 15% to 19% of children have a positive family history at the time of presentation. A murmur of mitral regurgitation is heard, usually caused by mitral annular dilation in association with left ventricular dilation. The mitral valve then becomes incompetent and leaks. Although this is in the differential, statistically it is less likely than congenital heart disease associated with left-to-right shunts and resultant heart failure. The true incidence of myocarditis is unknown and is probably more common than recognized. In the most fulminant forms, the left ventricle may not be dilated, but presentations are variable, and acute onset of symptoms of heart failure is common. Mitral regurgitation may be present. More than half of all cases of myocarditis are reported in infants younger than 1 year. Newborns and infants are more likely to be severely affected than older children, often presenting with cardiogenic shock. Myocarditis is the most frequent cause identified in children with known causes of dilated cardiomyopathy, with the second most common identified cause being neuromuscular disorders. This remains in the differential, but is not the most likely diagnosis for the infant in the vignette. The reported incidence of truncus arteriosus ranges from 3 to 10 per 100,000 live births. Infants typically present in the first days to weeks after birth depending on the fall in pulmonary vascular resistance and degree of truncal insufficiency. These infants frequently have a systolic click resulting from the truncal valve. The second heart sound is typically loud and single. The associated murmur is holosystolic and is among the signs and symptoms of heart failure. A truncal valve insufficiency murmur may be heard as well as diastolic flow rumble across the mitral valve.


Set pelajaran terkait

Which statement about fat is correct

View Set

HIST1111 - World History I - Lesson 2 Textbook Assignment

View Set

BUS X100 Ch. 1-4 Practice Questions

View Set

CUT 7e CH03 Working with System Software and File Storage

View Set

Troubleshooting Wireless Connection

View Set

Chapter 20 Assessment of respiratory

View Set